Immunology Final X 2021

Download as pdf or txt
Download as pdf or txt
You are on page 1of 491
At a glance
Powered by AI
The document discusses concepts related to the immune system including antibodies, T cells, B cells, cytokines and more.

Lectin- microbial sugars

B-cell receptor VDJ gene rearrangement

1. Which of the following statements about type I hypersensitivity is true?

a. It is also known as delayed-type hypersensitivity


b. it occurs when an IgE response is directed against pollens
c. it does not require IgE-sensitized mast cells
d. it is independent of pharmacologic mediators

2. Cessation of the immune response is usually mainly a result of?


a. Release of IL-10 and transforming growth factor beta
b. Immune cells binding to PAMPs and downregulating the innate immune response
c. Elimination of the pathogen
d. Immune cells binding to DAMPs

3. Antibodies that are able to provide protection from the effects of bacterial exotoxins do
so in which one of the following ways?
a. activate complement that, when activated, breaks down the toxin molecules into
harmless fragments
b. enable macrophage to present the toxin antigens to T-lymphocytes
c. bind to the toxin molecules and prevent the toxins from binding to body cell
surfaces

nolan x soju
d. cause blood vessels located near the site of the toxin to become slightly leaky, thus
allowing cells in the blood stream to enter the tissues and destroy the toxin
molecules

4. What are the two classes of antibody that are expressed together on the surface of
naive B- cells?
a. IgG and IgD
b. IgG and IgE
c. IgM and IgD
d. IgM and IgG

5. Germinal centers are the location of..?


a. T-cell receptor VDJ gene rearrangement
b. T-cell somatic hypermutation
c. B-cell receptor VDJ gene rearrangement
d. T follicular helper (Tfh) cells
6. The final damage to vessels in immune complex-mediated arthritis is due to
a. Histamine
b. NK cell
c. lysosomal enzymes of polymorphonuclear leukocytes
d. cytotoxic T cell

7. IgG binding to neutrophils is mediated by:


a. Fc-dependent cellular homing mechanisms
b. sensitization of mast cells and basophils
c. Fc receptors specific for IgG
d. ICAM’s

8. Which receptor does NOT vary greatly from person to person?


a. BCR
b. MHC
c. TCR
d. PRR

nolan x soju
9. A diagnostic marker for tumors of the colon is:
a. Alpha-fetoprotein
b. Common acute lymphoblastic leukemia antigen (CALLA)
c. EBV-related antigens
d. Carcinoembryonic antigen (CEA)

10. Which one of the following antibodies is NOT monomeric?


a. Cell surface IgM
b. IgG
c. IgD
d. Secretory IgA

11. Which complement pathway is NOT correctly matched with its activators?
a. Classical- antibody antigen complexes
b. Alternative- microbial sugars
c. Lectin- microbial sugars
d. Alternative- microbial structures
12. Poliomavirus cause tumors in “nude mice’ (which do not have a thymus, because of a
genetic defect) but not in normal mice. The BEST interpretation is that
a. macrophages are required to reject poliomavirus-induced tumors
b. natural killer cells can reject poliomavirus-induced tumors without help from T
lymphocytes
c. T lymphocytes play an important role in rejection of poliomavirus-induced tumors
d. B lymphocytes play no role in rejection of poliomavirus-induced tumors

13. Examples of primary T-cell deficiency disease include:


a. Hereditary angioneurotic edema
b. Chronic granulomatous disease
c. Wiskott-Aldrich syndrome
d. Leukocyte adhesion deficiency

14. What would be the expected result if a patient was unable to produce functional J
(joining) chain?
a. An increase in secretory IgA
b. A decrease in IgE response

nolan x soju
c. A decrease in circulating IgM
d. A decrease in cell surface IgM

15. Which of the following sequences correctly depicts the course of a B cell in a germinal
center?
a. B cell enters Dark Zone of germinal center, B cells proliferate and move to Light
Zone, B cells undergo somatic mutation and isotype switching. differentiation and
migration out of the germinal center as Dendritic cell or Helper T cell.
b. B cell enters Dark Zone of germinal center, B cells undergo dedifferentiation and
migration out of the germinal center as Memory B cell or Plasma cell
c. B cell enters Light Zone of germinal center, B cells proliferate and move to Dark
Zone. differentiation and migration out of the germinal center as Memory B cell or
Plasma cell
d. B cell enters Dark Zone of germinal center, B cells proliferate and move to Light
Zone, B cells undergo somatic mutation and isotype switching. differentiation and
migration out of the germinal center as Memory 5 cell or Plasma cell
16. The most likely clinical consequence of a genetic deficiency of complement component
C2 and C4 is increased
a. susceptibility to viral infections
b. susceptibility to fungal infections
c. susceptibility to bacterial infections
d. incidence of immune complex disease

17. NKT cells can characteristically recognize antigens presented by what?


a. CD2
b. CD1
c. CD4
d. CD8

18. T-cell-mediated immune responses CANNOT result in


a. formation of granulomas
b. induration at the reaction site
c. rejection of a kidney transplant
d. production of antibodies

nolan x soju
19. Removal of immune complexes is correctly described by which of the following
statements?
a. It is independent of C3 for particulate complexes
b. It occurs primarily in the spleen and liver
c. it is independent of size of the immune complexes
d. it occurs primarily in the thymus and spleen

20. The ‘giant cells’ that can be present in a granuloma are derived from?
a. Granulocytes
b. Macrophages
c. Fibroblasts
d. Th1 cells

21. An example of a molecule produced during the innate response is?


a. Antibody
b. RAG-1
c. Cytokine
d. Immunoglobulin

22. Adaptive immune response shows immunological memory. This can be defined as:
a. Upon a second encounter with an pathogen, a much faster and stronger response is
elicited
b. The inability to recognize an antigen in a specific way
c. The immune response is the same. no matter what pathogen is encountered
d. Production of cytokines and chemokines upon encountering a pathogen for the first
time

23. Type II hypersensitivity is correctly described by which of the following statements?


a. it is antibody independent
b. it is complement independent
c. it does not involve killer cells
d. it is antibody-dependent cytotoxic hypersensitive

24. Grafts between genetically identical individuals (1.e., identical twins)


a. are subject to acute rejection

nolan x soju
b. are subject to hyperacute rejection
c. are not rejected, even without immunosuppression
d. are not rejected if a kidney is grafted, but skin grafts are rejected

25. Which one of the following antibodies would be considered to have the greatest
specificity for an epitope?
a. the antibody with the lowest affinity for the epitope
b. the antibody that forms the fewest non-covalent bonds with the epitope
c. the antibody that has amino acids in the hypervariable regions that form no bonds
with the epitope
d. the antibody with the highest affinity for the epitope

26. Antibodies that are able to provide protection from the effects of bacterial exotoxins do
so in which one of the following ways?
a. activate complement that, when activated, breaks down the toxin molecules into
harmless fragments
b. enable macrophage to present the toxin antigens to T-lymphocytes
c. bind to the toxin molecules and prevent the toxins from binding to body cell
surfaces
d. cause blood vessels located near the site of the toxin to become slightly leaky, thus
allowing cells in the blood stream to enter the tissues and destroy the toxin
molecules

27. The following mechanism(s) may be involved in the clinical efficacy of injection therapy
(hyposensitization)
a. enhanced production of IgG, which binds allergen before it reaches mast cells
b. enhanced production of IgE, which binds IgE receptor before it reaches antigen
c. activation of mast cells that induces production of a large amount of anti-histamines
d. increased local recruitment of eosinophils

28. A macrophage that is stimulated to initiate phagocytosis of a bacterial cell that has a
capsule needs a specific antibody to an antigen on the capsule surface. The antibody
also attaches to which one of the following on the surface of the macrophage?
a. an antigen on the macrophage surface that is identical to the antigen on the capsule
surface for which the antibody is specific
b. MHC Class I molecules

nolan x soju
c. a receptor for a fragment of activated complement (the complement has been
activated by the binding of the antibody to the capsule antigen)
d. an Fe receptor

29. A cell expressing CD3, CD16 and CD56 is a


a. gamma/delta T cell
b. cytotoxic T ceil
c. natural killer T cell
d. regulatory T cell

30. Granulomatous hypersensitivity is correctly described by which of the following


statements?
a. It is not clinically important
b. it has not be encountered in schistosomiasis
c. it results from the presence of persistent microorganisms within macrophages
d. it does not cause pathological effects in diseases associated with T cell-mediated
immunity
31. Most autoimmune diseases are caused by
a. a T-cell defect
b. a constellation of genetic and environmental events
c. a known infectious organism
d. a single genetic defect

32. Hematopoietic stem cell differentiation down the lymphoid developmental pathway can
give rise to?
a. Eosinophils
b. Follicular dendritic cells
c. Natural killer (NK) cells
d. Monocytes

33. Opsonin-treated bacteria are more readily engulfed by phagocytes than are untreated
bacteria because
a. the capsule is removed by opsonin
b. opsonin digest the wall component
c. opsonin induces lysosomal enzymes

nolan x soju
d. the surface of a phagocyte contains receptors for the Fc portion of an antibody

34. Which of the following is NOT a way that the immune system eliminates pathogens?
a. Neutrophils and macrophages engulf pathogens
b. Pathogens are destroyed inside a phagocytic vacuole in an immune cell
c. Immune cells release factors that activate macrophages
d. immune cells direct pathogens to engulf neutrophils

35. A characteristic feature of rheumatoid arthritis is the presence of autoantibodies


specific for?
a. Fcu fragment
b. Fcy fragment
c. C1q component
d. Fab fragment

36. Hyperacute response differs from a first set response in that it involves
a. tissue damage by activated CTLs
b. antibodies, which mark the tissue for ADCC
c. coordination of the response by DTH cells
d. tissue damage by activated mast cells

37. ACD3 + cell that secrets perforin and granzyme is a


a. gamma/delta T cell
b. cytotoxic T cell
c. helper T cell
d. regulatory T cell

38. Which of the following viruses is NOT correctly associated with its associated
malignancy?
a. HBV- liver cancer
b. HPV 16- nasopharyngeal cancer
c. HTLV1- T cell leukemia

39. The first step in thymic education that occurs in the cortex of the thymus is...?

nolan x soju
a. negative selection: selecting out any T cells that are CD4 or CD8 negative
b. positive selection: T cells that recognize self MHC undergo apoptosis
c. positive selection: T cells that can recognize self MHC are saved from apoptosis
d. negative selection: T cells that recognize self MHC undergo apoptosis

40. In a patient, it was observed that macrophages are abundant but do not seem to be
phagocytizing antigen efficiently. What complement factor may be deficient in this
patient?
a. C5b6789
b. C5a
c. C3b
d. C3a

41. Which of the following autoimmune diseases is NOT correctly matched with its major
consequence?
a. Pernicious anemia - Vitamin B12 deficient anemia
b. Multiple sclerosis – demyelination
c. Type I diabetes – hyperglycemia
d. Graves disease - hypothyroidism

42. This cytokine-cytokine receptor interaction initiates extensive cell division of naive T-cell
in the paracortex of lymph node. This important cytokine-receptor interaction 1s which
one of the following?
a. IL-1 and low affinity IL-1 receptor
b. IL-7 and a high affinity IL-7 receptor
c. interferon-gamma and the interferon-gamma receptor
d. IL-2 and a high affinity IL-2? receptor

43. Antibody production by B-lymphocytes is initiated by direct contact between the B-


lymphocyte antigen receptor and the antigen for which that receptor is specific, and
which one of the following additional requirements?
a. help from a T-helper cell that has been stimulated by MHC Class I-presented
peptides
b. contact between the Fe portion of the antibody and an Fc receptor on the surface of
a T- helper 2 (TH2) lymphocytes
c. help from a macrophage that is presenting peptides using MHC Class II molecules
d. help from a T-helper 2 (TH2) lymphocyte

nolan x soju
44. Humans are most likely to mount a TH1 response to
a. intravesicular pathogen
b. a parasitic helminth (worm) infestation
c. pollen or cat dander allergens
d. self-antigen found in the bone marrow

45. How do red blood cells work in conjunction with antibodies in the immune system?
a. RBCs can bind to IgG and IgM to clear immune complexes by transporting them to
the liver, spleen, etc.
b. RBCs work in conjunction with ADCC to destroy target cells
c. RBCs mediate degranulation of mast cells
d. RBCs phagocytose the immune complex formed by IgG and an antigenic molecule

46. An example of an anaphylatoxin is?


a. C5a
b. IL-18
c. Histamine
d. Prostaglandin

47. An example of a disease that effects the nervous system is?


a. Celiac Disease
b. Myasthenia gravis
c. Type I diabetes

48. A graft-versus-host reaction is most commonly seen following transplantation of...?


a. Skin
b. Bone marrow
c. Cornea
d. Heart

49. Individuals with a genetic deficiency of C6 have


a. decreased resistance to viral infections
b. increased hypersensitivity reactions
c. increased frequency of cancer

nolan x soju
d. decreased resistance to Neisseria bacteremia

50. Th1 cells characteristically secrete:


a. TGF-beta
b. IL-22
c. IFN-gamma
d. IL-17

51. Activation of naive T-helper cells (specific for non-self peptides) by mature dendritic
cells (DC) in the lymph node involves which of the following cell interactions?
a. MHC Class IT on DC with CD4 on naive T-cell
b. CD28 on DC with CD56 on naive T-cell
c. CD28 DC with T-cell epitope-specific receptor on naive T-cell
d. CD40L on DC with CD4 on naive T-cell
52. The original specificity of any one B-lymphocyte is determined by which one of the
following?
a. the type of CD molecules on that B-cell’s surface
b. the specificity of the antigen receptors on that B-cell’s surface
c. the specificity of MHC class I molecules
d. MHC class IT molecules

53. Which of the following statements are characteristic of contact sensitivity?


a. The best therapy is administration of the antigen
b. Patch testing with the allergens is useless for diagnosis
c. Sensitization can be passively transferred with serum from an allergic individual
d. Some chemicals acting as haptens induce sensitivity by covalently binding to host
proteins acting as carriers

54. Chemically induced tumors antigens are


a. always the same for a given carcinogen
b. specific tumor associated antigens
c. very strong antigens
d. tumor associated antigens

nolan x soju
55. Which tissue transplant does NOT require cross matching and why?
a. Cornea - not vascularized. immune cells cannot mediate a rejection process
b. Blood - antigens and antibodies can be addressed using immunosuppressive therapy
c. Skin- not subject to immune mediated rejection
d. Liver- stand alone circulation, immune cells cannot mediate a rejection process

56. Immunologic tolerance is defined as


a. The removal of microbes expressing the antigen, so that the host can tolerate the
infection
b. Activation of only B cells, and not T cells, on exposure to an antigen
c. Unresponsiveness of the immune system to an antigen, which is induced by previous
exposure to that antigen
d. The production of memory B cells and T cells on primary exposure to an antigen

57. The major mechanism of host resistance to M leprae is


a. humoral antibodies
b. type IV hypersensitivity
c. high level of calcium in serum
d. type I hypersensitivity

58. Which of the following cells are NOT key to chronic inflammation?
a. Macrophages
b. Neutrophils
c. T-cells
d. Lymphocytes

59. Patients with Wiskott-Aldrich syndrome have:


a. a increased number of B cells
b. a increased number of suppressor T cells
c. high platelet level in blood
d. chronic eczema

60. Circulating lymphocytes will be found in which one of the following locations just before
they enter the blood circulation from the lymphatic system?

nolan x soju
a. a high endothelial venule
b. a lymph node
c. the thoracic duct
d. the bone marrow

61. Double positive T-cells in the thymus express both, which are?
a. CD3 and CD4
b. CD4 and CD8
c. MHC I and MHC II
d. CD3 and CD8

62. Your patient test is positive for the tuberculin antigen. You send him for a chest x-ray
because:
a. The tuberculin test is only presumptive, indicating that he has been exposed to a
tuberculosis antigen
b. He may have other lung infections
c. You are looking for fluid in his lungs due to inflammation caused by the bacillus
d. You are looking for granulomas with neutrophils inside
63. Against which pathogen IgE antibodies can provide immunity?
a. human hepatitis C virus
b. Schistosoma mansoni
c. Staphylococcus aureus
d. Salmonella spicies

64. Which of the following is NOT true regarding antibodies function?


a. Antibodies can mediate ADCC
b. Antibodies neutralize bacterial toxins
c. Antibodies cleave complement components
d. Antibodies can block viral binding

65. What crucial role do dendritic cells play in the differentiation of a Th0 cell?
a. Dendritic cells reverts Th1 and Th2 cells back to Th0 cells so that they can be reused
in an appropriate response
b. Dendritic cells release INFy in response to PAMPs- which influences Th0 cells.
c. Dendritic cells only present antigen to Th0 cells and have no other influence on their
differentiation.

nolan x soju
d. Dendritic cells release cytokines in response to PAMPs- these cytokines influence the
differentiation of the Th0 cell into the appropriate T cell lineage

66. A second set rejection is characterized by all of the following EXCEPT..?


a. Occurs after a first set rejection
b. Occurs rapidly
c. Has a period of latency before rejection
d. Requires immunological memory

67. At which stage of B-cell maturation does VDJ recombination occur?


a. Pre-B
b. Pro-B
c. Mature B
d. Stem cell
68. Antibody-dependent cellular cytotoxicity (ADCC) requires the involvement of which ONE
of the following?
a. antibodies of the IgM isotype that are specific for non-self-antigens/epitopes on the
surface of cancer cells
b. antibodies of the IgG isotype that are specific for non-self-antigens/epitopes on the
surface of cancer cells
c. antibodies of the IgD isotype
d. T-cytotoxic lymphocytes

69. What are the main differences between TCR and BCR?
a. TCR undergo genetic recombination, BCR is individually coded
b. TCR are highly peptide-MHC specific. BCR are antigen specific and can be secreted
from cell
c. TCR has a significant amount of protein diversity, BCR only has 12
d. TCR is part of the adaptive immune system, BCR is part of the innate immune system

70. What is the function of an adjuvant in a vaccine?


a. Neutralizes pathogenic nature of live-attenuated viruses
b. Stimulates antigen uptake and stimulate an immune response

nolan x soju
c. Functions as a subunit to an antigen
d. Facilitates recognition by dendritic cells

71. A patient is found to have a complete absence of C1 inhibitor. The most likely
consequence of this deficiency is
a. hereditary angioedema
b. immune complex disease
c. granuloma formation
d. deposition of immune complexes

72. TH1 cells most often “help” macrophages to become more efficient phagocytic cells.
This enhancement of phagocytosis is generally initiated by which one of the following
cytokines secreted by these TH1 cells during this macrophage-helping process?
a. interferon-gamma
b. interleukin-8
c. interleukin-2
d. CD40L
73. After direct or indirect alloantigen recognition._.?
a. T cells become activated CD4+ and CD&>+ effector T cells, targeting the alloantigen
on the graft
b. Cytokines produced by allogeneic CD4+ cells will prevent invasion of CD4+ and CD8+
effector T cells from host
c. Allogeneic effector B cells secrete cytokines which induce graft rejection
d. CD4+ effector T cells become activated and travel to allograft. CD8+ cells are not
involved

74. Complement that has been activated by pathogenic bacterial cells can not contribute
(directly or indirectly) to the destruction or elimination of those pathogenic bacterial
cells in which of the following ways?
a. by destroying the bacterial cell membrane
b. by assisting phagocytes to attach to bacterial cells
c. by attracting more phagocytes to the area of complement activation
d. by solubilization of immune complexes

75. Patients with DiGeorge syndrome

nolan x soju
a. have increased number of suppressor T cells
b. have selective IgA deficiency
c. have abnormal antigen-presenting cells
d. have a decreased number of helper T cells
76. Which of the following is a component of the lectin pathway of complement activation?
a. C1q
b. Factor D
c. Factor B
d. MASP-1

77. A T cell located at the epithelial barrier of the gut is a


a. gamma/delta T cell
b. helper T cell
c. cytotoxic T ceil
d. natural killer cell
78. A Tc cell can mount a cytotoxic attack on a viral-infected cell if it binds to a cell
displaying
a. insufficient MHC class I molecules
b. insufficient MHC class II molecules
c. MHC I bearing foreign antigen
d. MHC II bearing foreign antigen

79. Direct alloantigen recognition requires..?


a. Autoreactive T-cells
b. Allogeneic antigen presenting cells that express allogeneic MHC
c. Memory B cells specific for allograft
d. Host antigen presenting cells that express allogeneic MHC

80. Which one of the following cytokines are most closely involved in activating cytotoxic T
lymphocytes to mediate graft rejection?
a. IL-13
b. IL-10
c. IL-2
d. IL-4

nolan x soju
81. Which of the following 1s NOT correct regarding immunity to cancer?
a. despite the immune response tumors continue to grow
b. Th2 cells evoke anti-tumor immune response
c. vaccination can be used to treat cancer
d. destruction of tumor cells is achieved by T regulatory cells

82. Where do almost all mature, but naive, T-lymphocytes encounter the epitopes for which
they are specific during the priming phase?
a. in the HEV
b. in the lymph node
c. in the thoracic duct (just before entering the blood circulation)
d. in the medulla of the thymus (involves antigen presentation by dendritic cells)

83. Receptor editing as a mechanism of immunological tolerance occurs in?


a. B-cells
b. Plasma cells
c. Cytotoxic T-cells
d. Regulatory T-cells

84. What can lead to autoimmune disease


a. increase in anergy
b. Upregulation of genes for AIR
c. Peripheral lymphoid tissue destruction
d. A breakdown of tolerance

85. Anaphylactic reactions


a. can result in induration at the site reaction
b. are the consequences of TH1 activation
c. may involve components of mast-cell granule matrix
d. are the consequences of TH17 activation

86. The class of antibody that is most effective at participating in ADCC is:
a. IgM
b. IgG

nolan x soju
c. IgE
d. IgD

87. Class switching begins with what?


a. Signals from helper T cells
b. Recombination
c. Translation
d. Deletion of intervening genes

88. Choose the correct statement:


a. Class switching requires interaction between PD1 and B7
b. Wiskott-Aldrich syndrome is an example of impaired heavy chain switching
c. After class switching, the B cell begins to produce a new heavy-chain isotype with
the different specificity to an antigen
d. None of the above
89. Lupus erythematosus is
a. free from an immune complex disease component
b. associated with antibodies against nucleic acid
c. a disease of red and white cells
d. an organ-specific autoimmune disease

90. AIDS is caused by a human retrovirus that kills


a. B lymphocytes
b. lymphocyte stem cells
c. CD4-positive T lymphocytes
d. CD8-positive T lymphocytes

91. You have a patient who makes autoantibodies against his own red blood cells, leading to
hemolysis. Which one of the following mechanisms is MOST likely to explain the
hemolysis?
a. Perforins from cytotoxic T cells lyse the red cells
b. Neutrophils release proteases that lyse the red cells
c. Interleukin-2 binds to its receptor on the red cells, which results in lysis of the red
cells

nolan x soju
d. Complement is activated and membrane attack complexes lyse the red cells

92. Which antibodies can activate complement after binding to antigen


a. monomeric IgM on B cells
b. pentameric IgM in serum
c. monomeric IgA in serum
d. dimeric IgA in secretion

93. Antibody-mediated allergic reactions involve the attachment of one of the five isotypes
to which one of the following types of cells in the body?
a. macrophage (via Fc receptors for the IgG isotype)
b. mast cells
c. NK cells
d. any phagocytic cell

94. A Direct Coombs test detects...?


a. Free anti-RBC autoantibody in serum that 1s mixed with RBCs
b. Anti-RBC autoantibody that is already bound to RBCs using an anti-Human Ig
c. An exaggerated antibody response to anti-Human Ig
d. Destroyed or damaged RBCs

95. The first line of defense against viruses is


a. IgA antibodies directed against viruses at mucus surfaces
b. IgG directed to an internal viral antigen
c. IgM directed to an external viral antigen
d. interferon-gamma

96. Class switching requires:


a. CD40L on the surface of a B-cell
b. CD40L on the surface of a NK-cell
c. CD4OL on the surface of a T-cell
d. CD80 on the surface of a B-cell

97. Certain HLA genes are linked to autoimmune diseases such as ankylosing spondylitis.
This linkage has all the following characteristics EXCEPT

nolan x soju
a. it may be the result of closely linked genes
b. it may be the result of cross-reactivity between self-antigen and infection agent
c. it carries no increased risk for a specific disease for those individuals with the gene
d. it may be the result of a MHC molecule serving as an attachment site for an
infectious agent

98. Di George syndrome results from a defect in


a. Purine nucleoside phosphorylase
b. WASP
c. Thymic development
d. DNA repair

99. Regarding anaphylactic (type I) and immune complex (type IID hypersensitivities, which
of the following is the MOST accurate?
a. IgE is involved in both anaphylactic and immune complex hypersensitivities
b. Complement is involved in both anaphylactic and immune complex
hypersensitivities
c. Less dose of antigen is typically needed to trigger anaphylactic reaction than an
immune complex reaction
d. Neutrophils play more important role in anaphylactic reactions than in immune
complex

100. A patient is admitted with multiple bacterial infections and 1s found to have a
complete absence of C9. Which complement-mediated function would be mostly
affected in such a patient:
a. direct microbial killing
b. generation of neutrophil chemotactic factors
c. phagocytosis
d. opsonization of bacteria

nolan x soju
Immunology States Exam

1. Hyperacute response differs from a first set response in that it involves


a) Tissue damage by activated CTLs
b) Antibodies, which mark the tissue ADCC
c) Coordination of the response by DTH cells
d) Recognition of foreign MHC 1 and 2

2. The name given to the amino acids that actually form bonds between an
antibody molecule and an antigen for which that antibody molecule is
very strongly specific are called the, (one best answer)
a) Framework amino acids
b) The variable amino acids
c) The hypervariable amino acids
d) The domain amino acids

3. Which of the following is a direct consequence of antibody binding

nolan x soju
antigen?
a) Activation of B cells
b) Neutralization of antigen
c) Activation of complement
d) Phagocytosis of antigen

4. The original specificity of any one B lymphocyte is determined by which


one of the following?
a) The type of CD molecule on that B cell’s surface
b) The specificity of the antigen receptors on that B cell’s surface
c) The specificity of MHC class 1 molecules
d) The amino acid composition of two short tails extending from the tail
end of the constant portion of the B cell’s antigen receptor that are
embedded into the membrane surface of that B cell
5. Select incorrect answer. Complement that has been activated by
pathogenic bacterial cells can contribute (directly or indirectly) to the
destruction or elimination of those pathogenic bacterial cells
a) By destroying the bacterial cell membrane
b) By assisting phagocytes to attach to bacterial cells
c) By attracting more phagocytes to the area of complement activation
d) By stimulating somatic hypermutation in the phagocytes that are
already actively engaged in phagocytizing the bacterial cell

6. The total number of combinations of all gene fragments to produce


antigen specific receptors during somatic recombination is far less than
that actual number of different specificities that are actually produced.
This additional variable gene diversity is generated during somatic
recombination in which one of the following ways?
a) By looping out heavy chain isotype genes
b) By inserting nucleotides between variable portion gene fragment
nucleotide sequences before those fragments are ligated
c) By ADCC

nolan x soju
d) By modifying the epitope binding groove as the alpha chain of the
receptor folds into its final tertiary configuration

7. A CD3+ cell that secrets IL 2 when activated is a,


a) Regulatory T cell
b) Helper T cell
c) Cytotoxic T cell
d) Gamma T cell

8. What is a major consequence of the bone marrow transplantation in the


immunocompromised patients?
a) Potentially lethal graft versus host disease
b) High risk of T cell leukemia
c) Inability to use live donor
d) Delayed hypersensitivity
9. Virus causes tumors in “nude mice” (which do not have thymus, because
of genetic effect) but not in normal mice. The best interpretation is that,
a) Macrophages are required to reject poliomavirus induced tumors
b) Natural killer cells can reject poliomavirus induced tumors without
help from T lymphocytes
c) T lymphocyte play an important role in rejection of poliomavirus
induced tumors
d) B lymphocyte play no role in rejection of poliomavirus induced
tumors

10.Rheumatoid factor is correctly described as,


a) A synonym for auto anti IgG antibodies
b) Not involved in immune complex formation
c) A protease found in synovial fluid
d) Primarily a macrophage product

11.The much more rapid appearance of antibodies during a secondary


response compared to the primary response is due mainly to which one

nolan x soju
of the following?
a) A much improved presentation of peptide antigens to T helper 1 cells
by B lymphocytes during the secondary response
b) A much higher number of B lymphocytes at the beginning of the
secondary response specific for the antigen that stimulated both the
primary and secondary antibody response
c) Improved binding of antibody molecules to Fc receptors on the
surface of phagocytes which leads to a higher rate of peptide
presentation to B lymphocytes
d) Each B lymphocyte participating in a secondary antibody response is
specific for a very large number of different antigens while each B
cell participating in the primary response is specific for only one
antigen
12.Mechanism of antibacterial immunity include all of the following
EXCEPT?
a) Antibody and complement mediated bacteriolysis
b) Antibody and complement mediated opsonization
c) Neutralization of toxins by antibodies
d) Cytotoxic T cell mediated bacteriolysis

13.Patients with Wiskott Aldrich syndrome have,


a) A increased number of B cells
b) A increased number of suppresser T cells
c) High platelet level in blood
d) Chronic eczema

14.Which of the following human cell types could not be capable of


presenting peptide using MHC class 1?
a) Macrophages
b) Fully mature red cells
c) Dendritic cells

nolan x soju
d) B lymphocyte

15.Clonal selection
a) Begins with inflammation
b) Occurs for all leukocytes
c) Occurs in response to self-antigens
d) Occurs for all lymphocytes

16.Complement that has been activated by pathogenic bacterial cells can


not contribute ( directly or indirectly) to the destruction or elimination of
those pathogenic bacterial cells in which of the following ways
a) By destroying the bacterial cell membrane
b) By assisting phagocytes to attach to bacterial cells
c) By attracting more phagocytes to the area of complement
activation
d) By solubilization of immune complexes
17.Lymphocyte continually recirculate through peripheral lymphoid tissue in
order to,
a) Be killed before they cause autoimmunity
b) Efficiently encounter antigen
c) Mature from stem cells into lymphocytes
d) Phagocytose antigen and kill it

18.Which one of the following antibodies would be considered to have the


greatest specificity for an epitope
a) The antibody with the lowest affinity for the epitope
b) The antibody that forms the fewest non covalent bonds with the
epitope
c) The antibody that has amino acids in the hypervariable regions
that form no bonds with the epitope
d) The antibody with the highest affinity for the epitope

19.The IgG isotype is often used as an example of the “typical”


immunoglobulin. Identify the physical or biological characteristics of one

nolan x soju
antibody molecule that has the IgG isotype
a) Has one heavy chain and two identical light chains
b) Is able to bind two different antigen epitopes
c) Papain treatment will produce two identical Fab fragments and
two identical Fc fragments
d) Can bind (via the Fc portion) to phagocyte

20.A patient admitted with multiple bacterial infections and is found to have
a complete absence of C6. Which complement mediated function would
be mostly effected in such a patient?
a) Lysis of pathogen
b) Generation of neutrophil chemotactic factor
c) Generation of anaphylatoxins
d) Opsonization of bacteria
21. What is the role of class 2 MHC protein on donor cells in graft rejection
a) They are the receptors for interleukin 2 which is produced by
macrophages when they attack the donor cells
b) They are recognized by helper T cells which then activates
cytotoxic T cells to kill the donor cells
c) They induce the production of blocking antibodies that protect the
graft
d) They induce IgE which mediated graft rejection

22.Where do almost all mature, but naïve T lymphocytes encounter the


epitopes for which they are specific during priming phase?
a) In the HEV
b) In the lymph node
c) In the thoracic duct (just before entering the blood circulation)
d) In the medulla of thymus (involves antigen presentation by
dendritic cells)

23.Antibodies that are able to provide protection from the effects of

nolan x soju
bacterial exotoxins do so in which one of the following ways?
a) Activate complement that, when activated break down the toxin
molecules into harmless fragments
b) Enable macrophage to present the toxin antigens to T lymphocytes
c) Bind to the toxin molecules and prevent the toxins from binding to
body cell surfaces
d) Cause blood vessels located near the site of the toxin to become
slightly leaky thus allowing cells in the blood stream to enter the
tissues and destroy the toxin molecules

24.Which of the following clinical disease is most likely to involve a reaction


to a hapten in its etiology?
a) Goodpasture’s syndrome
b) Hemolytic anemia after treatment with penicillin
c) Rheumatoid arthritis
d) Serum sickness
25.Which one of the following types of cells is NOT capable of phagocytosis?
a) Dendritic cells
b) T helper cells
c) Macrophages
d) Neutrophils

26.Although IgE antibodies can cause quite serious allergic reactions this
isotype of antibody does appear to be very important in providing
immunity from which one of the following?
a) The smallpox virus
b) Parasitic worms
c) Tuberculosis bacteria that infect macrophages (and resist
destruction once inside the cell)
d) Bacteria that cause pneumonia (most of these bacteria have a
capsule)

27.A 5 years old child was revaccinated by measles, mumps and rubella
(MMR) vaccine. A blood test performed 2 months later showed the

nolan x soju
presents of high affinity IgG antibodies specific for antigens of all above
mention viruses. Which of the following proteins was NOT necessary for
the production of these high affinity IgG antibodies?
a) CD40 ligand
b) Activation induced deaminase (AID)
c) Class 2 MHC
d) C reactive protein (CRP)

28.Patients with Chediak-Higashi syndrome have phagocytes that,


a) Produce protein that interfere with chemotaxis
b) Lack C3b receptors that are needed for attachment to bacteria
c) Show a reduced ability of their lysosomes to fuse with
phagosomes to release microbicidal substances
d) Show defect for the engulfment of microbes
29.A CD3+ cell that secrets perforin and granzyme is a,
a) Gamma delta T cell
b) Cytotoxic T cell
c) Helper T cell
d) Regulatory T cell

30.Which of the following is NOT correct regarding immunity to cancer?


a) Despite the immune response tumor continue to grow
b) Abrogation of CD25+ cells leads to protective tumor immunity
c) Vaccination can be used to treat cancer
d) Destruction of tumor cells is achieved by T regulatory cells

31.Priming of naïve T helper cells (specific for non-self-peptide) by mature


dendritic cells (DC) in the lymph node involves which of the following cell
interactions?
a) MHC class 2 on DC with CD4 in naïve T cell
b) CD28 on DC with CD56 on naïve T cell
c) Peptide presented by DC with T cell epitope specific receptor on

nolan x soju
naïve T cell
d) CD40 on DC with CD40L on Naïve T cell

32.A patient is found to have a complete absence of C1 inhibitor. The most


likely consequent of this deficiency is
a) Hereditary angioedema
b) Immune complex disease
c) Granuloma formation
d) Deposition of immune complexes

33.Most autoimmune diseases are caused by


a) A T cell defect
b) A constellation of genetic and environmental events
c) A known infectious organism
d) A single genetic defect
34.Di George syndrome results from a defect in
a) Purine nucleotide phosphorylase
b) WASP
c) Thymic development
d) DNA repair

35.The following mechanism(s) may be involved in the clinical efficacy of


injection therapy (hyposensitization)
a) Enhanced production of IgG which binds allergen before it reaches
mast cells
b) Enhanced production of IgE which bind IgE receptor before it
reaches antigen
c) Activation of mast cells that induces production of a large amount
of anti-histamines
d) Increased local recruitment of eosinophil

36.Which ONE of the following is an important difference between innate


immune responses and adaptive immune responses?

nolan x soju
a) Innate response (e.g. phagocytosis, complement activation)
improves significantly with each exposure to pathogen
b) Innate responses often can take 1-2 weeks to begin destroying
infectious microorganisms, while adaptive responses destroy
infections almost immediately after the very first encounter with a
pathogen
c) An adaptive response does not improve significantly with repeated
exposure to the same non self-antigen
d) Each cell involve in innate responses (e.g. phagocytes) can be
stimulated to respond by many different molecules while each cell
involved in adaptive responses (e.g. Lymphocytes) can be
stimulated to respond by only one molecular structure (called an
antigen)
37.Your patient tests positive for the tuberculin antigen. You send him for a
chest X Ray because
a) The tuberculin test is only presumptive indicating that he has been
exposed to a tuberculosis antigen
b) He may have other lung infections
c) You are looking for fluid in his lungs due to inflammation caused by
bacillus
d) You are looking for looking for granulomas with neutrophils inside

38.In absence of antibodies phagocytes are not able to respond effectively


against
a) Capsulated bacteria
b) Bacterial cells to which the phagocytes firmly attach
c) Bacteria that are present in the body but outside of the blood
vessels
d) Protozoa

39.The differentiation of bone marrow stem cells into either B lymphocytes

nolan x soju
or T lymphocytes is often initiated or controlled by interactions between
the stem cell and cells that are permanent residents of the bone marrow
thymus. Which ONE of the following cell types is involved in initiating or
controlling T cell development in the thymus?
a) Stromal cells in the bone marrow that express cell adhesion
molecules
b) Epithelial cells in the thymus that express CD4 and CD8
c) Epithelial cells in the bone marrow that express CD4 and CD8
d) Cortical epithelial cells in the thymus that express MHC class 1
and MHC class 2

40.Antibody mediated allergic reaction involve the attachment of one of the


five isotypes to which one of the following types of cells in the body?
a) Macrophage (via Fc receptors for the IgG isotypes)
b) Mast cells
c) NK cells
d) Any phagocytic cell
41.Which of the following statement as (are) characteristics of contact
sensitivity?
a) The best therapy is administration of the antigen
b) Patch testing with the allergen is useless for diagnosis
c) Sensitization can be passively transferred with serum from an
allergic individual
d) Some chemicals acting as haptens induce sensitivity by covalently
binding to host proteins acting as carriers

42.Which molecules are responsible for initiation the rejection of a mis


matched transplant by the recipient?
a) HLA surface molecules that are also found in the body of the
recipient of the transplant
b) HLA DP DQ DR that are also found in the body of the recipient of
the transplant
c) MHC molecules that are not found in the body of the recipient of
the transplant
d) MHC class 1 molecules that are also found in the body of the

nolan x soju
recipient

43.Which one of the following cells is generally the first to encounter a


bacterial infections in the tissues is usually capable of phagocytizing
some (or even all) of those bacteria and secrete a wide range of
cytokines and chemokine that effect nearby blood vessels and recruit
and activate other cells that can also participate in eliminating the
infection?
a) A neutrophil
b) A dendritic cells
c) A T helper cell
d) A macrophage
44.Which one of the following surface molecule is expressed by T helper
cells that have just been primed in the lymph node that will participate in
sending a temporary inhibitory signal to that primed T cell?
a) B7
b) CD28
c) CTLA-4
d) CD4

45.The lesions in immune complex- induced glomerulonephritis


a) Are dependent on erythrocyte and complement
b) Result an increased production of urine
c) Require both complement and neutrophil
d) Are depended on the presence of macrophages

46.Which one of the following interactions between surface molecules must


take place for a T cytotoxic cell to be stimulated to begin to destroy a
virus infected cell?
a) MHC class 1 on virus infected cell and CD4 on T cytotoxic cell

nolan x soju
b) MHC class 1 in T cytotoxic cell and CD8 on virus infected cell
c) MHC class 2 on virus infected cell and CD4 on T cytotoxic cell
d) MHC class 1 on virus infected disease and CD8 in T cytotoxic cell

47.Immune complex formation can result from all EXCEPT?


a) Persistent infection
b) Infections of antigen material
c) Autoimmune disappear
d) Intradermal antigen

48.AIDS is caused by a human retrovirus that kills


a) B lymphocytes
b) Lymphocyte stem cells
c) CD4-positive T lymphocyte
d) CD8-positive T lymphocyte
49.The biological properties of an antibody molecule not directly associated
with specificity (e.g. complement activation, opsonization of phagocytes,
initiating allergic reactions involving mast cells, etc.) involve the
participation of amino acids in which one of the following locations?
Hint: this location also determines the isotype of the antibody
a) Fab
b) The constant portion of the light chain
c) The constant portion of the heavy chains
d) The domain that is at the amino terminal end of the heavy chain

50.Humans are most likely to mount a TH1 response to


a) Intravesicular pathogen
b) A parasitic helminth (worm) infection
c) Pollen or cat dander allergen
d) Self-antigen found in the bone marrow

51.Which one of the following type has the MOST influence on the antibody
response carried out by a B lymphocyte?

nolan x soju
a) A T cytotoxic cell
b) Cytokines released by a T helper 2 cell
c) Cytokines released by a T helper 1 cell
d) Phagocytes that are presenting peptides to T helper cells

52.If you wanted an adaptive immune response to lead to virus


neutralization and eventual phagocytosis. (But not destruction of virus-
infected cells) which of the following type of cells would have to be
involved in some part of that complete response (that eventually results
in degradation of the virus) ?
a) T cytotoxic cells, macrophage and dendritic cell
b) Macrophage, B lymphocyte and T helper cells
c) T helper cell, macrophage and dendritic cell
d) Macrophage, T cytotoxic cells and B lymphocytes
53.The most likely clinical consequence of a genetic deficiency of
complement component C2 and C4 is increased
a) Susceptibility to viral infection
b) Susceptibility to fungal infection
c) Susceptibility to bacterial infection
d) Incidence of immune complex disease

54.A cell expressing CD3+, CD25+ and FoxP3+ is a


a) Gamma/delta T cell
b) Cytotoxic T cell
c) Natural killer cell
d) Regulatory T cell

55.Phagocytes can NOT successfully attach to the cell wall surface of


bacterial cell in which of the following ways? Assume that the correct
lipopolysaccharides are present on the surface of the bacterial cell
a) Directly attaching to the bacterial cell wall lipopolysaccharides
b) Attaching to CD4 molecules on the bacterial cell surface using

nolan x soju
MHC class 2 molecules (on the phagocytes surface)
c) Using an antibody molecule that is specific for an antigen on the
surface of the bacterial cell
d) Using a fragment resulting from complement activation by the
bacterial cell

56.Activation of complement by an antibody-epitope interaction result in


several fragments many of which have biological significance. For
example, which one of the following is a fragment produced during
complement activation that can actually help a phagocyte to attach to
the surface of a bacterial cell to which that phagocyte could not attach in
the absence of this fragment?
a) C3
b) C3 convertase
c) C3a
d) C3b
57.The inflammatory response is one manifestation of the innate immune
system. The response is initiated by the presence of bacterial cells in the
extravascular tissues (i.e. outside of the blood vessels) phagocytes (e.g.
macrophages) that encounter these bacterial cells are stimulated to
release chemicals (cytokines and chemokine that can have a multitude of
effects on nearby blood vessels and the contents of those blood vessels.
which of the following will occur as a result of the release of these
cytokines and chemokine by the phagocytes?
a) The spaces between the cells that form the blood vessel will not
be increased, thus preventing fluid from the blood to leak into
the tissues
b) Some of phagocytes will be stimulate to also secrete antibodies
specific for antigens found on the surface of the bacterial cells
c) Some phagocytic cells in the nearby blood stream will actually be
attracted to the site of the infection, squeeze between the cells
that form the blood vessel and enter the tissues to assist in
fighting the invading bacteria present there
d) T lymphocytes that are in the blood circulation will leave the

nolan x soju
blood circulation right at the site of bacterial infection
immediately enter the lymphatic system and began to produce
antibodies specific for antigens associated with the infecting
bacteria

58.The first line of defense against viruses is


a) IgA antibodies directed against viruses at mucus surfaces
b) IgG directed to an internal viral antigens
c) IgM directed to an external viral antigens
d) Interferon gamma

59.A diagnostic marker for tumors of the colon is


a) Alpha fetoprotein
b) Common acute lymphoblastic leukemia antigen (CALLA)
c) EBV-related antigen
d) Carcinoembryonic antigen (CEA)
60.Clinical and experimental data indicates that certain forms of which one
of the following types of biomolecules may correlate with frequencies of
certain autoimmune disease?
a) Unusual forms of proteasome enzyme subunits
b) VDJ gene fragments used to produce receptors specific for cancer
associated peptides
c) Histocompatibility antigens
d) CD4 or CD8 surface molecule

61.Rapid but non antigen specific immune responses are produced by the
a) Adaptive immune system
b) Innate immune system
c) Lymphocytes
d) Lymphatic system

62.Which of the following is a common cause of autoimmune disease


a) Failure of lymphocyte maturation
b) Failure of self-tolerance

nolan x soju
c) Treatment with corticosteroids
d) Disseminated cancer

63.Against which pathogen IgE antibodies can provide immunity?


a) Human hepatitis C virus
b) Schistosoma mansoni
c) Staphylococcus aureus
d) Salmonella species

64.A T cell located at the epithelial barrier of the gut is a


a) Gamma/delta T cell
b) Helper T cell
c) Cytotoxic T cell
d) Natural killer cell
65.Circulating lymphocytes will be found in which one of the following
locations just before they enter the blood circulations from the lymphatic
system?
a) A high endothelial venule
b) A lymph node
c) The thoracic duct
d) The bone marrow

66.Failure to express class 2 HLA molecules on antigen presenting cells


causes
a) Th cell deficiency
b) Tc cell deficiency
c) B cell deficiency
d) NK cell deficiency

67.Removal of immune complexes is correctly described by which of the


following statements?
a) It is independent of C3 for particulate complexes

nolan x soju
b) It occurs primarily in the spleen and liver
c) It is independent of size of the immune complexes
d) It occurs primarily in the thymus and spleen

68.Although each isotype has a hinge region that can be important in


activating complement after an antibody molecule has bonded to its
antigen only two isotypes that can usually activate complement
a) IgM
b) IgD
c) IgA (the monomeric form in the serum)
d) IgE

69.Individuals with a genetic deficiency of C6 have


a) Decreased resistance to viral infection
b) Increased hypersensitivity reaction
c) Increased frequency of cancer
d) Decreased resistance to Neisseria bacteremia
70.Anaphylactic reactions
a) Can results in induration at the site reaction
b) Are the consequences of TH1 activation
c) May involve components of mast cell granule matrix
d) Are the consequences of TH17 activation

71.The final damage to vessels in immune complex mediated arthritis is due


to
a) Histamine
b) NK cell
c) Lysosomal enzymes of polymorphonuclear leukocytes
d) Cytotoxic T cell

72.Immunologic tolerance define as


a) The removal of microbes expressing the antigen, so that host can
tolerate the infection
b) Activation of only B cells and not T cells on exposure to an antigen

nolan x soju
c) Unresponsiveness of immune system to an antigen, which is
induced by previous exposure to that antigen
d) The production of memory B cells and T cells on primary exposure
to an antigen

73.Type 2 hypersensitivity is correctly described by which of the following


statements?
a) It is antibody independent
b) It is complement independent
c) It does not involved killer cells
d) It is antibody dependent cytotoxic hypersensitive

74. Patients with Di George syndrome


a) Have increased number of suppressor T cells
b) Have selective IgA deficiency
c) Have abnormal antigen presenting cells
d) Have a decreased number of helper T cells
75.Granulomatous hypersensitivity is correctly described by which of the
following statement?
a) It is not clinically important
b) It has not be encountered in schistosomiasis
c) It results from the presence of persistent microorganisms within
macrophages
d) It does not cause pathological effects in diseases associated with T
cell mediated immunity

76.The name given to the amino acids that actually form bond between an
antibody molecule and an antigen for which that antibody molecule is
very strongly specific are called the: (one best answer)
a) Framework amino acids
b) The variable amino acids
c) The hypervariable amino acids
d) The domain amino acids

77.A typical IgG isotype antibody molecule does not consist of which of the

nolan x soju
following?
a) Two identical light chains (220 amino acids in length)
b) Two identical heavy chains (440 amino acids in length)
c) A “J” (joining) chain
d) Two identical antigen binding sites

78.Certain HLA genes are linked to diseases such as ankylosing spondylitis


this linkage has all the following characteristics except:
a) It may be result from closely linked genes
b) It may be result from cross reactivity between self-antigen and
infection agent
c) It carries no increased risk for a specific disease for those
individuals with the disease
d) It may be the result of a MHC molecule serving as an attachment
site for an infectious agent
79.Peripheral lymphoid organs
a) Are located in the abdomen to protect their vital functions
b) Maximize contact between antigen and lymphocytes
c) Produce antigen specific lymphocytes from stem cells
d) Sequester antigen to minimize its damage to the body

80.Antibody production by B lymphocytes is initiated by direct contact


between the B lymphocyte antigen receptor and the antigen for which
that receptor is specific. And which one of the following additional
requirements?
a) Help from a T helper cell that has been stimulated by MHC class 1
presented peptides
b) Contact between the Fc portion of the antibody and an Fc receptor
on the surface of a T helper 2 (TH2) lymphocyte
c) Help from a macrophage that is presenting peptides using MHC
class molecules
d) Help from a T helper 2 (TH2) lymphocyte

nolan x soju
81.Which one of the following isotype of antibodies is responsible for
initiating the release of large amounts of histamine that results in the
symptoms most often associated with allergic reactions? Hint: this
isotype is normally found in very low amounts in the serum of non-
allergic persons
a) IgM
b) IgD
c) IgG
d) IgE
82.Which ONE of the following statements regarding various diseases is
true?
a) Tuberculoid leprosy is associated with a predominant Th2 profile
b) Epitheloid cells and giant cells are typical of immune complex
diseases
c) Lepromatous leprosy is associated with a predominated Th1
profile
d) Corhn’s disease is an inflammatory disease in which
granulomatous reaction causes stricture of the bowel and
penetrating fistulas into other organs

83.Tc cell can mount a cytotoxic attack on a viral infected cell if it binds to a
cell displaying
a) Insufficient MHC 1
b) Insufficient MHC 2
c) MHC 1 bearing foreign antigen
d) MHC 2 bearing foreign antigen

nolan x soju
84.Antibody dependent cellular cytotoxicity (ADCC) requires the
involvement of which ONE of the following?
a) Antibodies of the IgM isotype that are specific for non-self-
antigens/epitopes on the surface of cancer cells
b) Antibodies of the IgG isotype that are specific for non-self-
antigens/epitopes on the surface of cancer cells
c) Antibodies of the IgD isotype
d) T cytotoxic lymphocytes

85.You have a patient who makes autoantibodies against his own red blood
cells, leading to hemolysis. Which one of the following mechanisms is
MOST likely to explain the hemolysis?
a) Perforins from cytotoxic T cells lyse the red cells
b) Neutrophils release proteases that lyse the red cells
c) Interleukin 2 binds to its receptor on the red cells, which results in
lysis of the red cells
d) Complement is activated and membrane attack complexes lyse
the red cells

86.Which of the following statements about type 1 hypersensitivity is true?


a) It is also known as delayed type hypersensitivity reaction
b) It occurs when an IgE response is directed against pollens
c) It does not require IgE sensitized mast cells
d) It is independent of pharmacological mediators

87.Vaccination protects us from infectious disease by generating


a) Memory cells
b) Naïve lymphocytes
c) Macrophages
d) Regulatory cells

88.Identify the fundamental difference between the antigen recognition by


B and T cells?
a) Antigen must be presented in different ways

nolan x soju
b) Clonal selection after antigen recognition
c) Heterogeneity of TCR and BCR from one lymphocyte to the next
d) Membrane location of antigen specific receptors

89.IgG binding to neutrophils is mediated by


a) Fc dependent cellular homing mechanisms
b) Sensitization of mast cells and basophils
c) Fc receptors specific for IgG
d) ICAM’s

90.Strongly immunogenic tumors appear


a) In virtually all cancer
b) In immunosuppressed patients
c) Only in lymphoma and leukemia
d) In case liver cancer
91.Which experiment suggests that tumor cells express tumor specific
transplantation antigens?
a) When a syngeneic mouse is injected with the tumor cells accepts
the tumors and dies
b) When a syngeneic mouse is injected with the tumor cells reject the
tumor and survives
c) When an allogeneic mouse is injected with the tumor cells reject
the tumor and survives
d) When an allogeneic mouse is injected with the tumor cells accepts
the tumor and dies

92.The most important chemokine receptors that act as coreceptorsCXCR5


for HIV are
a) CXCR5 and CCR6
b) CXCR4 and CCR5
c) CXCR7 and CCR5
d) CXCR6 and CCR4

nolan x soju
93.IgE production depends on
a) Th1 cells
b) Th2 cells
c) Th17 cells
d) NK cells

94.If a naïve (unprimed) T cytotoxic cell encounters the antigen for which it
is specific without any co stimulation (e.g.B7, CD28) that T cell will then
do which one of the following?
a) Undergo significant cell division greatly expanding the number of
T cells specific for that antigen
b) Die (via apoptosis)
c) Become anergic
d) Express CTLA 4 in high concentration on its surface resulting in a
rapid increase in the secretion of cytokines by that T cytotoxic
cells
95.Which of the following statements BEST explains the relationship
between inflammation of the heart (carditis) and infection with group A
beta hemolytic streptococci
a) Streptococcal antigens induce antibodies cross reactive with heart
tissue
b) Streptococci are polyclonal activators of B cells
c) Streptococcal antigens binds to IgE on the surface of heart tissue
and histamine is released
d) Streptococci are ingested by neutrophils that release proteases
that damage heart tissue

96.Which ONE of the following isotypes of antibodies is involved in


providing protection to a fetus or the newborn infant?
a) IgM
b) IgG
c) IgE
d) IgD

nolan x soju
97.Antibodies that are able to provide protection from the effects of
bacterial exotoxins do so in which one of the following ways?
a) Activate complement that when activated break down the toxic
molecules into harmless fragments
b) Enable macrophage to present the toxic antigens to T lymphocytes
c) Bind to the toxin molecules and prevent the toxins from binding to
body cell surfaces
d) Cause blood vessels located near the site of the toxin to become
slightly leaky thus allowing cells in the blood stream to enter the
tissues and destroy the toxin molecules

98.A macrophage that is stimulate to initiate phagocytosis of a bacterial cell


that has a capsule needs to also involve an antibody specific for an
antigen on the capsule surface that also attaches to which one of the
following on the surface of the macrophage?
a) An antigen on the macrophage surface that is identical to the
antigen on the capsule surface for which the antibody is specific
b) MHC class 2 molecule
c) A receptor for a fragment of activated complement (the
complement has been activated by the binding of the antibody to
the capsule antigen)
d) An Fc receptor

99.When cells die by apoptosis as compared to by necrosis they potent to


induce inflammation and damage of the surrounding tissue because they
a) Do not release the cytoplasmic content
b) Are not subsequently phagocytized
c) Migrate into the blood stream prior to death
d) Are rarely malignant or infected by virus

100. Innate host defense mechanisms are critical to the protection of the
body because
a) They can be activated by other immune responses
b) The antibodies derived from the innate responses are critical to
neutralize bacterial toxins

nolan x soju
c) They are highly specific for the invading pathogens that avoid
PAMP receptor recognition
d) They provide immediate continuous protection in the absence of a
specific immune response
A patient is admitted with multiple bacterial infections and is found to have a complete absence
of C3. Which of the complement mediated function would remain intact in such a patient
A) Generation of neutrophil chemotactic factors
B) Opsonization of bacteria
C) Generation of anaphylatoxins
D) None of the above
Which of the following cells or molecules would have specificity that is unique for only one of
several millions of possible antigens
A) A macrophage
B) A dendritic cell
C) Complement
D) A B-lymphocyte
Which one of the following antibodies would be considered to have the greatest specificity for
an epitope
A) The antibody that forms the fewest non-covalent bonds with the epitope
B) The antibody that has amino acids in the hypervariable region that form no bonds with
the epitope
C) The antibody with the highest affinity for the epitope
D) The antibody with the lowest affinity for the epitope

nolan x soju
Which one of the following is an important difference between innate immune response and
adaptive immune responses?
A) Innate responses (eg phagocytosis, complement activation) improve significantly with
each exposure to pathogens
B) Innate responses often can take 1-2 weeks to begin destroying infectious
microorganisms, while adaptive responses destroy infections almost immediately after
the very first encounter with a pathogen
C) An adaptive response does not improve significantly with repeated exposure with the
same non-self antigen
D) Each cell involved in innate responses (eg. Phagocytes) can be stimulated to respond by
many different molecules, while each cell involved in adaptive responses (eg.
Lymphocytes) can be stimulated to respond by only one molecular structure (called an
antigen)

Mochii_
The total number of combination of all gene fragments to produce antigen-specific receptors
during somatic recombination is far less than that actual number of different specificities that
are actually produced. This additional variable gene diversity is generated during somatic
recombination in which one of the following ways
A) By ADCC
B) By modifying the epitope-binding groove as the alpha chain of the receptor folds into its
final tertiary configuration
C) By looping out heavy chain isotype genes
D) By inserting nucleotides between variable portion gene fragment nucleotide sequences
before those fragments are ligated
Which one of the following isotype of antibodies is involved in providing protection to a fetus or
a newborn infant
A) IgG
B) IgM
C) IgA
D) IgE
Although IgE antibodies can cause quite serious allergic reactions this isotype of antibody does
appear to be very important in providing immunity from which one of the following?
A) Parasitic worms

nolan x soju
B) Tuberculosis bacteria that infect macrophages (and resist destruction once inside the
cell)
C) The smallpox virus
D) Bacteria that cause pneumonia ( most of these bacteria have a capsule)
Circulating lymphocytes will be found in which one of the following locations just before they
enter the blood circulation from the lymphatic system?
A) A lymph node
B) The thoracic duct
C) The bone marrow
D) A high endothelial venule

Mochii_
The development of bone marrow stem cells to either B-lymphocytes or T-lymphocytes is often
initiated or controlled by interactions between the stem cell and cells that are permanent
residents of either the bone marrow or the thymus. Which ONE of the following cell types is
involved in initiating or controlling B-cells development in the bone marrow or T-cell
development in the thymus
A) Epithelial cells in the thymus express CD4 or CD8
B) Epithelial cells in the bone marrow that expresses CD4 or CD8
C) Stromal cells that express cell adhesion molecules
D) Cortical epithelial cells in the thymus that express MHC class I and MHC class II
The much more rapid appearance of antibodies during a secondary response, compared to the
primary response is due mainly to which of the following?
A) A much improved presentation of peptide antigens to T-helper 1 cells by B-lymphocytes
during the secondary response
B) A much higher number of B-lymphocytes at the beginning of the secondary response
specific for the antigen that stimulated both the primary and secondary antibody
responses
C) Improved binding of antibody molecules to Fc receptors on the surface of phagocytes
which leads to a higher rate of peptide presentation to B lymphocytes
D) Each B lymphocyte participating in a secondary antibody response is specific for a very
large number of different antigens, while each B-cell participating in the primary

nolan x soju
response is specific for only one antigen
An antibody specific for one epitope could have which of the following immunoglobulin types
as the constant portion of that antibody molecule>
A) M
B) A
C) G
D) All of these
Which one of the following cells is generally the first to encounter a bacterial infection in the
tissues, is usually capable of phagocytizing some (or even all) of those bacteria, and secrete a
wide range of cytokines and chemokines that affect nearby blood vessels that can also
participate in eliminating the infection
A) A macrophage
B) A T-helper cell
C) A neutrophil
D) A dendritic cell

Mochii_
Priming of naïve T-helper (specific for non-self peptides) by mature dendritic cell (DC in the
lymph node involves which of the following cell interactions)
A) CD28 on DC with CD56 on naïve T cell
B) MHC Class II on DC with CD4 on naïve T-cell
C) Peptide presented by DC with T-cell epitope-specific receptor on naïve T cell
D) CD40 on DC with CD40L on naïve T-cell
38 Virus neutralization (by antibodies) does NOT require the involvement of which of the
following?
a) MHC Class I antigen-presenting molecules
b) none of these
c) activated complement
d) phagocytes
29 The name given to the amino acids that actually form bonds between an antibody molecule
and an antigen for which that antibody molecule is very strongly specific are called the: (one
best answer)
a) framework amino acids
b) the hypervariable amino acids
c) the variable amino acids
d) the domain amino acids

nolan x soju
24 Interferon-gamma
a) Is synthesized by macrophages
b) Is synthesized by erythrocytes
c) Is released as a consequence of antigen- or mitogen- induced activation of T-
lymphocytes
d) Specifically binds to the antigen that induces its release

23 Phagocytes can NOT attach to the cell wall surface of bacterial cells in which of the following
ways? Assume that the correct lipopolysaccharides are present on the surface of the bacterial
cells
a) Attaching to CD4 molecules on the bacterial cell wall surface using MHC Class II
molecules (on the phagocyte’s surface)
b) Directly attaching to the bacterial cell wall lipopolysaccharides
c) Using an antibody molecule that is specific for an antigen on the surface of the bacterial
cell
d) Using a fragment resulting from complement activation by the bacterial cell

Mochii_
27 Which one of the following surface molecules is expressed by T-helper cells that have just
been primed in the lymph node that will participate in sending a temporary inhibitory signal to
that primed T-cell?
a) CTLA-4
b) CD4
c) CD28
d) B-7
17 Where do almost all mature, but naïve, T-lymphocytes encounter the epitopes for which
they are specific during the priming phase
a) In the thoracic duct (just before entering the blood circulation)
b) In the HEV
c) In the lymph node
d) In the medulla of the thymus (involves antigen presentation by dendritic cells)
22 If a naïve (unprimed) T-cytotoxic cell encounters the antigen for which it is specific without
any co-stimulation (e.g., B7-CD28), that T-cell will then do which one of the following?
a) Become anergic
b) Express CTLA-4 in high concentration on its surface, resulting in a rapid increase in the
secretion of the cytokines by that T-cytotoxic cell
c) Undergo significant cell division, greatly expanding the number of T-cells specific for

nolan x soju
that antigen
d) Die (via apoptosis)
A typical IgG isotype antibody molecule does not consist of which of the following?
A) Two identical antigen-binding sites
B) Two identical heavy chains (~440) amino acids in length)
C) A “J” (joining) chain
D) Two identical light chains (~220 amino acid in length)

Mochii_
The inflammatory response is one manifestation of the innate immune system. The response is
initiated by the presence of bacterial cells in the extravascular tissues (i.e outside of the blood
vessels) phagocytes (e.g macrophage) that encounter these bacterial cells are stimulated to
release chemicals (cytokines and chemokines) that can have multitude of effects on nearby
blood vessels and the contents of those blood vessels. Which of the following will occur as a
result of the release of these cytokines and chemokines by the phagocyte?
A) T-lymphocytes that are in the blood circulation will leave the blood circulation right at
the site of the bacterial infection, immediately enter the lymphatic system, and begin to
produce antibodies specific for antigens associated with the infecting bacteria
B) some phagocytic cells in the nearby blood stream will actually be attracted to the site of
the infection, squeeze between the cells that form the blood vessel and enter the tissue
to assist in fighting the invading bacteria present there
C) the spaces between the cells that form the blood vessel will not be increased, thus
preventing fluids from the blood to leak into the tissues
D) some of the phagocytes will be stimulated to also secrete antibodies specific for
antigens found on the surface of the bacterial cells
Activation of complement --- in antibody-epitope interaction results in several fragments,
many of which have biological significance. For example, which one of the following is a
fragment produced during complement activation that can actually help a phagocyte to attach
to the surface of a bacterial cell to which that phagocyte could not attach in the absence of this

nolan x soju
fragment?
A) C3b
B) C3 convertase
C) C3a
D) C3
The isotypes of antigen-specific receptors found on fully mature B-lymphocytes
A) IgD and secretory IgA
B) IgG and IgD
C) secretory IgA and IgG
D) IgM and IgD

Mochii_
The original specificity of any one B-lymphocyte is determined by which one of the following?
A) MHC Class I molecules
B) The amino acid composition of two short tails extending from the tail end of the
constant portion of B-cell’s antigen receptor that are embedded into the membrane
surface of that B-cells surface
C) The specificity of the antigen receptor on that B-cell’s surface
D) The type of CD molecules on that B-cell’s surface
Peptides from which of the following complex biomolecules would most likely eventually be
presented to T-cells using MHC Class 1?
A) bacterial proteins that were degraded in a phagolysosome
B) peptides produced by the degradation of an exotoxin during phagocytosis
C) a proteins of tuberculosis bacteria that infect macrophages and resist destruction once
inside the cell
D) virus proteins in a cell that has been infected by a virus
One isotype of antibody is uniquely designed to be highly resistant to degradation by proteases
(enzymes that degrade proteins). This isotype is found in the highest concentration in the
intestinal lumen, the fluids of the lungs, the vaginal canal and human breast milk. This isotype is
A) IgG
B) IgM

nolan x soju
C) monomeric IgA
D) dimeric IgA

Antibody production by B-lymphocytes is initiated by direct contact between the B-lymphocyte


antigen receptor and the antigen for which that receptor is specific, and which one of the
following additional requirements?
A) help from a T-helper 2 (TH2) lymphocyte
B) contact between the Fc portion of the antibody and an Fc receptor on the surface of a T-
helper 2 (TH2)
C) help from a macrophage that is presenting peptides using MHC Class II molecules
D) help from a T-helper cell that has been stimulated by MHC Class I-presented peptides

Mochii_
The biological properties of an antibody molecule not directly associated with specificity (e.g.,
complement activation, opsonization of phagocytes, initiating allergic reactions involving mast
cells, etc.) involve the participation of amino acids in which one of the following locations? Hint
this location also determines the isotype of the antibody
A) the constant portions of the heavy chains
B) the domain that is at the amino-terminal end of the heavy chain
C) the constant portions of the light chains
D) Fab
Antibody-mediated allergic reactions involve the attachment of one of the five isotypes to
which one of the following types of cells in the body?
A) any phagocytic cell
B) macrophage (via Fc receptors for the IgG isotype)
C) mast cells
D) NK cells
Which of the following human cell types would not be capable of presenting peptides using
MHC Class 1?
A) B-lymphocytes
B) dendritic cells
C) fully mature red blood cells

nolan x soju
D) macrophages
The source of all undifferentiated stem cells that can produce any of the cells involved in innate
or adaptive immune responses is which one of the following?
A) the spleen
B) the bone marrow
C) any lymph node
D) the thoracic duct

Mochii_
If you wanted an antibody that was capable of neutralizing a virus AND then contributing to the
destruction of that virus by phagocytosis, you would select which one of the following isotypes
for that antibody so that is worse be capable of carrying out these two functions (neutralization
and opsonization))?
A) IgE
B) IgA
C) IgG
D) IgM
A macrophage that is stimulated to initiate phagocytosis of a bacterial cell that has a capsule
needs to also involve an antibody specific for an antigen on the capsule surface that also
attaches to which one of the following on the surface of the macrophage?
A) an antigen on the macrophage surface that is Identical to the antigen on the capsule
surface for which the antibody is specific
B) a receptor for a fragment of activated complement (the complement has been activated
by the binding of the antibody to the capsule antigen
C) a MHC Class Il molecules
D) an Fc receptor
If you wanted an adaptive immune response to lead to virus neutralization and eventual
phagocytosis, destruction of virus infected cells) which of the following types of cells would

nolan x soju
have to be involved in some part of that complete response (that eventually results in the
degradation of the virus)?
A) T-cytotoxic cells, macrophage, and dendritic cell
B) T-helper cells macrophage and dendritic cell
C) B-lymphocytes, T-helper cells, and macrophage
D) Macrophage, T-cytotoxic cells, and B-lymphocytes
Natural Killer (NK) cells carry out an important process that results in the destruction of cancer
cells in the body known as antibody-dependent cellular cytotoxicity (ADCC). However, these NK
cells can not carry out this process unless which one of the following is also involved?
A) antibodies of the IgG isotype that are specific for non-self antigens on the surface of the
NK cell
B) antibodies of the IgG isotype that are specific for non-self antigens on the surface of the
cancer cell
C) antibodies of the IgE isotype that are specific for non-self antigens on the surface of the
NK cell
D) antibodies of the igE isotype that are specific for non-self antigens on the surface of the
cancer cell

Mochii_
Which one of the following isotypes of antibodies is responsible for initiating the release of
large amounts of histamine that results in the symptoms most often associated with allergic
reactions? Hint this isotype is normally found in very low amounts in the serum of non-allergic
persons,
A) IgD
B) IgM
C) IgG
D) IgE
Which one of the following types has the MOST Influence on the antibody response carried out
by a B lymphocyte?
A) phagocytes that are presenting peptides to T-helper cells
B) cytokines released by a T-helper1 cell
C) a T-cytotoxic cell
D) cytokines released by a T-helper2 cell
TH1 cells most often "help" macrophages to become more efficient phagocytic cells. This
enhancement of phagocytosis is generally initiated by which one of the following cytokines
secreted by these TH1 cells during this macrophage-helping process?
A) interferon-gamma
B) interleukin-2

nolan x soju
C) interleukin-8
D) CD40L
Which one of the following types has the MOST influence on the antibody response carried out
by a B lymphocyte?
A) phagocytes that are presenting peptides to T-helper cells
B) cytokines released by a T-helper1 cell
C) a T-cytotoxic cell
D) cytokines released by a T-helper2 cell

Mochii_
If a macrophage attaches to an IgG antibody molecule (that will eventually assist that
macrophage to bind to a bacterial cell), the receptor on the macrophage is binding to which
one of the following parts of the intact antibody molecule?
A) the first six amino acids at the N-terminal end of the heavy chain
B) the variable domain heavy chain
C) the constant portion of the light chain
D) the Fc portion of the antibody molecule
Which one of the following types of cells is NOT capable of phagocytosis?
A) T-helper cell
B) dendritic cell
C) macrophage
D) neutrophil
Almost immediately after a naive T-cell has been prior and, it will begin to express an important
receptor on its surface that will respond to a cytokine that is secreted by that same recently-
primed T-cell. This cytokine-cytokine receptor interaction will initiate extensive cell division of
that primed T-cell. This important cytokine receptor interaction is which one of the following?
A) IL-2 and a high affinity IL-2 receptor
B) IL-1 and low affinity IL-1 receptor
C) IL-7 and a high affinity IL-7 receptor

nolan x soju
D) interferon-gamma and the interferon-gamma receptor
Although each isotype has a hinge region that can be important in activating complement after
an antibody molecule has bonded to its antigen, only TWO isotypes that can usually activate
complement (select ONE answer that is correct)
A) IgA (the monomeric form in the serum)
B) IgE
C) IgM
D) IgD

Mochii_
Antibodies that are able to provide protection from the effects of bacterial exotoxins do so in
which one of the following ways?
A) cause blood vessels located near the site of the toxin to become slightly leaky, thus
allowing cells in the blood stream to enter the tissues and destroy the toxin molecules
B) bind to the toxin molecules and prevent the toxins from binding to body cell surfaces
C) activate complement that, when activated, breaks down the toxin molecules into
harmless fragments
D) enable macrophage to present the toxin antigens to T-lymphocytes

In the absence of antibodies, phagocytes are usually not able to effectively respond in any way
against which of the following?
A) exotoxins produced and secreted by bacteria
B) bacteria that are present in the body but outside of the blood vessels
C) bacterial cells to which the phagocytes firmly attach
D) none of these

Select incorrect answer. Complement that has been activated by pathogenic bacterial cells can
contribute (directly or indirectly to the destruction or elimination of those pathogenic bacterial

nolan x soju
cells
A) by attracting more phagocytes to the area of complement activation
B) by stimulating somatic hypermutation in the phagocytes that are already actively
engaged in phagocytizing the bacterial cells
C) by assisting phagocytes to attach to bacterial cells
D) by destroying the bacterial cell membrane
T-cell-mediated immune responses can result in
A) formation of granulomas
B) induration at the reaction site
C) rejection of a kidney transplant
D) all of the above

Mochii_
Although IgE antibodies can cause quite serious allergic reactions, this isotype of antibody does
appear to be very important in providing immunity from which one of the following?
A) tuberculosis bacteria that infect macrophages (and resist destruction once inside the
cell)
B) parasitic worms
C) bacteria that cause pneumonia (most of these bacteria have a capsule)
D) the smallpox virus
Concerning a monoclonal antibody, all of the following statements are correct EXCEPT that it
A) has antibody combining sites that are identical
B) may belong to the IgM, IgA, IgE, IgD, or IgG class,
C) can be obtained by the hybridoma technique
D) has found no diagnostic or therapeutic use
The rejection of a mis-matched transplant by the recipient of that transplant is initiated by
which one of the following that is part of the transplant?
A) HLA surface molecules that are also found in the body of the recipient of the transplant
B) HLA-DP, DQ, DR that are also found in the body of the recipient of the transplant
C) MHC molecules that are not found in the body of the recipient of the transplant
D) MHC Class I molecules that are also found in the body of the recipient, but these
transplant-associated MHCS have both of the chains that contribute to the formation of

nolan x soju
the peptide-binding groove degraded (which results in a lot of peptides that can be
bound by the recipient's own HLAS)
The function of the enzyme reverse transcriptase in HIV infection is to:
A) produce DNA that can be Incorporated Into the host cell's DNA from viral RNA
B) produce new viral RNA from the host cell's DNA
C) attach the virus to the host cell's plasma membranes
D) convert host RNA into viral RNA
Immediate hypersensitivity skin reactions
A) exhibit red flare due to vasodilation
B) can be elicited by monovalent haptens
C) usually occur within 15 minutes
D) all are correct

Mochii_
A positive DTH (delayed-type hypersensitivity) skin reaction involves the interaction of
A) antigen, macrophages and complement
B) antigen, antigen-sensitive lymphocytes, and macrophages
C) antigen, complements and lymphokines
D) antigen-antibody complexes, complement, and neutrophils
Rheumatoid factor is correctly described as
A) primarily a macrophage product
B) a protease found in synovial fluid
C) a synonym for auto-anti-IgG antibodies
D) not involved in immune complex formations
AIDS is caused by a human retrovirus that kills
A) B lymphocytes
B) CD4-positive Templates
C) lymphocyte stem cells
D) CD8-positive T lymphocytes
The lesions in immune complex induced glomerulonephritis
A) are dependent on the presence of macrophages
B) are dependent on erythrocytes and complement

nolan x soju
C) result in increased production of urine
D) require both complement and neutrophils
The first line of defense against viruses is
A) IgG directed to an internal viral antigens
B) IgM directed to an external viral antigens
C) antibodies directed against external viral antigens
D) interferon
A transplant of tissue between individuals of the same species, but with different genetic
background is called a(n):
A) Xenograft
B) autograft
C) isograft
D) allograft

Mochii_
Granulomatous hypersensitivity is correctly described by which of the following statements?
A) it has not be encountered in schistosomiasis
B) it results from the presence of persistent microorganisms within macrophages
C) it does not cause pathological effects in diseases associated with T cell-mediated
immunity
D) it is not clinically important
Which of the following statements BEST explains the relationship between inflammation of the
heart (carditis) and infection with group A beta-hemolytic streptococci?
A) Streptococcal antigens bind to IgE on the surface of heart tissue and histamine is
released
B) Streptococci are polyclonal activators of B cells
C) Streptococci are ingested by neutrophils that release proteases that damage heart
tissue
D) Streptococcal antigens induce antibodies cross-reactive with heart tissue
Patients with Wiskott-Aldrich syndrome have:
A) an increased number of suppressor T cells
B) an increased number of B cells
C) low platelet level in blood
D) selective igA deficiency

nolan x soju
Chemically induced tumors have tumor-associated transplantation antigens that
A) do not induce an immune response
B) are always the same for a given carcinogen
C) are different for two tumors of different histologic type ever if induced by the same
carcinogen
D) are very strong antigens
Certain HLA genes are linked to diseases such as ankylosing spondylitis. This linkage has all the
following characteristics except:
A) it may be the result of cross-reactivity between self-antigen and infection agent
B) it carries no increased risk for a specific disease for those individuals with the gene
C) it may be the result of a MHC molecule serving as an attachment site for an infectious
agent
D) it may be the result of closely linked genes

Mochii_
Circulating immune complexes are an etiologic factor in the following diseases
A) Goodpasture's syndrome
B) glomerulonephritis after treatment with horse anti-tetanus antiserum
C) farmer's lung
D) Myastenia gravis
Which of the following statements is (are) characteristic of contact sensitivity?
A) Some chemicals acting as haptens induce sensitivity by covalently binding to host
proteins acting as carriers
B) Sensitization can be passively transferred with serum from an allergic individual
C) The best therapy is administration of the antigen
D) Patch testing with the allergens is useless for diagnosis
The human immunodeficiency virus primarily infects:
A) plasma cells
B) helper cells
C) killer cells
D) epithelial cells lining the genitourinary tract

An IgA antibody to a red cell antigen is unlikely to cause autoimmune hemolytic anemia

nolan x soju
because
A) its Fc region would not bind receptors for Fc on phagocytic cells
B) it has too low affinity
C) it requires secretory component to work
D) it would be made only in the gastrointestinal tract
Opsonin-treated bacteria are more ready engulfed by phagocytes than are untreated bacteria
because
A) opsonin induces lysosomal enzymes
B) Opsonin digest the wall component
C) the surface of a phagocyte contains receptors for the Fc portion of an antibody
D) the capsule is removed by opsonin

Mochii_
The Arthus reaction involves
A) cross-linking of igE antibody
B) Formation of antigen-antibody precipitates on vessel walls
C) lymphocytic infiltration around veins
D) only 1 and 3 are correct

Which of the following experiments suggests that tumor cells express tumor-specific
transplantation antigens?
A) when an allogenic mouse is injected with the tumor cells, reject the tumor and survives
B) when a syngenic mouse is injected with the tumor cells, reject the tumor and survives
C) when a syngenic mouse is injected with the tumor cells, accepts the tumor and dies
D) when an allogenic mouse is injected with the tumor cells, accepts the tumor and dies

Clinical and experimental data indicate that certain forms of which one of the following types of
biomolecules may correlate with frequencies of certain autoimmune disease?
A) unusual forms of proteasome enzyme subunits
B) V,D,J gene fragments used to produce receptors specific for cancer-associated peptides
C) HLA molecules

nolan x soju
D) CD4 or CD8 surface molecules

Which of the following clinical cases is most likely to involve a reaction to a hapten in its
etiology?
A) Arthus reaction
B) Goodpasture's syndrome
C) hemolytic anemia after treatment with penicillin
D) rheumatoid arthritis
Which of the following statements correctly describes Arthus hypersensitivity?
A) it is not mediated by complement fixing lgG
B) it occurs 2-4 days after antigen challenge
C) It produces cell death and extensive local destruction
D) it cannot be manifested in the absence of high levels of IgE

Mochii_
The following mechanism(s) may be involved in the clinical efficacy of injection therapy
(hyposensitization)
A) enhanced production of CD8 T cells
B) induces large amounts of endogenous anti-histamines
C) directly affects stability of membranes
D) enhanced production of IgG, which binds allergen before it reaches

Mechanisms of antibacterial immunity include all of the following EXCEPT


A) antibody- and complement-mediated opsonization for enhanced phagocytosis
B) antibody neutralization of toxins
C) cytotoxic T cell-mediated bacteriolysis
D) antibody and complement-mediated bacteriolysis

Most autoimmune diseases are caused by


A) A constellation of genetic and environmental event
B) known infectious organism

nolan x soju
C) a T-cell defect
D) a single genetic defect
Bone marrow transplantation in immunocompromised patients presents which major problem?
A) potentially lethal graft versus host disease
B) high risk of T cell leukemia
C) inability to use a live donor
D) delayed hypersensitivity

Type II hypersensitivity is correctly described by which of the following statements?


A) It is complement independent
B) it does not involve killer cells
C) it is antibody independent
D) it is antibody-dependent cytotoxic hypersensitive

Mochii_
The most likely clinical consequence of a genetic deficiency of complement component C3 is
increased
A) incidence of malignancy
B) susceptibility to fungal infections
C) susceptibility to bacterial infections
D) susceptibility to viral infections

The major mechanism of host resistance to tuberculosis is


A) humoral antibodies
B) high level of calcium in serum
C) delayed hypersensitivity
D) massive proliferation of polymorphonuclear leukocytes

Grossly elevated serum levels of IgE can be found in persons with


A) brucellosis
B) parasitic worm infestations
C) Arthus hypersensitivity
D) Tuberculosis

nolan x soju
What is the role of class II MHC proteins on donor cells in graft rejection?
A) They induce IgE which mediate graft rejection
B) They are the receptors for interleukin-2, which is produced by macrophages when they
attack the donor cells
C) They are recognized by helper T cells, which then activate cytotoxic T cells to kill the
donor cells
D) They induce the production of blocking antibodies that protect the graft

Serum sickness is characterized by


A) phagocytosis of complexes by granulocytes 1)
B) consumption of complement
C) deposition of immune complexes in blood vessel walls
D) all of the above

Mochii_
Serum sickness occurs only
A) anti-basement membrane antibodies are present
B) when soluble immune complexes are formed
C) when igE antibody is produced
D) in the absence of neutrophils
Removal of immune complexes in correctly described by which of the following statements?
A) occurs primarily in the liver
B) it is independent of C3 for particulate complexes
C) it is independent of size of the immune complexes
D) it has not been associated of the Kupffer's cells

Which of the following does not involve CMI (cell mediated immunity)?
A) Rejection of a liver graft
B) serum sickness
C) Tuberkulin reaction
D) Contact sensitivity to lipstick

nolan x soju
destruction of tumor cells may be achieved by
A) antibody-dependent cell
B) cytotoxicity cytotoxic T lymphocytes
C) all are correct
D) antibodies and complement

You have a patient who makes autoantibodies against his own red blood cells. leading to
hemolysis. Which o of the following mechanisms is MOST likely to explain the hemolysis?
A) Complement is activated and membrane attack complexes lyse the red cells
B) Perforins from cytotoxic T cells lyse the red cells
C) Neutrophils release proteases that lyse the red cells
D) Interleukin-2 binds to its receptor on the red cells, which results in lysis of the red cells

Mochii_
Lupus erythematous is
A) none of these
B) a disease of red and white cells
C) free from an immune complex disease component
D) associated with antibodies against nucleic acid

Your patient is a child who has no delectable Tor B cells This immunodeficiency is most
probably the result of a defect in
A) the thymus
B) the bursal equivalent
C) T cell-B cell interaction
D) stem cells originating in the bone marrow

Clinical consequence(s) of C3 deficiency is (are):


A) increased susceptibility to viral Infections
B) Increased susceptibility to bacterial infection
C) Increased incidence of tumors
D) increased susceptibility to fungal infections

nolan x soju
Which one of the following is the BEST method of reducing the effect of graft versus-host
disease in a bone marrow recipient?
A) Removing pre-B cells from the graft
B) Matching the complement components of donor and recipient
C) Administering alpha interferon
D) Removing mature T cells from the graft
Antibodies that are able to provide protection from the effects of bacterial exotoxins do so in
which one of the following ways?
A) enable macrophage to present the toxin antigens to T-lymphocytes
B) Activate complement that, when activated breaks down the toxin molecules into
harmless fragments
C) bind to the toxin molecules and prevent the toxins from being the body cell surfaces
D) cause blood vessels located near the site of the toxin to become slightly leaky, thus
allowing cells in the blood stream to enter the tissues and destroy the toxin molecules

Mochii_
A patient is admitted with multiple bacterial infections and is found to have a complete absence
of C3 Which complement-mediated function would remain intact in such a patient
A) Lysis of bacteria B
B) opsonization of bacteria
C) generation of anaphylatoxins
D) none of the above

The final damage to vessels in immune complex-mediated arthritis this is due to


A) histamine
B) NK call
C) lysosomal enzyme of polymorphonuclear leukocytes
D) Cytotoxic T cell
Patients with Chediak-Higashi syndrome have phagocytes that
A) Lack C3b actors that are needed for attachment to bacteria
B) show a reduced ability of their lysosomes to fuse with a phagosomes to release

nolan x soju
microbicidal substances
C) show defects for the engulfment of microbes
D) produce protein that interfere with chemotaxis

Mochii_
1. One isotype of antibody is uniquely designed to be highly resistant to degradation by proteases
(enzymes that degrade proteins). This isotype is found in the highest concentration in the
intestinal lumen, the fluids of the lungs, the vaginal canal and human breast milk.This
isotype is:
a. Monomeric IgA
b. IgG
c. Dimeric IgA
d. IgM

2. Which of the following cells or molecules would have specificity that is unique for only one of
several millions of possible antigens/
a. A dendritic cell
b. A macrophage
c. A B-lymphocyte
d. Complement

3. The inflammatory response is one manifestation of the innate immune system. The response is
initiated by the presence of bacterial cells in the extravascular tissues (i.e.., outside of the blood
vessels). Phagocytes (e.g.. Macrophage) that encounter these bacterial cells are stimulated to
release chemical (cytokines and chemokines that can have a multitude of effects on nearby

nolan x soju
blood vessels and the contents of those blood vessels. Which of the following will occur as a
result of the release of these cytokines and chemokines by the phagocytes?
A. Some phagocytic cells in the nearby blood stream will actually be to the site of infection…….
between the cells that form blood vessel and enter the tissue to assist in fighting the
invading bacteria present here
B. T-lymphocytes that are in the blood circulation will leave the blood circulation right at the
site of the bacterial infection, immediately enter the lymphatic system, and begin to
produce antibodies specific for antigens associated with the infecting bacteria
C. The spaces between the cells that form the blood vessel will not be increasing thus
preventing fluids from the blood to leak into the tissue
D. Some of the phagocytes will be stimulated to also secrete antibodies specific for antigens
found on the surface of the bacterial cells

4. Which one of the following isotypes of antibodies is responsible for initiating the release of
large amounts of histamine that results in the symptoms most often associated with allergic
reactions? Hint: this isotype is normally found in very low amounts in the serum of non-allergic
persons.
a. IgD
b. IgG
c. IgM
d. IgE

5. Which one of the following antibodies would be considered to have the greatest specificity for
an antigen?
a. The antibody that forms the fewest non-covalent bonds with the…….
b. The antibody that has amino acids in the hypervariable region that form no bonds with
the…..
c. The antibody with the lowest affinity for the epitope
d. The antibody with the highest affinity for the epitope

6. Circulating lymphocytes will be found in which one of the following locations just before they
enter the blood circulation from the lymphatic system?
a. A lymph node
b. A high endothelial venule
c. The thoracic duct
d. The bone marrow

7. Select incorrect answer. Complement that has been activated by pathogenic bacterial cells can
contribute (directly or indirectly) to the destruction or elimination of those pathogenic bacterial
cells

nolan x soju
a. By assisting phagocytes to attach to bacterial cells
b. By attracting more phagocytes to the area of complement activation
c. By stimulating somatic hypermutation in the phagocytes that are already actively engaged
in phagocytizing thew bacterial cells
d. By destroying the bacterial cell membrane

8. Which one of the following surface molecules is expressed by T-helper cells that have just been
primed in the lymph node that will participate in sending a temporary inhibitory signal to that
primed T-cell?
a. CTLA-4
b. B-7
c. CD28
d. CD4

9. If a naïve (unprimed) T-cytotoxic cell encounters the antigen for which it is specific without any
co-stimulation (e.g., B7-CD28), that T-cell will then do which one of the following?
a. Die (via apoptosis)
b. Become anergic
c. Express CTLA-4 in high concentration on its surface, resulting in a rapid increase in the
secretion of cytokines by that T-cytotoxic cell
d. Undergo significant cell division, greatly expanding the number of T-cells specific for that
antigen

10. TH1 cells most often “help” macrophages to become more efficient phagocytic cells. This
enhancement of phagocytosis is generally initiated by which one of the following cytokines
secreted by these TH1 cells during this macrophage helping process?
a. CD40L
b. Interferon gamma
c. Interleukin-8
d. Interleukin-2

11. The source of all undifferentiated stem cells that can produce any of the cells involved in innate
or adaptive immune responses in which one of the following?
a. The bone marrow
b. Any lymph node
c. The spleen
d. The thoracic duct

nolan x soju
12. Which of the following human cell types would not be capable of presenting peptides using
MHC Class I?
a. Fully mature red blood cells
b. Macrophages
c. Dendritic cells
d. B-lymphocytes

13. If a macrophage attaches to an IgG antibody molecule (that will eventually assist that
macrophage to bind to a bacterial cell), the receptor on the macrophage is binding to which one
of the following parts of the intact antibody molecule?
a. The Fc portion of the antibody molecule
b. The variable domain of the heavy chain
c. The constant portion of the light chain
d. The first six amino acids at the N-terminal end of the heavy chain

14. The name given to the amino acids that actually form bonds between an antibody molecule and
an antigen for which that antibody molecule is very strongly specific are called the (one best
answer)
a. The variable amino acids
b. Framework amino acids
c. The hypervariable amino acids
d. The domain amino acids

15. Peptides from which of the following complex biomolecules would most likely eventually be
presented to T-cells using MHC Class I?
a. Bacterial proteins that were degraded in a phagolysosome
b. Virus proteins in a cell that has been infected by a virus
c. Peptides produced by the degradation of an exotoxin during phagocytosis
d. Proteins of tuberculosis bacteria that infect macrophages and resist destruction once inside
the cell

16. The original specificity of any one B-lymphocyte is determined by which one of the following?
a. The amino acid composition of two short tails extending from the tail end of the constant
portion of the B-cells antigen receptor that are embedded into the membrane surface of
that B-cell
b. The type of CD molecules on that B cells surface
c. The specificity of the antigen receptors on the B-cell’s surface
d. MHC Class I molecules

nolan x soju
17. If you wanted an antibody that was capable of neutralizing a virus AND then contributing to the
destruction of that virus by phagocytosis, you would select which one of the following isotypes
for that antibody so that is should be capable of carrying out these two functions (neutralizing
and opsonization)?
a. IgE
b. IgG
c. IgA
d. IgM

18. Antibody-dependent cellular cytotoxicity (ADCC) requires the involvement of which ONE of the
following?
a. Antibodies of the IgD isotype
b. Antibodies of the IgM isotype that are specific for non-self-antigen epitopes on the surface
of cancer cells
c. Antibodies of the IgG isotype that are specific for non-self-antigens epitopes on the surface
of cancer cells
d. T-cytotoxic lymphocytes
19. Antibodies that are able to provide protection from the effects of bacterial exotoxins do so in
which one of the following ways/
a. Bind to the toxin molecules and prevent the toxins from binding to body cell surfaces
b. Cause blood vessels located near the site of the toxin to become slightly leaky, thus allowing
cells in the blood stream to enter the tissues and destroy the toxin molecules
c. Activate complement that, when activated, breaks down the toxin molecules into harmless
fragments
d. Enable macrophage to present the toxin antigens to T-lymphocytes

20. Which one of the following interactions between surface molecules must take place for a T-
cytotoxic cell to be stimulated to begin to destroy a virus-infected cell?
a. MHC Class I on virus-infected cells and CD8 on T-cytotoxic cell
b. MHC Class I on T-cytotoxic cell and CD8 on virus infected cell
c. MHC Class I on virus infected cell and CD4 on T-cytotoxic cell
d. MHC Class II on virus infected cell and CD4 on T-cytotoxic cell

21. Which ONE of the following is an important difference between innate immune responses and
adaptive immune response?
a. Innate responses often can take 1-2 weeks to begin destroying infectious microorganisms,
while adaptive responses destroy infections almost immediately after the very first
encounter with a pathogen

nolan x soju
b. Innate responses (e.g., phagocytosis, complement activation) improve significantly with
each exposure to pathogens
c. An adaptive response does not improve significantly with repeated exposures to the same
non-self-antigen
d. Each cell involved in adaptive responses (e.g., lymphocytes) can be stimulated to respond by
only one molecular structure (called an antigen)

22. An antibody specific for one epitope could have which of the following immunoglobulin types as
the constant portion of that antibody molecule?
a. G
b. M
c. A
d. All of these

23. Interferon-gamma
a. Is released as a consequence of antigen or mitogen induced activation of T -lymphocytes
b. Is synthesized by macrophages
c. Is synthesized by erythrocytes
d. Specifically binds to the antigen that induces its release

24. A patient is admitted with multiple bacterial infections and is found to have a complete absence
of C3. Which complement-mediated function would remain intact in such a patient
a. Generation of anaphylatoxins
b. Generation of neutrophil chemotactic factors
c. None of the above
d. Opsonization of bacteria

25. The isotypes of antigen-specific receptors found in fully mature B-lymphocytes


a. IgM and IgD
b. Secretory IgA and IgG
c. IgG and IgD
d. IgD and secretory IgA

26. Antibody mediated allergic reactions involve the attachment of one of the five isotypes to which
one of the following types of cells in the body/
a. Any phagocytic cell
b. Macrophage (via Fc receptors for the IgG isotype)
c. Mast cells

nolan x soju
d. NK cells

27. If you wanted an adaptive immune response to lead to virus neutralization and eventual
phagocytosis, (but not destruction of virus-infected cells) which of the following types of cells
would have to be involved in some part of that complete response (that eventually results in
degradation of the virus)?
a. T-helper cells, macrophage and dendritic cell
b. T-cytotoxic cells,macrophage and dendritic cell
c. Lymphocytes,Macrophage, T-cytotoxic cells and B-lymphocytes
d. B-lymphocytes, T-helper cells and macrophage

28. Activation of complement by an antibody epitope interaction results in several fragments, many
of which have biological significance. For example, which one of the following is a fragment
produced during complement activation that can actually help a phagocyte to attach to the
surface of a bacterial cell to which that phagocyte could not attach in the absence of this
fragment?
a. C3
b. C3 convertase
c. C3a
d. C3b

29. Although each isotype has a hinge region that can be important in activating complement after
antibody molecule has bonded to its antigen, only TWO isotypes that can usually activate
complement. select ONE answer that is correct)
a. IgM
b. IgD
c. IgA (the monomeric form in the serum)
d. IgE

30. Although IgE antibodies can cause quite serious allergic reactions, this isotype of antibody does
appear to be very important in providing immunity from which one of the following/
a. Parasitic worms
b. Tuberculosis bacteria that infect macrophages (and resist destruction once inside the cell)
c. Bacteria that cause pneumonia (most of these bacteria have a capsule)
d. The smallpox virus

31. Antibody production by B-lymphocytes is initiated by direct contact between the B-lymphocyte
antigen receptor and the antigen for which that receptor is specific, and which one of the
following additional requirements?

nolan x soju
a. Help from a T-helper 2 (TH2) lymphocyte
b. Help from a T-helper cell that has been stimulated by MHC Class I-presented peptides
c. Contact between the Fc portion of the antibody and an Fc receptor on the surface of a T-
helper (TH2) lymphocyte
d. Help from a macrophage that is presenting peptides using MHC class II molecules

32. The biological properties of an antibody molecule not directly associated with specificity (e.g.,
complement activation, opsonization of phagocytes, initiating allergic reactions involving mast
cells, etc.) involve the participation of amino acids in which one of the following locations? Hint:
this location also determines the isotype of the antibody
a. The domain that is at the amino-terminal end of the heavy chain
b. Fab
c. The constant portions of the light chains
d. The constant portions of the heavy chains

33. Almost immediately after a nave T-cell has been primed,it will begin to express an important
receptor on its surface that will respond to a cytokine that is secreted by that same recently
primed T-cell.this cytokine-cytokine receptor interaction will interaction will initiate extensive
cell division of that primed T-cell.This important cytokine receptor is which one of the following/
a. IL-2 and a high affinity IL-2 receptor
b. Interferon gamma and the interferon gamma receptor
c. IL-7 and a high affinity Il-7 receptor
d. IL-1 and low affinity IL-1 receptor

34. The IgG isotype of an antibody molecule is often used as an example of the typical
immunoglobin/antibody. Which of the following is physical or biological characteristics of one
antibody molecule that is the igG isotype?
a. Papain treatment will produce two identical Fab fragments and two identical Fc fragments
b. Can bind (via the Fc portion) to phagocytes and mast cells
c. Is able to bind two different antigens/epitopes
d. Has one heavy chain and two identical light chains

35. Which ONE of the following isotypes of antibodies is involved in providing protection to a fetus
or a newborn infant/
a. IgM
b. IgA
c. IgG
d. IgD

nolan x soju
36. A macrophage that is stimulated to initiate phagocytosis of a bacterial cell that has a capsule
need to also involve an antibody specific for an antigen on the capsule surface that also attaches
to which one of the following on the surface of the macrophage/
a. An Fc receptor
b. A receptor for a fragment activated complement (the complement has been activated by
the binding of the antibody to the capsule antigen
c. An antigen on the macrophage …. that is identical to the antigen on the capsule surface for
which the antibody is specific
d. MHC Class II molecules

37. The much more rapid appearance of antibodies during a secondary response, compared to the
primary response is due to mainly to which one of the following?
a. A much higher number of B lymphocytes at the beginning of the secondary response specific
for the antigen that stimulated both the primary and secondary antibody responses
b. Improved binding of antibody molecules to Fc receptors on the surface of phagocytes, which
leads to a higher rate of peptide presentation to B-lymphocytes
c. Each B-lymphocyte participating in a secondary antibody response is specific for a very
larger number of different antigens, while each B-cell participating in the primary response
is specific for only one antigen
d. A much improved presentation of peptide antigens to T-helper I cells by B-lymphocytes
during the secondary response

38. Priming of naïve T-helper cells (specific for non-self-peptides) by mature dendritic cells (DC) in
the lymph node involves which of the following cell interactions?
a. Peptide presented by DC with T cell epitope-specific receptor on naïve -T cell
b. MHC Class II on DC with CD4 on naïve T cell
c. CD28 on DC with CD56 on naïve T-cell
d. CD40 on DC with CD40L on naïve T-cell

39. A typical IgG isotype antibody molecule does not consist of which of the following?
a. Two identical light chains (- 220 amino acids in length)
b. A “J” (joining) chain
c. Two identical heavy chains (-440 amino acids in length)
d. Two identical antigen binding sites

40. Which one of the following types of cells is NOT capable of phagocytosis?
a. T-helper cell
b. Dendritic cell
c. Macrophage

nolan x soju
d. Neutrophil

41. Which one of the following cells is generally the first to encounter a bacterial infection in the
tissues, is usually capable of phagocytizing some (or even all0 of those bacteria, and secretes a
wide range of cytokines and chemokines that affect nearby blood vessels and recruit and
activate other cells that can also participates in eliminating the infection
a. Neutrophil
b. A T-helper cell
c. A dendritic cell
d. A macrophage

42. Which one of the following types has the MOST influence on the antibody response carried out
by a B-lymphocyte?
a. Cytokines released by a T-helper1cell
b. A T-cytotoxic cell
c. Cytokines released by a T-helper2 cell peptides to helper cell
d. Phagocytes that are presenting peptides to T-helper cells
43. Natural killer (NK) cell carry out an important process that results in the destruction of cancer
cells in the body known as antibody-dependent cellular cytotoxicity (ADCC).However, these NK
cells can not carry out this process unless which one of the following is also involves?
a. Antibodies of the IgG isotype that are specific for non-self-antigens on the surface of the
cancer cell
b. Antibodies of the IgE isotype that are specific for non-self-antigens on the surface of the NK
cell
c. Antibodies of the IgE isotype that are specific that are specific for non-self-antigens on the
surface of the cancer cell
d. Antibodies of the IgG isotype that are specific for non-self-antigens on the surface of the NK
cell

44. Phagocytes can NOT attach to the cell wall surface of bacterial cells in which of the following
ways? Assume that the correct lipopolysaccharides are present on the surface of the bacterial
cells
a. Directly attaching to the bacterial cell well lipopolysaccharides
b. Attaching to CD4 molecules on the bacterial cell surface using MHC Class II molecules (on
the phagocytes surface)
c. Using an antibody molecule that is specific for an antigen on the surface of the bacteria cell
d. Using a fragment resulting from complement activation by the bacterial cells

nolan x soju
45. Where do almost all mature, but naïve, T-lymphocytes encounter the epitopes for which they
are specific during the priming phase?
a. In the medulla of the thymus (involves antigen presentation by dendritic cells)
b. In the HEV
c. In the lymph node
d. In the thoracic duct (just before entering the blood circulation)

46. The development of bone marrow stem cells into either B-lymphocytes or T-lymphocytes is
often initiated or controlled by interactions between the stem cell and cells that are permanent
residents of either the bone marrow or the thymus. Which ONE of the following cell types is
involved in initiating or controlling B-cell development in the bone marrow or T-cell
development in the thymus? That express MHC Class I and MHC Class II
a. Cortical epithelial cells in the thymus that express MHC Class I and MHC Class II
b. Epithelial cells in the bone marrow that express CD4 or CD8
c. Epithelial cells in the thymus that express CD4 or CD8
d. Stromal cells that express cell adhesion molecules

47. Virus neutralization (by antibodies) does NOT require the involvement of which of the
following?
a. Phagocytes
b. Activated complement
c. MHC Class I antigen presenting molecules
d. None of these

48. Which one of the following types has the MOST influence on the antibody response carried out
by a B-lymphocyte
a. Phagocytes that are presenting peptides to a T-helper cells
b. Cytokines released by a T-helper 2 cell
c. Cytokines released by a T-helper 1 cell
d. A T-cytotoxic cell

49. The total number of combinations of all gene fragments to produce antigen specific receptors
during somatic recombination is far less than that actual number of different specificities that
are actually produced. This additional variable gene diversity is generated during somatic
recombination in which one of the following ways?
a. By ADCC
b. By lopping out heavy chain isotype genes
c. By inserting nucleotides between variable portion gene fragment nucleotide sequences
before those fragments are ligated
d. By modifying the epitope-binding groove as the alpha chain of the receptor folds into its

nolan x soju
final tertiary configuration

50. In the absence of antibodies, phagocytosis are usually not able to effectively respond in any way
against which of the following?
a. None of these
b. Bacterial cells to which the phagocytes firmly attach
c. Exotoxins produced and secreted by bacteria
d. Bacteria that are present in the body but outside of the blood vessels

51. Antihistamines
a. Are more effective given before, rather than after the onset of allergic symptoms
b. Do not influence the activity of leukotrienes
c. Bind to receptors for histamine, thereby preventing the histamine from exerting a
pharmacologic effect
d. All are correct

52. Which of the following statements about type I hypersensitivity is true?


a. It occurs when an IgE response is directed against pollens
b. It does not require IgE sensitized mast cells
c. It is also known as delayed type hypersensitivity
d. It is independent of pharmacologic mediators

53. Lupus erythematosis is


a. None of those
b. Free from an immune complex disease component
c. Associated with antibodies against nucleic acid
d. A disease of red, white cells

54. AIDS is caused by a human retrovirus that kills


a. CD8 positive T lymphocytes
b. B lymphocytes
c. Lymphocyte stem cells
d. CD4 positive T lymphocyte

55. The function of the enzyme reverse transcriptase in HIV infection is to:
a. Convert host RNA into viral RNA
b. Produce new viral RNA from the host cell’s DNA
c. Attach the virus to the host cells plasma membranes

nolan x soju
d. Produce DNA that can be incorporated into the host cells DNA from viral RNA

56. Which of the following experiments suggests that tumor cells express tumor specific
transplantation antigens?
a. When an allogenic mouse is injected with the tumor cells reject the tumor and survives
b. When a syngeneic mouse is injected with the tumor cells reject the tumor and survives
c. When an allogenic mouse is injected with the tumor cells accepts the tumor dies
d. When a syngeneic mouse is injected with the tumor cells accepts the tumor and dies

57. Polyomavirus (a DNA virus) causes tumors in “nude mice do not have a thymus, because of a
genetic defect) but not in normal mice. The BEST interpretation is that
a. Natural killer cells can reject polyomavirus induced tumors without help from T lymphocytes
b. T lymphocytes play an important role in rejection of poliomavirus-induced tumors
c. Macrophages are required to reject polyomavirus induced tumors
d. B lymphocytes play no role in rejection of poliomavirus induced tumors

58. Mechanism of antibacterial immunity include all of the following EXCEPT


a. Antibody and complement mediated bacteriolysis
b. Antibody neutralization of toxins
c. Antibody and complement mediated opsonization for enhanced phagocytosis
d. Cytotoxic T cell mediated bacteriolysis

59. If you measure the ability of cytotoxic T cells from an HLA B27 person to kill virus X-infected
target cells, which one of the following statements is CORRECT?
a. Any virus X-infected target cells will be killed
b. Any HLA-B27 cell will be killed
c. Only virus X-infected cell of HLA-B27 type will be killed
d. No HLA B27 cell will be killed

60. Patients with Wiskott-Aldrich syndrome have:


a. Low platelet level in blood
b. Selective IgA deficiency
c. An increased number of suppressed T cells
d. An increased number of B cells

61. The maximal reaction time for tuberculin type hypersensitivity is


a. 12-20 hours
b. 48-72 hours

nolan x soju
c. 2-5 hours
d. At least 4 days

62. Which of the following statements is (are) characteristic of contact sensitivity?


a. Sensitization can be passively transferred with serum from an allergic individual
b. Patch testing with the allergens is useless for diagnosis
c. Some chemicals acting as haptens induce sensitivity by covalently binding to host proteins
acting as carriers
d. The best therapy is administration of the antigen

63. Patients with Chediak-higashi syndrome have phagocytes that


a. Lack C3B receptors that are needed from attachment to bacteria
b. Show a reduced ability of their lysosomes to fuse with phagosomes to release microbicidsal
substances
c. Shows defects for the engulfment of microbes
d. Produce proteins that interfere with chemotaxis
64. Opsonin-treated bacteria are more readily engulfed by phagocytes than untreated bacteria
because
a. The capsule is removed by opsonin
b. Opsonin induce lysosomal enzymes
c. Opsonin digest the wall component
d. The surface of a phagocyte contains receptor for the Fc portion of an antibody

65. Certain HLA genes are linked to diseases such as ankylosing spondylitis. This linkage has all the
following characteristics except:
a. It carries no increased risk for a specific disease for those individuals with the gene
b. It may be the result of a MHC molecule serving as an attachment site for an infectious agent
c. It may be the result of cross linked genes
d. It may be the result of cross-reactivity between self-antigen and infection agent

66. The first time of defense against virus is


a. IgM directed to an external viral antigen
b. Antibodies directed against external viral antigens
c. IgG directed to an internal viral antigen
d. Interferon

nolan x soju
67. Grossly elevated serum levels of IgE can be found in person with
a. Parasitic worm infestations
b. Brucellosis
c. Arthus hypersensitivity
d. Tuberculosis

68. T-cell mediated immune responses can result in


a. Formation of granulomas
b. Induration at the reaction site
c. Rejection of a kidney transplant
d. All of the above

69. An IgA antibody to a red cell antigen is unlikely to cause autoimmune hemolytic anemia because
a. It requires secretory component to work
b. It would be made only in the gastrointestinal tract
c. Its Fc region would not bind receptors for Fc on phagocytic cells
d. It has too low affinity
70. The following mechanism may be involved in the clinical efficacy of injection therapy
(hyposensitization)
a. Enhanced production of CD8 cells
b. Enhanced production of IgG which binds allergen before it reaches mast cells
c. Induces large amounts of endogenous anti histamines
d. Directly affects stability of membranes

71. The Arthus reaction involves


a. Only 1 and 3 are correct
b. Cross-linking of IgE antibody
c. Formation of antigen antibody precipitates on vessel walls
d. Lymphocytic infiltration around veins

72. Immediate hypersensitivity skin reactions


a. Can be elicited by monovalent haptens
b. All are correct
c. Usually occur within 15 minutes
d. Exhibit a red flare due to vasodilation

73. Although IgE antibodies can cause quite serious allergic reactions; this isotype of antibody does

nolan x soju
appear to be very important in providing immunity from which one of the following/
a. Bacteria that cause pneumonia (most of these bacteria have a capsule
b. The smallpox virus
c. Parasitic worms
d. Tuberculosis bacteria that infect macrophages (and resist destruction once inside the cell)

74. What is the role of class II MHC proteins on donor cells in graft rejection?
a. They induce IgE which mediate graft rejection
b. They are the receptors for interleukin-2 which is produced by macrophages when they
attack the donor cells
c. They are recognized by helper T cells, which then activate cytotoxic T cells to kill the donor
cells
d. They induce the production of blocking antibodies that protect the graft

75. Bone marrow transplantation in immunocompromised patients presents which major problem?
a. Potential lethal graft-versus-host disease
b. High risk of T cell leukemia
c. Inability to use a live donor
d. Delayed hypersensitivity
76. Most autoimmune diseases are caused by
a. A T-cell defect
b. A constellation of genetic and environmental events
c. A known infectious organism
d. A single genetic defect

77. Granulomatous hypersensitivity is correctly described by which of the following statements?


a. It results from the presence of persistent microorganisms within macrophages
b. It has not be encountered in achistosomiasis
c. It does not cause pathological effects in diseases associated with T cell-mediated immunity
d. It is not clinically important

78. Concerning a monoclonal antibody, all of the following statements are correct EXPECT that it
a. can be obtained by the hybridoma technique
b. has antibody combining sites that are identical
c. may belong to the IgM, IgA, IgE, IgD, or IgG class
d. has found no diagnostic or therapeutic use

79. destruction of tumor cells may be achieved by

nolan x soju
a. cytotoxic T lymphocytes
b. antibodies and complement
c. antibody-dependent cell cytotoxicity
d. all are correct

80. grafts between genetically identical individuals (i.e., identical twins)


a. are subject to hyperacute rejection
b. are subject to acute rejection
c. are not rejected, even without immunosuppression
d. are not rejected if a kidney is grafted, but skin grafts are rejected

81. A transplant of tissue between individuals of the same species, but with different genetic
background is called a(n):
a. Allograft
b. Isograft
c. Xenograft
d. Autograft
82. Your patient is a child who has no detectable T or B cells. This immunodeficiency is most
probably the result a defect bin
a. Stem cells originating in the bone marrow
b. The thymus
c. The bursal equivalent
d. T cell B cell interaction

83. Which one of the following is the BEST method of reducing the effect of graft-versus-host
disease in a bone marrow recipient?
a. Matching the complement components of donor and recipient
b. Removing mature T cells from the graft
c. Administering alpha interferon
d. Removing pre-B cells from the graft

84. Anaphylactic reactions


a. All of the above
b. May be followed by inflammatory sequelae hours later
c. Are the consequences of released pharmacologic agents
d. May involve components of mast-cell granule matrix

nolan x soju
85. Serum sickness occurs only
a. When soluble immune complexes are formed
b. When IgE antibody is produced
c. In the absence of neutrophils
d. Anti-basement membrane antibodies are present

86. Clinical consequence(s) of C3 deficiency is (are):


a. Increased susceptibility to viral infections
b. Increased susceptibility to bacterial infections
c. Increased incidence of tumors
d. Increased susceptibility to fungal infections

87. Patients with DiGeorge syndrome


a. Have increased number of suppressor T cells
b. Have selective IgA deficiency
c. Have abnormal antigen-presenting cells
d. Have a decreased number of helper T cells
88. The major mechanism of host resistance to tuberculosis is
a. Massive proliferation of polymorphonuclear leukocytes
b. Humoral antibodies
c. Delayed hypersensitivity
d. High level of calcium in serum

89. A patient is admitted with multiple bacterial infections and is found to have a complete absence
of C3. Which compliment-mediated function would remain intact in such a patient:
a. Opsonization of bacteria
b. Generation of anaphylatoxins
c. None of the above
d. Lysis of bacteria

90. You have a patient who makes autoantibodies against his own red blood cells, leading to
hemolysis. Which one of the following mechanisms is MOST likely to explain the hemolysis?
a. Interleukin-2 binds to its receptor on the red cells, which results in lysis of the red cells
b. Complement is activated and membrane attack complexes lyse the red cells
c. Perforins from the cytotoxic T cells lyse the red cells
d. Neutrophils release proteases that lyse the red cells

nolan x soju
91. Chemically induced tumors have tumor-associated transplantation antigens that
a. Are very strong antigens
b. Are always the same for a given carcinogen
c. Are different for two tumors of different histologic type even if induced by the same
carcinogen
d. Do not induce an immune response

92. Removal of immune complexes is correctly described by which of the following statements?
a. It occurs primarily in the river
b. It is independent of C3 for particular complexes
c. It is independent of size of the immune complexes
d. It has not been associated of the Kupffer’s cells

93. Which of the following clinical diseases is most likely to involve a reaction to a hapten in its
etiology
a. Hemolytic anemia after treatment with penicillin
b. Rheumatoid arthritis
c. Goodpasture’s syndrome
d. Arthus reaction
94. which of the following statements BEST explains the relationship between inflammation of the
heart (carditis) and infection with group A beta-hemolytic streptococci?
a. Streptococcal antigens induce antibodies cross-reactive with heart tissue
b. Streptococcal antigens bind to IgE on the surface of heart tissue and histamine is released
c. Streptococci are polyclonal activators of B cells
d. Streptococci are ingested by neutrophils that releases proteases that damage heart tissue

95. Which of the following statements correctly describes Arthus hypersensitivity?


a. It cannot be manifested in the absence of high levels of IgE
b. It produces cell death and extensive local destruction
c. It occurs 2-4 days after antigen challenge
d. It is not mediated by complement-fixing IgG

96. Complement that has been activated by pathogenic bacterial cells can contribute (directly or
indirectly) to the destruction or elimination of those pathogenic bacterial cells in which of the
following ways?
a. All are correct
b. By destroying the bacterial cell membrane
c. By assisting phagocytes to attach to bacterial cells
d. By attracting more phagocytes to the area of complement activation

nolan x soju
97. Immune complexes are involved in the pathogenesis of
a. All of the above
b. Pigeon breeder’s disease
c. Serum sickness
d. Post-streptococcal glomerulonephritis

98. Serum sickness is characterized by


a. Consumption of complement
b. All of the above
c. Deposition of immune complexes in blood vessels
d. Phagocytosis of complexes by granulocytes

99. Which of the following does not involve CMI (cell mediated immunity)?
a. Rejection of liver graft
b. Serum sickness
c. Tuberculin reaction
d. Contact sensitivity to lipstick
100. The final damage to vessels in immune complex-mediated arthritis is due to
a. Cytotoxic T cell
b. NK cell
c. Lysosomal enzymes of polymorphonuclear leukocytes
d. histamine

nolan x soju
1. In which of the following ways is the adaptive immune system able to
prevent or limit the spread of a virus infection?
- Complement activation that would then destroy bacterial cell membrane
- Destruction of newly-infected body cells by T-cytotoxic lymphocytes
before large numbers of viruses could be replicated by the infected cell
- Compliment activation that would then destroy the virus before it would
be capable of causing an infection
- Destruction of newly-infected body cells by T-helper lymphocytes before
large numbers of viruses could be replicated by the infected cell

2. When activated complement can have a wide variety of action that


contribute directly or indirectly to providing protection from infectious
microorganism? Which of the following is not result from complement
activation?
- Preventing the attachment of exotoxins to body cells
- Destruction of bacterial cell membranes
- Assisting phagocytes (e.g., macrophages) to attach to bacterial cells that
lack the common carbohydrates found on most bacterial cells
- Attracting other phagocytes to the location where the complement is

nolan x soju
being activated (in response to the presence of complement-activating
bacteria)

3. Antibody-dependent cellular cytotoxicity (ADCC) requires the


involvement of which of the following?
- Antibodies of the IgM isotype that are specific for non-self
antigens/epitopes on the surface of cancer cells
- Antibodies of the IgG isotype that are specific for non-self
antigen/epitopes on the surface of cancer cells
- Central I-reg cells
- T-cytotoxic lymphocytes

4. Which one of the following antibodies has two functions: neutralization


and opsonization?
- IgM
- IgG
- IgA
- IgE

5. The four polypeptide chains of one antibody molecule of the IgG isotype
are held together with which one of the following types of bonds so that
four chains act as a single antibody “unit”?
- Non-covalent hydrogen bonds
- Non-covalent disulfide bonds
- Covalent disulfide bond
- Peptide bonds (that are susceptible to degradation by the enzyme,
pepsin)

6. The random selection of gene fragments (e.g., V, J and V, D, J) during


somatic recombination determine ….. the specificity of a B-lymphocyte
for an epitope. This process (somatic recombination) occurs at which of

nolan x soju
the following times?
- After the B-cell has encountered the epitope for which it will become
specific for the first time
- After the B-cell has “improved” its epitope-specific receptor by somatic
hypermutation
- During the secondary antibody response at the same time as any isotope
switching is occurring
- During the initial development of the B-cell from a bone marrow stem
cell and before the developing B-cell ever encounter any
antigen/epitope

7. Epitopes that can be successfully presented by the form of MHC that


consists of one large chain that forms the peptide groove and a
stabilizing molecule (beta-2 microglobulin) that does not contribute to
the groove would have one of the following characteristics?
- They would be appropriately 15-25 amino acids in size
- They would be generated from the degradation of a larger complex
protein within a membrane-bound vesicle in the cell that would
eventually present the peptide
- They would be generated from the degradation of protein from
proteosome
- They would have at least 12 amino acid positions known as “anchor”
amino acids

8. The development of bone marrow stem cells into either B-lymphocytes


is often initiated or controlled by interaction between the stem cell and
cells that are permanent residents of either the bone marrow or the
thymus. Which of the following cell types are involved in initiating or
controlling B-cell development in the bone marrow?
- Stromal cells that express cell adhesion molecules
- Epithelial cells in the thymus that express CD4 or CD8
- Epithelial cells in the bone marrow that express MHC Class I and MHC
Class II
- Cortical epithelial cells in the thymus that express MHC Class I and MHC
Class II

nolan x soju
9. Under which one of the following condition would a developing B-cell be
signaled to kill itself (apoptosis) because it is likely that B-cell has
developed specificity for a self-epitope?
- If the B-cell’s receptors formed high affinity bonds with epitopes after
the B-cell has left the bone marrow
- If the B-cell’s receptors formed high affinity bonds with epitopes before
the B-cell has left the bone marrow
- If the B-cell’s receptors formed low affinity bonds with epitopes without
help from T cells
- If the B-cell’s receptors formed low affinity bonds with epitopes on non-
soluble material (e.g., on bacterial cell surfaces) after the B-cell left the
bone marrow
10.A double-positive (immature) developing T-cell will become a T-helper
cell if which one of following interactions occurs while that T-cell is still
in the thymus?
- A CD8 on the T-cell binds with an MHC Class I on a thymic medullary
epithelial cell
- A CD4 on a thymic cortical epithelial cell binds with an MHC Class II on
the T-cell
- A CD4 in the T-cell binds with an MHC Class I on a thymic epithelial cell
(cortical or medullary)
- A CD4 on the T cell binds with an MHC Class II on a thymic cortical
epithelial cell

11.Once a native T-cell has been primed, it is signaled to temporarily


express which one of the following surface molecules that bind to co-
stimulatory molecules, with this binding sending an inhibitory signal to
the T-cell?
- B-7.1 or B-7.2

nolan x soju
12.Which of the following is important difference between innate immune
responses and adaptive immune response?
- Innate response (e.g., phagocytosis, complement activation) improves
significantly with each exposure to pathogens
- Innate responses often can take 1-2 weeks to begin destroying infections
microorganisms, while adaptive responses destroy infections almost
immediately after the very first encounter with a pathogen
- An innate response improves significantly with repeated exposures to
the same non-self antigen
- Each cell involved in innate responses (e.g., phagocytes) can be
stimulated to respond by many different molecules, while each cell
involved in adaptive responses (e.g., lymphocytes) can be stimulated to
respond by only one molecular structure (called an antigen)

13.Which one of the following set of condition must be met to stimulate a


T-helper 2 (TH2) cell to provide “help” to the cell that needs that help?
- Macrophage presenting a carbohydrate antigen using MHC Class II
- B-lymphocytes presenting a peptide antigen using MHC Class I
- B-lymphocytes presenting a peptide antigen using MHC Class II
- Macrophage presenting a peptide using MHC Class II

14.Under which one of the following conditions would a phagocyte be


destroyed by T-cytotoxic cell?
- if the macrophage was presenting peptides originally located in a
membrane-bound vesicle within the macrophage using one of the MHC
Class presenting molecules
- if the macrophage was presenting peptides originally located in the
macrophage cell cytoplasm using MHC Class I presenting molecules
- if the macrophage was presenting peptide using MHC Class II presenting
molecules
- if the macrophage was presenting peptides originally located in the
macrophage cell cytoplasm using MHC Class II presenting molecules

15.which one of the following types of cells is NOT capable of phagocytosis?


- Dendritic cell

nolan x soju
- T-helper cell
- Macrophage
- Neutrophil

16.In the absence of antibodies, phagocytes are usually not able to


effectively respond in any way against which of the following?
- Bacteria, which is inside cell
- Exotoxins produced and secreted by bacteria
- Bacterial cells to which the phagocytes firmly attach
- Bacteria that are present in the body but outside of the blood vessels

17.The much more rapid appearance of antibodies during a secondary


response compared to the primary response is due mainly to which on
the following?
- A much improved presentation of peptide antigen to T-helper I cells by
B-lymphocytes during the secondary response
- A much higher number of B-lymphocytes at the beginning of the
secondary response specific for the antigen that stimulated both the
primary and secondary antibody response
- Improved binding of antibody molecules to Fc receptors on the surface
of phagocytes, which leads to a higher rate of peptides presentation to
B-lymphocytes
- Each B-lymphocyte participating in a secondary antibody response is
specific for a very large number of different antigens, while each B-cell
participate in the primary response is specific for only one antigen

18.Circulating lymphocytes will be found in which one of the following


locations just before they enter the blood circulation from the lymphatic
system?
- A high endothelial venule
- A lymph node
- The thoracic duct

19.HLA-DP, HLA-DQ and HLA-DR are generally found only in which on of the

nolan x soju
following types of cells in the human?
- Only those cells that present epitopes to T-cytotoxic cells
- Only those cells that present epitopes to T-helper cells
- Virtually all cells (except red blood cells)
- Only those cells that are often involved in autoimmune diseases

20.A macrophage that is stimulated to initiate phagocytosis of a bacterial


cell that has a capsule needs to also involve an antibody specific for an
antigen on the capsule surface that also attaches to which one of the
following on the surface of the macrophage?
- An antigen on the macrophage surface that is identical to the antigen on
the capsule surface for which the antibody is specific
- MHC Class II molecules
- A receptor for a fragment of activated complement (the complement
has been activated by the binding of the antibody to the capsule
antigen)
- An Fc receptor
21.Which on of the following interactions between surface molecules must
take place for a T-cytotoxic cell to be stimulated to begin to destroy a
virus-infected cell?
- MHC Class I on virus-infected cell and CD4 on T-cytotoxic cell
- MHC Class I on T-cytotoxic cell and CD8 on virus-infected cell
- MHC Class II on virus-infected cell and CD4 on T-cytotoxic cell
- MHC Class I on virus-infected cell and CD8 on T-cytotoxic cell
- MHC Class II , CD4+ , T-helper
- MHCClass I , CD8+ , T-cytotoxic

22.The original specificity of any one B-lymphocyte is determined by which


on of the following?
- The type of CD molecules on that B-cell’s surface
- The specificity of the antigen receptors on that B-cell’s surface
- Somatic hypermutation
- The amino acid composition of two short tails extending from the tail

nolan x soju
end of the constant portion of the B-cell’s antigen receptor that are
embedded into the membrane surface of that B-cell

23.The inflammatory response is one manifestation of the innate immune


system. The response is initiated by the presence of bacterial cells in the
extravascular tissues (i. e., outside of the blood vessels). Phagocytes
(e.g., macrophages) that encounter these bacterial cells are stimulated
to release chemicals (cytokines and chemokines) that can have a
multitude of effects on nearby blood vessels and the contents of those
blood vessels. Which of the following will occur as a result of the release
of these cytokines and chemokines by the phagocytes?
- T-lymphocytes will be stimulated to secrete antibodies
- Some of the phagocytes will be stimulated to also secrete antibodies
specific for antigens found on the surface of the bacterial cells
- Some phagocytic cells in the nearby blood stream will actually be
attracted to the site of the infection, squeeze between the cells that
form the blood vessels and enter the tissue to assist in fighting the
invading bacteria present here
- T-lymphocytes that are in blood circulation will leave the blood
circulation right at the site of the bacterial infection, immediately enter
the lymphatic system and begin to produce antibodies specific for
antigens associated with the infecting bacteria

24.Antibodies that are able to provide protection from the effects of


bacterial exotoxins do so in which one of the following ways?
- Activate complement that when activated breaks down the toxin
molecules into harmless fragments
- Enable macrophage to present the toxin antigen to T-lymphocytes
- Bind to the toxin molecules and prevent the toxins from binding to body
cell surface
- Cause blood vessels located near the site of the toxin to become slightly
leaky thus allowing cells in the blood stream to enter the tissues and
destroy the toxin molecules

nolan x soju
25. Somatic recombination is a process that results in the production of
literally hundreds of millions of individually specific lymphocytes. This
process is best described by which one of the following?
- The amount of genetic material available to use for generating specificity
is approximately 2/3 of the entire genome
- The specificity of the variable portion of each lymphocyte’s surface
antigen receptor is determined by a random selection of 2 or 3 short
gene fragments from a small amount of genetic material that is found in
every nucleated cell in the body
- The specificity of the constant portion of each lymphocyte’s surface
antigen receptor is determined by a random selection of 2 or 3 short
gene fragments from a small amount of genetic material that is found in
every nucleated cell in the body
- A very high rate pf mutation takes place within the short segment of
nucleotides that is used by every lymphocytes to generate the specificity
of the surface antigen receptor
26.Which one of the following types has the MOST influence on the
antibody response carried out by a B-lymphocytes?
- T-cytotoxic cell
- Cytokines released by a T-helper2 cell
- Cytokines released by a T-helper1 cell
- Phagocytes that are presenting peptides to T-helper cells

27.The biological properties of an antibody molecule not directly associated


with specificity involve the participation of amino acids in which one of
the following locations? Hint: this location also determines the isotype
of the antibody
- Fab
- The constant portion of the light chains
- The constant portions of the heavy chains
- The domain that is at the amino-terminal end of the heavy chain

28.Which one of the following isotypes of antibodies is responsible for


initiating the release of amounts of histamine that results in the

nolan x soju
symptoms most often associated with allergic reactions?
- IgM
- IgD
- IgE

29.MHC Class II molecules are also assembled in the ER but are prevented
from binding any peptides that are also in the ER which would interfere
with the normal binding of the Class II to epitopes later on. The molecule
that temporarily occupies the groove in the class II molecule while it is
the ER is which one of the following?
- CLIP
- HLA-DM
- Invariant chain
- Proteasomal peptides of lengths greater than15 amino acids
30.Double-positive developing T-cells will convert to T-helper cells under
which of the following sets of conditions?
- A CD4 on the double-positive T-cell binds to an MHC Class II on an
epithelial cell in the thymus
- The epitope-specific receptor on the double-positive T-cell forms a high
affinity bond with a peptide being presented by a macrophage in the
medullary area of the thymus

31.The name given to the amino acids that actually form bonds between an
antibody molecule …. antigen for which antibody molecule is very
strongly specific called the
- The variable amino acid

32. Damage to heart valve tissue that involves the immune system
following a prolonged strep through infection is due to which one of the
following?
- Antibodies produced in response to an epitope on the bacterium that is
causing the strep throat sym…. that cross react with a self-epitope on

nolan x soju
the heart valve tissue
1. Neutralization of viruses involves
- Superantigen
- Chemokines
- MHC Class I plus peptides
- Specific antibodies

2. Rapid but non-antigen specific immune response are produce…..


- Adaptive immune system
- Innate immune system
- Lymphocytes
- Lymphatic system

3. Cytokines are
- Not able to increase blood vessel permeability
- Not able to induce antigen-specific response
- Able to induce antigen-specific response
- Able to activate complement system

4. Pathological consequence of immune responses ….


- Elimination of immune complexes
- Endotoxic shock
- Artificial passive immunity

nolan x soju
- Natural active immunity

5. Inflammatory cytokines produced by macrophages activate


- Complement system via classical pathway
- Integrin on leukocyte to bind more strongly to vasc…
- Neutrophils to be more cytotoxic
- NK cells to kill virus-infected cells

6. All of the following are true or antigen receptors on both B….


- They are associated with signal transduction molecule
- They are members of the Ig gene superfamily
- They recognize antigen differently
- They are specific for a single antigen epitope

7. Endogenous antigen presentation requires delivery of antigen peptides to the endopla…..


- After degradation inside proteosome
- By HLA-DM
- After degradation inside phagosome
- By phagosome
8. Body effector functions include all of the following….
- Activation of complement on the bacterial ….
- Activation of proteolysis of endocytosed pro….
- Blocking of uptake of bacterial toxins
- Coating bacteria to promote their phagocytosis

9. The naive lymphocytes encounter the antigens for which they specifi….
- In the primary lymphoid organs
- In the thoracic duct
- In the secondary lymphoid organs
- In the blood

10. An inflammatory response


- Is characterized by a decrease in vascular permeability
- Is stimulated by cytokines produced by neutrophils
- Occurs only during a secondary response
- Attracts inflammatory cells to the infection site

11. Opsonin-treated bacteria are more readily engulfed by phagocytes than are unt….
- The capsule is removed by opsonin

nolan x soju
- Opsonin digest the wall component
- Opsonin induces lysosomal enzymes
- Phagocyte contains receptors that bind antibodies

12. A cell producing cytotoxic compounds following Th1 cell activation is a (an)
- Immature dendritic cell
- Macrophage
- Eosinophils
- Mature dendritic cell

13. Reaction to M.tuberculosis and eggs of schistosomes is


- An IgG-mediated response
- An IgE-mediated response
- A T cell mediated response
- Antibody mediated response

14. Which of the following statements can correctly describe type III hypersensitivity
- It occurs within 72 hours of re-exposure to antigen
- It is not mediated by complement fixing IgG
- Complement is an important mediator
- Desensitization is used for its control

15. Anaphylactic reactions


- Can result in induration at the site reaction
- Are the consequence of TH1 activation
- May involve components of mast-cell granule matrix
- Are the consequence of TH17 activation

16. Type IV hypersensitivity reactions result in


- Formation of granulomas
- Development of anaphylactic reaction
- Complement mediated lysis of infected cells
- Serum sickness

17. What is the major consequence of the bone marrow transplantation in ….


- Potentially lethal graft-versus-host disease
- High risk of T cell leukemia
- Inability to use a live donor
- Delayed hypersensitivity

nolan x soju
18. Chemically induced tumor have tumor-associated transplantation antigen
- Are always the same for a given carcinogen
- Are different for two tumors of different histological type even if in ….
- Are very strong antigens
- Do not induce an immune response

19. Failure to express class II HLA molecules on antigen-presenting cells cause


- Th cell deficiency
- Tc cell deficiency
- B cell deficiency
- NK cell deficiency

20. Type II hypersensitivity is


- Antibody independent
- Complement independent
- Caused by antibodies against soluble antigen
- Caused by antibodies against cell surface antigen..

21. The major mechanism of host resistance tp M.leprae is


- Humoral antibodies
- Type IV hypersensitivity
- High level of calcium in serum
- Type I hypersensitivity

22. Which antibodies are typically involved in both – Type II and Type III hypersensitivity…..
- IgA
- IgG
- IgD
- IgE

23. Autoimmunity results from


- Antigen driven self reactive lymphocy….
- Accerating clearence of apoptotic ce….
- Elimination of extracellular pathogens
- Activation of Treg cells

24. Isotype switching


- Decreases the functional diversity of Ig molecules
- Improves the antigen binding specificity of an Ig molecule
- Increases the affinity of antibodies in a process called affin….
- Increases the functional diversity of Ig molecules

nolan x soju
25. A diagnostic marker for tumors of the colon is
- Alpha fetoprotein
- Common acute lymphoblastic leukemia antigen (CALLA)
- EBV related antigen
- Carcinoembryonic antigen (CEA)

26. Product of vaccination is


- Naïve mature B cells
- Memory mature T cells
- Naïve mature T cells
- Plasma cells

27. The alternative pathway of complement activation


- Causes tissue damage in the absence of C1 inh
- Occurs only after the classic pathway is activated
- Occurs only after the lectin pathway is activated
- Requires bacterial surface
28. The cytokine secreted by TH1 cells which promotes the phagocytosis
- Interferon-gamma
- Interleukin-8
- Interleukin-2
- Interleukin-5

29. The region of the antibody molecules with the highest affinity to antigen are
- Hypervariable regions
- Fc regions
- Framework regions
- Hinge regions

30. Lymphocyte recirculation


- Activates inflammatory cytokines to promote antigen….
- Allows lymphocytes to encounter an appropriate antigen…..
- Circulates lymphokines efficiently throughout the body
- Circulates chemokines efficiently throughout the body

31. Immune system cell adhesion molecules


- Do not allow macrophages to leave the circulation
- Do not allow T cells to home to peripheral lymphoid tissue
- Help activated B cells to produce high affinity antibodies

nolan x soju
- Help cytotoxic T cells to bind specifically to their targets

32. Antibody-dependent cellular cytotoxicity (ADCC) requires the …….


- Dendritic cell and infected cell
- Macrophages and cancer cell
- Erythrocyte and antibodies
- Antibodies and follicular dendritic cell

33. T cells are MHC-restricted in their ability to respond to antigen because……


- During an infection all cells in the body present antigen on MHC….
- MHC binds antigen more specifically than TCR does
- TCR must recognize both antigen and MHC molecules
- The T cells should not respond to antigen on allogenic cells

34. Antibody effector functions include all of the following EXCEPT


- Activation of complement on the bacterial surface
- Activation of proteolysis of endocytosed proteins
- Blocking uptake of bacterial toxins
- Coating bacteria to promote then phagocytosis
35. Isotype switching resembles somatic recombination because both processes
- Are catalysed by the products of RAG1 and RAG2
- Are regulated by helper T cell cytokines
- Occurs in developing B cells in the bone marrow
- Results in loss of DNA from the B cell

36. Just before they enter the blood circulation from the lymphatic system circulating
lymphocytes will be found in
- A high endothelial venule
- A lymph node
- The thoracic duct
- The bone marrow

37. Activation of mast cell by anaphylatoxins (C3a and C5a) causes


- Apoptosis of mast cell
- Presentation of the antigen to Th cells
- Secretion of IgE antibodies
- Secretion of allergic mediators

38. Which Ig isotype is the most important for neutralizing pathogens caused intestinal

nolan x soju
infections before they infect intestinal cells
- Secretory IgA
- Serum IgA
- Serum IgM
- Membrane IgM

39. What is the role of class II MHC proteins on the donor cells in graft rejection?
- They are the receptors for interleukin-2 which is produced by macrophages when they
attack the donor cells
- They are recognized by helper T cells which then activates cytotoxic T cells to kill the donor
cells
- They induce the production of blocking antibodies that protect the graft
- They induce IgE which mediate graft rejection

40. Which cells are involved in DTH


- Th1 and macrophages
- NKT cells and mast cells
- NK cells and B cells
- B cells and eosinophils
Antibody effector functions include all of the following EXCEPT
A) Activation of complement on the bacterial surface
B) Activation of proteolysis of endocytosed proteins
C) Blocking uptake of bacterial toxins
D) Coating bacteria to promote their phagocytosis
What is the source of all undifferentiated cells that can produce any of the cells involved in
immune responses?

A) The spleen
B) The bone marrow
C) Any lymph node
D) The thoracic duct

Vaccines stimulate
A) Natural passive immunity
B) Artificial active immunity
C) Artificial passive immunity
D) Natural active immunity

Somatic recombination occurs


A) After affinity maturation

nolan x soju
B) In the progenitor cell as it is becoming a B cell
C) After isotype switching
D) In the plasma cell after antibody secretion
Which of the following human cell types are NOT capable of presenting peptides using MHC
Class I
A) Fully mature lymphocytes
B) Dendritic cells
C) Red blood cells
D) Macrophages
TCR most closely resembles

A) Class I MHC
B) Class II MHC
C) Fab region of immunoglobulin
D) Fc region of immunoglobulin
The antigen specificity of an adaptive response is due to
A) Activation of antigen specific lymphocytes
B) Activation of macrophages
C) Lysis of only certain pathogens by neutrophils
D) Phagocytosis of pathogens by macrophages
For the generation of TCR diversity T cells use
A) Isotype switching
B) Affinity maturation
C) N region addition of nucleotides
D) Somatic hypermutation
Engagement of the Fas receptor by Fas ligand (FasL) results in

A) Apoptotic cell death


B) Clonal proliferation
C) Cytotoxicity
D) Somatic hypermutation
Identify the cytokine that promotes the development of the Th2 response

A) Interleukin 17
B) Interleukin 4

nolan x soju
C) Interleukin 12
D) Interleukin 5

Infantile X linked agammaglobulinemia is associated with excessive infections of the


following types
A) Intracellular bacterial
B) Capsulated bacteria
C) Viral
D) Fungal
Antibody Fc fragments
A) Contain antigen binding sites
B) Contain complementarity determining regions
C) Bind to complement components
D) Binds to Toll like receptors
Isotype switching resembles somatic recombination because both processes
A) Are catalyzed by the products of RAG1 and RAG2
B) Are regulated by helper T cell cytokines
C) Occur in developing B cells in bone marrow
D) Result in loss of DNA from the B cell
What immune function is likely to be affected in case of mutation in Fc gamma receptor gene
A) Opsonization
B) Lymphocyte adhesion
C) Antigen binding
D) Intracellular killing
Activation of mast cell by anaphylatoxins (C3a and C5a) causes

A) Apoptosis of mast cell


B) Presentation of the antigen to Th cells
C) Secretion of IgE antibodies
D) Secretion of allergic mediators
Priming of naïve T-helper cells by mature dendritic cells in the lymph node involves

A) CD40 on DC with CD40L in T-cell


B) CTLA-4 on T-cell with peptide presented by DC

nolan x soju
C) CD56 on naïve T-cell with CD28 on DC
D) CD8 on naïve T-cell with MHC class I on DC

Human Class II MHC molecules


A) Are encoded by the genes HLA-A,B and C
B) Are found on all nucleated cells
C) Are encoded by the genes HLA-DP, DR and DQ
D) Present antigen to CD8 cytotoxic T cells
Identify an important cytokine receptor interaction which initiates extensive cell division of
the primed T-cell
A) IL-2 and a high affinity IL-2 receptor
B) IFN-gamma and the IFN-gamma receptor
C) IL-7 and a high affinity IL-7 receptor
D) IL-7 and low affinity IL-7 receptor
A cell which secretes large quantities of antibodies but does not express surface
immunoglobulin is a
A) Naïve mature B cell
B) Memory B cell
C) B-1 cell
D) Plasma Cell
TCR most closely resembles

A) Class I MHC
B) Class II MHC
C) Fab region of immunoglobulin
D) Fc region of immunoglobulin

It was observed that milkmaids, infected with cowpox, were later immune to smallpox
infections. This is an example of a(n)
A) Innate immunity to cross reactive antigen
B) Innate immunity of milkmaids to smallpox
C) Memory response to a cross reactive antigen
D) Passive immunization from contact with cow’s milk antibodies
Th17 cytokine that promotes killing of extracellular pathogen is a

nolan x soju
A) Interferon-gamma
B) Interleukin 17
C) Interleukin 10
D) Interleukin 4
Signal transduction is the process of converting
A) A B cell to a T cell
B) A binding signal to a chemical signal
C) A hapten to an antigen
D) IgA to secretory IgA
Just before they enter the blood circulation from the lymphatic system. Circulating
lymphocytes will be found in
A) A high endothelial venule
B) A lymph node
C) The thoracic duct
D) The bone marrow
Natural killer cells
A) Are inhibited to kill infected host cells by MHC class I molecules
B) Are activated to kill infected host cells by MHC class I molecules
C) Kill virus infected cells when the virus is acquired naturally but not by immunization
D) Secrete the complement MAC to lyse virus infected cells
Besides the direct contact between the B-lymphocyte and the antigen, antibody production
by B-lymphocytes require

A) Help from a T-helper cell


B) Help from a macrophage
C) Help from a NKT cell
D) Help from a T-cytotoxic cell

Genes for immunoglobulins are unlike other human genes in that


A) Antibody genes are composed of introns and exons
B) DNA for antibody molecules is inherited from only one parent
C) Gene segments must be spliced together to make each unique antibody molecule
D) Several introns encode each antibody molecule

Which cells in the body are involved in the antibody mediated allergic reactions
A) Macrophage and NK cell

nolan x soju
B) Mast cells and basophile
C) Gamma/delta T cells
D) Any phagocytic cell
Inflammatory cytokines produced by macrophages activate all of the following EXCEPT
A) Complement system via classical pathway
B) Integrin on leukocytes to bind more strongly to vascular CAMS’s
C) Neutrophils to be more cytotoxic
D) NK cells to kill virus infected cells
The IgG isotype is often used as an example of the typical immunoglobulin. Identify the
physical or biological characteristics of one antibody molecule that has the IgG isotype

A) Has one heavy chain and two identical light chains


B) Is able to bind two different antigens/epitopes
C) Papain treatment will produce two identical Fab fragments and two identical Fc
fragments
D) Is able to bind to specific antigen
Which of the following is a common cause of hypersensitivity diseases
A) Failure of lymphocyte maturation
B) Failure of self-tolerances
C) Treatment with corticosteroids
D) Disseminated cancer
Bacteria growing within macrophages are killed by
A) Reactive oxygen and nitrogen intermediates
B) Complement components
C) Cytotoxic T cells
D) Antibody dependent cell cytotoxicity
Which of the following is the neutralizing antibody

A) IgM
B) IgG
C) IgD
D) IgE
The most important chemokine receptors that act as coreceptors for HIV are

A) CXCR5 and CCR6


B) CXCR4 and CCR5

nolan x soju
C) CXCR7 and CCR5
D) CXCR6 and CCR4

A common strategy by which microbes survive their host’s immune responses involve
changing the structure of molecules they produce so that they are no longer recognized by
the host’s immune system. This strategy called antigenic variation is most likely to allow
evasion of which type of immune recognition

A) Toll like receptors Dependent recognition


B) Mannose receptor dependent recognition
C) Antibody recognition of microbial cell surface molecules
D) Natural killer cell inhibitory receptor recognition of Class I major histocompatibility
complex (MHC) molecules on infected cells
Th2 cells are responsible for fighting against
A) Tuberculosis bacteria
B) Flu virus
C) Tumor cells
D) Worms
Certain HLA genes are linked to autoimmune disease because
A) It may be the result of a single genetic defect
B) It may be the result of the decreased number of helper T cells
C) It may be the result of cross reactivity between self-antigen and infection agent
D) It carries no increased risk for a specific disease for those individuals with the gene
Lupus erythematosis is
A) Free from an immune complex disease component
B) Associated with antibodies against nucleic acid
C) A disease of red and white cells
D) An organ specific autoimmune disease
The lesions in immune complex induced glomerulonephritis are dependent on

A) Erythrocytes and complement


B) The production of urine
C) Complement and neutrophils
D) The presence of macrophages
Granulomatous hypersensitivity is correctly described by which of the following statements

A) Granuloma contains mast cells and neutrophiles


B) IgG has a major role in granuloma formation

nolan x soju
C) IFN-gamma is required granuloma formation
D) IL-4 is required for granuloma formation

A common strategy by which microbes survive their hosts immune responses involves
changing the structure of the molecules they produce so that they are no longer recognized
by the host’s immune system. This strategy called antigenic variation is most likely to allow
evasion of which type of immune recognition?

A) Toll like receptor dependent recognition


B) Mannose receptor dependent recognition
C) Antibody recognition of microbial cell surface molecule
D) Natural killer cell inhibitory receptor recognition of class I major histocompatibility
Complex (MHC) molecules on infected cells
A patient is found to have a complete absence of Cl inhibitor. The most likely consequence of
this deficiency is
A) Hereditary angioedema
B) Immune complex disease
C) Granuloma formation
D) Deposition of immune complexes
IgE production depends on
A) Th1 cells
B) Th2 cells
C) Th17 cells
D) NK cells
Which of the following is a common cause of hypersensitivity diseases
A) Failure of lymphocytes maturation
B) Failure of self-tolerances
C) Treatment with corticosteroids
D) Disseminated cancer
The lesions in immune complex induced glomerulonephritis are dependent on

A) Erythrocytes and complement


B) The production of urine
C) Complement and neutrophils
D) The presence of macrophages
Di George syndrome results from a defect in

A) Purine nucleoside phosphorylase


B) WASP

nolan x soju
C) Thymic development
D) DNA repair

Certain HLA genes are linked to autoimmune disease because


A) It may be the result of a single genetic defect
B) It may be the result of the decreased number of helper T cells
C) It may be the result of cross-reactivity between self-antigen and infection agent
D) It carries no increased risk for a specific disease for those individuals with the gene
Infantile X linked agammaglobulinemia is associated with excessive infections of the
following type
A) Intracellular bacterial
B) Capsulated bacteria
C) Viral
D) Fungal
Bacteria growing within macrophages are killed by
A) Reactive oxygen and nitrogen intermediates
B) Complement components
C) Cytotoxic T cells
D) Antibody dependent cell cytotoxicity
Granulomatous hypersensitivity is correctly described by which of the following statements?
A) Granuloma contain mast cells and neutrophiles
B) IgG has a major role in granuloma formation
C) IFN-gamma is required for granuloma formation
D) IL-4 is required for granuloma formation
Th2 cells are responsible for fighting against

A) Tuberculosis bacteria
B) Flu virus
C) Tumor cells
D) Worms
Isotype switching

A) Decreases the functional diversity of Ig molecules


B) Improves the antigen binding specificity Ig molecule

nolan x soju
C) Increases the affinity of antibodies in a process called affinity maturation
D) Increases the functional diversity of Ig molecules

Identify an important cytokine receptor interaction which initiates extensive cell division of
the primed T cell
A) IL-2 and a high affinity IL-2 receptor
B) IFN-gamma and the IFN-gamma receptor
C) IL-7 and a high affinity IL-7 receptor
D) IL-7 and a low affinity IL-7 receptor
The inflammation that results from a mosquito bite is different from the inflammation that
results from a positive TB skin test in that the reaction to the mosquito bite

A) Does not require prior sensitization


B) Takes more time to appear
C) Results primarily from antibody antigen interactions
D) Involves basophils, mast cells and Tc cells
A Tc cell can mount a cytotoxic attack on a viral infected cell if it binds to a cell displaying
A) Insufficient MHCI
B) Insufficient MHCII
C) MHC I bearing foreign antigen
D) MHC II bearing foreign antigen
Which of the following human cell types NOT capable of presenting peptides using Class I
A) Fully mature lymphocytes
B) Dendritic cells
C) Red blood cells
D) Macrophages
Autoimmunity results from

A) Antigen driven self-reactive lymphocytes


B) Accelerating clearance of apoptotic cells
C) Elimination of extracellular pathogens
D) Activation of Treg cells
Respiratory diseases and unconsciousness developed within minutes after penicillin injection
is probably mediated by
A) IgE antibody

nolan x soju
B) IgG antibody
C) Sensitized T cells
D) IgM antibody
Complement activated by pathogenic bacteria can contribute to the destruction of this
bacteria
A) By solubilization of immune complexes
B) By assisting phagocytes to attach to bacterial cells
C) By attracting more Treg cells to the site of infection
D) By deposition of immune complexes inside blood vessels
An example of innate immunity is

A) Antibody production by plasma cells


B) Pathogen recognition by dendritic cells
C) Destruction of infected cell by cytotoxic lymphocytes
D) Memory response to influenza virus
Secondary immune response compare to primary contains
A) Low level of specific IgG antibodies
B) High level of specific IgM antibodies
C) High level of specific IgG antibodies
D) High level of specific IgD antibodies
Which category of hypersensitivity BEST describes transfusion when a recipient has
antibodies against donor erythrocytes

A) Anaphylactic
B) Cytotoxic
C) Immune complex
D) Delayed

What is likely to be elevated in patient infected by Trichinella spiralis


A) Basophils
B) Neutrophils
C) Lymphocytes
D) Eosinophils

Antibody dependent cellular cytotoxicity (ADCC) requires the involvement of


A) Dendritic cell and infected cell

nolan x soju
B) Macrophage and cancer cell
C) Erythrocyte and antibodies
D) Antibodies and follicular dendritic cell
A CD3+ cell that secretes perforin granzyme is a
A) Gamma/delta T cell
B) Cytotoxic T cell
C) Helper T cell
D) Regulatory T cell
The ability of an antigen to induce an immune response depends on
A) The antigens ability to enter the thyroid
B) The antigens dose and size
C) Antigen processing mechanism
D) Antigen receptor gene rearrangement
Clonal selection
A) Begins with inflammation
B) Occurs for all leukocytes
C) Occurs in response to self antigens
D) Occurs for all lymphocytes
Phagocytes can NOT attach to the bacterial cell wall surface by which of the following ways
A) Directly attaching to the bacterial cell wall lipopolysaccharides
B) Attaching to molecules on the bacterial cell surface using MHC class II molecules on the
phagocyte
C) Using an antibody molecule that is specific for an antigen on the surface of the bacterial
cell
D) Using a fragment resulting from complement activation by the bacterial cells
Effects of bacterial toxin could best be counteracted by
A) Antibody binding and neutralization of the toxin
B) Antibody opsonization and phagocytosis of toxin producing bacteria
C) B cell binding to toxin producing bacteria
D) Cytotoxic T cell binding and lysis of toxin producing bacteria
Cytokines

nolan x soju
A) Are not able to increase blood vessel permeability
B) Not able to induce antigen specific response
C) Able to induce antigen specific response
D) Able to activate complement system
In case of C2 and C4 deficiencies
A) C5 can still be cleaved by the classical pathway
B) C3b will not be able to bind to bacteria
C) C9 will not polymerize and lyse host cells
D) C3b production will be reduced
A cell expressing CD3+, CD25+ and FoxP3+ is a

A) Gamma/deltaT cell
B) Cytotoxic T cell
C) Natural killer cell
D) Regulatory T cell
Which of the following is NOT a characteristic of IgG
A) It crosses the placenta
B) It neutralizes toxins
C) It is major antibody of saliva
D) It activates complement
Identify the part of the antibody molecule in which the amino acids form the most specific
bonds with an antigen

A) The framework
B) The variable
C) The hypervariable
D) The constant domain

Lymphocytes continually recirculate through peripheral lymphoid tissue in order to


A) Be killed before they cause autoimmunity
B) Efficiently encounter antigen
C) Mature from stem cells into lymphocytes
D) Phagocytose antigen and kill it

An antibody Fab fragment


A) Contains complementarity determining regions

nolan x soju
B) H and L chain constant regions
C) Different antigen binding regions
D) Binds to Fc receptors
Phagocytes that encounter bacterial cells in the extravascular tissues are stimulated to
release cytokines and chemokines, Choose the vent occurring as a result of the release of
these cytokines and chemokines by the phagocytes

A) The spaces between the cells that form the blood vessel do not increased
B) Some of the phagocytes are stimulated to secrete specific antibodies
C) Some phagocytic cells in the nearby blood stream are actually attracted to the site of
infection
D) T-lymphocyte leave the blood circulation at the site of infection to produce specific
antibodies
Peripheral lymphoid organs
A) Are located in the abdomen to protect their vital functions
B) Maximize contact between antigen and lymphocytes
C) Produce antigen specific lymphocytes from stem cells
D) Sequester antigen to minimize its damage to the body
All of the following are associated with secretion of proinflammatory cytokines except
A) Failure to make immune response following stimulation by antigen
B) Upregulation of adhesive molecules on the vascular endothelium
C) Migration of inflammatory cells into the damaged area
D) Increased permeability of vascular vessels

nolan x soju
1. What is the source of all undifferentiated stem cells that can produce any
of the cells involved in immune response?
- The spleen
- The bone marrow
- Any lymph node
- The thoracic duct

2. Somatic recombination occurs


- After affinity maturation
- In the progenitor cell as it is becoming a B cell
- After isotype switching
- In the plasma cell after antibody secretion

3. Antibody effector functions include all of the following EXCEPT


- Activation of complement on the bacterial surface
- Activation of proteolysis of endocytosed proteins
- Coating bacteria to promote their phagocytosis

nolan x soju
4. Vaccines stimulate
- Natural passive immunity
- Artificial active immunity
- Artificial passive immunity
- Natural active immunity

5. The antigen specificity of an adaptive immune response is due to


- Activation of antigen-specific lymphocytes
- Activation of macrophages
- Lysis of only certain pathogens by neutrophils
- Phagocytosis of pathogens by macrophages

6. Which of the following human cell types are NOT capable of presenting
peptides using MHC Class I?
- Fully mature lymphocytes
- Dendritic cells
- Red blood cells
- Macrophages

7. Isotype switching resembles somatic recombination because both


processes
- Are catalyzed by the products of RAG1 and RAG2
- Are regulated by helper T cell cytokines
- Occur in developing B cells in the bone marrow
- Results in loss of DNA from the B cell

8. Engagement of the Fas receptor by Fas ligand (FasL) results in


- Apoptotic cell death
- Clonal proliferation
- Cytotoxicity
- Somatic hypermutation

nolan x soju
9. For the generation of TCR diversity T cells use
- Isotype switching
- Affinity maturation
- N region addition of nucleotides
- Somatic hypermutation

10.Antibody Fc fragments
- Contain antigen binding sites
- Contain complementarity determining regions
- Bind to complement component
- Binds to Toll-like receptors

11.Activation of mast cell by anaphylatoxins (C3a and C5a) causes


- Apoptosis of mast cell
- Presentation of the antigen to Th cells
- Secretion of IgE antibodies
- Secretion of allergic mediators

12.TCR most closely resembles


- Class I MHC
- Class II MHC
- Fab region of immunoglobulin
- Fc region of immunoglobulin

13.Priming of native T helper cells by mature dendritic cells in the lymph


node involves
- CD40 on DC with CD40L on native T-cell
- CTLA-4 on T-cell with peptide presented by DC
- CD56 on native T-cell with CD28 on DC
- CD8 on native T-cell with MHC Class I on DC

14.Identify the cytokine that promotes the development of the Th2 response
- Interleukin 17

nolan x soju
- Interleukin 4
- Interleukin 12
- Interleukin 5

15.The IgG isotype is often used as an example of the “typical”


immunoglobulin. Identify the physical or biological characterisitics of one
antibody molecule that the IgG isotype?
- Has one heavy chain and two identical light chain
- Is able to bind two different antigens/epitopes
- Papain treatment will produce two identical Fab fragments and two
identical Fc fragments
- Is able to bind to specific antigen

16.Human class II MHC molecules


- Are encoded by the genes HLA, A, B, and C
- Are found on all nucleated cells
- Are encoded by the genes HLA-DP, DR, and DQ
- Present antigen to CD8 cytotoxic T cells

17.What immune function is likely to be affected in case of mutation in the Fc


gamma receptor gene?
- Opsonisation
- Lymphocyte adhesion
- Antigen binding
- Intracellular killing

18.Signal transduction is the process of converting


- a B cell to a T cell
- a binding signal to a chemical signal
- a hapten to an antigen
- IgA to secretory IgA

19.A cell which secretes large quantities of antibodies but does not express

nolan x soju
surface immunoglobulin is a
- Native mature B cell
- Memory B cell
- B-1 cell
- Plasma cell

20. Identify an important cytokine receptor interaction which initiates


extensive cell division of the ……..
- IL-2 and a high affinity IL-2 receptor
- IFN-gamma and the IFN-gamma receptor
- IL-7 and a high affinity IL-7 receptor
- IL-7 and low affinity IL-7 receptor

21.Natural killer cells


- Are inhibited to kill infected host cells by MHC class I molecules
- Are activated to kill infected host cells by MHC class I molecules
- Kill virus infected cells when the virus is acquired naturally but not by
immunization
- Secrete the complement MAC to lyse virus infected cells

22.Besides the direct contact between the B-lymphocyte and the antigen,
antibody production by B-lymphocytes requires
- Help from a T-helper cell
- Help from a macrophage
- Help from a NKT cell
- Help from a T-cytotoxic cell

23.Th17 cytokine that promote killing of extracellular pathogen is a


- Interferon-gamma
- Interleukin 17
- Interleukin 10
- Interleukin 4

nolan x soju
24.Just before they enter the blood circulation from the lymphatic system,
circulating lymphocytes will be found in
- A high endothelial venule
- A lymph node
- The thoracic duct
- The bone marrow

25.Inflammatory cytokines produced by macrophages activate all of the


following EXCEPT
- Complement system via classical pathway
- Integrin on leukocytes to bind more strongly to vascular CAMs
- Neutrophils to be more cytotoxic
- NK cells to kill virus-infected cells

26.Genes for immunoglobulin are unlike other human genes in that


- Antibody genes are composed of introns and exons
- DNA for antibody molecules is inherited from only one parent
- Gene segments must be spliced together to make each unique antibody
molecule
- Several introns encode each antibody molecule

27.It was observed that milkmaids, infected with cowpox, were later immune
to smallpox infections. This is an example of a(n)
- Innate immunity to a cross-reactive antigen
- Innate immunity of milkmaids to smallpox
- Memory response to a cross-reactive antigen
- Passive immunization from contact with cow’s milk antibodies

28.Which of the following is the neutralizing antibody?


- IgM
- IgG
- IgD
- IgI

nolan x soju
29.All of the following are associated with the secretion of proinflammatory
cytokines EXCEPT
- Failure to make immune response following stimulation by antigens
- Upregulation of adhesive molecules on the vascular endothelium
- Migration of inflammatory cells into the damaged area
- Increased permeability of vascular vessels

30.Isotype switching
- Decreases the functional diversity of Ig molecules
- Improves the antigen binding specificity of an Ig molecule
- Increases the affinity of antibodies in a process called affinity maturation
- Increases the functional diversity of Ig molecules

31.Phagocytes that encounter bacterial cells in the extravascular tissues are


stimulated to release cytokines and chemokines. Choose the event
occurring as a result of the release of these cytokines and chemokines by
the phagocytes?
- The spaces between the cells that form the blood vessel do not increased
- Some of the phagocytes are stimulated to secrete specific antibodies
- Some phagocytic cells in the nearby blood stream are actually attracted
to the site of the infection
- T-lymphocytes leave the blood circulation at the site of the infection to
produce specific antibodies

32.An antibody Fab fragment


- Contains complementarity determining regions
- H and L chain constant regions
- Different antigen binding regions
- Binds to Fc receptors

33.Peripheral lymphoid organs


- Are located in the abdomen to protect their vital functions
- Maximize contact between antigen and lymphocytes

nolan x soju
- Produce antigen specific lymphocytes from stem cells
- Sequester antigen to minimize its damage to the body

34.Clonal selection
- Begins with inflammation
- Occurs for all leukocytes
- Occurs in response to self antigens
- Occurs for all lymphocytes

35.Phagocytes can NOT attach to the bacterial cell wall surface by …….
- Directly attaching to the bacterial cell wall lipopolysaccharides
- Attaching to molecules on the bacterial cell surface using MHC Class II
molecules on the phagocyte
- Using an antibody molecule that is specific for an antigen on the surface
of the bacterial cell
- Using a fragment resulting from complement activation by the bacterial
cells

36.The ability of an antigen to induce an immune response depends on


- The antigen’s ability to enter the thyroid
- The antigen’s dose and size
- Antigen processing mechanism
- Antigen receptor gene rearrangement

37.Antibody-dependent cellular cytotoxicity (ADCC) requires the involvement


of
- Dendritic cell and infected cell
- Macrophage and cancer cell
- Erythrocyte and antibodies
- Antibodies and follicular dendritic cell

38.An example of innate immunity is

nolan x soju
- Antibody production by plasma cells
- Pathogen recognition by dendritic cells
- Distribution of infected cell by cytotoxic lymphocytes
- Memory response to influenza virus

39.Secondary immune response compare to primary contains


- Low level of specific IgG antibodies
- High level of specific IgM antibodies
- High level of specific IgG antibodies
- High level of specific IgD antibodies

40.What is likely to be elevated in the patient infected by Trichimella spiralis?


- Basophils
- Neutrophils
- Lymphocytes
- Eosinophils
41.A CD3+ cell that secrets perform and granzyme is a
- Gamma/delta T cell
- Cytotoxic T cell
- Helper T cell
- Regulatory T cell

42.Autoimmunity results from


- Antigen-driven self-reactive lymphocytes
- Accelerating clearence of apoptotic cells
- Elimination of extracellular pathogens
- Activation of Treg cells

43.Complement activated by pathogenic bacteria can contribute to the


destruction of ………
- By solubilization of immune complexes
- By assisting phagocytosis to attach to bacterial cells

nolan x soju
- By attracting more Treg cells to the site of infection
- By deposition of immune complexes insite blood vessels

44.Which category of hypersensitivity BEST describes transfusion reactions


when a recipient has antibodies against donor erythrocytes?
- Anaphylactic
- Cytotoxic
- Immune complex
- Delayed

45.Respiratory distress and unconsciousness developed within minutes after


penicillin injection is probably mediated by
- IgE antibody
- IgG antibody
- Sensitized T cells
- IgM antibody
46.A Tc cell can mount a cytotoxic attack on a viral-infected cell if it binds to a
cell displaying…….
- Insufficient MHCI
- Insufficient MHCII
- MHC I bearing foreign antigen
- MHC II bearing foreign antigen

47.The inflammation that results from a mosquito bite is different from the
inflammation that results from a positive TB skin test that the reaction to
the mosquito bite
- Does not require prior sensitization
- Takes more time to appear
- Results primarily from antibody-antigen interactions
- Involves basophils, mast cells and Tc cells

48. Patient with Wiskott-Aldrich syndrome have

nolan x soju
- An increased number of B cells
- An increased number of T cells
- Low platelet level in blood
- High platelet level in blood

49.Immune complex formation can result from all EXCEPT


- Persistent infection
- Inhalation of antigenic matherial
- Autoimmune disease
- Intradermal antigen

50.Opsonin treated bacteria are more readily engulfed by phagocytes than


are untreated bacteria because
- The capsule is removed by opsonin
- Opsonin digested the wall component
- Opsonin induces lysosomal enzymes
- Phagocyte contain receptors that bind antibodies
51. Lupus erythematosus is
- Free from an immune complex disease component
- Associated with antibodies against nucleic acid
- A disease of red and white cells
- An organ specific autoimmune disease

52.Di George syndrome results from a defect in


- Purine nucleoside phosphorylase
- WASP
- Thymic development
- DNA repair

53.IgE production depends on


- Th1 cells
- Th2 cells
- Th17 cells

nolan x soju
- NK cells

54.A patient is found to have a complete absence of C1 inhibitor. The most


likely consequence of this deficiency is
- Hereditary angioedema
- Immune complex disease
- Granuloma formation
- Deposition of immune complexes

55.The lesions in immune complex induced glomerulonephritis are


dependent on
- Erythrocytes and complement
- The production of urine
- Complement and neutrophils
- The presence of macrophages
56.Certain HLA genes are linked to autoimmune diseases because
- It may be the result of a single genetic defect
- It may be the result of the decreased number of helper T cells
- It may be the result of cross reactive between self antigen and infection
agent
- It carries no increased risk for a specific disease for those individuals with
the gene

57.Th2 cells are responsible for fighting against


- Tuberculosis bacteria
- Flu virus
- Tumor cell
- Worms

58.Granulomatous hypersensitivity is correctly described by which of the


following statements?

nolan x soju
- Granuloma contains mast cells and neutrophiles
- IgG has a major role in granuloma formation
- IFN-gamma is required for granuloma formation
- IL-4 is required for granuloma formation

59.A common strategy by which microbes survive their host’s immune


responses involves changing the structures of the molecules they produce
so that they are no longer recognized by the host’s immune system. This
strategy called antigenic variation is most likely to allow evasion of which
type of immune recognition?
- Toll like receptor dependent recognition
- Mannose receptor dependent recognition
- Antibody recognition of microbial cell surface molecules
- Natural killer cell inhibitory receptor recognition of class I major
histocompatibility complex (MHC) molecules on infected cells

60.Th most important chemokine receptor that act as coreceptors for HIV are
- CXCR5 and CCR6
- CXCR4 and CCR5
- CXCR7 and CCR5
- CXCR6 and CCR4

61.Infantile, X-linked agammaglobulinemia is associated with excessive


infection of the following type
- Intracellular bacterial
- Capsulated bacteria
- Viral
- Fungal

62.Bacteria growing within macrophages are killed by


- Reactive oxygen and nitrogen intermediates
- Complement components
- Cytotoxic T-cell
- Antibody dependent cell cytotoxicity

nolan x soju
63.Which of the following is a common cause of hypersensitivity diseases
- Failure of lymphocyte maturation
- Failure of self-tolerance
- Treatment with corticosteroids
- Disseminated cancer

64.In the thymus, T cells that recognise MHC class II molecules differentiate
to become
- CD8+ lymphocyte
- Gamma-delta T cell
- Natural killer cell
- CD4+ lymphocyte

65.IgA can be secreted from the body because


- Dimeric IgA binds poly-Ig receptor on mucosal epithelial cells
- Dimeric IgA has a specialized H chain called secretory chain
- IgA is small enough to pass between mucosal epithelial cells and leave the
body
- IgA is synthesized by mucosal epithelial cells and secreted directly into
the intestinal lumen

nolan x soju
Q: Hematopoietic stem cell differentiation down the lymphoid developmental pathway can give
rise to
A: Natural killer cells
Q: Which of the following autoimmune disease is NOT correctly matched with its major
consequence
A: upon a second encounter with an pathogen, a much faster and stronger response is elicited
Q: Which of the following autoimmune disease is NOT correctly matched with major
consequence
A: Graves disease - hypothyroidism
Q: Patients with DiGeorge syndrome
A: Have a decreased number of helper T cells
Q: Which of the following is NOT a way that the immune system eliminates pathogens?
A: ?
Q: How do red blood cells work in conjunction with antibodies in the immune system
A) RBCs can bind to IgG and IgM to clear immune complexes by transporting them to the
liver, the spleen, etc

nolan x soju
B) RBCs work in conjunction with AFCC to destroy target cells
C) RBCs mediate degranulation of mast cells
D) RBC’s phagocytose the immune complex formed by IgG and an antigenic molecule
Q: Receptor editing as a mechanism of immunological tolerance occurs in?
A) B-cells
B) Plasma cells
C) Cytotoxic T-cells
D) Regulatory T-cells
Q: Which of they following statements are characteristic of contact sensitivity?
A) The best therapy is administration of the antigen
B) Patch testing with the allergens is useless for diagnosis
C) Sensitization can be passively transferred with serum from an allergic individual
D) Some chemicals acting as haptens induce sensitivity by covalently binding to host
proteins actins as carriers

Mochii & Toba


Q: What structural feature organization allows immune cells access to all parts of body
A: The dual blood and lymphatic circulation
Q: removal of immune complexes is correctly described by which of the following statements?
A: it occurs primary of C3 for particulate complexes
Q: Granulomatous hypersensitivity is correctly described by which of the following statements?
A: it results from the presence of persistent microorganisms within macrophages
Q: A patients is found to have a complete absence of C1 inhibitor. The most likely consequence
of this deficiency is
A: hereditary angioedema
Q: TH1 cells most often “help” macrophages to become more efficient phagocytic cells. This
enhancement of phagocytosis is generally initiated by which one the following cytokines
secreted by these TH1 cells during this macrophages helping process?
A: interferon-gamma
Q: which tissue transplant does NOT require cross matching any
A: Cornea – not vascularized, immune cells cannot mediate a rejection

nolan x soju
Q: A second set rejection is characterized by all of the following EXCEPT?
A) Occurs after a first set rejection
B) Occurs rapidly
C) Has a period of latency before rejection
D) Requires immunological memory
Q: Antibodies that are able to provide protection from the effects of bacterial exotoxins do so in
which one of the following ways?
A: bind to the toxin molecules and prevent the toxins from binding to body cell surfaces
Q: at which stage of B-cell maturation does VDJ recombination occur?
A: Pre-B
Q: Patients with Chediak-Higashi syndrome have phagocytes which
A: show a reduced ability of their lysosomes to fuse with phagosomes to release microbicidal
substances

Mochii & Toba


Q: A macrophage that is stimulated to initiate phagocytosis of a bacterial cell that has a capsule
needs a specific antibody to an antigen on the capsule surface. The antibody also attaches to
which one of the following on the surface of the macrophage?
A: an Fc receptor
Q: A patient is admitted with multiple bacterial infections and is found to have a complete
absence of C9. Which complement-mediated function would be mostly affected in such a
patient:
A) Direct microbial killing
B) Generation of neutrophil chemotactic factors
C) Phagocytosis
D) Opsonization of bacteria
Q: A CD3+ cell that secretes perforin and granzyme is a
A: cytotoxic T cell
Q: Most autoimmune diseases are caused by
A: a constellation of genetic and environmental events
Q: cessation of the immune response is usually mainly a result of?
A) Release of IL-10 and transforming growth factor beta

nolan x soju
B) Immune cells binding to PAMPs and downregulating the innate immune response
C) Elimination of the pathogen
D) Immune cells binding to DAMPs
Q: Adaptive immune response shows immunological memory. This can be defined as
A: upon a second encounter with an pathogen, a much faster and stronger response is elicited
Q: IgG binding to neutrophil is mediated by
A: Fc receptor specific for IgG
Q: A Tc cell can mount a cytotoxic attack on a viral-infected cell if it binds to a displaying
A: MHC I bearing foreign antigen
Q: an example of a disease that effects the nervous system is?
A: myasthenia gravis
Q: Antibodies that are able to provide protection from the effects of bacterial exotoxins do so in
which one of the following ways?
A: bind to the toxin molecules and prevent the toxin from binding to body cell surfaces

Mochii & Toba


Q: Neutrophils of the following is NOT a way that the immune system eliminates pathogens?
A) Neutrophils and macrophages engulf pathogens
B) Pathogens are destroyed inside a phagocytic vacuole in a immunecell
C) Immune cells release factors that activate macrophages
D) Immune cells direct pathogens to engulf neutrophils
Q: T-cell mediated immune response CANNOT result in
A) Formation of granulomas
B) Induration at the reaction site
C) Rejection of a kidney transplant
D) Production of antibodies
Q: A Direct Coombs test detects…?
A: anti RBC autoantibody that is already bound to RBCs using an anti…
Q: Choose the correct statement
A) Class switching requires interaction between PD1 and B7
B) Wiskott-aldrich syndrome is an example of impaired heavy chain switching
C) After class switching the B cell begins to produce a new heavy chain isotype with the
different specificity to an antigen
D) None of the above

nolan x soju
Q: what crucial role do dendritic cells play in the differentiation Th0 cell?
A) Dendritic cells reverts Th1 and Th2 cells back to Th0 cells so that they can be reused in
an appropriate response
B) Dendritic cells release INF-gamma in response to PAMPs- which influences Th0 cells
C) Dendritic cells only present antigen to Th0 cells and have no other influence on their
differentiation
D) Dendritic cells release cytokines in response to PAMPs- these cytokines influence the
differentiation of the Th0 cell into the appropriate T cell lineage
Q: the original specificity of any one B-lymphocyte is determined by which of the following
A: the specificity of the antigen receptors on that B-cell’s surface
Q: which one of the following cytokines are most closely involved in activating cytotoxic T
lymphocytes to mediate graft rejection?
A: IL-2
Q: AIDS is caused by a human retrovirus that kills
A: CD4-positive T lymphocytes

Mochii & Toba


Q: The first step in thymic education that occurs in the cortex of the thymus is
A: Positive selection: T cell that can recognize self MHC are saved from apoptosis
Q: which one of the following antibodies would be considered to have the greatest specificity
for an epitope
A: the antibody with the highest affinity for the epitope
Q: Which receptor does NOT vary greatly from person to person?
A: PRR
Q: Class switching requires
A: CD40L on the surface of a T-cell
Q: Which of the following is a common cause of autoimmune diseases
A: Failure of self-tolerances
Q: T-cell-mediated immune responses CANNOT result in
A: production of antibodies
Q: The chemokine receptors that act as coreceptors for HIV is
A) CXCR5

nolan x soju
B) CXCR4
C) CXCR7
D) CXCR6
Q: which of the following cells are NOT key to chronic inflammation
A: Neutrophils
Q: Adaptive immune response shows immunological memory. This can be defined as
A: Upon a second encounter with an pathogen, a much faster and stronger response is
elicited
Q: where do almost all mature but naïve, T-lymphocyte encounter the epitopes for which they
are specific during the priming phase?
A: in the lymph node
Q: peptides from which of the following complex biomolecules would most likely eventually be
presented to T-cells using MHC Class I?
A: virus proteins in a cell that has been infected by a virus

Mochii & Toba


Q: The first step in thymic education that occurs in the cortex of the thymus is…?
A) Negative selection: selecting out any T cells that are CD4 or CD8 negative
B) Positive selection: T cells that recognize self MHC undergo apoptosis
C) Positive selection: T cells that can recognize self MHC are saved from apoptosis
D) Negative selection: T cells that recognize self MHC undergo apoptosis
Q: The ‘giant cells’ that can be present in a granuloma are derived from
A: macrophages
Q: complement that has been activated by pathogenic bacterial cells can not contribute
(directly or indirectly) to the destruction or elimination of those pathogenic bacterial cells in
which of the following ways?
A: by solubilization of immune system
Q: Humans are most likely to mount a TH1 response to
A: intravesicular pathogen
Q: A T cell located at the epithelial barrier of the gut is a
A: gamma/delta T cell

nolan x soju
Q: Rheumatoid factor is correctly described as
A: a synonym for auto-anti-IgG antibodies
Q: A characteristic feature of rheumatoid arthritis is the presence of autoantibodies is specific
for ?
A: Fcγ fragment
Q: Activation of naïve T-helper cells (specific for non-self-peptides) by mature dendritic (DC) in the
lymph node involves which of the following cell interactions?
A: MHC class II on DC with CD4 on naïve T-cell
Q: This cytokine-cytokine receptor interaction initiates extensive cell division of naïve T-cell in the
paracortex of lymph node. This important cytokine-receptor interaction is which one of the following?
A: IL-2 and a high affinity IL-2 receptor

Q: Which of the following viruses is NOT correctly associated with its associated malignancy?
A: HPV 16- nasopharyngeal cancer
Q: Which of the following viruses is NOT correctly associated with its associated malignancy?
A) HBV- liver cancer

Mochii & Toba


B) HPV 16- nasopharyngeal cancer
C) HTLV1-T cell leukemia
Q: Which of the following is NOT true regarding antibodies function?
A) Antibodies can mediate ADCC
B) Antibodies neutralize bacterial toxins
C) Antibodies cleave complement components
D) Antibodies can block viral binding
Q: Receptor editing as a mechanism of immunological tolerance occurs in?
A) B-cells
B) Plasma cells
C) Cytotoxic T-cells
D) Regulatory T-cells
Q: A second set rejection is characterized by all of the following EXCEPT?

A) Occurs after a first set rejection


B) Occurs rapidly
C) Has a period of latency before rejection
D) Requires immunological memory

Q: A T cell located at the epithelial barrier of the gut is a


A: gamma/delta T cell

Q: the following mechanism(s) may be involved in the clinical efficacy injection therapy

nolan x soju
(hyposensitization)
A: enhanced production of IgG, which binds allergen before it reaches m….
Q: immunologic tolerance is defined as
A: unresponsiveness of the immune system to an antigen, which is induced by previous
exposure to that antigen
Q: Which class of the antibody class has 2 subclasses
A: IgA
Q: Direct alloantigen recognition requires..?
A: Allogenic antigen presenting cells that express allogeneic MHC
Q: In a patient, it was observed that macrophages are abundant but do not seem to be
phagocytizing antigen efficiently. What complement factor may be deficient in this patient?
A: C3b
Q: A second set rejection is characterized by al of the following EXCEPT.
A: Has a period of latency before rejection

Mochii & Toba


Q: Which one of the following antibodies is NOT monomeric
A) Cell surface IgM
B) IgG
C) IgD
D) Secretory IgA
Q) Your patient test is positive for the tuberculin antigen. You should send him for a chest x-ray
because:
a) The tuberculin test is the only presumptive, indicating that he has been exposed to a
tuberculosis antigen
b) He may have other lung infection
c) You are looking for fluid in his lungs due to inflammation caused by the bacillus
d) You are looking for granulomas with neutrophils inside
Q) a graft-versus-host reaction is most commonly seen following transplantation of…?
A) Bone marrow
Q) ‘Double positive’ T-cells in the thymus express both which are?
A) CD4 and CD8

nolan x soju
Q) How do red blood cells work in conjunction with antibodies in the immune system?
A) RBCs can bind to IgG and IgM to clear immune complexes by transporting them to the
liver, the spleen, etc.
Q) what is the function of an adjuvant in a vaccine?
a) Neutralizes pathogenic nature of live-attenuated viruses
b) Stimulates antigen uptake and stimulate an immune response
c) Functions as a subunit to an antigen
d) Facilitates recognition by dendritic cells
Q) Class switching begins with what?
A) Signals from helper T cells
Q) the major mechanism of host resistance to M.leprae is
A) type IV hypersensitivity
Q) Hematopoietic stem cell differentiation down the lymphoid developmental pathway can give
rise to?

Mochii & Toba


A) Natural killer (NK) cells
Q) Against which pathogen IgE antibodies can provide immunity?
A) Schistosoma mansoni
Q) Which of the following is a component of the lectin pathway of complement activation?
A) MASP-1
Q) Which of the following statements are characteristic of contact sensitivity?
A) Some chemicals acting as haptens induce sensitivity by covalently binding to host proteins
acting as carriers
Q) which of the following sequences correctly depicts the course of a B cell in a germinal
center?
A) B cell enters Dark Zone of germinal center, B cells proliferate and move to Light Zone, B
cells undergo somatic mutation and isotype switch, differentiation and migration out of the
germinal center as memory B cell or Plasma cell
Q) You have a patient who makes autoantibodies against his own red blood cells, leading to
hemolysis. Which one of the following mechanisms is the MOST likely to explain the hemolysis?
A) Complement is activated and membrane attack complexes lyse the red cells

nolan x soju
Q) A second set rejection is characterized by all of the following EXCEPT..?
a) Occurs after a first set rejection
b) Occurs rapidly
c) Has a period of latency before rejection
d) Requires immunological memory
Q) What are the two classes of antibody that are expressed together on the surface of naïve B-
cells?
A) IgM and IgD
Q) A cell expressing CD3, CD16 and CD56 is a
A) natural killer T cell
Q) Th1 cells characteristically secrete:
A) IFN-gamma
Q) Activation of naïve T-helper cells (specific for non-self peptides) by mature dendritic cells
(DC) in the lymph node involves which of the following cells interactions

Mochii & Toba


A) MHC Class II on DC with CD4 on naïve T-cell
Q) Which complement pathway is NOT correctly matched with its activators?
a) Classical- antibody antigen complexes
b) Alternative- microbial sugars
c) Lectin- microbial sugars
d) Alternative- microbial structures
Q) Chemically induced tumors antigens are
a) Always the same for a given carcinogen
b) Specific tumor associated antigens
c) Very strong antigens
d) Tumor associated antigens
Q) Direct alloantigen recognition requires
A) Allogeneic antigen presenting cells that express allogeneic antigens
Q) Cessation of the immune response is usually mainly a result of?
A) Elimination of the pathogen
Q) Which one of the following cytokines are most closely involved in activating cytotoxic T
lymphocytes to mediate graft rejection?

nolan x soju
A) IL-2s
Q) An example of an anaphylatoxin is?
A) C5a
Q) NKT cells can characteristically recognize antigen presented by what?
a) CD2
b) CD1
c) CD4
d) CD8
Q) An example of a molecule produced during the innate response is?
A) Cytokine
Q) Which of the following antibodies is NOT monomeric?
a) Cell surface IgM
b) IgG
c) IgD

Mochii & Toba


d) Secretory IgA
Q) A Direct Coombs test detects…?
a) Free anti-RBC autoantibody in serum that is mixed with RBCs
b) Anti-RBC autoantibody that is already bound to RBCs using an anti-Human Ig
c) An exaggerated antibody response to anti-Human Ig
d) Destroyed or damaged RBCs
Q) A patient is admitted with multiple bacterial infections and is found to have a complete
absence of C9. Which complement-mediated function would be mostly affected in such a
patient:
A) direct microbial killing
Q) Regarding anaphylactic (type I) and immune complex (type III) hypersensitivities, which of
the following is the MOST acurate?
A) Less dose of antigen is typically needed to trigger anaphylactic reaction than in immune
complex reaction
Q) Patients with Wiskott-Aldrich syndrome have:
A) chronic eczema
Q) Chemically induced tumors antigens are

nolan x soju
A) specific tumor associated antigens
Q) Which of the following human cell types would not be capable of presenting peptides using
MHC Class I?
A) fully mature red blood cells
Q) Which of the following is NOT a way that the immune system eliminates pathogens?
A) Immune cells direct pathogens to engulf neutrophils
Q) Choose the correct statement:
a) Class switching requires interaction between PD1 and B7
b) Wiskott-Aldrich syndrome is an example of impaired heavy chain switching
c) After class switching, the B cell begins to produce a new heavy-chain isotype with the
different specificity to an antigen
d) None of the above
Q) After direct or indirect alloantigen recognition…?

Mochii & Toba


A) T cells become activated CD4+ and CD8+ effector T cells, targeting the alloantigen on the
graft
Q) The first line of defense against viruses is
A) IgA antibodies directed against viruses at mucus surfaces

Q) Opsonin-treated bacteria are more readily engulfed by phagocytes than are untreated
because
a) The capsule is removed by opsonin
b) Opsonin digest the wall component
c) Opsonin induces lysosomal enzymes
d) The surface of a phagocyte contains receptors for the Fc portion of an antibody
Q) which of the following viruses is NOT correctly associated with its associate malignancy?
a) HBV- liver cancer
b) HPV 16- nasopharyngeal cancer
c) HTLV1-T cell leukemia
Q) Class switching requires:

nolan x soju
a) CD40L on the surface of a B-cell
b) CD40L on the surface of a NK-cell
c) CD40L on the surface of a T-cell
d) CD80 on the surface of a B-cell
Q) Which of the following statements about type I hypersensitivity is true?
A) it occurs when an IgE response is directed against pollens
Q) what would be the expected result if a patient was unable to produce functional J (joining)
chain?
A) A decrease in circulating IgM
Q) Interferon-gamma
A) is released as a consequence of antigen- or mitogen-induced activation of T-lymphocytes
Q) The class of antibody that is most effective at participating in ADCC is:
A) IgG

Mochii & Toba


Q) which of the following autoimmune diseases is NOT correctly matched with its major
consequence?
A) Graves disease – hypothyroidism
Q) which of the following is NOT correct regarding immunity to cancer?
A) destruction of tumor cells is achieved by T regulatory cells
Q) Which receptor does NOT vary greatly from person to person?
A) PRR
Q) Antibody production by B-lymphocytes is initiated y direct contact between the B-
lymphocyte antigen receptor and the antigen for which that receptor is specific, wand which
one of the following additional requirements?
A) help from a T-helper 2 (TH2) lymphocyte
Q) Di George syndrome results from a defect in:
A) Thymic development
Q) Which antibodies can activate complement after binding to antigen
A) pentameric IgM in serum

nolan x soju
Q) certain HLA genes are linked to autoimmune diseases such as ankylosing spondylitis. This
linkage has all the following characteristics EXCEPT
A) It carries no increased risk for a specific disease for those individuals with the HLA-gene
Q) Antibody-dependent cellular cytotoxicity (ADCC) requires the involvement of which ONE of
the following?
A) Antibodies of the IgG isotype that are specific for non-self antigens/epitopes on the
surface of cancer cells
Q) what are the main differences between TCR and BCR?
A) TCR are highly peptide-MHC specific, BCR are antigen specific and are secreted from cell
Q) The final damage to vessels in immune complex-mediated arthritis is due to
A) lysosomal enzymes of poly morphonuclear leukocytes

Mochii & Toba


Q: How do red blood cells work in conjunction with antibodies in the immune system
A) RBCs can bind to IgG and IgM to clear immune complexes by transporting them to the
liver, the spleen, etc
B) RBCs work in conjunction with AFCC to destroy target cells
C) RBCs mediate degranulation of mast cells
D) RBC’s phagocytose the immune complex formed by IgG and an antigenic molecule
Q: Receptor editing as a mechanism of immunological tolerance occurs in?

A) B-cells
B) Plasma cells
C) Cytotoxic T-cells
D) Regulatory T-cells

Q: Which of they following statements are characteristic of contact sensitivity?


A) The best therapy is administration of the antigen
B) Patch testing with the allergens is useless for diagnosis
C) Sensitization can be passively transferred with serum from an allergic individual
D) Some chemicals acting as haptens induce sensitivity by covalently binding to host
proteins actins as carriers
Q: A second set rejection is characterized by all of the following EXCEPT?

nolan x soju
A) Occurs after a first set rejection
B) Occurs rapidly
C) Has a period of latency before rejection
D) Requires immunological memory
Q: A patient is admitted with multiple bacterial infections and is found to have a complete
absence of C9. Which complement-mediated function would be mostly affected in such a
patient:
A) Direct microbial killing
B) Generation of neutrophil chemotactic factors
C) Phagocytosis
D) Opsonization of bacteria
Q: cessation of the immune response is usually mainly a result of?
A) Release of IL-10 and transforming growth factor beta
B) Immune cells binding to PAMPs and downregulating the innate immune response
C) Elimination of the pathogen
D) Immune cells binding to DAMPs
Q: Neutrophils of the following is NOT a way that the immune system eliminates pathogens?
A) Neutrophils and macrophages engulf pathogens
B) Pathogens are destroyed inside a phagocytic vacuole in a immunecell
C) Immune cells release factors that activate macrophages
D) Immune cells direct pathogens to engulf neutrophils
Q: T-cell mediated immune response CANNOT result in
A) Formation of granulomas
B) Induration at the reaction site
C) Rejection of a kidney transplant
D) Production of antibodies
Q: what crucial role do dendritic cells play in the differentiation Th0 cell?

A) Dendritic cells reverts Th1 and Th2 cells back to Th0 cells so that they can be reused in
an appropriate response
B) Dendritic cells release INF-gamma in response to PAMPs- which influences Th0 cells
C) Dendritic cells only present antigen to Th0 cells and have no other influence on their
differentiation
D) Dendritic cells release cytokines in response to PAMPs- these cytokines influence the
differentiation of the Th0 cell into the appropriate T cell lineage

nolan x soju
Q: The chemokine receptors that act as coreceptors for HIV is
A) CXCR5
B) CXCR4
C) CXCR7
D) CXCR6
Q: The first step in thymic education that occurs in the cortex of the thymus is…?

A) Negative selection: selecting out any T cells that are CD4 or CD8 negative
B) Positive selection: T cells that recognize self MHC undergo apoptosis
C) Positive selection: T cells that can recognize self MHC are saved from apoptosis
D) Negative selection: T cells that recognize self MHC undergo apoptosis
Q: Which of the following viruses is NOT correctly associated with its associated malignancy?

A) HBV- liver cancer


B) HPV 16- nasopharyngeal cancer
C) HTLV1-T cell leukemia
Q: Which of the following is NOT true regarding antibodies function?

A) Antibodies can mediate ADCC


B) Antibodies neutralize bacterial toxins
C) Antibodies cleave complement components
D) Antibodies can block viral binding

Q: Receptor editing as a mechanism of immunological tolerance occurs in?

A) B-cells
B) Plasma cells
C) Cytotoxic T-cells
D) Regulatory T-cells

Q: A second set rejection is characterized by all of the following EXCEPT?

A) Occurs after a first set rejection


B) Occurs rapidly
C) Has a period of latency before rejection
D) Requires immunological memory

Q: Which one of the following antibodies is NOT monomeric


A) Cell surface IgM
B) IgG
C) IgD
D) Secretory IgA

nolan x soju
Q) Your patient test is positive for the tuberculin antigen. You should send him for a chest x-ray
because:
a) The tuberculin test is the only presumptive, indicating that he has been exposed to a
tuberculosis antigen
b) He may have other lung infection
c) You are looking for fluid in his lungs due to inflammation caused by the bacillus
d) You are looking for granulomas with neutrophils inside
Q) what is the function of an adjuvant in a vaccine?
a) Neutralizes pathogenic nature of live-attenuated viruses
b) Stimulates antigen uptake and stimulate an immune response
c) Functions as a subunit to an antigen
d) Facilitates recognition by dendritic cells
Q) A second set rejection is characterized by all of the following EXCEPT..?
a) Occurs after a first set rejection
b) Occurs rapidly
c) Has a period of latency before rejection
d) Requires immunological memory
Q) Which complement pathway is NOT correctly matched with its activators?
a) Classical- antibody antigen complexes
b) Alternative- microbial sugars
c) Lectin- microbial sugars
d) Alternative- microbial structures
Q) Chemically induced tumors antigens are
a) Always the same for a given carcinogen
b) Specific tumor associated antigens
c) Very strong antigens
d) Tumor associated antigens
Q) NKT cells can characteristically recognize antigen presented by what?

a) CD2
b) CD1
c) CD4
d) CD8
Q) A Direct Coombs test detects…?

a) Free anti-RBC autoantibody in serum that is mixed with RBCs


b) Anti-RBC autoantibody that is already bound to RBCs using an anti-Human Ig

nolan x soju
c) An exaggerated antibody response to anti-Human Ig
d) Destroyed or damaged RBCs

Q) Choose the correct statement:


a) Class switching requires interaction between PD1 and B7
b) Wiskott-Aldrich syndrome is an example of impaired heavy chain switching
c) After class switching, the B cell begins to produce a new heavy-chain isotype with the
different specificity to an antigen
d) None of the above
Q) Opsonin-treated bacteria are more readily engulfed by phagocytes than are untreated
because

a) The capsule is removed by opsonin


b) Opsonin digest the wall component
c) Opsonin induces lysosomal enzymes
d) The surface of a phagocyte contains receptors for the Fc portion of an antibody
Q) which of the following viruses is NOT correctly associated with its associate malignancy?
a) HBV- liver cancer
b) HPV 16- nasopharyngeal cancer
c) HTLV1-T cell leukemia
Q) Class switching requires:
a) CD40L on the surface of a B-cell
b) CD40L on the surface of a NK-cell
c) CD40L on the surface of a T-cell
d) CD80 on the surface of a B-cell

nolan x soju
3.2 Immunology State Questions
(July) 2020
1. Hematopoietic stem cell differentiation down the lymphoid
development pathway can give rise to:
c. Natural killer (NK) cells

2. Antibodies that are able to provide protection from the effects of


bacterial exotoxins do so in which of the following ways?
b. Bind to the toxin molecules and prevent the toxins from binding to
the body cell surfaces

3.What can lead to autoimmune disease


d. A breakdown of tolerance

nolan x soju
4. T-cell-mediated immune responses CANNOT result in
d. production of antibodies
or
d. immune complex disease

5. You have a patient who makes autoantibodies against his own red
blood cells, leading to hemolysis. Which one of the following
mechanisms is MOST likely to explain the hemolysis?
d. Complement is activated and membrane attack complexes lyse the red
cells
6. In a patient, it was observed that macrophages are abundant but do not
seem to be phagocytizing antigen efficiently. What complement factor
may be deficient in this patient ?
c.C3b

7. Select incorrect answer. Complement that has been activated by


pathogenic bacterial cells can contribute ( directly or indirectly ) to the
destruction or elimination of those pathogenic bacterial cells
d. by stimulating somatic hypermutation in the phagocytes that are
already actively engaged in phagocytizing the bacterial cells

8. Activation of naive T-helper cells (specific for non-self peptides) by


mature dendritic cells (DC) in the lymph node involves which of the
following cell interactions?

nolan x soju
a. MHC Class II on DC with CD4 on naive T-cell

9. A T cell located at the epithelial barrier of the gut is a


a. gamma/delta T cell

10. Which of the following is NOT a way that the immune system
eliminates pathogens?
d. Immune cells direct pathogens to engulf neutrophils

11. A cell expressing CD3, CD16 and CD56 is a


c. Natural killer T cell
12.Humans are most likely to mount a TH1response to
a. intravesicular pathogen
13. Where do almost all mature, but naive, T-lymphocytes encounter the
epitopes for which they are specific during the priming phase?
b. in the lymph node

14. Which of the following antibodies is NOT monomeric?


d. Secretory IgA

15. Patients with DiGeorge syndrome


d. have a decreased number of helper T cells

nolan x soju
16. Which of the following is NOT a way that the immune system
eliminates pathogens?
d. Immune cells direct pathogens to engulf neutrophils

17.What would be the expected result is a patient was unable to produce


functional J (joining) chain?
c. A decrease in circulating IgM

18. Class switching requires:


c. CD40L on the surface of a T-cell
19. Which class of the antibody class has 2 subclasses
a. IgA

20. An example of disease that effects the nervous system is ?


c. Myasthenia gravis

21.Which receptor does NOT vary greatly from person to person?


d. PRR

22. A graft-versus-host reaction is most commonly seen following


transplantation of…?
b. Bone marrow

nolan x soju
23. Receptor editing as a mechanism of immunological tolerance occurs
in?
a. B-cells

24. Which of the following viruses is NOT correctly associated with its
associated malignancy?
b. HPV 16-nasopharyngeal cancer

25.What structural organization allows immune cells access to all parts


of the body
b. The dual blood and lymphatic circulation
26.A CD3+ cell that secrets perforin and granzyme is a
b. cytotoxic T cell
27. Adaptive immune response shows immunological memory. This can
be defined as:
a. Upon a second encounter with an pathogen, a much faster and
stronger response is elicited

28. Which of the following autoimmune disease is NOT correctly


matched with its major consequences?
d. Graves disease – hypothyroidism

29. Choose the correct statement:

nolan x soju
a. class switching requires interaction between PD 1 and B7
b. Wiskott-Aldrich syndrome is an example of impaired heavy chain
switching
c. After class switching, the B cell begins to produce a new heavy-chain
isotype with the different specificity to an antigen
d. None of the above

30. Which of the following statements about type I hypersensitivity is


true?
b. it occurs when an IgE response is directed against pollens
31. Removal of immune complexes is correctly described by which of
the following statements?
b. it occurs primarily in the spleen and liver

32. Cessation of the immune response is usually mainly a result of?


c. Elimination of the pathogen

33. Di George syndrome results from a defect in


c. Thymic development
34. AIDS is caused by a human retrovirus that kills
c. CD4-positive T lymphocytes

nolan x soju
35. The class of antibody that is most effective at participating in ADCC
is:
b. IgG

36. Patients with Wiskott-Aldrich syndrome have:


d. chronic eczema

37. The original specificity of any on B-lymphocyte is determined by


which of the following?
b. the specificity of antigen receptors on that B-cell’s surface
38. A second set rejection is characterized by all of the following
EXCEPT:
c. Has a period of latency before rejection

39. A characteristic feature of rheumatoid arthritis is the presence of


autoantibodies specific for?
b. Fcγ fragment

40. Which of the following sequences correctly depicts the course of a B


cell in a germinal center?
d. B cell enters Dark Zone of germinal center, B cells proliferate and
move to Light Zone, B cells undergo somatic mutation and isotype
switching, differentiation and migration out of the germinal center as
Memory B cell or Plasma cell

nolan x soju
41. Antibody-dependent cellular cytotoxicity (ADCC) requires the
involvement of which ONE of the following?
b. antibodies of the IgG isotype that are specific for non-self
antigens/epitopes on the surface of cancer cells

42. Direct alloantigen recognition requires...?


b. Allogeneic antigen presenting cells that express allogeneic MHC

43. This cytokine-cytokine receptor interaction initiates extensive cell


division of naïve T-cell in the paracortex of lymph node. This important
cytokine-receptor interaction is which of the following?
d. IL-2 and a high affinity IL-2 receptor

44. A T cell located at the epithelial barrier of gut is a


a. gamma/delta T cell

45. The ‘giant cells’ that can be present in a granuloma are derived from:
b. Macrophages

46. A Direct Coombs test detects…?


b. Anti-RBC autoantibody that is already bound to RBCs using an anti-
Human Ig

nolan x soju
47. A macrophage that is stimulated to initiate phagocytosis of a
bacterial cell that has a capsule needs a spesific antibody to an antigen
on the capsule surface. The antibody also attaches to which one of the
following on the surface of the macrophage?
d. an Fc receptor

48. Regarding anaphylactic (type I) and immune complex (type III)


hypersensitivities, which of the following is MOST accurate?
c. Less dose of antigen is typically needed to trigger anaphylactic
reaction than an immune complex reaction

49. Which tissue transplant does NOT require cross matching and why
a. Cornea – not vascularized, immune cells cannot mediate a rejection
50. What are the main difference between TCR and BCR?
b. TCR are highly peptide-MHC specific, BCR are antigen specific and
can be secreted from cell

51. Which of the following is a common cause of autoimmune diseases


b. Failure of self-tolerance

52. How doe red blood cells work in conjunction with antibodies in the
immune system?
a. RBCs can bind to IgG and IgM to clear immune complexes by
transporting them to the liver, the spleen, etc.

nolan x soju
53. Th1 cells characteristically secrete:
c. IFN-gamma

54. Certain HLA genes are linked to autoimmune diseases such as


ankylosing spondylitis. This linkage has the following characteristics
EXCEPT
c. it carries no increased risk for a specific disease for those individuals
with the gene

55. Poliomavirus cause tumors in “nude mice” (which do not have a


thymus, because of a genetic defect) but not in normal mice. The BEST
interpretation is that
c. T lymphocytes play an important role in rejection of poliomavirus
induced tumors

56. ‘Double positive’ T-cell in the thymus express both, which are?
b. CD4 and CD8
57. Chemically induced tumors antigens are
b. specific tumor associated antigens

58. Which of the following cells are NOT key to chronic inflammation
b. Neutrophils

59. Which one of the following antibodies would be considered to have


the greatest specificity for an epitope?

nolan x soju
d. the antibody with the highest affinity for the epitope

60. Against which pathogen IgE antibodies can provide immunity?


b. Schistosoma mansoni

61. The first line of defense against viruses is


a. IgA antibodies directed against viruses at mucus surfaces

62. Rheumatoid factor is correctly described as


a. a synonym for auto-anti-IgG antibodies
63. Peptides from which of the following complex biomolecules would
most likely eventually be presented to T-cells using MHC Class I?
a. virus proteins in a cell that has been infected by a virus

64. The final damage to vessels in immune complex-mediated arthritis is


due to
c. lysosomal enzymes of polymorphonuclear leukocytes

65. Which of the following cytokines are most closely involved in


activating cytotoxic T lymphocytes to mediate graft rejection?
c. IL-2

66. After direct or indirect alloantigen recognition…?

nolan x soju
a. T cells become activated CD4+ and CD8+ effector T cells, targeting
the alloantigen on the graft

67. The major mechanism of host resistance to M.leprae is


b. type IV hypersensitivity

68. Interferon-gamma
c. is released as a consequence of antigen- or mitogen-induced activation
of T-lymphocytes

69. What are the two classes of antibody that are expressed together on
the surface of naive B-cells?
c. IgM and IgD

70. Antibody production by B-lymphocytes is initiated by direct contact


between the B-lymphocytes antigen receptor and the antigen for which
that receptor is specific, and which one of the following additional
requirements?
d. help from a T-helper 2 (TH2) lymphocyte

71. Most autoimmune diseases are caused by


b. a constellation of genetic and environmental events

72. Class switching begins with what?


a. Signals from helper T cells

nolan x soju
73. Which of the following human cell types would not be capable of
presenting peptides using MHC Class I?
b. fully mature red blood cells

74. Which of the following is NOT correct regarding immunity to


cancer?
d. destruction of tumor cells is achieved by T regulatory cells

75. Which of the following is a component of the lectin pathway of


complement activation?
d. MASP-1
76. A patient is admitted with multiple bacterial infections and is found
to have a complete absence of C9. Which complement-mediated
function would be mostly affected in such a patient:
a. direct microbial killing

77. Complement that has been activated by pathogenic bacterial cells


can not contribute (directly or indirectly) to the destruction or
elimination of those pathogenic bacterial cells in which of the following
ways?
d. by solubilization od the immune complexes

78. An example of a molecule produced during the innate response is?


c. Cytokine

nolan x soju
79. TH1 cells most often “help” macrophages to become more efficient
phagocytic cells. This enhancement of phagocytosis is generally initiated
by which one of the following cytokines secreted by these TH1 cells
during macrophage helping process?
a. interferon-gamma

80. Immunologic tolerance is defined as


c. Unresponsiveness of the immune system to an antigen , which is
induced by previous exposure to that antigen
81. The following mechanism(s) may be involved in the clinical efficacy
injection therapy (hyposensitization)
a. enhanced production of IgG , which binds allergen before it reaches
mast cells

82. Circulating lymphocytes will be found in which one of the following


just before they enter the blood circulation from the lymphatic system
c. the thoracic duct

83. Germinal centers are the location of..?


d. T follicular helper (Tfh) cells

84. Which of the following is NOT true regarding antibodies functions?

nolan x soju
c. Antibodies cleave complement components

85. Which antibodies can activate complement after binding to antigen


b. pentameric IgM in serum

86. Opsonin-treated bacteria are more readily engulfed by phagocytes


than are untreated bacteria because
d. the surface of a phagocyte contains receptors for the Fc portion of an
antibody

87. The chemokine receptors that act as coreceptors for HIV is


b. CXCR4
88. A diagnostic marker for tumors of the colon is:
d. Carcinoembryonic antigen (CEA)

89. What crucial role do dendritic cells play in differentiation of a Th0


cell?
d. Dendritic cells release cytokines in response to PAMPs- these
cytokines influence the differentiate of the Th0 cell into the appropriate
T cell lineage

90. Antibody-mediated allergic reactions involve the attachment of one


of the five isotypes to which one of the following types of cells in the
body?
b. mast cells

nolan x soju
91. Which of the following statements BEST explains the relationship
(?) cardiomyopathy and infection with Coxsakie B virus ?
a. Coxsakie B virus antigens induce antibodies cross-reactive with heart
tissue

92. What is the function of an adjuvant in a vaccine?


b. Stimulates antigen uptake and stimulate an immune response

93. Which complement pathway is NOT correctly matched with its


activation
b. Alternative- microbial sugars

94. NKT cells can characteristically recognize antigens presented by


what
b. CD1
95. IgG binding to neutrophils is mediated by:
c. Fc receptors specific for IgG

96. A Tc cell can mount a cytotoxic attack on a viral-infected if it binds


to a (?) displaying
c. MHC I bearing foreign antigen

97. At which stage of B-cell maturation does VDJ recombination occur?

nolan x soju
a. Pre-B

98. Patients with Chediak-Higashi syndrome have phagocytes which


c. show a reduced ability of their lysosomes to fuse with phagosomes to
release microbial substances

99. Granulomatous hypersensitivity is correctly described by which of


the following statements?
c. it results from the presence of persistent microorganisms within
macrophages
100. A patient is found to have a complete absence of C1 inhibitor. The
most likely consequence of this deficiency is
a. hereditary angioedema

101. The most likely clinical consequence of genetic deficiency of


complement component C2 and C4 is increased
d. incidence of immune complex disease

102. The first step in thymic education that occurs in the cortex of the
thymus is… ?
c.positive selection: T cells that can recognize self MHC are saved from
apoptosis
103. Your patient test is positive for the tuberculin antigen. You send
him for a chest x-ray because:

nolan x soju
a. The tuberculin test is only presumptive, indicating that he has been
exposed to the tuberculosis antigen

104. Which of the following statements are characteristic of contact


sensitivity?
d. some chemicals acting as haptens induce sensitivity by covalently
binding to host proteins acting as carriers

105. Anaphylactic reactions


c. may involve components of mast-cell granule matrix
106. An example of an anaphylatoxin is?
a. C5a

107. Individuals with genetic deficiency of C6 have


d. decreased resistance to Neisseria bacteremia

108. Type II hypersensitivity is correctly described by which of the


following statements?
d. it is antibody-dependent cytotoxic hypersensitive

109. Grafts between genetically identical individuals (i.e., identical


twins)
c. are not rejected, even without immunosuppression

nolan x soju
110. Lupus erythematous is
b. associated with antibodies against nucleic acid

111. Examples of primary T-cell deficiency disease include:


c. Wiskott-Aldrich syndrome

112. Which of the following surface molecules is expressed by T-helper


cells that have just been activated in the lymph node and that will
participate in sending a temporary inhibitory signal to that activated T-
cell?
c. CTLA-4
113. Hyperacute response differs from first set response in that it
involves
b. antibodies, which mark the tissue for ADCC

( questions from class tests)


114. When cells die by apoptosis, as compared to by necrosis, they don’t
potent to induce inflammation and damage of the surrounding tissue
because they
a. do not release the cytoplasmic contents

115. Which of the following clinical diseases is most likely to involve a


reaction to a hapten in its etiology?

nolan x soju
d. hemolytic anemia after treatment with penicillin

116. Patients with severely reduced C3 levels tend to have


b. increased numbers of severe bacterial infections.

117. Function of the enzyme reverse transcriptase in HIV infection is to:


c. produce DNA that can be incorporated into the host cell’s DNA from
viral RNA

118. Bone marrow transplantation in immunocompromised patients


presents which major problem?
a. potentially lethal graft-versus-host disease
119. Immune complexes are NOT involved in pathogenesis of
d. tuberculosis

120. Although IgE antibodies can cause quite serious allergic reaction:
this isotype of antibody does not appear to be very important in
providing immunity from which one of the following?
b. parasitic worms

121. A positive DTH (delayed-type hypersensitivity) skin reaction


involves the interaction of
b. antigen, antigen-sensitive lymphocytes, and macrophages

nolan x soju
122. Antihistamines
a. bind to receptors for histamine, thereby preventing the histamine from
exerting a pharmacologic effect
b. are more effective given before, rather than after the onset of allergic
reaction
c.do not influence the activity of leukotrienes
d. all are correct

123. Concerning a monoclonal antibody, all of the following statements


are correct EXCEPT that it
d. has found no diagnostic or therapeutic use
124. Chemically induced tumors have tumor-associated transplantation
antigens that
b. are different for two tumors of different histological type even if
induced by same carcinogen
125. Circulating immune complexes are an etiologic factor in the
following diseases:
b. glomerulonephritis after treatment with horse anti-tetanus antiserum

126. Clinical and experimental data indicate that certain forms of which
of which one of the following types of biomolecules may correlated with
frequencies of certain autoimmune disease?
c. HLA molecules

nolan x soju
127. The destruction of tumor cells may be achieved by
a. antibodies and complement
b. cytotoxic T lymphocytes
c. antibody-dependent cell cytotoxicity
d. all are correct

128. A patient is found to have a complete absence of C6. The most


likely consequence of this deficiency is defect in
a. lysis of bacteria
129. A patient with multiple bacterial infections and is found to have a
complete absence of C3. Which complement-mediated function would
remain intact in such a patient:
a. none of them
b. generation of neutrophil chemotactic factors
c. generation of anaphylatoxins
d. opsonization of bacteria

130. Grossly elevated serum level of IgE can be found in persons with
c. parasitic worm infestations

131. Delayed hypersensitivity skin reactions


d. can result in induration at the reaction site

nolan x soju
132. Serum sickness occurs only
b. when soluble immune complexes are found

133. The rejection of a mis-matched transplant by the recipient is


initiated by which one of the following that is part of the transplant?
c. MHC molecules that are not in the body of the recipient

134. Which one of the following does not involve CMI (cell mediated
immunity)?
a. Serum sickness
135. The major mechanism of host resistance to tuberculosis is
b. delayed hypersensitivity

136. The lesions in immune complex-induced glomerulonephritis


c. are caused by complement and neutrophils
137. Serum sickness is not characterized by
d. Th1 immune response

138.The Arthus reaction does not involve


d. IgE- crosslink induced degranulation of mast cells

nolan x soju
139. The most likely clinical consequence of a genetic deficiency of
complement component C3 is increased
c. susceptibility to bacterial infections

140. An IgA antibody to a red cells antigen is unlikely to cause


autoimmune hemolytic anemia because
c. its Fc region would not bind receptors for Fc for phagocytic cells
1. Which of the following statements about type I hypersensitivity is true?
a. it is also known as delayed-type hypersensitivity
b. it occurs when an IgE response is directed against pollens
c. it does not require IgE-sensitized mast cells
d. it is independent of pharmacologic mediators

2. Cessation of the immune response is usually mainly a result of?


a. Release of IL-10 and transforming growth factor beta
b. Immune cells binding to PAMPs and downregulating the innate immune response
c. Elimination of the pathogen
d. Immune cells binding to DAMPS

3. Antibodies that are able to provide protection from the effects of bacterial exotoxins do so in
which one of the following ways?
a. activate complement that, when activated, breaks down the toxin molecules into harmless
fragments
b. enable macrophage to present the toxin antigens to T-lymphocytes
c. bind to the toxin molecules and prevent the toxins from binding to body cell surfaces
d. cause blood vessels located near the site of the toxin to become slightly leaky, thus allowing
cells in the blood stream to enter the tissues and destroy the toxin molecules.

nolan x soju
4. What are the two classes of antibody that are expressed together on the surface of naive B
cells?
a. IgG and IgD
b. IgG and IgE
c. IgM and IgD
d. IgM and IgG

5. Germinal centers are the location of..?


a. T-cell receptor VDJ gene rearrangement
b. T-cell somatic hypermutation
c. B-cell receptor VDJ gene rearrangement
d. T follicular helper (Tfh) cells

6. The final damage to vessels in immune complex-mediated arthritis is due to


a. Histamine
b. NK cell
c. Lysosomal enzymes of polymorphonuclear leukocytes
d. cytotoxic T cell
7. IgG binding to neutrophils is mediated by
a. Fc-dependent cellular homing mechanisms
b. sensitization of mast cells and basophils
c. Fc receptors specific for lgG
d. ICAM's

8. Which receptor does NOT vary greatly from person to person?


a. BCR
b. MHC
c. TCR
d. PRR

9. A diagnostic marker for tumors of the colon is:


a. Alpha-fetoprotein
b. Common acute lymphoblastic leukemia antigen (CALLA)
c. EBV-related antigens
d. Carcinoembryonic antigen (CEA)

10. Which one of the following antibodies is NOT monomeric?


a. Cell surface IgM

nolan x soju
b. IgG
c. IgD
d. Secretory IgA

11. Which complement pathway is NOT correctly matched with its activators
a. Classical- antibody antigen complexes
b. Alternative- microbial sugars
c. Lectin- microbial sugars
d. Alternative- microbial structures

12. Poliomavirus cause tumors in “nude mice” (which do not have a thymus, because of a genetic
defect) but not in normal mice. The BEST interpretation is that
a. macrophages are required to reject polomavirus-induced tumors
b. natural killer cells can reject poliomavirus-induced tumors without help from T lymphocytes
c. T lymphocytes play an important role in rejection of poliomavirus-induced tumors
d. B lymphocytes play no role and rejection of poliomavirus-induced tumors

13. Examples of primary T-cell deficiency disease include:


a. Hereditary angioneurotic edema
b. Chronic granulomatous disease
c. Wiskott-Aldrich syndrome
d. Leukocyte adhesion deficiency

14. What would be the expected result if a patient was unable to produce functional Goining)
chain?
a. An increase in secretory IgA
b. A decrease in IgE response
c. A decrease in circulating IgM
d. A decrease in cell surface IgM

15. Which of the following sequences correctly depicts the course of a B cell in a germinal center?
a. B cell enters Dark Zone of germinal center, B cells proliferate and move to Light Zone, B
cells undergo somatic mutation and isotype switching, differentiation and migration out of
the germinal center as Dendritic cell or Helper T cell
b. B cell enters Dark Zone of germinal center B cells undergo dedifferentiation and migration
out of the germinal center as Memory B cell or Plasma cell
c. B cell enters Light Zone of germinal center B cells proliferate and move to Dark Zone,
differentiation and migration out of the germinal center as Memory B cell or Plasma cell
d. B cell enters Dark Zone of germinal center B cells proliferate and move to Light Zone B cells

nolan x soju
undergo somatic mutation and isotype switching differentiation and migration out of the
germinal center as Memory B cell or Plasma cell

16. The most likely clinical consequence of a genetic deficiency of complement component C2 and
C4 is increased
a. susceptibility to viral infections
b. susceptibility to fungal infections
c. susceptibility to bacterial infections
d. incidence of immune complex disease

17. NKT cells can characteristically recognize antigens presented by what?


a. CD2
b. CD1
c. CD4
d. CD8

18. T-cell-mediated immune responses CANNOT result in


a. formation of granulomas
b. induration at the reaction site
c. rejection of a kidney transplant
d. production of antibodies

19. Removal of immune complexes is correctly described by which of the following statements?
a. it is independent of C3 for particulate complexes
b. it occurs primarily in the spleen and liver
c. it is independent of size of the immune complexes
d. it occurs primarily in the thymus and spleen

20. The ‘giant cells that can be present in a granuloma are derived from?
a. Granulocytes
b. Macrophages
c. Fibroblasts
d. Th1 cells

21. An example of a molecule produced during the innate response is?


a. Antibody
b. RAG-1
c. Cytokine

nolan x soju
d. Immunoglobulin

22. Adaptive immune response shows immunological memory. This can be defined as:
a. Upon a second encounter with an pathogen a much faster and stronger response is elicited
b. The inability to recognize an antigen in a specific way
c. The immune response is the same, no matter what pathogen is encountered
d. Production of cytokines and chemokines upon encountering a pathogen for the first time

23. Type II hypersensitivity is correctly described by which of the following statements?


a. it is antibody independent
b. it is complement independent
c. it does not involve killer cells
d. it is antibody-dependent cytotoxic hypersensitive

24. Grafts between genetically identical individuals (ie, identical twins)


a. are subject to acute rejection
b. are subject to hyperacute rejection
c. Are not rejected, even without immunosuppression
d. are not rejected if a kidney is grafted but skin grafts are rejected

25. Which one of the following antibodies would be considered to have the greatest specificity for
an epitope?
a. the antibody with the lowest affinity for the epitope
b. the antibody that forms the fewest covalent bonds with the epitope
c. the antibody that has amino acids in the hypervariable regions that form no bonds with the
epitope
d. the antibody with the highest affinity for the epitope

26. Antibodies that are able to provide protection from the effects of bacterial exotoxins do so in
which one of the following ways?
a. activate complement that, when activated, breaks down the toxin molecules into harmless
fragments
b. enable macrophage to present the toxin antigens to T-lymphocytes
c. bind to the toxin molecules and prevent the toxins from binding to body cell surfaces
d. cause blood vessels located near the site of the toxin to become slightly leaky, thus allowing
cells in the blood stream to enter the tissues and destroy the toxin molecules

27. The following mechanism may be involved in the clinical efficacy of injection therapy

nolan x soju
(hyposensitization)
a. Enhanced production of IgG, which binds allergen before it reaches mast cells
b. enhanced production of IgE, which binds IgE receptor before it reaches antigen
c. activation of mast cells that induces production of a large amount of anti-histamines
d. increased local recruitment of eosinophils

28. A macrophage that is stimulated to initiate phagocytosis of a bacterial cell that has a capsule
needs a specific antibody to an antigen on the capsule surface. The antibody also attaches to
which one of the following on the surface of the macrophage?
a. an antigen on the macrophage surface that is identical to the antigen on the capsule surface
for which the antibody is specific
b. MHC Class I] molecules
c. a receptor for a fragment of activated complement the complement has been activated by
the binding of the antibody to the capsule antigen)
d. an Fc receptor

29. A cell expressing CD3, CD16 and CD56 is a

a. gamma delta T cell


b.cytotoxic T cell

c. Natural killer T cell

d. regulatory T cell

30. Granulomatous hypersensitivity is correctly described by which of the following

a. it is not clinically important

b. it has not be encountered in schistosomiasis

c. it results from the presence of persistent microorganisms within macrophages

d. it does not cause pathological effects in diseases associated with T cell-mediated immunity

nolan x soju
31. Most autoimmune diseases are caused by

a. a T-cell defect
b. a constellation of genetic and environmental events
c. a known infectious organism
d. a single genetic defect

32. Hematopoietic stem cell differentiation down the lymphoid developmental pathway can give rise to?

a. Eosinophils

b. Follicular dendritic cells

c. Natural killer (NK) cells

d. Monocytes

33. Opsonin-treated bacteria are more readily engulfed by phagocytes than are untreated bacteria

Because
a. the capsule is removed by opsonin

b. opsonin digest the wall component

c. opsonin induces lysosomal enzymes

d. the surface of a phagocyte contains receptors for the Fe portion of an antibody

34. Which of the following is NOT a way that the immune system eliminates pathogens?

a. Neutrophils and macrophages engulf pathogens

b. Pathogens are destroyed inside a phagocytic vacuole in an immune cell

c. immune cells release factors that activate macrophages

d. Immune cells direct pathogens to engulf neutrophils

35. Acharacteristic feature of rheumatoid arthritis is the presence of autoantibodies specific for?

a. Fcu fragment

b. Fcy fragment

c. Clq component

nolan x soju
d. Fab fragment

36. Hyperacute response differs from a first set response in that it involves

a. tissue damage by activated CTLs

b. Antibodies, which mark the tissue for ADCC

c. coordination of the response by DTH cells

d. tissue damage by activated mast cells

37. ACD3+ cell that secrets perforin and granzyme is a

a. gamma delta T cell

b. cytotoxic T cell

c. helper T cell

d. regulatory T cell
38. Which of the following viruses is NOT correctly associated with its associated malignancy?

a. HBV-liver cancer

b. HPV 16- nasopharyngeal cancer

c. HTLV1- T cell leukemia

39. The first step in thymic education that occurs in the cortex of the thymus is.?

a. negative selection: selecting out any T cells that are CD4 or CD8 negative

b. positive selection. T cells that recognize self MHC undergo apoptosis

c. positive selection: T cells that can recognize self MHC are saved from apoptosis

d. negative selection: T cells that recognize self MHC undergo apoptosis

40. In a patient, it was observed that macrophages are abundant but do not seem be phagocytizing

antigen efficiently. What complement factor may be deficient in this Patient?

a. C5b6789

b.C5a

nolan x soju
c. C3b

d. C3a

41. Which of the following autoimmune diseases is NOT correctly matched with its major

consequence?

a. Pernicious anemia - Vitamin B12 deficient anemia

b. Multiple sclerosis - demyelination

c. Type I diabetes - hyperglycemia

d. Graves’ disease — hypothyroidism

42. This cytokine-cytokine receptor interaction initiates extensive cell division of naive T-cell in the

paracortex of lymph node. This important cytokine receptor interaction is which one of the following?

a. IL-1 and low affinity IL-1 receptor

b. IL-7 and a high affinity IL-7 receptor


c. interferon-gamma and the interferon-gamma receptor

d. IL-2 and a high affinity IL-2 receptor

43. Antibody production by B-lymphocytes is initiated by direct contact between the B lymphocyte

antigen receptor and the antigen for which that receptor is specific and which one of the following
additional requirements?

a. help from a T-helper cell that has been stimulated by MHC Class I-presented peptides

b. contact between the Fc portion of the antibody and an Fc receptor on the surface of a T- helper 2
(TH2) lymphocyte c

c. help from a macrophage that is presenting peptides using MHC Class Il molecules

d. help from a T-helper 2 (TH2) lymphocyte

44. Humans are most likely to mount a TH1 response to

a. Intravesicular pathogens

b. a parasitic helminth (worm) infestation

c. pollen or cat dander allergens

nolan x soju
d. self-antigen found in the bone marrow

45. How do red blood cells work in conjunction with antibodies in the immune system?

a.RBCs can bind to IgG and IgM to clear immune complexes by transporting them to the liver, the
spleen, etc.

b. RBCs work in conjunction with ADCC to destroy target cells

c. RBCs mediate degranulation of mast cells

d. RBCs phagocytose the immune complex formed by IgG and an antigenic molecule

46. An example of an anaphylatoxin is?

a. C5a

b. IL-18

c. Histamine

d. Prostaglandin
47. An example of a disease that effects the nervous system is?

b. Celiac Disease

c. Myasthenia gravis

d. Type I diabetes

48. A graft-versus-host reaction is most commonly seen following transplantation of

a. Skin

b. Bone marrow

c. Cornea

d. Heart

49. Individuals with a genetic deficiency of C6 have :

a. decreased resistance to viral infections

b. increased hypersensitivity reactions

c. increased frequency of cancer

nolan x soju
d. decreased resistance to Neisseria bacteraemia

50. Th1 cells characteristically secrete:

a. TGF-beta

b.IL-22

c. IFN-gamma

d. IL-17

51. Activation of naive T-helper cells (specific for non-self peptides) by mature dendritic cells (DC) in the

lymph node involves which of the following cell interactions?

a.MHC Class Il on DC with CD4 on naive T-cell

b.CD28 on DC with CD56 on naive T-cell

c.CD28 DC with T-cell epitope-specific receptor on naive T cell

d.CD40L on DC with CD4 on naive T-cell


52 The original specifity of any one B-lymphocyte is determined by which one of the following?

a.The type of CD molecules on that B-cell’s surface

b.The specifity of the antigen receptors on that B-cell’s surface

c.The specifity of MHC class | molecules

d.MHC class II molecules

53. Which of the following statement as (are) characteristics of contact sensitivity?

a. The best therapy is administration of the antigen

b. Patch testing with the allergen is useless for diagnosis

c. Sensitization can be passively transferred with serum from an allergic individual

d. Some chemicals acting as haptens induce sensitivity by covalently binding to host proteins acting as
carriers

54. Chemically induced tumours antigens are

nolan x soju
a.always the sa,e for a given carcinogen

b.specific tumor associated antigens

c.very strong antigens

d.tumor associated antigens

55. Which tissue transplant does NOT require cross matching and why?

a. Cornea-not vascularised,immune cells cannot mediate a rejection process

b. Blood-antigens and antibodies can be addressed using immunosupressive therapy

c. Skin-not subject to immune mediated rejection

d. Liver-stand alone circulation, immune cells cannot mediate a rejection process

56 Immunologic tolerance define as

a) The removal of microbes expressing the antigen, so that host can tolerate the infection

b) Activation of only B cells and not T cells on exposure to an antigen


c) Unresponsiveness of immune system to an antigen, which is induced by previous exposure to that
antigen

d) The production of memory B cells and T cells on primary exposure to an antigen

57.The major mechanism of host resistance to M.leprae is

a. Humoral antibodies

b. Type lV hypersensitivity

c. High level of calcium in serum

d. Type! hypersensitivity

58.which of the following cells are NOT key to chronic inflammation?

a. Macrophages

b. Neutrophils

c. T-cells

d. Lymphocytes

nolan x soju
59.Patients with Wiskott-Aldrich syndrome have:

A. Increased number of B cells

B. Aincreased number of suppressor Tcells

C. High platelet level in blood

D. Chronic eczema

60. Circulating lymphocytes will be found in which one of the following locations just before they

enter the blood circulations from the lymphatic system?

a) A high endothelial venule

b) A lymph node

c) The thoracic duct

d) The bone marrow


61.Double positive T-cells in the thymus express both, which are?

a. CD3 and CD4

b. CD4 and CD8

c. MHCland MHCII

d. CD3 and CD8

62.Your patient tests positive for the tuberculin antigen. You send him for a chest X Ray because:

a)The tuberculin test is only presumptive indicating that he has been exposed to a tuberculosis antigen.

b) He may have other lung infections

c) You are looking for fluid in his lungs due to inflammation caused by bacillus

d) You are looking for looking for granulomas with neutrophils inside

63.Against which pathogen IgE antibodies can provide immunity?

a. Human hepatitis C virus

b. Schistosoma mansoni

nolan x soju
c. Staphylococcus aureus

d. Salmonella species

64.Which of the following is NOT true regarding antibodies function?

a. Antibodies can mediate ADCC.

b. Antibodies neutralize bacterial toxins.

c. Antibodies cleave complement components.

d. Antibodies can block viral binding.

65.What crucial role do dendritic cells play in the differentiation of a Th0 cell?

a. Dendritic cells revert Th1 and Th2 cells back to ThO cells so that they can be reused in an

appropriate response.

b. Dendritic cells release INFy in response to PAMPs-which influences ThO cells.

c. Dendritic cells only present antigens to ThO cells and have no other influence on their differentiation.
d. Dendritic cells release cytokines in response to PAMPs-these cytokines influence the

differentiation of the ThO cell into the appropriate T cell lineage.

66.A second set rejection is characterised by all the following EXCEPT?

a. Occurs after a first set rejection.

b. Occurs rapidly.

c. Hasa period of latency before rejection

d. Requires immunological memory.

67.At which stage of B-cell maturation does VDJ recombination occur?

a. Pre-B

b. Pro-B

c. MatureB

d. Stem Cell

nolan x soju
Qn.68. Antibody dependent cellular cytotoxicity (ADCC) requires the involvement of which ONE of the

following?

a) Antibodies of the IgM isotype that are specific for non-self antigens/epitopes on the surface of cancer
cells

b) Antibodies of the IgG isotype that are specific for non-self antigens/epitopes on the surface of cancer
cells

c) Antibodies of the IgD isotype

d) T cytotoxic lymphocytes

69. What are the main differences between TCR and BCR?

a. TCR undergo genetic recombination is individually coded

b. TCR are highly peptide-MHC specific, BCR are antigen specific and can be secreted from cell

c. TCR has a significant amount of protein diversity, BCR has only 12

d. TCR is part of the adaptive immune system, BCR is part of the innate immune system
70.What is the function of an adjuvant in vaccine?

a. Neutralizes pathogenic nature of live-attenuated viruses

b. Stimulates antigen uptake and stimulate an immune response

c. Functions as a subunit to an antigen

d. Facilitates recognition by dendritic cells

71 .A patient is found to have a complete absence of C1 inhibitor. The most likely consequent of this
deficiency is

a) Hereditary angioedema

b) Immune complex disease

c) Granuloma formation

d) Deposition of immune complexes

72.TH1 cells most often” help” macrophages to become more efficient phagocytotic cells. This

enhancement of phagocytosis is generally iniated by which one of the following cytokines secreted by
these TH1 cells during this macrophage-helping process?

nolan x soju
a. Interferon-gamma

b. Interleukin-8

c. Interleukin-2

d. CD4OL

73.After direct or indirect alloantigen recognition?

a. Tcells become activated CD4+ and CD8+ effector T cells,targeting the alloantigen on the graft

b. Cytokines produced by allogenic CD4+ cells will prevent invasion of CD4+ and CD8+ effector T

cells from host

c. Allogeneic effector B cells secrete cytokines which induce graft rejection

CD4+ effector T cells become activated and travel to allograft.CD8+ cells are not involved

74. Complement that has been activated by pathogenic bacterial cells can contribute (directly or
indirectly) to the destruction or elimination of those pathogenic bacterial cells
a) By destroying the bacterial cell membrane

b) By assisting phagocytes to attach to bacterial cells

c) By attracting more phagocytes to the area of complement activation

d) by solubilization of immune complexes

75.Patients With DiGeorge syndrome

a. Have increased number of suppressor T cells

b. Have selective IgA deficiency

c. Have abnormal; antigen-presenting cells

d. Have a decreased number of helper T cells

76.Which of the following is a component of the lectin pathway of complement activation?

a. Clq

b. Factor D

c. FactorB

nolan x soju
d. MASP-1

77. A patient is admitted with multiple bacterial infections and is found to have a complete absence of

C9. Which complement-mediated function would be mostly affected in such a patient

A) direct microbial killing

b) generation of neutrophil chemotactic factors

c) phagocytosis

d) opsonization of bacteria

78. Regarding anaphylactic (type |) and immune complex (type III) hypersensitivities, which of the

following is the MOST accurate?

A ) IgE is involved in both anaphylactic and immune complex hypersensitivites

B) Complement is involved in both anaphylactic and immune complex hypersensitivities


C) Less dose of antigen is typically needed to trigger anaphylactic reaction than an immune complex
reaction

d) Neutrophils play more important role in anaphylactic reactions than in immune complex reactions

79. Di George syndrome results from a defect in

A) Purine nucleoside phosphorylase

B) WASP

C) Thymic development

D) DNA repair

80. Certain HLA genes are linked to autoimmune diseases such as ankylosing spondylitis. This linkage has

all the following characteristics EXCEPT

A) it may be the result of closely linked genes

b) it may be the result of cross-reactivity between self antigen and infection agent

C) it carries no increased risk for a specific disease for those individuals with the gene

nolan x soju
D) it may be the result of a MHC molecule serving as an attachment site for an infectious agent

81. Class switching requires:

A) CD40L on the surface of a B-cell

B) CD4OL on the surface of a NK-cell

C) CD40L on the surface of a T-cell

D) CD80 on the surface of a B-cell

82. The first line of defense against viruses is

a) IgA antibodies directed against viruses at mucus surfaces

b) IgG directed to an internal viral antigens

c) IgM directed to an external viral antigens

d) interferon-gamma
83. A Direct Coombs test detects...?

A) Free anti-RBC autoantibody in serum that is mixed with RBCS

B) Anti-RBC autoantibody that is already bound to RBCs using an anti-Human lg

C) An exaggerated antibody response to anti-Human lg

D) Destroyed or damaged RBCS

84. Antibody-mediated allergic reactions involve the attachment of one of the five isotypes to which one

of the following types of cells in the body?

a) macrophage (via Fc receptors for the IgG isotype)

b) mast cells

C) NK cells

D) any phagocytic cell

85.Which antibodies can activate complement after binding to antigen

Answers:

nolan x soju
A) monomeric IgM on B cells

b) pentameric lgM in serum

c) monomeric IgA in serum

d) dimeric IgA in secretion

86.You have a patient who makes autoantibodies against his own red blood cells, leading to hemolysis.

Which one of the following mechanisms is MOST likely to explain the hemolysis?

A) Perforins from cytotoxic T cells lyse the red cells

B) Neutrophils release proteases that lyse the red cells

C) Interleukin-2 binds to its receptor on the red cells, which results in lysis of the red cells

D) Complement is activated and membrane attack complexes lyse the red cells

87.AIDS is caused by a human retrovirus that kills

A) B lymphocytes
B) lymphocyte stem cells

C) CD4-positive T lymphocytes

D) CD8-positive T lymphocytes

88. Lupus erythematosus is

A) free from an immune complex disease component

b) associated with antibodies against nucleic acid

C) a disease of red and white cells

D) an organ-specific autoimmune disease

89. Choose the correct statement:

A) Class switching requires interaction between PD1 and B7

b) Wiskott-Aldrich syndrome is an example of impaired heavy chain switching

c) After class switching, the B cell begins to produce a new heavy-chain isotype with the different

specificity to an antigen

nolan x soju
D) None of the above

90. Class switching begins with what?

a) Signals from helper T cells

b) Recombination

C) Translation

d) Deletion of intervening genes

91. The class of antibody that is most effective at participating in ADCC is:

A) IgM

B) IgG

c) IgE

d) lgD
92. Anaphylactic reactions

A) can result in induration at the site reaction

b ) are the consequences of TH1 activation

c) may involve components of mast-cell granule matrix

D) are the consequences of TH17 activation

93. What can lead to autoimmune disease

a) Increase in anergy

b ) Upregulation of genes for AIR

c ) Peripheral lymphoid tissue destruction

d) A breakdown of tolerance

94. Receptor editing as a mechanism of immunological tolerance occurs in?

a) B-cells

B) Plasma cells

nolan x soju
c) Cytotoxic T-cells

d) Regulatory T-cells

95. Where do almost all mature, but naive, T-lymphocytes encounter the epitopes for which they are

specific during the priming phase?

a) in the HEV

b) in the lymph node

c) in the thoracic duct (just before entering the blood circulation)

d) in the medulla of the thymus (involves antigen presentation by dendritic cells)

96. Which of the following is NOT correct regarding immunity to cancer?

a) despite the immune response tumors continue to grow

b) Th2 cells evoke anti-tumor immune response

c) vaccination can be used to treat cancer


d) destruction of tumour cells is achieved by T regulatory cells

97. Which one of the following cytokines are most closely involved in activating cytotoxic T lymphocytes

to mediate graft rejection?

A) IL-13

B) IL-10

C) IL-2

D) IL-4

98.Direct alloantigen recognition requires..?

A) Autoreactive T-cells

B) Allogeneic antigen presenting cells that express allogeneic MHC

C) Memory B cells specific for allograft

D) Host antigen presenting cells that express allogeneic MHC

nolan x soju
99. A Tc Cell can mount a cytotoxic attack on a viral-infected cell if it binds to a cell displaying

A) insufficient MHC class I molecules

B) insufficient MHC class II molecules

c) MHC I bearing foreign antigen

d) MHC Il bearing foreign antigen

100. AT cell located at the epithelial barrier of the gut is a Answer

a) gamma delta T cell

b) helper T cell

c) cytotoxic T cell

d) Natural killer cell


nolan x soju
1. What is the source of all undifferentiated stem cell that can produce any of the cells
involved in immune responses?
a. The spleen
b. The bone marrow
c. The lymph node
d. The thoracic duct

2. Somatic recombination occurs


a. after affinity maturation
b. in the progenitor cell as it is becoming a B cell
c. after isotype switching
d. in the plasma cell after antibody secretion

3. Vaccines stimulate
a. Natural passive immunity
b. Artificial active immunity
c. Artificial passive immunity
d. Natural active immunity

nolan x soju
4. The antigen specificity of an adaptive immune response is due to
a. Activation of antigen specific lymphocytes
b. Activation of macrophages
c. Lysis of only certain pathogens by neutrophils
d. phagocytosis of pathogens by macrophages

5. Antibody Fc fragments
a. Contain antigen binding sites
b. Contain complementarity determining regions
c. Bind to complement component
d. Binds to Toll-like receptors

6. TCR most closely resembles


a. Class I MHC
b. Class II MHC
c. Fab region of immunoglobulin
d. Fc region of immunoglobulin
7. Identify the cytokine that promotes the development of the Th2 response
a. interleukin 17
b. interleukin 4
c. interleukin 12
d. interleukin 5

8. The IgG isotype is often used as an example of the “typical” immunoglobulin. Identify
the physical or biological characteristics of one antibody molecule that has the IgG
isotype?
a. has one heavy chain and two identical light chains
b. is able to bind two different antigens/epitopes
c. papain treatment will produce two identical Fab fragments and two identical Fc
fragments
d. is able to bind to specific antigen

9. Human Class II MHC molecules


a. Are encoded by the genes HLA A, B and C
b. Are found on all nucleated cells

nolan x soju
c. Are encoded by the genes HLA DP, DR and DQ
d. Present antigen to CD8 cytotoxic T cells

10. Signal transduction is the process of converting


a. B cell to a T cell
b. A binding signal to a chemical signal
c. A hapten to an antigen
d. IgA to secretory IgA

11. A cell which secretes large quantities of antibodies but does not express surface
immunoglobulin is a
a. Naive mature B cell
b. Memory B cell
c. B-1 cell
d. Plasma cell
12. Identify an important cytokine receptor interaction which initiates extensive cell division
of the primed T cell
a. IL-2 and a high affinity IL-2 receptor
b. IFN gamma and the IFN gamma receptor
c. IL-7 and a high affinity IL-7 receptor
d. IL-7 and low affinity IL-7 receptor

13. Besides the direct contact between the B-lymphocyte and the antigen, antibody
production by B-lymphocytes requires
a. Help from a T-helper cell
b. Help from a macrophage
c. Help from a NKT cell
d. Help from a T-cytotoxic cell

14. Th17 cytokine that promotes killing of extracellular pathogen is a


a. interferon-gamma
b. interleukin I7
c. interleukin 10
d. interleukin 4

nolan x soju
15. Genes for immunoglobulins are unlike other human genes in that
a. Antibody genes are composed of introns and exons
b. DNA for antibody molecules is inherited from only one parent
c. Gene segments must be spliced together to make each unique antibody molecule
d. Several introns encode each antibody molecule

16. Which of the following is the neutralizing antibody?


a. IgM
b. IgG
c. IgD
d. IgE

17. All of the following are associated with the secretion of proinflammatory cytokines
EXCEPT
a. Failure to make immune response following stimulation by antigen
b. upregulation of adhesive molecules on the vascular endothelium
c. Migration of inflammatory cells into the damaged area
d. Increased permeability of vascular vessels

18. Phagocytes that encounter bacterial cells in the extravascular tissues are stimulated to
release cytokines and chemokines. Choose the event occurring as result of the release
of these cytokines and chemokines by the phagocytes?
a. the spaces between the cells that form the blood vessel do not increased
b. some of the phagocytes are stimulated to secrete specific antibodies
c. some phagocytic cells m the nearby blood stream are actually attracted to the site of
the infection
d. T-lymphocytes leave the blood circulation at the site of the infection to produce
specific antibodies

19. An antibody fab fragment


a. Contains complementarity determining regions
b. Ha and L chain constant regions
c. different antigen binding regions
d. binds to Fc receptors

nolan x soju
20. Clonal selection
a. Begins with inflammation
b. Occurs for all leukocytes
c. Occurs in response to self-antigens
d. Occurs for all lymphocytes

21. Phagocytes can NOT attach to the bacterial cell wall surface by which of the following
ways?
a. Directly attaching to the bacterial cell wall lipopolysaccharides
b. Attaching to molecules on the bacterial cell surface using MHC Class II molecules on
the phagocyte
c. Using an antibody molecule that is specific for an antigen on the surface of the
bacterial cell
d. Using a fragment resulting from complement activation by the bacterial cells

22. The ability of an antigen to induce an immune response depends on


a. The antigen’s ability to enter the thyroid
b. The antigen’s dose and size
c. Antigen processing mechanism
d. Antigen receptor gene rearrangement

23. An example of innate immunity is


a. Antibody production by plasma cells
b. Pathogen recognition by dendritic cells
c. Destruction of infected cell by cytotoxic lymphocytes
d. Memory response to influenza virus

24. Secondary immune response compare to primary contains


a. Low level of specific IgG antibodies
b. High level of specific IgM antibodies
c. High level of specific IgG antibodies
d. High level of specific IgD antibodies

25. What is likely to be elevated in patient infected by Trichinella spiralis?


a. Basophils

nolan x soju
b. Neutrophils
c. Lymphocytes
d. Eosinophils

26. Autoimmunity results from


a. antigen-driven self-reactive lymphocytes
b. accelerating clearance of apoptotic cells
c. elimination of extracellular pathogens
d. activation of Treg cells

27. which category of hypersensitivity Best describes transfusion reactions when a recipient
has antibodies against donor erythrocytes?
a. Anaphylactic
b. Cytotoxic
c. Immune complex
d. Delayed
28. Respiratory distress and unconsciousness developed within minutes after penicillin
injection is probably mediated by
a. IgE antibody
b. IgG antibody
c. sensitized T cells
d. IgM antibody

29. A Tc cell can mount a cytotoxic attack on a viral infected cell if it binds to a cell
displaying
a. Insufficient MHC I
b. Insufficient MHC II
c. MHC I bearing foreign antigen
d. MHC II bearing foreign antigen

30. Patients with Wiskott Aldrich syndrome Have:


a. An increased number of B cells
b. An increased number of T cells
c. Low platelet level in blood
d. High platelet level in blood

nolan x soju
31. immune complex formation can result from all, EXCEPT
a. persistent infection
b. inhalation of antigenic material
c. autoimmune disease
d. intradermal antigen

32. Di George syndrome results from a defect in


a. Purine nucleoside phosphorylase
b. WASP
c. Thymic development
d. DNA repair

33. IgE production depends on


a. Th1 cells
b. Th2 cells
c. Th17 cells
d. NK cells

34. A patient is found to have a complete absence of C1 inhibitor. The most likely
consequence of this deficiency is
a. Hereditary angioedema
b. Immune complex disease
c. Granuloma formation
d. Deposition of immune complexes

35. The lesions in immune complex induced glomerulonephritis are dependent on


a. Erythrocytes and complement
b. The production of urine
c. Complement and neutrophils
d. The presence of macrophages

36. Certain HLA genes are linked to autoimmune disease because


a. It may be the result of a single genetic defect
b. It may be result of decreased number of helper T cells

nolan x soju
c. It may be the result of cross reactivity between self-antigen and infection agent
d. It carries no increased risk for a specific disease for those individuals with the gene

37. Th2 cells are responsible for fighting against


a. Tuberculosis bacteria
b. Flu virus
c. Tumor cells
d. Worms

38. Granulomatous hyperse3nsitivity is correctly described by which of the following


statements?
a. Granuloma contains mast cells and neutrophils
b. IgG has a major role in granuloma formation
c. IFN gamma is required for granuloma formation
d. IL-4 is required for granuloma formation
39. A common strategy by which microbes survive their hosts immune response involves
changing the structures of the molecules they produce so that they are no longer
recognized by the hosts immune system. This strategy called antigenic variation is most
likely to allow evasion of which type of immune recognition?
a. Toll-like receptor dependent recognition
b. Mannose receptor dependent recognition
c. Antibody recognition of microbial cell surface molecules
d. Natural killer cell inhibitory receptor recognition of class I major histocompatibility
complex (MHC) molecules on infected cells

40. The most important chemokine receptors that act as coreceptors for HIV are
a. CXCR5 and CCR6
b. CXCR4 and CCR5
c. CXCR7 and CCR5
d. CXCR6 and CCR4

41. Infantile X-linked agammaglobulinemia is associated with excessive infections of the


following type
a. Intracellular bacterial

nolan x soju
b. Capsulated bacteria
c. Viral
d. Fungal

42. Bacteria growing within macrophages are killed by


a. Reactive oxygen and nitrogen intermediates
b. Complement components
c. Cytotoxic t cells
d. Antibody dependent cell cytotoxicity

43. Which of the following is a common cause of hypersensitivity diseases


a. Failure of lymphocyte maturation
b. Failure of self tolerances
c. Treatment with corticosteroids
d. Disseminated cancer
Priming of naïve T helper cells by mature dendritic cells in lymph nodes involves:
A) CD40 on DC with CD40L on naïve T cell
B) CTLA-4 on T cell with peptide presented by DC
C) CD56 on naïve T cell with CD28 on DC
D) CD8 on naïve T cell with MHC class 1 on DC
The specificity of any lymphocyte is determined by
A) The type of CD molecules on the cell
B) The specificity of the antigen receptors on that cell
C) MHC class 1 molecules
D) PAMP recognition receptors
Which is NOT characteristic of for both Type II and Type III hypersensitivity reactions
A) Involves attack by Tc cells
B) Involves IgG antibodies
C) Is likely to result in kidney damage
D) Can be evoked by exposure to pathogen
If pathogen blocks the fusion of lysosomes with phagocytotic vesicle, they can be killed through
the effector function of
A) Memory B cells

nolan x soju
B) Complement
C) Cytotoxic T cells
D) Opsonizing antibody
Which of the following has the MAJOR influence on the antibody response carried out by a B-
lymphocyte?
A) A T-cytotoxic cell surface molecule
B) Cytokine released by T helper cell
C) Anaphylatoxins
D) Phagocytes presenting peptides to T helper cells
T cells are MHC restricted in their ability to respond to antigen because
A) During an infection, all cells in the body present antigen on MHC class I
B) MHC binds antigen more specifically than TCR does
C) TCR must recognize both antigen and MHC molecules
D) The T cells should not respond to antigen on allogenic cells
Which of the following is not correct regarding immunity in cancer cells
A) Despite the immune response tumors continue to grow
B) Abrogation of CD25+ cells leads to protective tumor immunity
C) Vaccination can be used to treat cancer
D) Patients with cancer have successful tumor immunity
Positive selection includes apoptosis in developing T cells that bind to
A) Both MHC I and II
B) Neither MHC I and II
C) Self-antigen
D) Foreign antigen
All of the following are true for immune hemolytic anemia EXCEPT
A) It is the clinical condition in which IgG antibodies bind to red cell surface antigens
B) It is the clinical condition in which IgM antibodies bind to red cell surface antigens
C) Antibodies initiate red cell destruction via the complement system and phagocytes
D) Red cell destruction occurs by T cytotoxic and NK cells
Which of the following is not correct regarding autoimmune diseases
A) Genetic factors play a role in development of autoimmune diseases
B) Abrogation of CD25+ cells leads to downregulation of the autoimmune response

nolan x soju
C) An individual can have more than one autoimmune disease
D) Autoantibody production alone does not equal autoimmune disease
The inflammation that results from a mosquito bite is different from the inflammation that
results from a positive TB skin test tin that the reaction to the mosquito bite
A) Does not require prior sensitization
B) Takes more time to appear
C) Results primarily from antibody antigen interaction
D) Involves basophil, mast cells and Tc cells
Which one of the following types of cells is NOT capable of phagocytosis
A) Dendritic cells
B) NKT cell
C) Macrophage
D) Neutrophil
Which of the following immune effector mechanism is the most important in completely
clearing of infection caused by streptococcus pneumoniae
A) Antibody dependent cell cytotoxicity
B) Complement mediated opsonization
C) Cytotoxic T cell lymphocytes
D) Natural killer cells
To elicit the best antibodies to rat self antigen you should inject rat cells into
A) An animal of different species
B) A rat of same strain
C) A rat of different strain
D) The rat you isolated from
Which statement about antigen epitope is false
A) An epitope may be shared by two different antigens
B) A protein molecule usually contains multiple enzymes
C) Epitope is a part of an antibody which recognizes and binds to an antigen
D) Some epitopes are more immunogenic than others
An antibody Fab fragment
A) Contains complementarity determining regions

nolan x soju
B) H and L chain constant regions
C) Different antigen binding regions
D) Binds to Fc receptors
Positive skin test showing delayed type hypersensitivity such as for mumps or tuberculosis
indicate that
A) A humoral immune response has occurred
B) A cell mediated immune response has occurred
C) Only B cell system is functional
D) The patient has immune deficiency
A transplant of tissue between individuals of different species is called an
A) Autograft
B) Isograft
C) Allograft
D) Xenograft
Which of the following statements correctly describe type III hypersensitivity
A) It occurs within 72 hours of re-exposure to antigen
B) It is not mediated by complement fixing IgG
C) Complement is an important mediator
D) Desensitization is used for its control
Example of primary T cell deficiency disease include
A) Hereditary angioneurotic edema
B) Chronic granulomatous disease
C) Wiskott-Aldrich syndrome
D) Leukocyte adhesion deficiency
Type IV hypersensitivity reactions result in
A) Formation of granulomas
B) Development of anaphylactic reaction
C) Complement mediated lysis of infected cells
D) Serum sickness
Which of the following is NOT correct regarding autoimmune diseases
A) Genetic factors play a role in development of autoimmune diseases
B) Abrogation of CD25+ cells leads to downregulation autoimmune response

nolan x soju
C) An individual can have more than one autoimmune disease
D) Autoantibody production alone does not equal autoimmune disease
Type II hypersensitivity is
A) Antibody independent
B) Complement independent
C) Caused by antibodies against soluble antigen
D) Caused by antibodies against cell surface antigen
Hemolytic disease of the newborn caused by Rh blood group incompatibility requires maternal
antibody to enter fetal blood stream therefore the mediator of the disease is
A) IgE antibody
B) IgG antibody
C) IgM antibody
D) IgA antibody
When cells die by apoptosis, they do not induce inflammation and damage of the surround
tissue because they
A) Do not release cytoplasmic contents
B) Are not subsequently phagocytized
C) Migrate into the blood stream prior to death
D) Are rarely malignant or infected by virus
Antibody dependent cell mediated cytotoxicity (ADCC) is a proves in which antibody coated
cells are killed by
A) Cells with Fc receptors for IgM
B) Cells with Fc receptors for Complement components
C) Cells with Fc receptors for secretory IgA
D) Cells with Fc receptors for IgG
Inflammation does NOT involve:
A) Cytokine production by macrophages
B) Migration of leukocytes out of the circulation
C) Swelling of tissues and pain
D) Production and maturation of lymphocytes
Select INCORRECT answer. Complement that has been activated by bacterial cells can

nolan x soju
contribute to the destruction or elimination of these cells
A) By destroying the bacterial cell membrane
B) By assisting phagocytes to attach to bacterial cells
C) By attracting more phagocytes to the area of complement activation
D) By assisting phagocytes to produce memory cells
Complement receptors
A) Activate complement system on the surface of pathogen
B) Bind only activated complement proteins
C) Are involved in elimination of immune complexes
D) On macrophages signal host cells to make opsonins
What is the role of the macrophage during antibody formation
A) Development of allergic reactions
B) Lysis of virus infected cells
C) Antigen presentation
D) Synthesis of antibodies
Identify the fundamental difference between the antigen recognition by B and T cells
A) Antigen must be presented in different ways
B) Clonal selection after antigen recognition
C) Heterogeneity of TCR and BCR from one lymphocyte to the next
D) Membrane location of antigen specific receptors
Which of the following correctly describes Type III hypersensitivity
A) It occurs within 72 hours of re-exposure to antigens
B) It is not mediated by complement fixing IgG
C) Complement is an important mediator
D) Desensitization is used for its control
Options without questions:
Macrophage and cancer cells
By assisting phagocytes to produce memory cells

nolan x soju
1- TCR most closely resembles:
a- Class I MHC
b- Class II MHC
c- Fab region of immunoglobulin
d- Fc region of immunoglobulin

2- Rapid but non-antigen specific immune responses are produced by the:


a- Adaptive immune system
b- Innate immune system
c- Lymphocytes
d- Lymphatic system

3- Interferon gamma:
a- Is released as a consequence of antigen-induced activation of Th2
lymphocytes
b- Is released as a consequence of antigen-induced activation of Th1
lymphocytes
c- Is released as a consequence of antigen-induced activation of Th17

nolan x soju
lymphocytes
d- Is released as a consequence of antigen-induced activation of treg
lymphocytes

4- Which of the following statements is FALSE:


a- As example of passive humoral immunity is treatment with horse anti-
snake venin
b- Antigen recognized by Th cells must be associated with class II MHC
molecules
c- Each lymphocyte has many antigen binding receptor, each receptor
capable of binding the same antigen
d- The cytokines produced by T cells subunits determine their effector
functions
e- The cytokines that drive the development of T cell subunits are NOT
produced by antigen-presenting cells
5- A cell producing cytotoxic compounds following Th1 cell activation is a
(an):
a- Immature dendritic cell
b- Macrophage
c- Eosinophils
d- Mature dendritic cell

6- Somatic recombination occurs:


a- After affinity maturation
b- In the progenitor cells as it is becoming a B cell
c- After isotype switching
d- In the plasma cells after antibody secretion

7- Identify the NK marker(s):


a- CD3
b- CD14
c- CD16 and CD36
d- CD19 and CD20

nolan x soju
8- The immunoglobulin isotype is determined by the:
a- Antigen specificity
b- H chain constant region
c- L chain variable region
d- Number of antigen-binding sites

9- A CD3+ cell which secretes IL2 when activated is:


a- Respiratory T cell
b- Helper T cell
c- Cytotoxic T cell
d- Gammadelta T cell
10- Select INCORRECT answer. Complement that has been activated
bu bacterial cells can contribute to the destruction and elimination of
these cells:
a- By destroying the bacterial cell membrane
b- By assisting phagocytes to attach to bacterial cells
c- By attracting more phagocytes to the area of complement activation
d- By assisting phagocytes to produce memory cells

11- Complement receptors (CRs):


a- Activate complement system on the surface of pathogens
b- Bind only activated complement proteins
c- Are involved in elimination of immune complexes
d- On macrophages signal host cells to make openings

12- For specific antigen recognition by T cells:


a- CD25 molecules are required
b- HLA molecules are required
c- Soluble antigen is bound directly without processing

nolan x soju
d- Antigen exposure during T cell maturation is required

13- Amoxicillin is a small molecule that does not induce antibody


formation. However it binds to serum protein and forms a complex
that in several people induces allergic reactions. Amoxicillin is
therefore:
a- An antigen
b- A hepatin
c- An immunogen
d- A super antigen

14- Identify the component that does NOT actually help a phagocyte
to attach to the surface of a bacterial cell?
a- IgG
b- C4b
c- C3b
d- C3a

15- Which of the following antibodies would be considered to have


the greatest specificity for an epitope?
a- The antibody with the lowest affinity for the epitope
b- The antibody that forms the fewest non-covalent bonds with the epitope
c- The antibody that forms no bonds with the epitope
d- The antibody with the highest affinity for the epitope

16- Just before they enter the blood circulation from the lymphatic
system, circulating lymphocytes will be found in the:
a- A high endothelial venule
b- A lymph node
c- The thoracic duct
d- The bone marrow

17- A product of vaccination is:

nolan x soju
a- Naïve mature B cells
b- Memory B cells
c- Naïve mature T cells
d- Plasma cells

18- Immune system cell adhesion molecule:


a- Do not allow macrophages to leave circulation
b- Do not allow T cells to come to peripheral lymphoid tissue
c- Help activated B cell producing high affinity antibody
d- Help cytotoxic T cell to bind specific target

19- Lymphocyte circulation:


a- Activates inflammatory cytokines to promote antigen presentation to T
cells
b- Allows lymphocytes to encounter an appropriate antigen
c- Circulates lymphokines efficiently throughout the body
d- Circulates chemokines efficiently throughout the body

20- Which one of the following on the surface of the macrophages


initiates phagocytosis of capsulated bacteria?
a- An antigen on the macrophage surface that is identical to the antigen on
the capsule surface
b- MHC class II molecules on the macrophage surface
c- A receptor for a fragment of activated complement
d- An Fc receptor on the macrophage surface

21- Identify the marker on T cells in the blood sample:


a- CD3
b- CD4
c- CD8
d- CD20

22- In case of C2 and C4 deficiencies:

nolan x soju
a- C5 can still be cleaved by the chemical pathway
b- C3b will not be able to bind to bacteria
c- C9 will not polymerize and lyse host cells
d- C3b production will be reduced

23- Lymphocytes completely recirculate through peripheral


lymphoid in order to:
a- Be killed before they cause autoimmunity
b- Efficiently encounter antigen
c- Mature from stem cells into lymphocytes
d- Phagocytose antigen and kill it

24- T cells are MHC-restricted in their ability to respond to antigen


because:
a- During an infection, all cells in the body present antigen on MHC class I
b- MHC binds antigen more specifically than TCR does
c- TCR must recognize both antigen and MHC molecules
d- The T cells should not respond to antigen on allogenic cells

25- In the thymus, T cells that recognize MHC class II molecules


differentiate to become:
a- CD8 lymphocyte
b- Gamma-delta T cell
c- Natural killer cell
d- CD4+ lymphocyte

26- The signal transduction molecules associated with TCR are:


a- CD21 and CD81
b- Iga and Igb
c- IgD and IgM
d- ITAMs and ITIMs

27- It was observed that milkmaids infected with cowpox were later

nolan x soju
immune to smallpox infections, this is an example of:
a- Innate immunity to a cross-reactive agent
b- Innate immunity of milkmaids to smallpox
c- Memory response to a cross-reactive agent

28- The IgG isotype is often used as an example of the typical


immunoglobulin. Identify the physical or biological characteristics of
one antibody molecules that has the IgG isotype:
a- Has one heavy chain and two identical light chains
b- Is able to bind two different antigens/epitopes
c- Papain treatment will produce two identical Fc fragments
d- Is able to bind to specific antigen

29- Which of the following is the neutralizing antibody?


a- IgM
b- IgG
c- IgD
d- IgE

30- Both class I and class II MHC molecules are:


a- Expressed constantly on all nucleated cells
b- Expressed on the surface of antigen-presenting cells
c- Part of the T cell receptor for antigen
d- Synthesized in response to antigen processing

31- A cell which secretes large quantities of antibodies but does not
express surface immunoglobulin is a:
a- Naïve mature B cell
b- Memory B cell
c- B-1 cell
d- Plasma cell

32- Which cells in the body are involved in the antibody-mediated

nolan x soju
allergic reactions?
a- Macrophages and NK cells
b- Mast cells and basophils
c- Gammadelta T cells
d- Any phygotic cell

33- Clonal selection


a- Begins with inflammation
b- Occurs for all leukocytes
c- Occurs in response to cell antigen
d- Occurs for all lymphocytes

34- IgA can be secreted from the body because:


a- Dimeric IgA binds poly-Ig receptor on mucosal epithelial cells
b- Dimeric IgA has a specialized II chain called secretory chain
c- IgA is small enough to pass between mucosal epithelial cells and leave
the body
d- IgA is synthesized by mucosal epithelial cells and secreted directly into
the intestinal ….

35- Class II MHC does not efficiently present endogenous antigen


because:
a- Antigen synthesized inside the cell never makes it to the endosomal
compartments
b- Endogenous antigens cannot be processed into peptides small enough
c- HLA DM transport class II to the surface before it can bind endogenous
peptide
d- Invariant chain blocks binding endogenous peptide in the ER

36- Exogenous peptides are presented in association with:


a- IgG heavy chains
b- HLA-A, B or C molecules
c- HLA-DR, DP or DQ molecules

nolan x soju
d- T cell receptor

37- A typical IgM isotype antibody does NOT consist of:


a- Two identical light chains
b- Two identical heavy chains
c- Secretory component
d- A “J” (joining) chain

38- Identify a major interleukin produced by naïve CD+ T helper


lymphocytes:
a- IL-1
b- IL-2
c- IL-4
d- IL-8
39- Somatic hypermutation:
a- Occur by somatic recombination
b- Occur during T cell proliferation
c- Occur in the B cell following antigen stimulation
d- Result in decreased affinity of antibodies

40- Antibody effector functions include all of the following EXCEPT:


a- Activation of complement on the bacterial surface
b- Activation of proteolysis of endocytosed proteins
c- Blocking uptake of bacterial toxins
d- Coating bacteria to promote their phagocytosis

41- Identify the surface molecule expressed on the antigen-


presenting cells participating in active naïve T cell:
a- B-7
b- CD28
c- CTLA-4
d- CD4

nolan x soju
42- A CD3+ cell that secretes perforin and granzyme is a:
a- Gamma/delta T cell
b- Cytotoxic T cell
c- Helper T cell
d- Regulatory T cell

43- Allergy symptoms are produced when antigen binds to IgE on


FcR on:
a- NKT cells
b- Macrophages
c- Basophils
d- Neutrophils
44- Inflammatory cytokines produced by macrophages activate all of
the following EXCEPT:
a- Complement system via classical pathway
b- Integrin on leukocytes to bind more strongly to vascular CAMs
c- Neutrophils to be more cytotoxic
d- NK cells to kill virus infected cells

45- Choose the distinction between innate and adaptive immune


responses:
a- Adaptive responses (phagocytosis, complement activation) improve
significantly with each exposure to pathogens
b- Innate responses often can take 1-2 weeks to begin destroying infection
microorganisms, while adaptive responses destroy infections about
immediately after the very first encounter with a pathogen
c- An adaptive response does not improve significantly with repeated
exposure to the same non-…….
d- Each cell involved in innate responses (e.g. phagocytes) can be
stimulated to respond by many …… molecules, while each cell involved
in adaptive responses (e.g. lymphocytes) can be stimulated to respond

nolan x soju
by only one molecular structure (called an antigen)

46- Phagocytes attach to the bacterial cell wall surface using any of
these mechanisms, EXCEPT:
a- Directly attaching to the bacterial cell wall polysaccharides
b- Attaching to molecules on the bacterial cell surface using MHC class II
molecules on the …..
c- Using an antibody molecule that is specific for an antigen on the surface
of the bacterial cell wall
d- Using a fragment resulting from complement activation by the bacterial
cells

47- Isotype switching:


a- Decreases the functional diversity of Ig molecules
b- Improves the antigen binding specificity of an Ig molecule
c- Increases the affinity of antibodies in a process called affinity maturation
d- Increases the functional diversity of Ig molecules

48- Identify molecules on the surface of an infected cell that are


responsible for recognition by host CD8+ cell:
a- CD4
b- CD8
c- MHC class I
d- MHC class II

49- Antibody dependent cellular cytotoxicity (ADCC) requires the


involvement of :
a- Dendritic cell and infected cell
b- Macrophage and cancer cell
c- Erythrocyte and antibodies
d- Antibodies and follicular dendritic cell

50- Which marker or markers is present on the B class and could be

nolan x soju
used to specifically identify such cells in a flow cytometric analyser?
a- CD3
b- CD8
c- CD16 and CD56
d- CD19 and CD20

51- Antigens normally expressed only on embryonic cells but also


found in certain tumors are known as:
a- Cryptic antigen
b- Oncofetal antigen
c- Superantigen
d- HTLV-1

52- Autoimmunity results from:


a- Antigen driven self reactive lymphocytes
b- Accelerating clearance of apoptotic cells
c- Elimination of extracellular pathogens
d- Activation of Treg cells

53- What is the major consequences of the bone marrow


transplantation in an immunocompromised patients?
a- Potentially lethal graft-versus-host disease
b- High risk of T cell leokemia
c- Inability to use a liver donor
d- Delayed hypersensitivity

54- Which of the following statements BEST explains the relationship


between inflammation of the heart and infection with group A beta
haemolytic streptococci?
a- Streptococcal antigens induce antibodies cross-reactive with heart tissue
b- Streptococci are polyclonal activators of b cells
c- Streptococcal antigens bind to IgE on the surface of heart tissue and
histamine is released
d- Streptococci are ingested by neutrophils that release proteases that

nolan x soju
damage heart tissue

55- Which of the following is not correct regarding immunity to


cancer?
a- Despite immune responses tumour continues to grow
b- Activation of CD25+ cell lead to protective tumour immunity
c- Vaccination can be used to treat cancer
d- Patient with cancer have successful tumour immunity

56- Complement activated by pathogenic bacteria can contribute to


destruction of this bacteria:
a- By solubilisation of immune complexes
b- By assisting phagocytes to attack to bacterial cells
c- By attracting more Treg cells to the site of infection
d- By depression of immune complexes inside blood vessels
57- The principal difference between cytotoxic (type II) and immune
complex (type III) hypersensitivity:
a- The clone (isotype) of antibody
b- The site where antigen-antibody complexes are formed
c- The participation of complement
d- The participation of T cells

58- Lupus erythematosus is:


a- Free from an immune complex disease component
b- Associated with antibodies against neucleic acid
c- A disease of red and white cells
d- An organ specific autoimmune disease

59- The most important receptors that act as coreceptors for HIV are
a- CXCR5 and CCR6
b- CXCR4 and CCR5
c- CXCR7 and CCR5
d- CXCR6 and CCR4

nolan x soju
60- An example of a known oncogenic virus is:
a- Herpes zoster
b- HIV-2
c- Epstein-Barr virus
d- Proteus mirabilis

61- Which is NOT characteristic for both type II and type III
hypersensitivity reactions?
a- Involve attack by Tc cells
b- Involve IgG antibodies
c- Is likely to result in kidney damage
d- Can be evoked by exposure to a pathogen
62- Which cells die by apoptosis they do not induce inflammation
and damage to the surrounding tissue because they:
a- Do not release the cytoplasmic contents
b- Are not subsequently phagocytized
c- Migrate into the blood stream prior to death
d- Are rarely malignant or infected by virus

63- A delayed hypersensitivity reactions is characterized by:


a- Edema without a cellular infiltrate
b- An infiltrate composed of neutrophils
c- Differentiation of macrophages into epitheloid cells
d- An infiltrate composed on eosinophils

64- Which histomolecules may correlate with frequencies of certain


autoimmune diseases?
a- Forms of proteasome enzyme subunits
b- V,D,J gene fragments used to produce receptors specific for cancer-
associated peptides

nolan x soju
c- Histocompatibility antigens
d- CD4 or CD8 surface molecules

65- Hypersensitivity to ……… and hypersensitivity to nickel are both:


a- Mediated by IgE antibodies
b- Mediated by TH17 cytokines
c- Initiated by haptines
d- Initiated by Th2 cells

66- A transplant of tissue between individuals of different species is


called:
a- Autograft
b- Isograft
c- Allograft
d- Xenograft
67- Individuals with a genetic deficiency of C6 have:
a- Decreased resistance to viral infections
b- Increased hypersensitivity reactions
c- Increased frequency of cancer
d- Decreased resistance of Neisseria bacteraemia

68- Which of the following is the common cause of hypersensitivity


diseases?
a- Failure of lymphocyte maturation
b- Failure of self-tolerance
c- Treatment with corticosteroids
d- Diseminated cancer

69- Viruses cause tumours in “nude mice”(which do not have a


thymus because of genetic defect) but …………… normal mice. The BEST
interpretation is that:
a- Macrophages are required to reject polymavirus-induced tumours
b- Natural killers can reject polymavirus-induced tumours without help

nolan x soju
from T lymphocytes
c- T lymphocytes play an important role in rejection of polymavirus-
induced tumours
d- B lymphocytes play no role in rejection of polymavirus-induced tumours

70- Laboratory examination results IgG antibodies specific to


cytomegalovirus (CMV) in the serum of pregnant woman. What is your
conclusion?
a- A person was infected with CMV sometime during his/her life
b- A person has primary CMV infection
c- A person was not exposed to CMV
d- A person has an autoimmune disease

71- Which is the best method of reducing the effect of graft-versus-


host disease in a bone marrow?
a- Matching a complement component of donor and recipient
b- Administrating alpha interferons
c- Removing mature T cells from the graft
d- Removing mature B cells from the graft

72- Why do …….. treat patients by injecting them with an allergen


they are sensitive to?
a- If allergen is present in high enough levels, it maybe mistaken for a self
protein and ignored
b- Over a longer period of time there is a progressive decrease in IgE in the
serum
c- An injection into muscle will direct the immune responses from the
upper respiratory tract
d- This prevents the allergen from decreasing as it crosses the epithelial
barrier

73- Which category of hypersensitivity best describes the transfusion


reactions when a recipient has antibodies against donor erythrocytes?
a- Anaphylactic

nolan x soju
b- Cytotoxic
c- Immune complex
d- Delayed

74- B cells in bone marrow undergo negative selection if they:


a- Bind antigen
b- Do not bind antigen
c- Recognize class II MHC protein
d- Recognize class I MHC protein

75- Immunologic tolerance is defined as:


a- The removal of microbes expressing the antigen, so that the host can
tolerate the infection
b- Activation to only B cells, and not T cells, on exposure to an antigen
c- Unresponsiveness of the immune system to an antigen, which is induced
by previous exposure to the antigen
d- The production of memory B cells and T cells on primary exposure to an
antigen

76- A 59-year-old woman was treated with antibiotic for acute


respiratory infection, ten minutes after the ingestion patient
developed pruritus, generalized urticarial, wheezing and edema of the
larynx. This is most likely to be due to:
a- Systemic anaphylaxis
b- Serum sickness
c- Deposition of immune complex
d- Cytotoxic hypersensitivity

77- Which of the following statements is characteristic of contact


sensitivity?
a- The best therapy is administration of the antigens
b- It requires secretory component to work
c- Can be transferred with a serum from an allergic individual
d- It is an eczematous skin reaction at the site of contact with allergen

nolan x soju
78- Grafts between genetically identical individuals (e.g. identical
twins)
a- Are subject to acute rejection
b- Are subject to hyperactive rejection
c- Are not rejected, even without immunosuppression
d- Are not rejected if a kidney is grafted, but skin grafts are rejected

79- Chemically-induced tumours have tumour-associated


transplantation antigens that:
a- Are always the same for a given carcinogen
b- Are different for two tumours of different biological type, even if
induced by the same carcinogen
c- Are very strong antigens
d- Do not induce an immune response
80- Which one of the following mechanism maybe involved in the
clinical efficacy of immunotherapy with allerger
extracts(hyposynsetization)
a- Enhanced production of IgG, which binds allergen before it reaches mast
cells
b- Enhanced production of IgE, which binds IgG receptor before it reaches
antigen
c- Activation of mast cells that induces production of large amount of anti-
histamines
d- Local recruitment of eosinophils

81- The tuberculin skin test is performed by injecting 0.1 ml of


tuberculin purified protein derivative (PPD) into the inner surface of
the forearm. Histologically the reaction site would MOST probably
show:
a- Mast cells
b- Neutrophils
c- Th cell and macrophage
d- T and B cells

nolan x soju
82- Certain HLA genes are linked to autoimmune diseases because:
a- It maybe the result of a single genetic defect
b- It maybe the result of decreased helper T cells
c- It maybe the result of cross-reactivity between self antigen and
infectious agent
d- It carries no increased risk for a specific disease for those individuals
with the gene

83- Against which pathogen IgE antibodies can provide immunity?


a- Human hepatitis C virus
b- Shistosoma mansoni
c- Staphylococcus aureus
d- Salmonella species
84- Which antibodies are typically involved in both type II and type
III hypersensitivity?
a- IgA
b- IgG
c- IgD
d- IgE

85- The major mechanism of host resistance to M.Leprae is:


a- Humoral antibodies
b- Type IV hypersensitivity
c- High level of calcium in serum
d- Type I hypersensitivity

86- Di George syndrome results from a defect in:


a- Purine nucleoside phosphorylase
b- WASP
c- Thymic development
d- DNA repair

nolan x soju
87- Th2 cells are responsible for fighting against:
a- Tuberculosis bacteria
b- Flu virus
c- Tumour cells
d- Worms

88- The most likely clinical consequence of a genetic deficiency of


complement component C3 is increased:
a- Susceptibility to viral infections
b- Susceptibility to mycobacteria
c- Susceptibility to bacterial infections
d- Incidence of malignancy
89- Which of the following statements about type I hypersensitivity
is true?
a- It is also known as delayed type hypersensitivity
b- Cytokines involved in this type of reactions are IL-4, IL-13, IL-5
c- Cytokines involved in this type of reactions are IFN-gamma, IL-17, IL-7
d- It is independent from pharmacologic mediators

90- Which components of MICs transplant can initiate graft


rejection?
a- Histocompatibility antigen that are also found in the body of the
recipient
b- HLA DP,DR,DQ that are also found in the body of the recipient
c- Histocompatibility antigens that are not found in the body of the
recipient
d- Histocompatibility antigens that can induce donor-specific tolerance

91- Which of the following statement are correcty describe type 3


hypersensitivity?

nolan x soju
a- It occurs within 72 hours of re-exposure to antigen
b- It is not mediated by compliment-fixing IgG
c- Compliment is an important mediator
d- Desensitization is used for its control

92- Examples of primary T-cell deficiency disease includes:


a- Hereditary angioneurotic edema
b- Chronic granulation disease
c- Wiskott-Aldrich syndrome
d- Leukocyte adhesion deficiency

93- Reactions of M.tuberculosis and eggs of schistosomiasis is:


a- An IgG-mediated response
b- An IgE-mediated response
c- A T cell-mediated response
d- Antibody mediated response
94- Hyperacute responses differs from a first set response in that it
involves:
a- Tissue damage by activated CTLs
b- Antibodies, which mark the tissue for ADCC
c- Coordination of the response by DTH cells
d- Recombination of foreign MHC I and II

95- Removal of immune complex is correctly described by which of


the following statements?
a- It is independent of C3 for particular complexes
b- It occurs primarily in the spleen and liver
c- It is independent of size of immune complexes
d- It occurs primarily in the thymus and spleen

96- A diagnostic marker for tumours of the colon is:


a- Alpha-fetoprotein
b- Common acute hyphoblastic leukemia antigen (CALLA)

nolan x soju
c- EBV-related antigen
d- Carcinoembryonic antigen (CEA)

97- All of the following are true for immune haemolytic anemia
EXCEPT:
a- It is the clinical condition in which IgG antibodies bind to red cell surface
antigens
b- Is is the clinical condition in which IgM antibodies bind to red cell surface
antigens
c- Antibodies initiate red cell destruction via the complement system and
phagocytes
d- Red cell destruction occurs by T cytotoxic and NK cells.

98- Type II hypersensitivity differs from type III in that:


a- Mast cells trigger II, neutrophils trigger III
b- In II phagocytes attack cells with surface antigen antibody, in III they
engulf immune complexes.
c- Pencillin has not been identified as a trigger of 2 ,but it does trigger 3
d- 2 puts a person at risk for kidney damage,3 does not

99- Strongly immunogenic tumours appear:


a- In virtually all cancers
b- In immunosuppressed patients
c- Mostly in lymphoma and leukemia
d- In case liver cancer

100- Patients with Di George syndrome have:


a- Increased number of T cells
b- Selective IgA deficiency
c- Abnormal antigen-presenting cells
d- A decreased number of T cells

nolan x soju
1. Which experiment suggests that tumor cells express tumor-specific transplantation antigens?
a. When a syngeneic mouse is injected with the tumor cells, accepts the tumor and dies
b. When a syngeneic mouse is injected with the tumor cells, reject the tumor and survives
c. When an allogenic mouse is injected with the tumor cells, reject the tumor and survives
d. When an allogenic mouse is injected with the tumor cells, accepts the tumor and dies

1. A macrophage that is stimulated to initiate phagocytosis of a bacterial cell that has a capsule
needs to also involve…… antigen on the capsule surface that also attaches to which one of the
following on the surface of the macrophage?
a. An antigen on the macrophage surface that is identical to the antigen on the capsule surface
for which the…….
b. MHC Class II molecules
c. A receptor for a fragment of activated complement (the complement has been activated by
the binding of antibodies
d. And Fc receptor

2. Most autoimmune diseases are caused by


a. A T-cell defect
b. A constellation of genetic and environmental events
c. A known infectious organism
d. A single genetic effect

nolan x soju
3. Which is the best method of reducing the effect of graft-versus host disease in a bone marrow
patient?
a. Matching the complement components of donor and recipient
b. Administering alpha interferon
c. Removing mature T cells from the graft
d. Removing mature B cells from the graft

4. In the absence of antibodies, phagocytes are not able to respond effectively against
a. Capsulated bacteria
b. Bacterial cells to which the phagocytes firmly attach
c. Bacteria that are present in the body but outside the blood vessels
d. Protozoa

5. Your patient tests positive for the tuberculin antigen. You send him for a chest x-ray because
a. The tuberculin test is only presumptive, indicating that he has been exposed to tuberculin
b. He may have other lung infections
c. You are looking for fluid in his lungs due to inflammation caused by the bacillus
d. You are looking for granulomas with neutrophils inside

6. The final damage to vessels in immune complex mediated arthritis is due to


a. Histamine
b. NK cell
c. Lysosomal enzymes of polymorphonuclear leukocytes
d. Cytotoxic t cell

7. One isotype of antibody is uniquely designed to be highly resistant to degradation by


protease…… found in the highest concentration in the intestinal lumen, the fluids of the lungs,
the vagina……
a. Monomeric IgA
b. Dimeric IgA
c. IgG
d. IgM

8. Lupus erythematosus is
a. Free form an immune complex disease component
b. Associated with antibodies against nucleic acid
c. A disease of red and white cells

nolan x soju
d. An organ-specific autoimmune disease

9. Interferon-gamma
a. Is synthesized by macrophages
b. Is synthesized by erythrocytes
c. Is released as a consequence of antigen or mitogen induces activation of T-lymphocytes
(NOT SURE)
d. Specifically binds to the antigen that induces its release

10. Granulomatous hypersensitivity is correctly described by which of the following statements


a. It is not clinically important
b. I has not be encountered in schistosomiasis
c. It results from the presence of persistent microorganism within macrophages
d. It does not cause pathological effects in diseases associated with T cell-mediated immunity

11. If you wanted an adaptive immune response to lead to virus neutralization and eventual
phagocytosis (but not destruction of virus infected cells) which of the following types of cells
would have to be involved in some part of that complete response (that eventually results in
degradation of the virus)?
a. T-cytotoxic cells, macrophage and dendritic cell
b. Macrophage, B-lymphocytes and T-helper cells
c. T-helper cells, macrophage and dendritic cell
d. Macrophage, T-cytotoxic cells and B-Lymphocytes

12. Peptides from which of the following complex biomolecules would most likely eventually be
presented to?
a. Virus proteins in a cell that has been infected by a virus
b. Peptides produced by the degradation of an exotoxin during phagocytosis
c. Proteins, NOT degraded inside the proteasome
d. Proteins of microbes that resist phagocytosis

13. Patients with Chediak-Higashi syndrome have phagocytes that


a. Produce protein that interfere with chemotaxis
b. Lack C3b receptors that are needed for attachment to bacteria
c. Show a reduced ability of their lysosomes to fuse with phagosomes to release
d. Show defects for the engulfment of microbes

nolan x soju
14. Which of the following is a direct consequence of antibody binding to antigen?
a. Activation of B cells
b. Neutralization of the antigen
c. Activation of complement
d. Phagocytosis of the antigen

15. The biological properties of an antibody molecule not directly associated with specificity (e.g.,
complement…... phagocytes, imitating allergic reactions involving mast cells, etc.) involve the
participation of amino acids…. Locations? Hint: this location also determine the isotype of the
antibody
a. Fab
b. The constant potions of the light chains
c. The constant portions of the heavy chains
d. The domain that is at the amino-terminal end of the heavy chain

16. You have a patient who makes autoantibodies against his own red blood cells, leading to
hemolysis. Which one of the following is MOST likely to explain the hemolysis?
a. Perforins from cytotoxic T cells lyse the red cells
b. Neutrophils release proteases that lyse the red cells
c. Interleukin-2 binds to its receptor on the red cells, which results in lyses of the red blood
cells
d. Complement is activated and membrane attack complexes lyse the red cells

17. Individuals with a genetic deficiency of C6 have


a. Decreased hypersensitivity reactions
b. Increased frequency of cancer
c. Decreased resistance to Neisseria bacteremia
d. Decreased resistance to viral infections

18. Which of the following cells or molecules would have a specificity that is unique for only one of
several millions?
a. Complement
b. A dendritic cell
c. A macrophage
d. a B- lymphocyte

19. which one of the following types has the MOST influence on the antibody response carried out
by a B-lymphocyte?
a. A T-cytotoxic cell

nolan x soju
b. Cytokines released by a T-helper2 cell
c. Cytokines released by a T-helper1 cell
d. Phagocytes that are presenting peptides to T-helper cells

20. Certain HLA genes are linked to diseases such as ankylosing spondylitis. This linkage has all the
following……...
a. It may be result of closely linked genes
b. It may be result of cross-reactivity between self-antigen and infection agent
c. It carries no increased risk for a specific disease for those individuals with the genes
d. It may be result of a MHC molecule serving as an attachment site for an infectious agent

21. IgG binding to neutrophils is mediated by:


a. Fc-dependent cellular homing mechanism
b. Sensitization of Mast cells and basophils
c. Fc receptors specific for IgG
d. ICAM’s
22. A 5 years-old child was revaccinated by measles, mumps and rubella (MMR) vaccine. A blood
test performed……...presence of high affinity IgG antibodies specific for antigens of all above
mentioned viruses. Which of the following……...necessary for the production of the se high
affinity IgG antibodies?
a. CD40 Ligand
b. Activation induced deaminase (AID)
c. Class II MHC
d. C reactive protein (CRP)

23. The differentiation of bone marrow stem cells into either B-lymphocytes or T-lymphocytes is
often…. between the stem cell and cells that are permanent residents of the bone
marrow/thymus. Which ONE……initiating or controlling T-cell development in the thymus?
a. Stromal cells in the bone marrow that express cell adhesion molecules
b. Epithelial cells in the thymus that express CD4 or CD8
c. Epithelial cells in the bone marrow that express CD4 or CD8
d. Cortical epithelial cells in the thymus that express MHC Class 1 and MHC Class II

24. Chemically induced tumors have tumor-associated transplantation antigens that


a. Are always the same for a given carcinogen
b. Are different for two tumors of different histological type even if induced by the same
carcinogen

nolan x soju
c. Are very strong antigens
d. Do not induce an immune response
e. SPECIFIC TUMOUR ASSOCIATED ANTIGEN

25. What is the role of class II MHC proteins on donor cells in graft rejection?
a. They are the receptors for interleukin-2, which is produced by macrophages when they
attack the donor
b. They are recognized by helper T cells, which then activate cytotoxic T cells to kill the donor
cells
c. They induce the production of blocking antibodies that protect the graft
d. They induce IgE which mediate graft rejection

26. Antibody production by B-lymphocytes is initiated by direct contact between the B-lymphocyte
antigen receptor……...receptor is specific, and which one of the following additional
requirements?
a. Help from a T-helper cell that has been stimulated by MHC Class I-presented peptides
b. Contact between the Fc portion of the antibody and an Fc receptor on the surface of a T-
helper (TH2) lymphocyte
c. Help from a macrophage that is presenting peptides using MHC Class II molecules
d. Help from a T-helper 2 (TH2) lymphocyte

27. Which one of the following isotypes of antibodies is responsible for initiating the release of
larger amounts of histamine most often associated with allergic reactions? Hint: this isotype is
normally found in very low …………persons
a. IgM
b. IgD
c. IgG
d. IgE

28. Grafts between genetically identical individuals (i.e., identical twins)


a. Are subject to acute rejection
b. Are subject to hyperacute rejection
c. Are not rejected, even without immunosuppression
d. Are not rejected if a kidney is grafted, but skin grafts are rejected

29. Where do almost all mature, but naive, T-lymphocytes encounter the epitopes for which they
are specific during the printing….
a. In the HEV
b. In the lymph node

nolan x soju
c. In the thoracic duct (just before entering the blood circulation)
d. In the medulla of the thymus (involve antigen presentation by dendritic cells)

30. Which of the following statements BEST explains the relationship between inflammation of the
heart (carditis) and………beta-hemolytic streptococci?
a. Streptococcal antigens induce antibodies cross-reactive with heart tissue
b. Streptococci are polyclonal activators of B cells
c. Streptococcal antigens bind to IgE on the surface of heart tissue and histamine is released
d. Streptococci are ingested by neutrophils that release proteases that damage heart tissue

31. The following mechanism(s) may be involved in the clinical efficacy of injection therapy (hypo
sensitization)
a. Enhanced production of IgG, which binds allergen before it reaches mast cells
b. Enhanced production of IgE which binds IgE receptor before it reaches antigen
c. Activation of mast cells that induces production of a large amount of antihistamines
d. Increased local recruitment of eosinophils
32. Regarding anaphylactic (type I and immune complex (type III) hypersensitivities, which of the
following in the ………?
a. IgE is involved in both anaphylactic and immune complex hypersensitivities
b. Complement is involved in both anaphylactic and immune must take hypersensitivities
c. Less dose of antigen I typically needed to trigger anaphylactic reaction than an immune
complex reaction
d. Neutrophils play more important role in anaphylactic reactions than in immune complex
reactions

33. Failure to express class II HLA molecules on antigen-presenting cells causes


a. Th cell deficiency
b. Tc cell deficiency
c. B-cell deficiency
d. NK-cell deficiency

34. Which of the following statements as (are) characteristic of contact sensitivity?


a. The best therapy is administration of the antigen
b. Patch testing with the allergens is useless for diagnosis
c. Sensitization can be passively transferred with serum from an allergic individual
d. Some chemical acting as haptens induce sensitivity by covalently binding to hist proteins
acting as carriers

nolan x soju
35. Which ONE of the following is an important difference between innate immune responses and
adaptive immune responses?
a. Innate response (e.g., phagocytosis, complement activation) improves significantly with
each exposure to pathogens
b. Innate responses often can take 1-2 weeks to begin destroying infectious microorganisms,
while adaptive responses destroy…...immediately after the very first encounter with a
pathogen
c. An adaptive response does not improve significantly with repeated exposures to the same
non-self-antigen
d. Each cell involved in innate responses (e.g., phagocytes) can be stimulated to respond by
many different molecules while each cell involved in adaptive responses (e.g.,
lymphocytes) can be stimulated to respond by only one molecular structure (called an
antigen)

36. A T cell located at the epithelial barrier of the gut is a


a. Gamma/delta T cell
b. Helper T cell
c. Cytotoxic T cell
d. Natural killer cell

37. Type II hypersensitivity is correctly described by which of the following statements?


a. It is antibody independent
b. It is complement independent
c. I does not involve killer cells
d. It is antibody-dependent cytotoxic hypersensitive

38. AIDS is caused by a human retrovirus that kills


a. B lymphocytes
b. Lymphocyte stem cells
c. CD4-positive T lymphocytes
d. CD8-positive T lymphocytes

39. A patient is admitted with multiple bacterial infections and is found to have a complete absence
of C6. which complement……. would be mostly affected in such a patient:
a. Lysis of pathogen
b. Generation of neutrophil chemotactic factors
c. Generation of anaphylatoxins
d. Opsonization of bacteria

nolan x soju
40. Virus neutralization requires the involvement of which of the following?
a. Phagocytes
b. Activated complement
c. MHC Class I antigen-presenting molecules
d. Antibodies

41. Which of the following statements about type I hypersensitivity is true?


a. It is also known as delayed type hypersensitivity
b. It occurs when an IgE response is directed against pollen
c. It does not require IgE-sensitized mast cells
d. It is independent of pharmacologic mediators

42. The much more rapid appearance of antibodies during a secondary response, compared to the
response is……. the following?
a. A much improved presentation of peptide antigens to T-helper 1 cells by B-lymphocytes
during the secondary
b. A much higher number of B-lymphocytes at the beginning of the secondary response
specific for the antigen that stimulated……and secondary antibody responses
c. Improved binding of antibody molecules to Fc receptors on the surface of phagocytes, which
leads to a higher rate of peptide………. lymphocytes
d. Each B-lymphocyte participating in a secondary antibody response is specific for a very large
number of different antigens………participating in the primary response is specific for only
one antigen

43. Although each isotype has region that can be important in activating complement after an
antibody molecule……. only two isotypes that can usually activated complement
a. IgM
b. IgD
c. IgA (the monomeric form in the serum)
d. IgE

44. Which of the following clinical diseases is most likely to involve a reaction to a hapten in its
etiology?
a. Goodpasture’s syndrome
b. Hemolytic anemia after treatment with penicillin
c. Rheumatoid arthritis
d. Serum sickness

nolan x soju
45. Antibody mediated allergic reactions involve the attachment of one of the five isotypes to which
one of the following…………
a. Macrophage (via Fc receptors for the IgG isotype)
b. Mast cells
c. NK cells
d. Any phagocytic cell

46. Antibodies are able to provide protection from the effects of bacterial exotoxins do so in which
one of the following…...
a. Activate complement that, when activated, breaks down the toxin molecules into harmless
fragments
b. Enable macrophage to present the toxin antigens to T-lymphocytes
c. Bind to the toxin molecules and prevent the toxins from binding to body cell surfaces
d. Cause blood vessels located near the site of the toxin to become slightly leaky, thus allowing
cells in the bloodstream to enter the……...destroy the toxin molecules
47. The total number of combinations of all gene fragments to produce antigen-specific receptors
during somatic recombination actual number of different specificities that are actually
produced, this additional variable gene diversity is generated during………recombination in
which one of the following ways?
a. By looping out heavy chain isotype genes
b. By inserting nucleotides between variable portion gene fragment nucleotide sequences
before those fragments are ligated
c. By ADCC
d. By modifying the epitope-binding groove as the alpha chain of the receptor folds into its
final tertiary configuration

48. A typical IgG isotype antibody molecule does not consist of which of the following
a. Two identical light chains (- 220 amino acids in length)
b. Two identical heavy chains (- 440 amino acids in length)
c. A “J” (joining) chain
d. Two identical antigen-binding sites

49. The original specificity of any one B-lymphocyte is determined by which one of the following?
a. The type of CD molecules on that B-cell’s surface
b. The specificity of the antigen receptors on that B-cell’s surface
c. The specificity of MHC class I molecules

nolan x soju
d. The amino acid composition of two short tails extending from the tails end of the constant
portion of the B-cell’s antigen receptor that we embedded into the membrane surface of
that B-cell

50. Phagocytes can NOT successfully attach to the cell wall surface of bacterial cells in which of the
following ways?………lipopolysaccharides are present on the surface of the bacterial cells
a. Directly attaching to the bacterial cell wall lipopolysaccharides
b. Attaching to CD4 molecules on the bacterial cell surface using MHC Class II molecules (on
the phagocyte’s surface)
c. Using an antibody molecule that is specific for an antigen on the surface of the bacterial cell
d. Using a fragment resulting from complement activation by the bacterial cells

51. Which one of the following interactions between surface molecules must take place for a T
cytotoxic cell to be …………. virus-infected cell?
a. MHC Class I on virus infected cell and CD4 on T Cytotoxic cell
b. MHC Class I on T-cytotoxic cell and CD8 on virus-infected cell
c. MHC Class II on virus-infected cell and CD4 on T-cytotoxic cell
d. MHC Class I on virus-infected cell and CD8 on t-cytotoxic cell
52. poliomaviruses cause tumors in “nude mice” (which do not have a thymus because of a genetic
defect) but not in normal mice. The BEST interpretation is that
a. Macrophages are required to reject poliomavirus induced tumors
b. Natural killer cells can reject polyomavirus-induced tumors without help from T lymphocytes
c. T lymphocytes play an important role in rejection of poliomavirus-induced tumors
d. B lymphocytes play no role in rejection of poliomavirus-induces tumors

53. Which class of antibodies is responsible for neutralization a virus AND then contributes to
the……...
a. IgM
b. IgG
c. IgA
d. IgE

54. Complement that has been activated by pathogenic bacterial cells can not contribute (directly or
indirectly) to the destruction………. those pathogenic bacterial cells in which of the following
ways?
a. By destroying the bacterial cell membrane
b. By assisting phagocytes to attach to bacterial cells

nolan x soju
c. By attracting more phagocytes to the area of compliment activation
d. To solubilization of immune complexes

55. Although IgE antibodies can cause quite serious allergic reactions, this isotype of antibody does
appear to………immunity from which one of the following/
a. The smallpox virus
b. Parasitic worms
c. Tuberculosis bacteria that infect macrophages (and resist destruction once inside the cell)
d. Bacteria that cause pneumonia (most of these bacteria have a capsule)
Q. In case of C2 and C4 de ciencies
- C5 can s ll be cleaved by the classical pathway
- C3b will not be able to bind to bacteria
- C9 will not polymerize and lyse host cells
- C3b produc on will be reduced

Q. e ects of bacterial toxin could best be counteracted by


- an body binding and neutraliza on of the toxin
- an body opsoniza on and phagocytosis of toxin-producing bacteria
- B cell binding to toxin-producing bacteria
- cytotoxic T cell binding and lysis of toxins-producing bacteria

Q. which statement about Treg cells is FALSE?


- natural Treg cells di eren ate in the thymus
- iTreg cells di eren ate in the periphery
- Tregs secrete immunosuppressive cytokines

nolan x soju
- Tregs secrete proin ammatory cytokines

Q. a cell expressing CD3+, CD25+ and FoxP3+ is a


- gamma/delta T cell
- cytotoxic T cell
- Natural killer cell
- Regulatory T cell

1. Q. Lymphocytes con nually recirculate through peripheral lymphoid


ssue in order to
- Be killed before they cause autoimmunity
- E ciently encounter an gen
- Mature from stem cells into lymphocytes
- Phagocytose an gen and kill it
ti
ffi
ti
ti
ff
ti
ti
ff

ti
ti
ti
fl

ff

ti
fi
ti
ti

ti

2. Cytokines are
- Not able to increase blood vessel permeability
- Not able to induce an gen speci c response
- Able to induce an gen speci c response
- Able to ac vate complement system

3. Which of the following is NOT a characteris c of IgG?


- It crosses the placenta
- It neutralizes toxins
- It is major an body of saliva
- It ac vates complement

nolan x soju
4. Iden fy the part of the an body molecule in which the amino acids form
the most speci c bonds with an an gen
- The framework
- Th variable
- The hypervariable
- The constant domain
ti
ti
ti

ti

fi

ti

ti

ti

fi
fi
ti

ti

Pick the one BEST answer for each question by clicking on the letter of the correct
choice.

1. Central (primary) lymphoid organs

a. are efficient in exposing T cells to foreign antigen.


b. are the primary site of antibody synthesis and release.
c. filter blood and trap blood-borne antigens.
d. provide the microenvironment for maturation of T and B cells.
e. line the mucosal surfaces of the body for efficient antigen contact.

2. Hematopoietic stem cells are pluripotent, which means that they are

a. antigen-specific cells.
b. capable of developing into any blood cells.
c. committed to produce cells of a single lineage.
d. not self-renewing.
e. T and B lymphocytes of many different antigen specificities.

3. Lymphocytes continually recirculate through peripheral lymphoid tissue in


order to

nolan x soju
a. be killed before they cause autoimmunity.
b. efficiently encounter antigen.
c. mature from stem cells into lymphocytes.
d. phagocytose antigen and kill it.
e. go where no cell has gone before.

4. Peripheral lymphoid organs

a. are centrally located in the abdomen to protect their vital functions.


b. are designed to maximize contact between antigen and lymphocytes.
c. produce antigen-specific lymphocytes from stem cells in response to
antigen.
d. sequester antigen to minimize its damage to the body.
e. store large numbers of activated effector cells for a rapid response to
antigen.

5. The PRIMARY purpose of the adaptive immune system is to


a. block all pathogens from entering the body.
b. cure allergic reactions.
c. kill tumor cells.
d. protect from disease upon re-infection with a specific pathogen.
e. reject foreign transplants.

6. Rapid but non-antigen specific immune responses are produced by the

a. adaptive immune system.


b. innate immune system.
c. leukocytes.
d. lymphatic system.
e. memory response.

7. Vaccination protects us from infectious disease by generating memory

a. antigen.
b. lymphocytes.
c. macrophages.
d. PMNs.

nolan x soju
e. stem cells.

8. Which situation below describes an example of innate immunity?

a. antibody production by plasma cells.


b. antigen removal by cilia in the respiratory tract.
c. complement activation by antibody bound to the surface of a bacterium.
d. memory response to influenza virus
e. recognition and killing of virus-infected cells by cytotoxic T cells.

9. The antigen specificity of an adaptive immune response is due to

a. activation of antigen-specific lymphocytes.


b. folding of antibody to fit the pathogen.
c. lysis of only certain pathogens by neutrophils.
d. phagocytosis of only certain pathogens by macrophages.
e. production of cytokines by antigen-specific macrophages
10. Clonal selection

a. begins with inflammation.


b. occurs for all leukocytes.
c. occurs in response to self antigens.
d. results in innate immunity.
e. results in proliferation of antigen-specific lymphocytes.

11. Cytokines are NOT

a. able to induce increased blood vessel permeability.


b. antigen-specific.
c. made in response to bacterial antigens.
d. signals from one cell that affects the behavior of another cell.
e. secreted by macrophages.

12. A fundamental difference between the antigen receptors on B cells (BCR)


and on T cells (TCR) is their

a. different requirements for antigen presentation.

nolan x soju
b. function following antigen binding.
c. heterogeneity from one lymphocyte to the next.
d. heterogeneity on each lymphocyte.
e. membrane location.

13. Genes for immunoglobulins (antibodies) are unlike other human genes in
that

a. antibody genes are composed of introns and exons.


b. DNA for antibody molecules is inherited from only one parent.
c. gene segments must be spliced together to make each unique antibody
molecule.
d. several exons encode each antibody molecule.
e. none of the above is true.

14. Humoral immunity can be acquired passively by

a. catching a virus from a friend by shaking hands.


b. receiving a vaccine of influenza virus grown in eggs.
c. receiving serum from someone who has recovered from an infection.
d. receiving leukocytes from an immune family member.
e. sharing a soda with someone who has a cold.

15. Inflammation does NOT involve

a. cytokine production by macrophages.


b. migration of leukocytes out of the circulation.
c. pain.
d. secretion of antibodies.
e. swelling at the site of infection.

16. Innate immune responses are most effective against

a. antigens resembling self antigens.


b. common antigens on bacteria.
c. genetically engineered antigens.
d. viruses.
e. viruses that have previously caused infection.

nolan x soju
17. Lymphocytes acquire their antigen specificity

a. as they enter the tissues from the circulation.


b. before they encounter antigen.
c. depending on which antigens are present.
d. from contact with self antigen.
e. in the secondary lymphoid organs.

18. A secondary immune response is NOT

a. faster than a primary response.


b. larger than a primary response.
c. longer lasting than a primary response.
d. more likely to result in increased adaptive immunity than a primary
response.
e. preceded by a longer lag period than a primary response.

19. Antibody effector functions include all of the following EXCEPT


a. activating complement on bacterial surfaces to promote phagocytosis by
neutrophils.
b. binding extracellular viruses to block their entry into host cells.
c. binding intracellular viruses to initiate cytotoxicity.
d. blocking uptake of bacterial toxins by host cells.
e. coating bacteria to promote their phagocytosis by neutrophils.

20. Effector functions of complement include all of the following EXCEPT

a. attracting phagocytes to the site of infection.


b. facilitating phagocytosis of complement-coated bacteria.
c. increasing blood vessel permeability to plasma proteins.
d. lysing bacterial cells.
e. presenting antigen to B cells.

21. Jenner observed that milkmaids who were infected with cowpox were later
immune to smallpox infections. This is an example of a(n)

a. acquired immunity of barrier skin cells.


b. active immunization with a non-related organism that causes similar
symptoms.

nolan x soju
c. innate immunity of milkmaids to smallpox.
d. memory response to a cross-reactive antigen.
e. passive immunization from contact with cow's milk antibodies.

22. Macrophages generally kill the bacteria they phagocytose by fusing


lysosomes containing digestive enzymes with the phagocytic vesicle. In the
case of pathogens which block this fusion, pathogen killing can still be
achieved through the effector function of

a. B cells.
b. complement.
c. cytotoxic T cells.
d. opsonizing antibody.
e. Th1 cells.

23. Phagocytosis
a. can be stimulated by antigen binding to complement or antibody.
b. is an antigen-specific process.
c. must be preceded by antigen processing.
d. rids the body of virus-infected cells.
e. only occurs after plasma cells begin secreting antibody.

24. Several friends who went on a picnic together developed vomiting and
diarrhea from eating potato salad contaminated with Staphylococcus
aureus enterotoxin. Effects of the toxin could best be counteracted by

a. antibody binding and neutralization of the toxin.


b. antibody opsonization and phagocytosis of S. aureus.
c. antibody opsonization and phagocytosis of the toxin.
d. B cell binding to S. aureus.
e. cytotoxic T cell binding and lysis of S. aureus.

25. Which of the following statements is FALSE?

a. An example of passive humoral immunity is treatment with horse anti-snake


venin.

nolan x soju
b. Antigen recognized by helper T cells must be associated with Class II MHC
molecules on the surface of professional APC .
c. Each lymphocyte has many antigen binding receptors, each receptor
capable of binding the same antigen.
d. Recognition and killing of virus-infected cells by cytotoxic T cells is an
example of adaptive immunity.
e. The innate immune system does not deal with endogenous antigen.

1. The ability of an antigen to induce an immune response does NOT depend


on the antigen's

a. ability to enter the thyroid.


b. degree of aggregation.
c. dose.
d. size.
e. usual presence in the body.
2. Alum is an effective adjuvant because it

a. disaggregates the antigen.


b. is immunogenic for stem cells
c. is immunogenic for T cells.
d. slows the release of antigen.
e. transports antigen into the cytoplasm of antigen-presenting cells.

3. Antibody cross-reactivity is demonstrated by antibody binding to

a. a cell surface marker.


b. a hapten.
c. a hapten-carrier complex.
d. an antigen that is structurally similar to the immunogen
e. the immunogen.

4. The antibiotic penicillin is a small molecule that does not induce antibody
formation. However, penicillin binds to serum proteins and forms a complex
that in some people induces antibody formation resulting in an allergic
reaction. Penicillin is therefore

a. an antigen.

nolan x soju
b. a hapten.
c. an immunogen.
d. both an antigen and a hapten.
e. both an antigen and an immunogen.

5. Antigen entering the body in a subcutaneous injection activates its specific


lymphocytes in the

a. blood circulation.
b. draining lymph nodes.
c. MALT.
d. skin.
e. spleen.

6. To detect a humoral immune response to influenza virus, you would


measure

a. cytotoxicity of virus-infected cells in the lung.


b. cytotoxicity of virus-infected cells in tissue culture.
c. dividing T cells in the draining lymph nodes.
d. plasma cytokine levels.
e. serum antibody titer.

7. During the lag period between antigen contact and detection of adaptive
immunity,

a. antigen is hidden from the immune system in macrophages.


b. cellular immunity can be detected but antibodies cannot.
c. innate immune effectors are eliminating antigen.
d. innate immunity blocks the activation of adaptive immune effector cells.
e. new B and T cells with the appropriate antigen specificity must be produced
in the bone marrow.

8. To elicit the best antibodies to mouse MHC I, you should inject it into

a. a goat.
b. a mouse of the same genetic background (strain).
c. a mouse of a different strain.
d. a rat.
e. the mouse you isolated it from.

9. For specific antigen recognition by T cells,

nolan x soju
a. antigen is bound by a T cell membrane antibody.
b. denaturation of antigen does not reduce epitope recognition.
c. MHC molecules are not required.
d. soluble antigen is bound directly without processing.
e. antigen exposure during T cell maturation is required.

10. The immune response to a booster vaccine is called a(n)

a. cellular response.
b. humoral response.
c. innate response.
d. primary response.
e. secondary response.

11. Immunogenicity

a. depends on the ability of the native antigen to be presented by MHC.


b. is usually a property of "self" antigens such as eye tissue.
c. is not a property of antibodies.
d. is not a property of haptens.
e. only applies to antigens that are composed of proteins.

12. Lymphocytes are activated by antigen in the

a. blood stream.
b. bone marrow.
c. liver.
d. lymph nodes.
e. skin.

13. A molecule that can be covalently linked to a non-immunogenic antigen to


make it an immunogen is called a(n)

a. adjuvant.
b. carrier.
c. hapten.
d. mitogen.
e. superantigen.

14. A polyclonal antibody response

nolan x soju
a. is not antigen-specific.
b. is produced only in response to polymeric antigens.
c. is produced by several B cells recognizing different epitopes on the same
antigen.
d. occurs during the lag phase of the immune response.
e. violates clonal selection.

15. Very low doses of antigen may induce

a. a secondary response.
b. hypersensitivity.
c. immunological ignorance.
d. low zone tolerance.
e. low zone immunity.

16. A virus vaccine that can activate cytotoxic T cells MUST contain

a. a high dose of virus particles.


b. an adjuvant to stimulate T cell division.
c. foreign MHC.
d. live virus.
e. virus peptides.

17. Which statement about antigen epitopes is FALSE?

a. An epitope may be shared by two different antigens.


b. A protein molecule usually contains multiple epitopes.
c. B cells bind only processed antigen epitopes.
d. Epitopes may be linear or assembled.
e. Some epitopes are more immunogenic than others

18. CD antigens

a. allow leukocytes to recognize antigen.


b. are each expressed on only one cell type.
c. are expressed on immune cells by immunologists to "mark" them for
separation.
d. are found only on leukocytes.
e. function as receptors for cytokine and CAMs.

19. A patient desperately needs a bone marrow transplant, and a perfect


match cannot be found. The rejection response in unmatched marrow is

nolan x soju
primarily due to the presence of mature T cells that recognize the recipient's
cells as foreign. To minimize this rejection response, the marrow can be
treated before transfusion into the recipient with complement plus antibody to
human

a. CD3.
b. CD4.
c. CD8.
d. CD28.
e. CD154.

20. Antibody to membrane receptors sometimes inhibits receptor function and


sometimes mimics the action of the normal receptor ligand. (For example,
some antibodies to insulin receptor block the action of insulin and some mimic
the action of insulin.) An antibody which should NOT either block or stimulate
B cell function would be anti-

a. CD21.
b. CD56.
c. CD80.
d. Ig.
e.  chain.

1. Cytokines may exhibit __________ action, signaling the cells that produce
them.

a. antagonistic
b. autocrine
c. endocrine
d. paracrine.
e. synergistic

2. Cytokines are NOT

a. antigen specific.
b. capable of activating more than one cell type.
c. made by lymphocytes.
d. small protein molecules.

nolan x soju
e. synthesized de novo in response to antigen or other cytokines.

3. Several cytokines may have the same effect on the cells they bind. This is
an example of

a. a cascade.
b. antagonism.
c. pleiotropism.
d. redundancy.
e. synergy.

4. Characterization of cytokine activities is NOT made more difficult by their

a. gene structure.
b. pleiotropism.
c. redundancy.
d. secretion close to target cell membranes.
e. short half-lives.

5. Interferons
a. activate B cells to make virus-specific antibodies.
b. are Th2 cytokines.
c. are virus proteins that interfere with activation of cytotoxic T cells.
d. block virus infection of host cells.
e. inhibit virus replication by infected cells.

6. A cytokine can do all of the following EXCEPT

a. bind to receptors which do not share cytokine-binding subunits.


b. bind to its specific receptor on the same cell that produced it.
c. bind to receptor antagonists produced by pathogenic viruses.
d. compete with other cytokines whose receptors share signal-transducing
subunits
e. upregulate (increase) synthesis of high affinity subunits for its receptor.

7. Members of a cytokine receptor family

a. all bind the same cytokines.


b. are grouped together because they share antigen specificity
c. are often found on the same cells
d. are similar in protein structure and sometimes in regions of amino acid
sequence.

nolan x soju
e. are specific for cytokines produced by a single cell type

8. The ability of a cytokine to change gene expression in the target cell is


influenced by all of the following EXCEPT

a. presence of high-affinity receptors on the target cell.


b. presence of soluble cytokine receptors.
c. proximity of the producing and target cells.
d. rate of transport of cytokine-receptor complexes into the cytoplasm.
e. simultaneous production of another cytokine whose receptor uses the same
signal transducing subunit.

9. Cytokines are NOT

a. able to inhibit the function of other cytokines.


b. able to stimulate the synthesis of other cytokines.
c. produced by more than one cell type.
d. small protein molecules.
e. stored in the cell for quick release.
10. The IL-2R subfamily consists of receptors for IL-2, IL-4, IL-7, IL-9, and IL-
15. This group of cytokine receptors

a. bind all five cytokines to promote synergistic action on target cells.


b. bind cytokines which are produced by the same cell.
c. each has a unique high affinity cytokine-specific  chain.
d. shift the immune response towards cellular immunity.
e. each has a unique signal-transducing  chain.

11. An antagonist for cytokine X may NOT be

a. cytokine A competing for a shared receptor subunit.


b. cytokine B which acts synergistically with cytokine X.
c. cytokine C which inhibits the activation of the cell that produces cytokine X.
d. made by microorganisms.
e. soluble cytokine X receptors.

12. A knock-out mouse for a particular cytokine allows immunologists to


characterize cytokine function

a. by doing a dose-response study with competing cytokines.


b. in the absence of all other cytokines.

nolan x soju
c. on all cell types simultaneously.
d. under controlled conditions of local cytokine concentrations.
e. with defined cell populations.

13. Activated Tc can regulate immune responses by signaling activated


lymphocytes to undergo

a. apoptosis.
b. clonal deletion.
c. clonal proliferation.
d. cytotoxicity.
e. somatic hypermutation.

1. Complement
a. is a group of active proteolytic enzymes found in serum.
b. is secreted by macrophages and hepatocytes in response to antigen
binding.
c. participates in both innate and adaptive immune responses.
d. prevents lysis of virus-infected cells.
e. All of the above statements about complement are true.

2. Complement is involved in all of the following except

a. attraction of neutrophils to an infection site.


b. increased presence of serum proteins in the infected tissues.
c. lysis of bacteria in the absence of specific antibodies.
d. opsonization of microorganisms for phagocytosis.
e. sensitization of T cells to antigen

3. Complement is

a. activated by binding to specific complement receptors.


b. antigen-specific.
c. a potent promoter of virus entry into host cells.
d. a series of intracellular proteins which work with antibody to eliminate
endogenous antigen.

nolan x soju
e. present in the circulation in an inactive form.

4. The alternative pathway of complement activation

a. causes tissue damage in the absence of C1INH.


b. occurs after the classic pathway is activated.
c. occurs only if the classical pathway is ineffective in pathogen clearance.
d. requires C3.
e. requires C4.

5. If a person is born without C2 and C4,

a. C5 can still be cleaved by the classical pathway.


b. C3b will not be able to bind to bacteria.
c. C9 will polymerize inappropriately and lyse host cells.
d. the classical pathway will be changed into the alternative pathway.
e. the amount of C3b produced during bacterial infections will be reduced.

6. Which of the following are least sensitive to complement-mediated lysis?


a. Enveloped viruses
b. Erythrocytes
c. Gram negative bacteria
d. Gram positive bacteria
e. Leukocytes

7. In the membrane attack phase of the classical complement pathway, the


role of C5b is to

a. activate the C5 convertase activity.


b. attract neutrophils to lyse the pathogen.
c. initiate formation of the MAC.
d. polymerize into a membrane-spanning channel.
e. All of these are activities of C5b.

8. Complement receptors (CR)

a. activate complement on the surface of pathogens.


b. bind only activated complement proteins.
c. inhibit complement activation on the surface of host cells.
d. on erythrocytes remove immune complexes from the circulation.
e. on macrophages signal host cells to make opsonins.

nolan x soju
9. As complement is activated by complexes of antibody-coated bacteria,
bystander lysis of nearby host cells is prevented by

a. a long-lived thioester bond on active complement proteins.


b. covalent attachment of all active complement proteins to the pathogen
surface.
c. plasma proteins that inactivate the anaphylatoxins.
d. proteins on host cell membranes that inhibit MAC formation.
e. the slow catalytic rates of complement proteases.

10. Complement activity is restricted by all of the following EXCEPT

a. dissociation of C3 and C5 convertases.


b. Gram positive cell walls that are resistant to MAC polymerization.
c. host cell plasma proteins that inactivate C3a, C4a, and C5a activity.
d. LPS in the outer membrane of Gram negative bacteria that inactivates C3b.
e. proteolytic cleavage of complement proteins into smaller fragments.

11. A deficiency in complement proteins or in their regulators can result in


a. blood in the urine from erythrocyte lysis.
b. decreased levels of certain complement proteins in the circulation.
c. immune complex disease.
d. increased numbers of infections.
e. All of the above can result from complement deficiencies.

1. Phagocytosis must be preceded by

a. antigen binding to the phagocyte.


b. chemotaxis.
c. extravasation.
d. integrin binding to Ig superfamily CAMs.
e. oxidative burst.

2. Phagocytes bind antigen using receptors for

a. C5a.
b. chemokines.

nolan x soju
c. glucose.
d. LPS.
e. selectins.

3. Pathogens engulfed by macrophages

a. are completely degraded by hydrolytic enzymes into their component amino


acids and sugars.
b. are degraded to small peptides and carbohydrates which are presented on
Class I MHC to Tc.
c. may survive and replicate in the macrophage phagocytic vesicles.
d. stimulate macrophages to adhere to B cells.
e. stimulate vascular endothelium to upregulate selectin expression..

4. An inflammatory response

a. is characterized by a decrease in vascular permeability.


b. is stimulated by cytokines produced by neutrophils.
c. occurs only during a secondary response.
d. recruits phagocytes to the infection site.
e. usually lasts for many weeks to ensure antigen is completely removed

5. Natural Killer cells

a. are stimulated to kill infected host cells via carbohydrate-binding receptors.


b. kill normal host cells with high levels of membrane MHC Class I.
c. kill virus-infected cells when the virus is acquired naturally but not by
immunization.
d . recognize virus-infected cells by the presence of viral peptide on MHC
Class II.
e. secrete the complement MAC to lyse virus-infected cells.

6. Interferons and  do NOT

a. activate NK cells to kill virus-infected cells.


b. get synthesized by virus-infected cells in response to infection.
c. induce macrophages to increase expression of Class II MHC.
d. inhibit virus replication in infected cells.
e. stimulate expression of molecules required for Class I MHC presentation of
viral proteins.

nolan x soju
7. Immune system cell adhesion molecules do NOT

a. allow macrophages to leave the circulation.


b. allow T cells to home specifically to peripheral or mucosal lymphoid tissue.
c. attract leukocytes to an infection site.
d. help cytotoxic T cells to bind to their targets.
e. signal neutrophils that they have arrived at an infection site.

8. Early induced immune responses are like adaptive immunity in that they

a. are antigen-specific
b. demonstrate immune memory.
c. involve macrophages and complement.
d. involve T and B lymphocytes
e. use pre-synthesized proteins which can be released quickly upon cell
activation.

9. Selectins
a. are present on both leukocytes and vascular endothelial cells.
b. bind Ig-like vascular addressins.
c. include ICAM, VCAM, and MAdCAM.
d. select antigen-specific lymphocytes to extravasate into the infection site.
e. select antigen-specific macrophages to extravasate into the infection site.

10. Lymphocyte recirculation

a. activates inflammatory cytokines to promote antigen presentation to T cells.


b. allows B cells to go to the site of infection to produce antibody.
c. circulates lymphokines efficiently throughout the body.
d. occurs for both naïve and effector lymphocytes
e. only occurs during an infection.

11. Phagocytes kill bacteria using all of the following EXCEPT

a. H2O2.
b. hydrolytic enzymes.
c. low pH
d. lysozyme.
e. strong reducing agents.

nolan x soju
12. For a circulating neutrophil to reach the site of inflammation, it must bind
to blood vessel endothelial cell and then pass between the endothelial cells in
a process called

a. addressinazition.
b. chemotaxis.
c. extravasation.
d. marginalization.
e. opsonization.

13. Macrophages are attracted to the site of infection by all of the following
EXCEPT

a. bacterial peptides.
b. chemokines.
c. C5a.
d. IL-8.
e. MAdCAM.
14. Inflammatory cytokines produced by macrophages activate all of the
following EXCEPT

a. B cells to secrete acute phase proteins.


b. integrin on leukocytes to bind more strongly to vascular CAMs.
c. neutrophils to be more cytotoxic.
d. NK cells to kill virus-infected cells.
e. vascular endothelium to increase expression of CAMs.

1. An antibody Fab contains

a. complementarity determining regions.


b. H and L chain variable regions.
c. one antigen binding region.
d. one H-L interchain disulfide bond.
e. all of the above.

2. Myeloma proteins are

nolan x soju
a. abnormally formed antibodies secreted from cancerous plasma cells.
b. cancerous plasma cells that divide without requiring antigen activation.
c. cell lines that secrete specific antibodies for a short time, then die.
d. homogeneous antibody molecules secreted by plasma cell tumors.
e. protein signaling molecules that make a plasma cell become a multiple
myeloma.

3. The regions of the antibody molecule which contribute MOST to the affinity
of the antibody for antigen are the

a. CDR.
b. Fab regions.
c. Fc regions.
d. framework regions.
e. hinge regions.

4. Antibody Fc fragments contain

a. antigen-binding sites.
b. CDR.
c. complement-binding sites.
d. framework residues.
e. light chain variable domains.

5. The immunoglobulin isotype is determined by the

a. antigen specificity.
b. H chain constant region.
c. L chain variable region.
d. number of antigen-binding sites.
e. number of VH domains.

6. Which statement about antigen epitopes is FALSE?

a. An epitope may be shared by two different antigens.


b. A protein molecule usually contains multiple epitopes.
c. B cells bind only processed antigen epitopes.
d. Epitopes may be linear (composed of sequential amino acids) or
assembled by protein folding from amino acids far apart in the protein primary
amino acid sequence.
e. Some epitopes are more immunogenic than others.

nolan x soju
7. An example of an antigen epitope from an infectious organism would be

a. a bacterial endotoxin (LPS) molecule.


b. a fungal cell wall protein.
c. a peptide on the surface of a virus capsid protein.
d. a whole virus.
e. All of the above are antigen epitopes.

8. Antibody affinity for antigen depends on

a. the antibody isotype.


b. the complementary shape and charge of each antibody V region for its
antigen epitope.
c. the number of Fab regions in each antibody molecule.
d. whether the antibody is in the serum or on the cell surface.
e. whether the light chains are kappa or lambda.

9. Avidity
a. is a pathogenic agent, causing a very serious disease.
b. occurs when the ratio of antibody to antigen is optimal.
c. refers to the strength of interactions between a multivalent antibody and a
multivalent antigen.
d. results in a loss of antibody reactivity.
e. results in cross-reactivity when antibody binds two different antigens.

10. A colleague sends you an antibody to polio virus capsid protein. You
perform equilibrium dialysis on the antibody to measure its affinity. Plotting r/c
versus r gives you a curved line with K= 2.5 X 108 L/mole and an r intercept of
4. From these results, you conclude that the antibody is probably

a. a cross-reactive antibody.
b. a monoclonal anti-polio virus antibody.
c. a polyclonal IgG antibody.
d. IgA anti-polio virus.
e. not specific for polio virus.

11. Allotypic determinants are

a. constant region determinants that distinguish each Ig class and subclass


within a species.

nolan x soju
b. expressed only from the paternal chromosome.
c. generated by the conformation of antigen-specific VH and VL sequences.
d. Not immunogenic in individuals who do not have that allotype.
e. amino acid differences encoded by different alleles for the same H or L
chain locus.

12. Which of the following is NOT a characteristic of IgG?

a. It contains 2 and 2 L chains


b. It crosses the placenta.
c. It is the predominant immunoglobulin in blood, lymph, and peritoneal fluid.
d. It is the largest of all the Igs.
e. Its L chains are either  or.

13. Human serum IgA is isolated and injected into a rabbit. The rabbit anti-IgA
antibodies will react against all of the following EXCEPT human

a. chain.
b. IgG.
c. chain.
d chain.
e. secretory component.

14. You have purified some Fab from an IgG myeloma protein. Under
appropriate conditions, you could use this Fab to generate antibodies to

a. both  and chain.


b. chain hinge region.
c. J chain.
d. chain allotypic determinants.
e. the idiotype of this myeloma.

15. The Ig isotype which would be most important for neutralizing polio virus
before it could infect intestinal cells would be

a. secretory IgA.
b. serum IgA.
c. serum IgD.
d. serum IgG.
e. membrane IgM.

16. Which of the following changes to a serum IgM antibody molecule would

nolan x soju
definitely DECREASE its avidity?

a. Increase noncovalent antigen-antibody interactions in the CDR.


b. Remove the secretory component.
c. Replace the Fc portion of the mu chains with the Fc portion of alpha
chains.
d. Replace VH and VL framework regions with those from a different antibody.
e. Use limited enzyme digestion to make Fab fragments.

17. IgA can be secreted from the body because it

a. binds poly-Ig receptor on mucosal epithelial cells.


b. has a specialized H chain called secretory chain.
c. has a special secretory idiotype.
d. is small enough to pass between mucosal epithelial cells and leave the
body.
e. is synthesized by mucosal epithelial cells and secreted directly into the
intestinal lumen.
18. The ability to make antibody with the same antigen specificity but different
Fc regions

a. causes allelic exclusion of Ig molecules.


b. does not occur against bacterial antigens.
c. improves the antigen binding specificity of an Ig molecule.
d. increases the effector functions of Ig molecules.
e. requires clonal elimination.

19. Allergy symptoms are produced when antigen binds to IgE on FcR on

a. A cells.
b. macrophages.
c. mast cells.
d. neutrophils.
e. Th1 cells.

20. One amino acid difference in the Fc region of different human chains is
the epitope recognized by anti-

a. allotype.
b. idiotype.

nolan x soju
c. isotype.
d. IgG.
e.  chain.

1. Genes for immunoglobulins are unlike other human genes in that

a. each polypeptide chain is encoded by several exons.


b. Ig genes are composed of introns and exons
c. somatic recombination occurs before mRNA is transcribed
d. there is less Ig genetic material in mature B cells than in other somatic
cells
e. both c and d are true.

2. The gene segments needed to encode the variable region of a chain are
a. one J plus one D.
b. one J plus one C.
c. one V plus one D.
d. one V plus one J.
e. one V plus one J plus one D.

3. Pseudogenes are DNA sequences which look very similar to functional


genes except for the presence of a(n)

a. intron.
b. leader sequence.
c. promoter codon.
d. signal sequence.
e. stop codon.

4. Combinatorial diversity says that by random combination of 40 functional


V segments with five J segments, the number of possible different  chains
that could be made are

a. 40.
b. 45.
c. 70.

nolan x soju
d. 200.
e. 1200.

5. Which does NOT contribute to Ig antigen-binding diversity

a. Any L chain can combine with any H chain to form a functional antibody.
b. Any V can be joined to any J to encode the light chain V region.
c. Many CH genes are present in the germline DNA.
d. Random numbers of N nucleotides can be added during somatic
recombination.
e. VJL and VDJH joining is imprecise.

6. The proper joining of one VL to one JL is regulated by

a. heptamer and nonamer sequences.


b. leader sequences.
c. P-nucleotide addition sites.
d. 12 and 23 nucleotide spacers between heptamer and nonamer sequences.
e. TdT binding site for DNA.
7. Since each B cell productively rearranges a single H and L chain allele, it
exhibits

a. affinity.
b. allelic exclusion
c. antibody restriction.
d. antigen-binding diversity.
e. cross-reactivity

8. Primary mRNA for H chain encodes

a. one VH, one DH, and one JH segment.


b. one VH, one DH, and multiple JH segments.
c. multiple VH, one DH, and one JH segments.
d. multiple VH, one DH, and multiple JH segments.
e. multiple VH, DH, and JH segments.

9. Somatic recombination occurs

a. in the bone marrow stem cell.


b. in the progenitor cell as it is becoming a B cell.
c. in the mature B cell following antigen contact.

nolan x soju
d. in the plasma cell after antigen contact.
e. in the plasma cell after antibody secretion.

10. Junctional diversity affects primarily the amino acid sequence in

a. all CDR equally.


b. CDR1.
c. CDR2.
d. CDR3.
e. FR3.

11. Isotype switching

a. changes the leader sequence exon so the antibody is secreted.


b. improves the antigen binding specificity of an Ig molecule.
c. increases the affinity of antibodies in a process called affinity maturation.
d. increases the functional diversity of Ig molecules.
e. occurs randomly between switch regions.
12. Isotype switching resembles somatic recombination because both
processes

a. are catalyzed by the products of RAG1 and RAG2


b. are regulated by helper T cell cytokines.
c. can result in stop codons in coding sequences.
d. occur in developing B cells in the bone marrow.
e. result in the irreversible loss of DNA from the B cell.

13. Alternative mRNA splicing

a. allows the B cell to improve its antigen-binding fit after antigen contact.
b. allows the B cell to make membrane IgM from the mature mRNA for
secreted IgD.
c. can be used for the simultaneous production of any two Ig isotypes.
d. is a process by which a B cell can simultaneously
synthesize  and  chains.
e. occurs in response to T cell cytokines.

14. Because of the order of the CH gene segments (C, C, C3, C1,
pseudogene C, C1, C2, C4, C, and C2), a human B cell which undergoes
isotype switching from IgM to IgG1 can never in the future secrete

nolan x soju
a. IgA.
b. IgE.
c. IgG2.
d. IgG3.
e. IgG4.

15. Isotype switching is always productive because

a. B cells produce all isotypes simultaneously.


b. isotype switching does not involve recombination of DNA gene segments.
c. no DNA is deleted from the chromosome in isotype switching.
d. no effector diversity results from isotype switching.
e. recombination between switch sites occurs in introns so it cannot introduce
stop codons into coding regions.

16. Somatic hypermutation does NOT

a. occur by somatic recombination.


b. occur during B cell proliferation.
c. occur in the B cell following antigen stimulation.
d. result in increased affinity of antibodies secreted later in immune
responses.
e. result in the death of some B cells which no longer bind antigen.

1. Which of the following is NOT True about TCR?

a. All TCRs on a particular T cell have identical idiotypes.


b. CDR3 of TCR has the most sequence variability from molecule to
molecule.
c. TCR has binding sites for both antigen and self MHC.
d. TCR is a disulfide-bonded heterodimer.
e. The or  isotype of TCR determines the biological function of its
secreted form.

2. The antigen-binding region of TCR is formed by the folding of

a. V and V chains.

nolan x soju
b. V, V, and CD3 chains.
c. V and V2-microglobulin chains.
d. V and V chains.
e. VL and VH chains.

3. Which of the following properties are NOT shared by TCR and BCR?

a. Antigen-binding avidity is increased by the presence of two antigen binding


regions on each receptor.
b. Antigen-binding diversity is generated through gene rearrangement.
c. Folding of protein domains is maintained by intrachain disulfide bonds.
d. Membrane expression and lymphocyte activation by antigen require
receptors to be associated with signal transduction molecules.
e. Receptor antigen-binding sites are formed from two polypeptide chains.

4. TCR most closely resembles

a. Class I MHC.
b. Class II MHC.
c. Fab region of immunoglobulin.
d. Fc region of immunoglobulin.
e. light chain of immunoglobulin.

5. Rearrangement of both TCR and BCR gene segments does NOT

a. generate diversity of antigen binding by recombination of a large pool of


germline V, D, and J segments.
b. lead to CDR3 being the most hypervariable region in the receptor chains.
c. require RAG-1, RAG-2, and TdT expression.
d. result in allelic exclusion of membrane receptors.
e. result in isotype switching after antigen stimulation of the mature
lymphocytes.

6. The amount of diversity in TCR generated within one individual by somatic


recombination

a. is higher than BCR diversity.


b. is about the same as for BCR diversity.
c. is lower than BCR diversity.
d. is lower than Class I MHC diversity.
e. is lower than Class II MHC diversity.

nolan x soju
7. T cells use all of the following for generating antigen-recognition diversity
on the TCR, except

a. combinatorial association of chains.


b. combinatorial association of segments.
c. large germline pool of gene sequences.
d. N region addition of nucleotides.
e. somatic hypermutation.

8. CD8 is a co-receptor on T cells that binds

a. CD3.
b. endogenous antigen peptide.
c. the constant region of Class I MHC.
d. the constant region of TCR.
e. the variable region of Class I MHC.

9. All of the following are true for antigen receptors on both B cells and T cells
EXCEPT
a. associated with signal transduction molecules in the membrane.
b. generated by somatic recombination during lymphocyte development.
c. members of the Ig gene superfamily.
d. MHC-restricted in their ability to bind antigen.
e. specific for a single antigen epitope.

10. Which of the following statements is FALSE?

a. TCR is allelically excluded on individual T cells.


b. CD4 and CD8 co-receptors are also signal transducing molecules for T cell
activation.
c. The arrangement of  chain gene segments most closely resembles that
of chain.
d. The gene segments for the  chain are interspersed with those for
the  chain.
e. The T cells that are most likely to react against allogeneic kidney cells are
CD8+ cytotoxic T cells.

1. Exogenous antigen includes all of the following EXCEPT

nolan x soju
a. bacterial toxins.
b. extracellular protozoan parasites.
c. most bacteria.
d. ragweed pollen.
e. viruses.

2. Human Class I MHC  chain molecules are

a.2-microglobulin.
b. H-2 D, K, and L.
c. H-2 IA and IE
d. HLA-A. -B, and -C.
e. HLA-DR, -DP, and -DQ.

3. Cells which have MHC Class II are _________________, which present


_____________ antigen to Th cells.

a. antigen presenting cells, endogenous


b. antigen presenting cells, exogenous
c. infected cells, inflammatory
d. target cells, endogenous
e. target cells, exogenous

4. Signaling to a cytotoxic T cell that a liver cell is infected with hepatitis virus
depends on

a. binding of Ii to Class I MHC until the peptide is loaded.


b. binding of TCR on the cytotoxic T cell to Class II MHC on the infected cell.
c. binding of processed antigen to liver cell Class I MHC.
d. processing the hepatitis virus peptides to the correct size and anchor
residues in the endosomal pathway.
e. both c and d are correct.

5. Endogenous antigen presentation requires delivery of antigen peptides to


the endoplasmic reticulum by

a. Class I MHC and invariant chain.


b. calnexin and tapasin.
c. HLA-DM.
d. leader sequence.
e. TAP-1 and TAP-2.

nolan x soju
6. Following virus infection, peptides produced from the proteasome are more
likely to be presented on the surface of the target cell because

a. MHC Class I is synthesized in response to virus infection.


b. proteasomal enzymes which produce shorter peptides are synthesized in
response to virus infection.
c. TAP-1 and TAP-2 specifically bind virus peptides.
d. virus amino and carboxyl terminal amino acids bind better to Class I MHC
than peptides from self proteins.
e. virus infection induces expression of proteases which cut proteins at sites
which bind best to TAP-1 and TAP-2.

7. Exogenous antigen is processed

a. after presentation by antigen presenting cells.


b. by nearly every nucleated cell.
c. by the cytosolic processing pathway.
d. in the presence of 2-microglobulin.
e. in acidified endosomes.
8. Class II MHC does not efficiently present endogenous antigen because

a. antigen synthesized inside the cell never makes it to the endosomal


compartment.
b. endogenous antigen cannot be processed into peptides small enough.
c. HLA DM transports Class II to the surface before it can bind endogenous
peptide.
d. invariant chain blocks binding of endogenous peptide in the ER.
e. phagocytosed antigen binds Class II as rapidly as Class II is synthesized.

9. MIIC is a specialized intracellular compartment where

a. HLA DM promotes the release of CLIP and peptide binding to Class II


MHC.
b. invariant chain binds to Class II MHC  and  chains.
c. peptides are transported into the ER for binding to Class II.
d. proteins are broken down into peptides by proteasomes.
e. some pathogens live protected from lysosomal enzymes.

10. In order to have pathogen peptide plus Class II MHC molecules expressed
on the membrane of host cells, all of the following are required EXCEPT

nolan x soju
a. 2-microglobulin.
b. CLIP.
c. HLA-DM.
d. HLA-DR, -DP, and -DQ alpha chains.
e. Ii .

11. Invariant chain (Ii)

a. inhibits binding of endogenous peptide to Class I MHC.


b. is degraded in the MIIC compartment to CLIP.
c. is released from Class II upon binding of2-microglobulin.
d. is the constant region of Class I peptide binding site.
e. prevents exogenous peptide binding to Class II MHC in the ER.

12. Antigen binding by Class I MHC molecules

a. accommodates many different peptides.


b. preferentially occurs for peptides 13-18 amino acids in length.
c. occurs at a site on Class I MHC formed by folding of 1 and2-
microglobulin domains.
d. occurs only on antigen presenting cells.
e. takes place at the plasma membrane of the infected cell.

13. Both Class I and Class II MHC molecules are

a. composed of and  chains with variable and constant regions.


b. expressed constitutively on all nucleated cells.
c. expressed on the B cell membrane.
d. part of the T cell receptor for antigen.
e. synthesized in response to antigen processing.

14. The major histocompatibility complex has

a. dozens of loci for Class I and Class II proteins.


b. genes that encode proteins associated with antigen processing.
c. only genes encoding Class I and Class II molecules.
d. single loci for Class I and Class II proteins.
e. three regions encoding Class I, Class II, and Class III receptors.

15. MHC polymorphism

a. is generated by recombination of HLA A, B, and C gene segments.

nolan x soju
b. is present primarily in the peptide-binding regions of MHC proteins.
c. is the result of random association of many alpha and beta genes.
d. restricts the ability of B cells to bind antigen.
e. results in expression of dozens of MHC alleles on each APC.

16. T cells are MHC-restricted in their ability to respond to antigen because

a. all antigen must be processed and presented to activate lymphocytes.


b. during an infection, all cells in the body present antigen on MHC Class I.
c. MHC binds antigen more specifically than TCR does.
d. TCR must recognize both antigen and MHC molecules.
e. the T cells should not respond to antigen on allogeneic cells.

17. Linkage of a disease to an HLA allele means that

a. everyone with that allele will eventually get the disease.


b. people with that allele have a higher risk for the disease.
c. the MHC protein encoded by that allele is defective.
d. the allele will eventually disappear from the population.
e. None of the above is true.
18. All of the following are associated with the expression of Class I MHC
molecules EXCEPT

a. antigen peptide presentation on membrane Class I MHC to Tc.


b. graft rejection.
c. increased risk of certain autoimmune diseases.
d. lysis of virus-infected cells.
e. stimulation of antibody production.

19. Human Class II MHC molecules

a. are encoded by the genes HLA-A, B, and C.


b. are found on all nucleated cells.
c. have an antigen binding site formed from regions of two polypeptide
chains.
d. must be associated with 2-microglobulin molecules to bind peptide.
e. present antigen to CD8 cytotoxic T cells.

20. Humans inherit from each of their parents

a. a random set of MHC Class I, Class II, and Class III genes.
b. enough diversity in MHC to present epitopes from most pathogens.

nolan x soju
c. enough diversity in MHC to present every possible antigen epitope.
d. genes for  and  chains that can be recombined to increase their diversity.
e. the same Class I and Class II MHC genes as their siblings.

21. The  chain of HLA-DR

a. can be expressed with the chain of any MHC molecule.


b. can be expressed with the  chain of any Class II MHC molecule.
c. can be expressed with the  chain of any Class II DR molecule.
d. must be expressed with2-microglobulin.
e. must be expressed with the chain of Class II DR from the same
chromosome.

22. Which of the following statements is TRUE?

a. Each individual expresses all the diversity of MHC protein structure.


b. If a family has four children, no two of them will have the same MHC
genotype.
c. Someone with bare lymphocyte syndrome who expressed no MHC proteins
would die in infancy.
d. TCR on Tc cells binds 1 and 2 domains of Class I MHC protein.
e. The chances of finding a tissue match are much higher between children
and their parents than between siblings.

23. Which of the following statements is FALSE?

a. All MHC alleles in the population have been counted.


b. CD4 T cells see antigen on self Class II MHC but not on self Class I MHC.
c. Human Class II MHC proteins are called HLA DP, HLA DQ, and HLA DR.
d. Class I and Class II MHC are less antigen-specific than Ig.
e. Peptides presented by Class I MHC must be 8-10 amino acids long.

24. Which of the following statements is FALSE?

a. A peptide binding to Class I must have certain amino- and carboxyl-terminal


amino acids to bind tightly to the ends of the Class I binding cleft.
b. A transplant is most likely to be successful between people who share the
same alleles at all Class I and Class II MHC loci.
c. Identical twins share all their Class I and Class II MHC alleles.
d. Peptide binding to TCR is influenced by both its own conformation and the
conformation of the MHC protein to which it is bound.
e. The gene for 2-microglobulin is in the Class I region of the MHC.

nolan x soju
1. An antigen binding signal at the membrane results in the mature B
lymphocyte changing its

a. antigen-binding specificity.
b. color.
c. Ig V-D-J gene rearrangement.
d. gene expression.
e. signal transduction molecules.

2. Signal transduction is the process of converting

a. a B cell to a T cell.
b. a binding signal to a chemical signal.
c. a hapten to an antigen.
d. IgA to secretory IgA.
e. a kinase to a phosphatase.

3. A ligand is

a. a cytokine.
b. a molecule that specifically binds a receptor.
c. an antigen.
d. an enzyme.
e. all of the above are ligands.

4. A tyrosine kinase which is activated by antigen binding is found in the


__________ of the BCr or TCR complex.

a. cytoplasmic domain
b. extracellular domain.
c. Ig superfamily domain.
d. transmembrane domain.
e. variable domain.

5. The ligand for TCR is

nolan x soju
a. BCR.
b. MHC
c. MHC + peptide.
d. peptide.
e. TCR ligand.

6. An oncogene is a gene that is associated with

a. apoptosis.
b. cancer.
c. ITIMs.
d. TCR and BCR signal transduction.
e. viruses.

7. Antigen binding to B cells is most effective at sending an activation signal to


the B cell if it causes

a. antigen processing and presentation on Class II MHC.


b. BCR clustering.
c. BCR internalization.
d. inflammation.
e. opsonization.

8. An enzyme which puts a phosphate group on a protein molecule is called a

a. co-receptor.
b. ITAM.
c. kinase.
d. phosphatase.
e. receptor.

9. Gene expression does NOT necessarily involve

a. changes in a cell's activities (phenotype).


b. mRNA synthesis.
c. protein synthesis.
d. DNA synthesis.
e. transcription factors.

10. The signal transduction molecules associated with TCR are

a. CD1.

nolan x soju
b. CD3.
c. CD4.
d. CD8.
e. CD22.

11. The signal transduction molecules associated with BCR are

a. CD21 and CD81.


b. Ig and Ig
c. IgD and IgM.
d. ITAMs and ITIMs.
e. RAG-1 and RAG-2.

12. The second messenger IP3 increases the cytoplasmic concentration of

a. antigen.
b. calcium.
c. Class I MHC.
d. phosphate.
e. sodium.
13. DAG and IP3 are released from PIP2 by the action of

a. adaptor protein.
b. phospholipase C (PLC).
c. protein kinase C (PKC).
d. small G protein.
e. TdT.

14. Small G proteins (like Ras) convert GTP to GDP by their ___________
activity.

a. GEF.
b. kinase.
c. phosphatase.
d. polymerase.
e. protease.

15. Transcription factors

a. increase synthesis of mRNA.


b. increase synthesis of DNA.
c. inhibit synthesis of mRNA.

nolan x soju
d. promote DNA phosphorylation.
e. synthesize mRNA.

16. An enzyme cascade is a

a. case where the enzyme catalyzes its own inactivation, like small G proteins.
b. pair of enzymatic reactions that have opposite effects, like kinases and
phosphatases.
c. series of enzymatic reactions that result in cancer.
d. series of enzymatic reactions where the product of one reaction catalyzes
the next reaction.
e. small waterfall.

17. Signal transduction complex associates with TCR in the membrane


through

a. agonist peptides.
b. covalent bonds.
c. enzyme cascades.
d. reverse phosphorylation.
e. salt bridges.

18. If IgIg cannot be made, B cells

a. cannot express BCR.


b. cannot express Class II MHC.
c. express 1,000-fold less BCR than usual
d. synthesize CD3 and become T cells.
e. require 1,000-fold more antigen to be activated.

19. The immune system of a person who had a mutation in CD3 could NOT
fight a viral hepatitis A infection by

a. blocking Hepatitis A virus from infecting liver cells with neutralizing IgG
antibodies.
b. generating cytotoxic T cells to lyse infected liver cells
c. lysing virus-infected cells with NK cells.
d. phagocytosing complement-opsonized Hepatitis A virus.
e. Both 1 and 2 are correct.

20. Amino acid sequences in lymphocyte signal transduction complexes which

nolan x soju
are phosphorylated following antigen binding are called

a. ITAMs.
b. ITIMs.
c. MAPs.
d. PTKs.
e. syks.

21. An immune deficiency resulting from a defective PTK in the activation


cascade in B cells would probably be characterized by

a. high numbers of circulating B cells.


b. high numbers of circulating lymphocytes.
c. high concentrations of plasma immunoglobulins.
d. low concentrations of plasma immunoglobulins.
e. low numbers of circulating T cells.

22. B cell co-receptor complex CD19, CD22, and CD81


a. allows B cells to be activated with 1,000-fold less complement-coated
antigen.
b. allows B cells to be activated with 1,000-fold more complement-coated
antigen.
c. decreases B cell expression of BCR.
d. increases B cell expression of BCR.
e. prevents B cell activation by self antigen.

23. The anti-rejection drugs cyclosporin A and FK506 block rejection of


transplanted organs by interfering with

a. activation of a T cell transcription factor required for T cell activation.


b. antibody synthesis required for ADCC of transplanted cells.
c. CD3 expression.
d. MHC Class I expression.
e. processing of graft peptides and presentation on Class I MHC.

24. Antagonist peptides

a. fail to bind to T cells.


b. fully activate T cells.
c. interfere with T cell activation by agonist peptides.

nolan x soju
d. partially activate T cells.
e. require partial agonist peptides to fully activate T cells.

25. Antibody-dependent cell-mediated cytotoxicity (ADCC) is a process in


which antibody-coated cells are killed by

a. the antibodies.
b. complement.
c. cytotoxic T cells.
d. cells with Fc receptors for IgG3.
e. cells with Fc receptors for IgE.

26. When IgE on mast cell FcR is cross-linked by, antigen, the mast cell
responds by

a. apoptosis.
b. presenting the antigen to Th cells.
c. secreting IgE.
d. secreting histamine and other allergic mediators.
e. stimulating macrophage and neutrophil phagocytosis of the coated antigen.
27. Homeostasis is

a. macrophage activation by bacterial antigens.


b. programmed cell death.
c. the normal process of signal transduction.
d. the synthesis from all leukocytes from bone marrow stem cells.
e. the regulation of biological systems within normal limits.

28. STAT proteins are NOT

a. cytosolic proteins.
b. involved in cytokine signaling.
c. JAK kinases.
d. signal transducers.
e. transcription activators.

29. Cells receive a death signal through

a. bcl-2 receptor.
b. death receptor.
c. Fas.
d. Fas ligand.

nolan x soju
e. STAT ligand.

30. The most important receptor through which lymphocytes receive life and
death signals is

a. antigen receptor.
b. bcl-2 receptor.
c. Fas receptor.
d. FcR.
e. growth factor receptor.

1. B cell differentiation begins with the expression of

a. membrane chain + surrogate L chain.


b. membrane IgD.
c. Membrane IgM.
d. germline IgM.
e. RAG-1, RAG-2 and TdT.

2. Bone marrow stromal cells

a. are important because they provide physical support for B cells (hence,
their name from the Greek word for mattress).
b. are present only in the center of the marrow.
c. present foreign antigen to B cells to stimulate somatic hypermutation .
d. present self antigen on self MHC to B cells for negative selection.
e. secrete cytokines such as IL-7 that signal developing B cells to divide and
differentiate.

3. Cell adhesion molecules (CAMs)

a. are found only on bone marrow stromal cells.


b. are specific receptors for cytokines that promote cell-cell binding.
c. function primarily to hold developing B cells in one location until they are
fully developed.
d. signal developing B cells to divide and differentiate.
e. signal developing B cells to die because they have bound self.

nolan x soju
4. The developmental step that commits a cell to the B lineage is

a. expression of both membrane IgM and IgD.


b. expression of membrane  chain.
c. expression of recombinase enzymes.
d. joining of a VH gene segment to a DH gene segment.
e. joining of a DH gene segment to a JH gene segment

5. Which statement about B cell development is FALSE?

a. Cells which fail to synthesize and express  chains usually die.


b. Each DNA joining event in Ig genes has a 67% probability of resulting in a
nonproductive rearrangement.
c. The earliest developing B cell which could be stained with FITC-anti- chain
would be a pro B cell.
d. The earliest developing B cell which could be stained with FITC-anti-
 chain would be an immature B cell.
e. The enzyme which can add nucleotides not encoded in the DNA to Ig
genes during recombination is TdT.
6. Once H chain genes have been productively rearranged and expressed on
the pre-B cell membrane, the next event to occur in the cell is

a. death of cells binding self antigen.


b. expression of membrane IgD.
c. expression of membrane of IgM.
d. proliferation of the pre-B cells.
e. somatic recombination of light chain genes.

7. In a productive rearrangement of Ig DNA, what is produced must be

a. a functional membrane Ig protein chain.


b. a loop of DNA that is then removed.
c. an mRNA for H or L chain.
d. a shorter piece of DNA.
e. successful antibody secretion in response to antigen.

8. Proliferation of large pre-B cells

a. is part of clonal selection.


b. makes the pre-B cells more susceptible to apoptosis following self antigen
binding

nolan x soju
c. results in production of many B cells with the same antigen specificity.
d. results in the production of many B cells with the same VH chain but
different antigen specificities due to different VL regions.
e. requires the presence of RAG-1 and RAG-2.

9. Which of the following statements is TRUE?

a. Isotypic exclusion on individual B cells pertains to expression of a single


heavy chain isotype on each mature naive B cell.
b. Pre-B cells must receive a signal from specific antigen binding to pre-B
receptor before they can proceed to the next stage in development.
c. Membrane  chain is always expressed with Ig and Ig
d. Membrane  chain is always expressed with VpreB and 5.
e. Transgenic mice for recombined H and L immunoglobulin genes have
germline H and L genes in non-B cells.

10. Light chain rescue

a. allows self-specific B cells to repeat somatic recombination of light chain


gene segments.
b. is a signal received through binding to the surrogate light chain that rescues
the developing B cell from death.
c. results from multiple V-J joining events on a single chromosome until
productive rearrangement of light chain occurs or all J segments have been
recombined.
d. signals the developing B cell through IgIg to begin recombination of light
chain gene segments.
e. None of the above is true.

11. Regulatory nucleotide sequences in the DNA that control Ig protein


synthesis are

a. inducers and promoters.


b. initiation sites and enhancers.
c. promoters and enhancers.
d. promoters and switch regions.
e. recombination signal sequences and promoters.

12. Transgenic mice for BCR

a. are given a completely new set of Ig genes.


b. express the recombined H chain gene on every cell in the body.

nolan x soju
c. have germline H chain gene segments in all their cells except B cells.
d. suppress their own H chain V-D-J recombination if they have been given a
recombined H chain gene.
e. synthesize Ig with the same amount of diversity as non-transgenic mice.

13. If the Ig V regions encoded in the transgenes were specific for self MHC,
the transgenic mice

a. might be able to produce low numbers of non-self-specific B cells through


receptor editing.
b. would produce normal numbers of B cells because MHC is not expressed
in the marrow.
c. would die of autoimmunity.
d. would fail to produce any B cells.
e. would not die of autoimmunity because only T cells bind MHC.

14. In general, a knock-out mouse

a. does not show allelic exclusion of Ig.


b. has a mutant TdT that removes (knocks out) N nucleotides instead of
inserting them during somatic recombination.
c. has no Ig gene segments.
d. has a normal gene replaced with a nonfunctional gene for the same protein.
e. is often used to produce monoclonal antibody.

16. Knock-out mice which have  gene segments from which the membrane
domain has been removed

a. cannot synthesize any  chain.


b. can splice the C segments to the membrane segment for  and express
normal amounts of membrane  chain.
c. have normal pro-B cells but no later stages of developing or mature B cells.
d. have normal pro-B cells and pre-B cells but no mature B cells.
e. have no cells with rearranged Ig genes.

17. B cells which express Ig with a truncated cytoplasmic domain (lacking
the ITAMS)

a. become "stuck" in the immature B cell stage of development.


b. can express pre-B cell receptor but not proceed to divide and then
recombine light chain genes.
c. develop normally since Ig has its ITAM sequences.

nolan x soju
d. express both IgM and IgD but cannot be activated by antigen.
e. require more antigen for activation.

18. The ability of an individual B cell to express only one H chain allotype is
called

a. allelic exclusion.
b. co-dominant expression.
c. isotypic exclusion.
d. nonproductive rearrangement.
e. survival of the fittest.

19. Once they leave the marrow, in order to survive mature naive B cells must
enter the

a. circulation
b. lymph microenvironment.
c. primary lymphoid follicles.
d. secondary lymphoid follicles.
e. thymus.
20. Negative selection of B cells occurs when

a. immature B cells bind self antigen and undergo apoptosis.


b. immature B cells fail to bind self MHC and die.
c. lymphoid progenitor cells become committed to becoming T cells and leave
the marrow for the thymus.
d. mature B cells fail to bind antigen in the lymphoid follicles and die.
e. pre-B cells bind ligand with their pre-B receptor and stop rearranging H
chain genes.

21. A mature lymphocyte which has specific antigen receptors but cannot
respond to that antigen is called

a. anergic.
b. apoptotic.
c. deleted.
d. lazy.
e. memory.

22. Immature B cells which bind soluble self antigen

a. become apoptotic.

nolan x soju
b. escape clonal deletion and can potentially cause autoimmune disease in
adults.
c. go on to express normal levels of IgM and IgD but cannot respond to self
antigen.
d. undergo clonal deletion in the bone marrow.
e. usually cannot enter the primary lymphoid follicles and soon die in the
periphery.

23. Immature B cells from inbred mouse strain q (expressing the q allele of
every MHC molecule: Dq, Kq, Lq, IAq, and IEq) which also express transgenic H
and L chains specific for H-2 IAs (the s allele of Class II MHC IA) will undergo
clonal deletion when

a. exposed to macrophages from an "s" strain mouse (expressing the "s"


allele of every MHC molecule).
b. exposed to T cells from an s strain mouse.
c. exposed to their own macrophages.
d. exposed to their own T cells.
e. Both 1 and 2 are correct.
24. Chickens have very few functional V, D, and J gene segments and
therefore

a. are very susceptible to disease.


b. have many self-specific lymphocytes and usually die of autoimmune
disease.
c. insert diverse sequences from VH pseudogenes into their H chains while B
cells are immature.
d. make mature B cells which are all specific for the same antigen.
e. use gene conversion to introduce diversity into Ig gene sequences as B
cells divide in response to foreign antigen.

25. The B cells with the longest life span are the

a. anergized B cells.
b. immature B cells.
c. mature naive B cells.
d. memory cells.
e. plasma cells.

26. Receptor editing

nolan x soju
a. allows self-specific B cells to repeat somatic recombination.
b. is responsible for allelic exclusion of Ig on individual B cells.
c. kills self-specific B cells before they leave the marrow.
d. occurs during the small pro-B cell stage of development.
e. occurs only for light chains of self-specific B cells.

27. B-1 B cells are more _____________ than conventional B (B-2) cells.

a. antigen-specific
b. high affinity
c. likely to be protein-specific
d. numerous in adults
e. polyreactive

28. The antibody secreted by B-1 B cells often

a. binds common bacterial carbohydrates.


b. has a high affinity for antigen.
c. is IgG.
d. neutralizes common viral receptor proteins.
e. neutralizes self antigens.

29. As cells become mature naive B cells, they

a. become more antigen-independent.


b. leave the marrow and never return.
c. stay in one location and wait for antigen and T cells to come to them.
d. stay in the same locations as T cell to be able to receive helper signals.
e. use CAMs to enter the appropriate lymphoid organ locations.

30. A germinal center is where B cells

a. become mature.
b. divide in response to antigen.
c. first bind antigen.
d. secrete antibody.
e. undergo somatic recombination.

31. If you wanted to check for the presence of memory B cells from your
vaccine subjects by taking some of their B cells and re-stimulating them in
vitro (outside the body) with antigen, the best place to get some memory cells

nolan x soju
would be from

a. blood.
b. bone marrow.
c. lymph nodes.
d. spleen.
e. tonsils.

32. Multiple myeloma is a cancer which has arisen from a

a. B-1 B cell.
b. lymphoid progenitor.
c. mature B cell.
d. plasma cell.
e. pre-B cell.

33. If a B cell tumor is monoclonal, it can be differentiated from normal B cells


of the same person by its unique membrane Ig
a. allotype.
b. H chain.
c. idiotype.
d. isotype.
e. L chain.

34. An oncogene is a

a. gene for a virus protein that causes cancer.


b. gene for tumor antigens on B cells.
c. mutated gene that causes cancer.
d. normal gene that can cause cancer if its normal function is disrupted.
e. translocated Ig gene.

1. T cells differ from B cells by expressing

a. antigen receptor of only one idiotype.


b. antigen receptor of only one isotype.
c. RAG-1 and RAG-2 during somatic recombination.
d. a surrogate chain before somatic recombination of the second receptor

nolan x soju
chain is complete.
e. signal transduction molecules with their antigen receptor.

2. The step that commits a cell to becoming a T cell is

a. expression of CD4 and CD8.


b. expression of CD3.
c. migration to the thymus.
d. somatic recombination of  chain.
e. somatic recombination of  chain.

3. The thymic equivalent of the bone marrow stromal cells are the thymic

a. dendritic cells.
b. epithelial cells.
c. hormones.
d. macrophages.
e. thymocytes.

4. Of the cells that enter the thymus, ___% become mature naive T cells.
a. 100
b. 75
c. 50
d. 33
e. 3

5. CD25 is

a. a CAM.
b. a co-receptor for TCR.
c. part of the receptor for IL-2.
d. a signal for positive selection.
e. a signal transduction molecule for TCR.

6. Double positive T cells express

a. both  and  TCR.


b. both CD3 and CD4.
c. both CD3 and CD8.
d. both CD4 and CD8.
e. TCR of more than one antigen specificity.

nolan x soju
7. A developing T cell has more chances to productively rearrange  chain
than a developing B cell has for H chain because

a. if T cells fail to rearrange , they can rearrange  chain.


b. there are many more V than VH segments.
c. T cells have four alleles of each set of  gene segments, while B cells have
only two alleles of each set of H chain segments.
d. there are two sets of D, J, and C  loci so that if rearrangement at the first
locus fails, rearrangement can still occur at the second.
e. a developing T cell and developing B cell have exactly the same
opportunities for productive rearrangements.

8. The surrogate light chain that is expressed with newly rearranged  chain

a. binds self MHC.


b. can signal the T cell without CD3.
c. is CD44.
d. is unique for each developing T cell.
e. is pT chain.
9.  chains are rearranged

a. after the T cell binds self antigen.


b. before  chains.
c. before T cells become CD3+.
d. in double negative T cells.
e. in double positive T cells.

10. Because individual T cells can rearrange more than one  chain, each cell

a. can bind multiple antigens.


b. can bind both Class I and Class II MHC.
c. can participate in both cellular and humoral immunity.
d. is not allelically excluded for  chain.
e. is not isotypically excluded for  chain

11. Cells that have just successfully expressed both  and  chains

a. are called single positive thymocytes because they have a single specificity
of TCR on each cell.
b. are all committed to becoming either Th or Tc.
c. divide rapidly before undergoing positive selection.

nolan x soju
d. have lower amounts of membrane TCR than mature T cells.
e. have only  receptors, while mature T cells express  and  receptors.

12. Single positive T cells

a. are either mature Th or mature Tc cells.


b. are found only in the thymus.
c. are ready to undergo positive selection.
d. divide rapidly before leaving the thymus.
e. undergo apoptosis if they leave the thymus and encounter foreign antigen.

13. Most of our T cells are produced when we are

a. asleep.
b. less than 12 years old.
c. between 12 and 22 years old.
d. between 22 and 42 years old.
e. making a cellular response.

14. Nude mice have no


a. B cells.
b. clothes.
c. RAG-1.
d. skin.
e. thymic stroma.

15. Nude mice would be unable to protect themselves from infection using

a. antibody.
b. antigen-specific cytotoxicity.
c. complement.
d. inflammation.
e. natural killer cells.

16. To give SCID mice a normal immune system, one would have to

a. inject antibody.
b. infuse lymphoid progenitors.
c. infuse mature T cells.
d. transplant a thymus.
e. None of the above would restore a functional immune system to a SCID
mouse.

nolan x soju
17. The term "MHC restriction" refers to the

a. ability of MHC genes to regulate TCR rearrangement.


b. inheritance of a limited number of MHC antigens.
c. problems associated with allograft rejection.
d. requirement for antigen to be recognized in association with MHC
molecules.
e. requirement for MHC molecules in order to reject grafts

18. T cells from an MHCaxb F1 mouse

a. all recognize antigen presented only on MHCa.


b. all recognize antigen presented only on MHCb.
c. each recognize antigen on both MHCa and MHCb.
d. each recognize antigen on either MHCa or MHCb.
e. fail to recognize antigen on any MHC except MHCaxb.

19. A radiation chimera made by giving bone marrow cells from an


MHCaxb mouse to an irradiated MHCa mouse produces T cells that
a. are positively selected on MHCaxb cells in the thymus.
b. can only see foreign antigen presented on MHCa APC.
c. can only see foreign antigen presented on MHCaxb APC.
d. will reject MHCa skin grafts.
e. will reject MHCb skin grafts.

20. Positive selection for the production of HLA A7-restricted CD8+ T cells
requires all of the following EXCEPT

a. expression of CD8 that binds the invariant part of HLA-A7.


b. expression of foreign peptide on HLA- A7.
c. expression of HLA -A7 on thymic epithelial cells.
d. rearrangement of TCR V gene segments that bind HLA -A7.
e. TAP-1 and TAP-2.

21. Mice which were deficient in H-2M and presented CLIP on all their class II
MHC molecules

a. developed autoimmunity.
b. failed to produce any CD4+ T cells.
c. failed to produce any CD8+ T cells.
d. produced CD4+ T cells that could only recognize CLIP presented on

nolan x soju
syngeneic MHC of normal mice.
e. produced CD4+ T cells that could recognize self peptides presented on
syngeneic MHC of normal mice.

22. Which of the following is NOT associated with negative selection?

a. Apoptosis of thymocytes.
b. Bone-marrow derived APC.
c. Recognition of self-MHC molecules.
d. Recognition of foreign antigen.
e. Self tolerance.

23. Double positive T cells with a transgenic TCR specific for H-Y (male)
antigen on self Class I MHC

a. would develop normally because H-Y is a membrane molecule and its


peptides would not be presented on Class I.
b. would develop into both CD4+ T cells and CD8+ T cells in female mice but
would not develop into mature T cells in male mice.
c. would develop into CD8+ T cells in female mice but would not develop into
mature T cells in male mice.
d. would develop into CD8+ T cells in both female and male mice.
e. None of the above is true.

24. Expression of endogenous MMTV superantigen during T cell development

a. has no effect on because MMTV is not a self antigen.


b. negatively selects T cells with the V regions that bind superantigen.
c. positively selects T cells with the V regions that do not bind superantigen.
d. results in mice that have very strong reactivity against MMTV.
e. results in production of T cells with normal TCR diversity.

25. Positive and negative selection of T cells

a. involve different MHC molecules.


b. involve different peptides.
c. must involve signals that differ either in amount or in type.
d. occur in the same thymus microenvironment.
e. result in T cells which can never recognize self antigen.

26. The maximum number of Class I and Class II MHC genes we have is
probably NOT influenced by

nolan x soju
a. the maximum number of MHC genes that can be carried in the DNA.
b. the maximum number of MHC molecules that can be expressed on the cell
surface.
c. the maximum number of peptide-MHC complexes that will not result in
excessive negative selection of T cells.
d. the minimum number of peptide-MHC complexes that must be bound by a
T cell to send a strong enough signal to activate the T cell.
e. the minimum number of pathogen peptides that must be recognized for a
person to survive.

27. A monoclonal T cell tumor which developed from a normal T cell would
NOT express the normal T cell's

a. cell surface markers.


b. division rate.
c. organ location.
d. TCR gene rearrangement.
e. All of the above would resemble the normal parent cell.
28. Most of the cells that enter the thymus

a. become MHC-restricted  TCR cells.


b. become MHC-restricted  TCR cells.
c. become monoclonal T cell tumors at various stages of development.
d. die because they cannot make a self-MHC binding TCR.
e. die because they bind self peptide.

29.  T cells

a. are made before  T cells during fetal development.


b. are more diverse than  T cells.
c. are precursors of  T cells.
d. outnumber  T cells 19:1 in the spleen.
e. probably bind common pathogen peptides on Class II MHC.

30. The step that commits a cell to becoming a  T cell is

a. development outside the thymus.


b. expression of more  than  receptors on its surface.
c. migration out of the thymus to the skin.
d. somatic recombination of  chain before  chain.

nolan x soju
e. somatic recombination of  chain before  chain.

1. Adaptive cell-mediated immunity involves the generation of activated


antigen-specific effector

a. antibodies.
b. B cells.
c. cytokines.
d. macrophages
e. T cells.

2. T cells are activated to armed effector cells when they encounter antigen on

a. blood-borne pathogens.
b. bone marrow stromal cells.
c. professional APC.
d. virus-infected tissues.
e. all of the above.

3. T cells move from the circulation into the peripheral lymphoid tissues when
they bind

a. antigen.
b. APC.
c. B7.
d. homing molecules.
e. vascular addressins.

4. LFA-1 is a(n)

a. co-receptor on T cells.
b. co-stimulatory molecule on APC.
c. cytokine receptor on APC.
d. integrin on T cells.
e. vascular addressin on HEV.

5. The antigen specificity of Th activation by macrophages is enhanced by the

nolan x soju
a. ability of each macrophage to present only specific epitopes to T cells.
b. binding of T cell adhesion molecules only to macrophages presenting the
antigen recognized by the T cell.
c. specificity of CD4 binding to Class II MHC on the macrophage.
d. upregulation of CD4 or CD8 co-receptors following antigen binding.
e. upregulation of co-stimulatory molecules only on macrophages which have
contacted foreign antigen

6. Membrane events required for Th activation include all of the following


EXCEPT binding of T cell

a. CD4 to APC Class II MHC.


b. CTLA-4 to APC B7.
c. high affinity IL-2 receptor to IL-2.
d. LFA-1 to APC ICAM.
e. TCR to peptide on APC Class II MHC.

7. Cytoplasmic signals generated in response to antigen binding and co-


stimulatory signals to Tc cells include all of the following EXCEPT
a. activation of transcription factors that upregulate synthesis of IL-2.
b. aggregation of TCR and CD8 in the T cell membrane.
c. increased free intracellular Ca++.
d. induction of RAG synthesis.
e. tyrosine kinase binding to the cytoplasmic domains of CD3 and CD8.

8. Th1 cells secrete cytokines which

a. activate macrophages.
b. increase susceptibility to allergic reaction
c. inhibit macrophage secretion of inflammatory cytokines
d. stimulate B cells to secrete neutralizing antibodies
e. stimulate proliferation of Th2 cells

9. A T cell which binds antigen without undergoing a co-stimulatory signal


from an APC

a. becomes inactivated.
b. can become a helper cell but not a killer cell.
c. can get a co-stimulatory signal from a different APC
d. dies.
e. divides but doesn't make effector molecules.

nolan x soju
10. Dendritic cells are the best APCs for activating

a. CTL.
b. helper T cells.
c. naïve T cells.
d. Th1 cells.
e. Th2 cells.

11. Macrophages normally have low levels of B7 and MHC, but express more
after they phagocytose

a. bacterial antigens.
b. peptide antigens.
c. Tc cytokines.
d. Th cytokines.
e. viral antigens.

12. B cells differ from the other professional APC because B cells
a. activate T cells in the secondary lymphoid organs.
b. present exogenous antigen to T cells on Class II MHC.
c. present only peptides from antigens they bind with their membrane Ig.
d. use B7 as their co-stimulatory molecule.
e. None of the above is true.

13. Once T cells have received antigen plus co-stimulatory signals, they divide
in response to IL-2 secreted by

a. B cells.
b. dendritic cells.
c. macrophages.
d. themselves.
e. whichever APC activated them.

14. The  chain of the IL-2 receptor

a. causes the receptor to associate with TCR in the T cell membrane.


b. changes the specificity of the receptor to bind IFN.
c. increases the affinity of the receptor for IL-2.
d. increases the signal transduction ITAMs of the receptor.
e. All of the above are true.

nolan x soju
15. It takes ___________________ after antigen contact before T cells are
armed effector cells.

a. 5 minutes.
b. 30 minutes.
c. 24 hours.
d. 4-5 days.
e. 7-10 days.

16. Armed effector T cells differ from naïve T cells in that effector cells

a. are not antigen-specific.


b. do not need to bind co-stimulatory molecules.
c. do not need to bind peptide plus MHC.
d. do not use CAMs to bind other cells.
e. have CD28 instead of CTLA-4 to bind B7.

17. The effector molecules of cytotoxic T cells do NOT include


a. Fas.
b. granzymes.
c. IFN.
d. perforin.
e. TNF.

18. Interferons

a. activate B cells to make virus-specific antibodies.


b. are Th2 cytokines.
c. are virus proteins that interfere with activation of cytotoxic T cells.
d. interfere with virus entry into host cells.
e. inhibit virus replication by infected cells.

19. The ability of a cytokine to change gene expression in the target cell is
influenced by all of the following EXCEPT

a. presence of high-affinity receptors on the target cell.


b. presence of soluble cytokine receptors.
c. proximity of the producing and target cells.
d. rate of transport of cytokine-receptor complexes into the cytoplasm.
e. simultaneous production of another cytokine whose receptor uses the same

nolan x soju
signal transducing subunit.

20. IL-1 is produced by macrophages and acts on the hypothalamus to induce


fever. This is an example of _____ cytokine action.

a. autocrine.
b. endocrine.
c. paracrine.
d. pleiotropic.
e. synergistic.

21. CD40 on macrophages binding to CD40L on Th1 cells signals the


macrophages to

a. die.
b. kill cytoplasmic viruses.
c. phagocytose the Th1 cells.
d. respond to IFN by expressing more membrane MHC.
e. secrete IL-2.
22. Activated CTL can regulate immune responses by signaling activated
lymphocytes to undergo

a. apoptosis.
b. clonal deletion.
c. clonal proliferation.
d. cytotoxicity.
e. somatic hypermutation.

23. Cellular immunity includes all of the following except

a. activated macrophages killing phagocytosed bacteria.


b. CD8 T cells inducing apoptosis using Fas-FasL binding.
c. complement-mediated lysis of bacterial cells.
d. MHC-restricted cytotoxic T cells killing virus-infected cells.
e. NK cells performing natural killing of tumor cells.

24. CTL binding and destruction of target cells depends on

a. a co-stimulatory signal from the target cell.


b. antigen presentation on the surface of the target cell.
c. lack of a negative signal from MHC to prevent cytotoxicity.

nolan x soju
d. presence of virus proteins in the membrane of the target cell.
e. secretion of cytokines by the infected cell to attract CTL.

25. CTL use perforin to

a. activate the classical complement cascade to lyse the target.


b. allow granzymes to enter the target cell and initiate apoptosis.
c. promote target cell lysis by macrophages.
d. stimulate the target cells to express Fas on their membranes
e. substitute for C9 in the alternative complement pathway.

26. CTL do not damage uninfected cells because

a. CTL do not bind uninfected cells using their CAMs.


b. granzymes and perforin are secreted only in the space between the CTL
and the infected target cells.
c. uninfected cells cannot undergo apoptosis.
d. uninfected cells have no membrane Class I MHC because they have no
pathogen peptides to present.
e. None of the above is true.
27. Granzymes are

a. enzymes which cut DNA into ladders to induce apoptosis.


b. present in an inactive form in resting CD8 cells.
c. secreted in large amounts by CTL to kill many target cells simultaneously.
d. structurally similar to macrophage cytokines that kill vesicular pathogens.
e. synthesized by CTL before they bind to target cells.

28. Which of the following would you NOT expect to result in reduced CTL
killing activity?

a. absence of the gene for 2 microglobulin.


b. absence of the gene for HLA DM.
c. absence of the gene for perforin.
d. presence of anti-CD8 antibodies.
e. presence of monoclonal antibodies specific for multiple TCR V and
V specificities.

29. IFN does all of the following EXCEPT

a. activate macrophages.
b. cut DNA in target cells into 200bp fragments.

nolan x soju
c. directly inhibit viral replication in infected cells.
d. induce macrophages to express higher levels of MHC on their membranes.
e. starve intracellular parasites of tryptophan.

30. Macrophage activation by Th1 cells is an important immune mechanism


for eliminating

a. bacteria which can resist lysosomal degradation.


b. bacteria whose capsule makes them resistant to phagocytosis.
c. enveloped viruses.
d. parasites that infect T cells.
e. viruses that infect macrophages.

31. T cells which activate macrophages do all of the following EXCEPT

a. activate macrophages at the site of infection.


b. activate only macrophages presenting specific antigen on MHC.
c. become activated effector cells in response to antigen plus co-stimulation.
d. have membrane CD4.
e. provide granzymes that the macrophages use to kill vesicular pathogens.
32. Th1 cells promote cellular immunity by secreting primarily

a. IL-1 and IL-6.


b. IL-2 and IFN.
c. IL-4 and IL-5.
d. IL-10 and IL-12.
e. TNF and FasL.

33. Macrophages kill pathogens using all of the following EXCEPT

a. nitric oxide.
b. oxygen radicals.
c. perforins.
d. peroxides.
e. proteolytic enzymes.

34. Th1 cells make IL-3 and GM-CSF, which

a. activate macrophages.
b. induce apoptosis in old macrophages.
c. stimulate macrophages to kill vesicular pathogens.
d. stimulate macrophage production in the bone marrow.

nolan x soju
e. None of the above is a function of IL-3 and GM-CSF.

35. Granulomas

a. are commonly found in tissues infected with viruses.


b. are granulocyte tumors.
c. are a sign that the body has successfully eliminated all pathogen
d. contain infected macrophages surrounded by T cells.
e. contain infected T cells surrounded by macrophages.

1. T independent antigens do NOT

a. bind to BCR.
b. get presented on MHC Class II.
c. have repeating epitopes.
d. induce B cell proliferation.
e. provide co-stimulatory signals to B cells

2. The humoral immune response to T-independent antigens includes


production of

a. IgA.
b. IgM.
c. memory B cells.
d. memory T cells.
e. all of the above.

3. T-independent antigens are often

a. components of self cell membranes.


b. polyclonal B cell activators.
c. repeating peptide epitopes
d. too small to be phagocytosed and presented.
e. none of the above

4. Before a B cell can receive T cell help, the B cell must

nolan x soju
a. express membrane B7.
b. express membrane CD40L.
c. express membrane IFN receptors.
d. go to the site of infection.
e. process and present peptide on Class II MHC.

5. Antigen-binding B cells entering the secondary lymphoid organs initially go


to the

a. B cell areas where they can bind antigen presented by follicular dendritic
cells.
b. B cell areas where they can process and present antigen to T cells.
c. plasma cell areas where they can secrete antibody.
d. T cell areas where they can be the predominant activator of naïve T cells.
e. T cell areas where they can find specific helper T cells.

6. Germinal centers are

a. areas of lymphocyte generation in the bone marrow.


b. common in unimmunized mice.
c. sites of rapid antigen-induced B cell division in the lymph nodes and spleen.
d. surrounded by naïve T cells waiting to be activated.
e. None of the above is true

7. In order for T cells to provide help to B cells, T cell and B cell epitopes must
be

a. covalently linked.
b. identical.
c. non-identical.
d. peptides.
e. T-dependent.

8. B cell isotype switching is important for

a. increasing the avidity of the antibody.


b. providing antibodies of many different antigen specificities.
c. providing antibodies which can perform different effector functions.
d. signaling the B cells to become activated by follicular dendritic cells
e. signaling B cells to leave the lymph nodes and secrete antibody at the site
of infection.

nolan x soju
9. Affinity maturation of the humoral immune response is due to

a. continued stimulation of B cells by high levels of antigen on the FDC.


b. DNA recombination by products of the RAG genes.
c. isotype switching.
d. negative selection of T cells with the lowest helper potential.
e. positive selection of B cells with the highest affinity for antigen.

10. Isotype switching by B cells occurs in response to T cell

a. IFN.
b. IL-4.
c. IL-5.
d. IL-6.
e. all of the above.

11. Isotype switching resembles somatic recombination because both


processes
a. are catalyzed by the products of RAG-1 and RAG-2.
b. are regulated by helper T cell cytokines.
c. can result in stop codons in coding sequences.
d. occur in developing B cells in the bone marrow.
e. result in the irreversible loss of DNA from the B cell.

12. Somatic hypermutation results in

a. antibody with different CDR than the membrane Ig which originally bound
antigen.
b. B cell apoptosis.
c. B cells with higher affinity for the stimulating antigen.
d. B cells which can no longer bind the stimulating antigen.
e. All of the above result from somatic hypermutation

13. Plasma cells

a. are all very long-lived.


b. divide and differentiate into memory B cells.
c. produce most of their antibody at the site of infection.
d. secrete antibodies as long as antigen binds their membrane Ig receptors.
e. None of the above are true.

nolan x soju
14. Which of the following is NOT a similarity between the cellular and
humoral immune responses?

a. Antigen-specific lymphocytes undergo clonal selection and expansion.


b. Cytokine signals promote effector cell differentiation.
c. Memory cells are generated.
d. Macrophage cytotoxicity is increased.
e. Receptor isotype switching occurs.

15. Humoral immunity involves all of the following EXCEPT

a. antibody-dependent cell-mediated cytotoxicity.


b. antibody secretion by plasma cells.
c. B cell activation by antigen plus cytokines.
d. immunoglobulin isotype switching.
e. macrophage activation by Th1 cells
16. In the problem below dealing with linked T and B cell immune responses
to haptens, the investigators chose to look at production of IgG rather than
IgM because

a. IgG antibodies are easier to detect than IgM.


b. IgG is made before IgM in an immune response.
c. no IgM antibody can be produced to a hapten.
d. production of IgG requires T cell help but production of IgM does not.
e. None of the above is true.

17. In the practice problem below dealing with linked T and B cell immune
responses to haptens, IgG is not produced in experiment #4 because

a. HEL is not an effective carrier for PC.


b. PC cannot be presented to B cells.
c. T and B cell epitopes need to be identical in order for the T cells to provide
help.
d. T and B cell epitopes need to be physically linked in order for the T cell to
provide help.
e. the same antigen epitopes were not given in both primary and secondary
exposures.

nolan x soju
18. Epithelial cells which secrete IgA to block pathogen entry are NOT found
in the

a. digestive tract.
b. mammary glands.
c. respiratory tract.
d. salivary glands.
e. skin.

19. Different Ig isotypes are found in different body locations because they

a. are secreted in different tissues.


b. bind to different FcR that allow them to cross tissue barriers.
c. have different addressins.
d. have different affinities for antigen.
e. None of the above is true.

20. Neutralizing antibody provides effective immunity to


a. bacterial infection.
b. bacterial toxins.
c. virus infection.
d. Both a and b are correct.
e. All of the above can be blocked by neutralizing antibodies.

21. An inactivated toxin used in a vaccine is called a(n)

a. adjuvant.
b. attenuated vaccine.
c. hemagglutinin.
d. neutralizing antigen.
e. toxoid.

22. FcRI on binds with highest affinity to

a. antigen-bound IgG1 and IgG3.+


b. complement-activated IgG1 and IgG3.
c. free IgE.
d. free IgG1 and IgG3.
e. None of the above.

nolan x soju
23. Binding of ligand to FcRI on macrophages and neutrophils does NOT
signal the cells to

a. acidify their phagocytic vesicles.


b. engulf antigen.
c. release their histamine-containing granules to initiate inflammation.
d. produce lactoferrin to compete with microbes for iron.
e. undergo oxidative burst.

24. Successful immune responses to bacteria which resist phagocytosis


because of a polysaccharide capsule depends on the production of

a. active complement anaphylatoxins.


b. armed effector CTL.
c. neutralizing antibodies.
d. opsonizing antibodies.
e. oxidative burst.

25. Successful immune responses to bacteria which adhere to mucosal


surfaces in order to initiate infection depends on the production of
a. active complement.
b. armed effector CTL.
c. neutralizing antibodies.
d. opsonizing antibodies.
e. oxidative burst.

26. Successful immune responses to bacterial toxins depend on the


production of

a. active complement.
b. armed effector CTL.
c. neutralizing antibodies.
d. opsonizing antibodies.
e. oxidative burst.

27. In order for NK cells to do ADCC, they must bind

a. antibodies to virus proteins expressed on infected cell membranes


b. B7.
c. CD16.
d. toxic oxygen radicals.
e. virus peptides expressed on infected cell membrane MHC.

nolan x soju
28. NK cells kill their targets in ADCC using

a. complement-mediated lysis.
b. Fas and FasL.
c. oxidative burst.
d. perforins and granzymes.
e. all of the above.

29. Mast cells release their granule contents to stimulate inflammation in


response to

a. antigen binding to IgE on mast cell FcR.


b. antigen binding to IgG on mast cell FcR.
c. antigen binding to mast cell antigen receptors.
d. antigen binding to mast cell FcR.
e. IgE-coated antigen binding to mast cell FcR.

30. Successful immune responses to helminth parasites depends on the


production of
a. active complement.
b. armed effector CTL.
c. neutralizing IgA.
d. opsonizing IgE.
e. opsonizing IgG.

1. Innate immune responses are important in the initiation of adaptive responses


because

a. antigen cannot bind B cells until it has first bound complement.


b. inflammatory chemokines attract naïve B and T cells to the infection site to be
activated.
c. inflammatory cytokines and acute phase proteins stimulate the expression of co-
stimulatory molecules on dendritic cells.
d. lymphocytes cannot enter the secondary lymphoid organs until inflammatory
cytokines upregulate CAM expression on HEV.
e. T cells are stimulated to express co-stimulatory molecules by acute phase proteins.

2. Th0 differentiation into Th1 or Th2 cells is influences by all of the following EXCEPT

a. antigen density on the APC MHC.

nolan x soju
b. avidity of peptide binding to TCR.
c. cytokines being produced by nearby dendritic cells.
d. cytokines being secreted by nearby T cells.
e. whether the T0 cell has already begun to make IL-2.

3. Armed effector CTL find and kill their target cells in

a. draining lymph nodes.


b. the site of pathogen entry into the body.
c. the spleen.
d. whatever tissues have APC to provide co-stimulatory signals.
e. whatever tissues are infected.

4. Naïve lymphocytes enter the lymph nodes

a. any time they pass by the lymph node HEV.


b. any time inflammatory chemokines signal them to go to the lymph nodes.
c. only during infection.
d. only when their specific antigen is NOT present in the tissues.
e. only when their specific antigen is present.

5. Effector B cells (plasma cells) secrete antibody in the


a. bone marrow.
b. germinal centers.
c. primary follicles.
d. secondary follicles.
e. T cell follicles.

6. Immune memory is provided by persisting

a. antibody molecules.
b. effector CTL.
c. memory cells that can be activated by antigen to become effectors.
d. plasma cells
e. All of the above contribute to immune memory.

7. Lymphocytes which are part of the MALT do not generally recirculate through the
peripheral lymph nodes because

a. MALT lymphocytes do not recirculate.


b. the antigens for which they are specific cannot go to the lymph nodes.
c. the APC that activate them are found only in the MALT.
d. they are negatively selected against MALT self antigens in the bone marrow and
must stay in the MALT to avoid inducing autoimmunity
e. they bind better to MAdCAM on the mucosal vascular endothelium than they do to
CD34 on the lymph node HEV.

nolan x soju
8. All of the following are part of the gut-associated lymphoid tissue EXCEPT the

a. adenoids.
b. appendix.
c. enterocytes.
d. M cells.
e. Peyer's Patches.

9. The function of the M cells is to

a. induce apoptosis in food antigens.


b. kill infected enterocytes.
c. migrate to the Peyer's Patches and present antigen to T cells.
d. phagocytose or endocytose antigen.
e. secrete neutralizing IgA.

10. The mucosal equivalent of the lymph node, where naïve T and B cells are activated,
is the

a. appendix.
b. crypt.
c. MAdCAM-1 vesicle.
d. Peyer's Patches.
e. submucosal lymphoid tissue.

11. Plasma cells activated in the gut-associated lymphoid tissue secrete IgA in the

a. bone marrow.
b. lamina propria lymphoid tissues.
c. mucosa of gut, lungs, and genital tracts.
d. Peyer's Patches.
e. spleen

12. Specialized  T cells in the mucosal epithelium

a. are educated in the thymus to be tolerant to food antigens.


b. become  T cells when stimulated by Th1 cytokines.
c. kill infected enterocytes expressing MIC-1 and MIC-2.
d. recognize peptides presented on Class I MHC.
e. remove food antigens before they can be phagocytosed by M cells.

13. The principal effector function that eliminates intestinal pathogens is

a. CTL in the Peyer's Patches.


b. macrophages in the intestinal lumen.

nolan x soju
c. neutralizing secretory IgA in the mucus.
d. opsonizing secretory IgA on M cells.
e. tolerance.

14. Normal flora in the intestine

a. are neither harmful nor beneficial.


b. block adherence of pathogens by secreting toxins that kill them.
c. cannot cause disease.
d. induce no immune response unless they damage the intestinal epithelium.
e. None of the above is true.

15. Intestinal pathogens

a. cannot survive in the stomach because of the low pH there.


b. mimic food antigens to avoid the immune response.
c. must enter the body to cause disease.
d. stimulate inflammatory cytokine secretion to suppress immune responses.
e. use M cells as an entry way into the body.

16. Food antigens induce tolerance by


a. activating regulatory cells that suppress immune responses.
b. entering the body through enterocytes instead of M cells.
c. failing to induce inflammatory cytokine production.
d. inducing clonal proliferation in specific T cells.
e. Both a and c are correct.

17. A secondary humoral response

a. becomes detectable in serum overnight following antigen stimulation.


b. is T cell independent.
c. produces antibody of lower average affinity than a primary response
d. requires the presence of antibody from the primary response to initiate activation of
memory cells.
e. results in high serum levels of IgG.

18. Primary and secondary humoral immune responses do NOT differ in their

a. antibody affinities.
b. effector mechanisms.
c. kinetics.
d. peak antibody titers
e. ratio of IgM:IgG.

19. An Elvis impersonator comes to you with a sore throat. You run some tests and

nolan x soju
learn that his C3 levels are low, IgM levels are moderate, and IgG levels are very high.
This laboratory data tells you that your patient

a. does not have an infection, because complement levels rise during an infection.
b. has not previously been exposed to the infectious organism.
c. is probably undergoing a secondary immune response.
d. must have a bacterial infection, since viruses cannot induce an IgG response.
e. must be the King himself.

20. Original antigenic sin

a. activates memory cells more quickly than naïve cells.


b. indicates that memory responses suppress the activation of naïve cells.
c. occurs in response to eating apples.
d. occurs when an autoimmune response is produced.
e. only occurs during responses to influenza virus.

1. Pathogens evade immune detection by

a. altering their antigens periodically.


b. binding the Fc region of IgG to block complement activation and opsonization.
c. degrading IgA directed against them
d. living in the cytoplasm of host cells.
e. All of the above are ways pathogens evade immunity.

2. Which has NO effect on immune function?

a. Age.
b. Body temperature.
c. Nutrition.
d. Steroid hormones.
e. All of the above affect immune function.

3. Superantigens

a. avoid immune elimination by only activating T cells.


b. fit into the antigen-combining site of most TCR.
c. induce a very strong (super) immune response against the pathogens that produce
them.
d. induce CD4 T cells to produce cytokines that inhibit an immune response.
e. suppress immunity by inducing clonal proliferation.

4. A virus that can remain latent in a cell avoids elimination by

a. ADCC.
b. CTL.

nolan x soju
c. macrophages.
d. neutralizing antibodies.
e. opsonizing antibodies.

5. Staphylococcus aureus protein A, that binds the Fc region of Ig  chains, allows the
bacterium to avoid elimination by

a. ADCC.
b. antibody opsonization.
c. classical complement activation.
d. antibody-mediated phagocytosis.
e. All of the above.

6. A virus could avoid CTL recognition by inhibiting the expression of

a.2 microglobulin.
b. chaperones.
c. proteasomal enzymes.
d. TAP.
e. All of the above.

7. Mycobacterium leprae that induce a Th2 response avoid immune elimination by


a. activated macrophages.
b. ADCC.
c. antibodies.
d. CTL.
e. NK cells.

8. Tissue damage caused by the immune system responding to pathogens is usually


due to

a. cytotoxicity by CTL.
b. cytotoxicity by NK cells.
c. inflammation.
d. memory cells.
e. superantigens.

9. Expression of endogenous MMTV superantigen on host cells

a. activates host cells to produce virus.


b. deletes host T helper cells specific for superantigen.
c. eliminates the host immune response to MMTV.
d. both b and c are correct.
e. a, b, and c are correct.

10. HIV can kill CD4 T cells by

nolan x soju
a. diverting all their protein synthesis into virus production.
b. expressing gp120 on T cell membranes to make them targets for ADCC.
c. inducing apoptosis by binding to CD4 without being presented on Class II MHC.
d. inducing their lysis by HIV-specific CTL
e. All of the above probably occur.

11. A person has AIDS when s/he

a. becomes infected with HIV.


b. dies from HIV.
c. has fewer than 200 CD4 T cells/l of blood.
d. has more than 200 HIV particles/l of blood.
e. is seropositive for HIV.

12. Clinical latency means that

a. a balance exists between virus production and elimination.


b. anti-viral medications are still effective against HIV.
c. HIV is latent in host cells.
d. no disease symptoms are experienced.
e. no free virions are detectable in the blood.
13. People with AIDS

a. are always highly infectious.


b. die from opportunistic infections.
c. live a normal life-span if they take their medications on schedule.
d. rarely suffer side effects from the anti-viral medications.
e. would be expected to live on average another 10 years.

14. Current medications used to treat HIV infection interfere with all of the following
EXCEPT

a. CCR5.
b. integrase.
c. opportunistic infections.
d. protease.
e. reverse transcriptase.

15. All of the following contribute to the difficulty in developing an HIV vaccine EXCEPT
the

a. antigenic variation of the virus.


b. difficulty in inducing sterilizing immunity.
c. ethical considerations that require prevention and disease treatment during any
vaccine trial.

nolan x soju
d. need to induce opsonizing antibodies for protective immunity.
e. potential danger of a vaccine to HIV.

1. Combined cellular and humoral immune deficiencies result from lack of all of the
following EXCEPT

a. a thymus.
b. Class II MHC.
c. HIV infection of CD4 T cells.
d. RAG-1 or RAG-2 products.
e. TAP.

2. If Class II MHC is not expressed in the thymus, the resulting immune deficiencies
would include all of the following EXCEPT reduced

a. alternative complement activation.


b. CD8 T cell-mediated cytotoxicity.
c. macrophage activation to vesicular pathogens.
d. IgG synthesis.
e. All of the above would be involved.

3. Infants are most susceptible to bacterial infection due to low circulating levels of IgG

a. in utero (before birth).


b. at 0-3 months of age.
c. at 3-12 months of age.
d. at 12-24 months of age.
e. after the age of 2 years.

4. Chronic granulomatous disease results from a failure to perform oxidative burst. This
deficiency would be most likely to interfere with

a. CTL killing of viruses.


b. dendritic cell activation to become a mature APC.
c. infected cell processing of virus peptides.
d. macrophage intracellular killing of bacteria.
e. M cell uptake of mucosal antigens.

5. Bare lymphocyte syndrome due to lack of Class I MHC expression would be


expected to result in an inability to

a. activate Th1 cells to promote macrophage killing of vesicular pathogens.

nolan x soju
b. educate any T cells in the thymus.
c. make an inflammatory response to bacterial LPS.
d. produce any CD8 T cells.
e. process endogenous antigen for presentation.

6. X-linked Hyper IgM syndrome, resulting in high levels of serum IgM and low levels of
serum IgG, is caused by a defect in CD40L expression. The specific immune event that
would be prevented by a defective CD40L would be

a. activation of B cells by T-independent antigens.


b. failure of B cells to provide co-stimulation for Th2 activation.
c. failure of dendritic cells to provide co-stimulation for Th2 activation.
d. failure of Th2 cells to provide co-stimulation for B cell isotype switching.
e. failure of Th2 cells to provide co-stimulation for B cell proliferation.

7. DiGeorge's syndrome is characterized by the lack of a thymus. The mouse model


closest to this human disease would be a

a. knock-out mouse for RAG-1 and RAG-2.


b. knock-out mouse for a thymus.
c. nude mouse.
d. recombinant mouse for CD3.
e. SCID mouse.

8. A selective IgA deficiency would be expected to result in problems with

a. bacterial infections.
b. infections following dental work due to bacteria entering the bloodstream.
c. mucosal pathogens.
d. pathogens which can survive inside macrophages.
e. viral infections.

9. Bone marrow given to an infant with SCID must

a. be irradiated to eliminate GVHD.


b. contain hematopoietic stem cells that have been transduced with corrected copies of
defective genes.
c. contain mature T cells that can begin making immune responses immediately.
d. come from a donor that shares some MHC alleles with the recipient.
e. come from one of the child's parents.

10. Difficulties with somatic gene therapy arise from all of the following EXCEPT

a. GVHD caused by mature T cells in the transplanted cells.


b. inserting a gene so that it will function properly.

nolan x soju
c. limited life span of more mature hematopoietic cells.
d. obtaining enough stem cells.
e. transducing genetic material into stem cells.

1. Upon initial exposure to allergen, plasma cells secrete antigen-specific IgE that binds
to mast cell FcRI. The mast cells are said to be

a. activated.
b. allergenic.
c. anaphylactic.
d. sensitized.
e. tolerized.

2. An immediate allergic mediator released by mast cells is

a. epinephrine.
b. IgE.
c. IL-4.
d. histamine.
e. prostaglandin.
3. Humans probably make IgE responses because

a. IgE binds more efficiently to low doses of antigen than IgG


b. IgE is protective against dangerous pollens.
c. IgE triggers eosinophils to release products toxic to helminth parasites
d. their T cells were not properly tolerized to self IgE in the thymus.
e. they cannot produce enough IgG to protect themselves against allergens.

4. All of the following are Type I hypersensitivities EXCEPT

a. an allergy to peanuts.
b. an anaphylactic reaction to bee stings.
c. a blood transfusion reaction.
d. asthma induced by cat dander.
e. hay fever.

5. Fran walks outside on a beautiful day and takes a deep breath of ragweed pollen, to
which she has a strong Type I hypersensitivity. Which event below will NOT occur within
30 minutes due to this hypersensitivity?

a. A local inflammatory response in the nose is induced, resulting in a runny or stuffy


nose.
b. IgE specific for ragweed pollen is synthesized by B cells in the local lymph nodes.
c. Mast cells respond to the antigen-IgE signal by releasing preformed histamine

nolan x soju
d. Ragweed pollen antigen binds to IgE present on mast cell FcRI in the respiratory
tract.
e. Systemic effects of hypersensitivity such as anaphylactic shock may occur.

6. Type II hypersensitivities involve

a. anaphylactic shock.
b. complement-mediated lysis of antibody-coated cells.
c. cytotoxic T cell mediated lysis of antibody coated cells
d. chemotaxis of eosinophils.
e. IgE-mediated degranulation of mast cells.

7. All of the following are Type II hypersensitivities EXCEPT

a. a blood transfusion reaction to AB antigens on erythrocytes.


b. autoimmune hemolytic anemia, production of autoantibodies to erythrocyte antigens.
c. drug-induced hemolytic anemia, production of antibodies to medications which can
bind to erythrocytes.
d. Grave's disease, production of autoantibodies to TSH receptor on thyroid cells.
e. serum sickness, production of antibodies to passively administered foreign
antibodies.
8. Type II hypersensitivity results in all of the following EXCEPT

a. attraction and activation of inflammatory cells.


b. increased vascular permeability.
c. lysis of antibody coated cells by NK cells.
d. mediator release by CTL.
e. release of cytokines by macrophages.

9. A Type III hypersensitivity reaction is mediated by

a. antibody reacting with membrane antigen epitopes.


b. autoimmune reactions to self tissue antigens.
c. complement activation by immune complexes deposited in the blood vessel walls,
kidneys, and joints.
d. cytokine release by Th1 cells.
e. the cell-mediated branch of the immune system.

10. As he cleared brush near his home, Frank was bitten by a rattlesnake. He went to
the emergency room for treatment with horse IgG anti-rattlesnake venom. About a week
after the treatment, Frank experienced a rash, fever, swollen lymph nodes, and pains in
his joints, all symptoms of serum sickness. These symptoms are probably due to

a. a T cell memory response to horse IgG.


b. cross reactivity between horse IgG and human IgG.

nolan x soju
c. late phase damage caused by the rattlesnake venom
d. production of IgG anti-horse immunoglobulin, which triggered a Type III
hypersensitivity.
e. production of IgG anti-rattlesnake venom, which triggered a Type III hypersensitivity.

11. In the situation described in Question 10 above, Frank can be treated with

a. antiserum to complement to block its activation.


b. human anti-horse IgG to more quickly clear the horse antibody.
c. immunosuppressive drugs to block B cell production of antibody.
d. plasmapheresis to remove antigen-antibody complexes from the blood.
e. rattlesnake venom to absorb the horse anti-venom antibody.

12. Type IV hypersensitivity (DTH)

a. can be passively transferred with CD4 T cells.


b. causes chicken pox.
c. involves cell damage induced by IgG antibodies which are produced late in an
immune response.
d. is mediated by memory macrophages.
e. occurs 1-2 weeks after antigen exposure.
13. During Delayed Type Hypersensitivity reactions, macrophages

a. are not antigen specific.


b. are stimulated by IFN.
c. do not depend on antibody for antigen recognition.
d. kill neighboring cells, whether infected or not.
e. all of the above are true.

14. A positive skin reaction to tuberculin means that one has

a. an active case of tuberculosis.


b. an allergy to Mycobacterium tuberculosis.
c. antibodies specific for M. tuberculosis.
d. macrophages containing M. tuberculosis in their phagolysosomes.
e. memory CD4 T cells specific for M. tuberculosis.

15. A common feature of all hypersensitivities is

a. activation of CTL.
b. activation of Th2 cells.
c. antibody synthesis.
d. inflammation.
e. all of the above.

nolan x soju
1. All of the following probably contribute to the development of autoimmunity EXCEPT

a. cross-reactivity of pathogen and self antigens.


b. expression of self antigen in the thymus or bone marrow.
c. low avidity presentation of some self peptides in the thymus.
d. random generation of TCR and BCR specificities.
e. tissue injury which releases normally hidden self antigens.

2. The elimination of self-reactive T cells during thymus education is called

a. apoptosis.
b. clonal selection.
c. clonal anergy.
d. negative selection.
e. positive selection.

3. An autoimmune disease mediated by Type III hypersensitivity is

a. autoimmune hemolytic anemia caused by anti-A blood group antigen.


b. Grave's Disease caused by anti-TSH receptor antibodies.
c. IDDM caused by CTL to islet antigens.
d. Lyme arthritis caused by immune complexes resulting from a persistent infection with
Borrelia burgdorferi.
e. Myasthenia gravis, caused by antibodies to the acetylcholine receptor

4. Tolerance is induced by all of the following EXCEPT

a. administration of antigen with adjuvant.


b. clonal anergy of self-reactive mature T cells which bind antigen without co-
stimulation.
c. clonal deletion of self-reactive immature B cells whose BCR is extensively cross-
linked by antigen.
d. failure of particular MHC alleles to present certain peptides.
e. regulatory T cells that suppress immune responses.

5. Autoimmune diseases which are Type II hypersensitivities include all of the following
EXCEPT

a. Goodpasture's syndrome cause by autoantibodies to Type IV collagen.


b. insulin-resistant diabetes caused by autoantibodies to insulin receptor.
c. multiple sclerosis caused by Th1 cells specific for proteins in the brain and spinal
cord.
d. pemphigus vulgaris caused by autoantibodies to epidermal cadherin.
e. rheumatic fever caused by antibodies to streptococcal antigens that cross-react with

nolan x soju
heart tissue.

6. Induction of autoimmunity usually involves

a. activation of antigen-specific T cells.


b. anergy of antigen-specific B cells.
c. development of autoimune disease in everyone with certain HLA alleles.
d. occurrence of infection in the absence of inflammation.
e. recruitment of antigen-specific dendritic cells to T cell areas of the lymph nodes.

7. The chances of developing a given autoimmune disease are linked to all of the
following EXCEPT

a. contracting an infectious disease.


b. gender.
c. having an identical twin with the disease.
d. inheriting certain HLA alleles.
e. total absence of thymus education.

8. Autoantibodies to a cell surface receptor


a. bind the cell to mimic the action of the natural ligand for that receptor
b. block cell activation via that receptor.
c. cause the destruction of the cell by ADCC.
d. cause the destruction of the cell via complement-mediated lysis.
e. All of the above are possible consequences.

9. Organ-specific autoimune diseases include all of the following EXCEPT

a. autoimmune hemolytic anemia.


b. Hashimoto's thyroiditis.
c. insulin-dependent diabetes mellitus (IDDM).
d. myasthenia gravis.
e. rheumatoid arthritis.

10. Increased risk of developing Type 1 diabetes (IDDM) associated with HLA DR3 and
DR4

a. can be lowered by the presence of HLA DR2.


b. has been discovered to be due to a genetic mutation in those alleles induced by
closely linked DQ.
c. means that everyone with those HLA alleles will develop diabetes during his or her
lifetime.
d. occurs because DR3 and DR4 cannot present self antigen in the thymus.
e. protects individuals with IDDM from malaria.

nolan x soju
11. Of the following, it is usually the easiest to identify specific

a. antigens for which autoimmune T cells are specific.


b. antigens to which autoantibodies are being produced.
c. environmental factors which promoted development of autoimmunity.
d. pathogen antigens which initiated the autoimmune response.
e. All of the above are equally easy to determine.

12. Determinant spreading occurs when

a. antigenic epitopes move from one cell to another.


b. B cells specific for one epitope provide co-stimulation to Th2 cells specific for different
epitopes.
c. CTL killing leads to release of new antigens to which B cells can produce antibodies.
d. Th2 cells specific for one epitope on an antigen complex activate B cells specific for
several different epitopes on that same complex.
e. Answers b, c and d are examples of how determinant spreading could occur.

13. Feeding of myelin basic protein (MBP) to mice prevents them from later developing
experimental autoimmune encephalomyelitis when they are injected with MBP and
adjuvant. The mechanism by which antigen feeding prevents EAE is though to be
a. activation of regulatory T cells producing IFN.
b. activation of regulatory T cells producing TGF.
c. clonal anergy of all MBP-specific T cells.
d. clonal deletion of all MBP-specific T cells.
e. induction of an IgE response instead of an IgG response to MBP.

14. Immunological ignorance to an antigen occurs when

a. all lymphocytes specific for that antigen have undergone clonal deletion or clonal
anergy.
b. the antigen is absent from the body (non-self).
c. the antigen is presented on self MHC with such high avidity that specific T cells are
activated to undergo apoptosis.
d. the antigen is presented on self MHC at such low avidity that specific T cells are not
activated.
e. the antigen is sequestered in the cell cytoplasm.

15. Self-specific B cells are not generally responsible for the induction of autoimmunity
because they

a. are not present in the body.


b. can be activated by self antigens but not produce any self-specific antibodies.
c. cannot be activated by binding cell surface antigen.
d. require activation by self-specific T cells, whose activates are more tightly regulated

nolan x soju
than those of B cells.
e. undergo somatic recombination, which eliminates self-specific B cells.

16. Immune deviation might alleviate autoimmune disease by

a. activating cytotoxic T cells to become regulatory T cells.


b. distracting the immune system to respond to a different, less harmful self antigen.
c. substituting allogeneic T cells for autoimmune T cells via bone marrow
transplantation.
d. switching the autoimmune response from a Th1 response to a less damaging Th2
response.
e. targeting the immune response to an immunologically privileged site.

17. The link between infection and autoimmunity is related to four of the following
observations. Find the exception.

a. Antigens on pathogens may induce the production of antibodies which bind self
antigen.
b. Infection stimulates the production of inflammatory cytokines which induce dendritic
cells to express more B7.
c. Infection stimulates the production of inflammatory cytokines which induce dendritic
cells to express more MHC molecules.
d. Molecular mimicry of self antigen by pathogens antigen results in clonal deletion of
self-reactive T cells that might otherwise cause autoimmunity.
e. People who suffer more parasite infections are less likely to develop SLE than people
of similar genetic backgrounds who are not infected by those parasites.

1. Cancer is caused by

a. chemicals.
b. radiation.
c. spontaneous mutations.
d. viruses.
e. all of the above.

2. Tumor-associated antigens CANNOT be

a. expressed on non-tumor cells.


b. onco-fetal antigens.
c. over-expressed self antigens.
d. presented on MHC Class I.
e. tumor-specific.

3. Tumors do NOT escape immune surveillance by

nolan x soju
a. downregulating their expression of MHC Class I.
b. expressing antigens which are not found on other self tissues.
c. failing to express B7.
d. producing immunosuppressive cytokines like TGF.
e. shedding their membrane antigens.

4. A virus molecule expressed on the membrane of a virus-induced tumor cell is an


example of a(n)

a. angiogenic molecule.
b. carcinoembryonic antigen.
c. proto-oncogene.
d. tumor-associated antigen.
e. tumor-specific antigen.

5. Proto-oncogenes can be activated to become oncogenes and cause cancer by

a. carcinogens in cigarette smoke.


b. overexposure to UV radiation in sunlight.
c. spontaneous mutations.
d. virus infection.
e. all of the above.

6. Enhancing antibodies

a. enhance the ability of CTL to eliminate tumor cells.


b. enhance tumor growth by blocking access of CTL to tumor cells.
c. kill tumor cells when administered to the cancer patient.
d. promote APC uptake of tumor antigen via FcR.
e. promote tumor detection when used to generate a HAMA response.

7. Experimental approaches that may result in new immunotherapies for cancer include
all of the following EXCEPT

a. monoclonal antibodies to tumor antigens coupled with cytotoxic drugs.


b. patient dendritic cells pulsed with tumor heat shock proteins.
c. vaccines containing live tumor cells to induce vigorous CTL responses.
d. vaccines containing tumor cells transfected with a gene for B7.
e. vaccines containing tumor cells transfected with an antisense gene for TGF.

1. John Doe needs a kidney transplant. The best MHC match for John would probably
be

nolan x soju
a. an unrelated cadaver.
b. his brother
c. his cousin.
d . his mother.
e. his wife.

2. The serological tissue typing test is

a. an ELISA for donor antibodies to recipient MHC antigens.


b. the best test to predict if the recipient will reject the donor tissue.
c. used to match Class I MHC for a heart transplant.
d. used to match Class I MHC for a kidney transplant.
e. used to match Minor H antigens between donor and recipient.

3. One of the ways by which allograft rejection is prevented is through administration of

a. antibodies to CD3.
b. antibody to the foreign MHC.
c. IL-2.
d. IgE.
e. Rhogam.
4. Hyperacute graft rejection

a. always occurs if HLA-B or HLA-DR alleles are mismatched.


b. can be controlled with Cyclosporin A and anti-CD3.
c. causes most transplantation failures.
d. is due to the presence of antibodies against graft surface antigens.
e. is due to T cell recognition of more than three differences in MHC alleles.

5. The best test to determine whether a kidney graft from a relative would be rejected
would be

a. agglutination of donor kidney cells by anti-MHC antibodies from the recipient.


b. 51Cr release assay using donor target cells and recipient T cells.
c. ELISA of donor kidney molecules with enzyme-labeled recipient anti-kidney
antibodies.
d. flow cytometry of donor cells using antibodies to recipient MHC alleles.
e. limit dilution assays of the frequency of recipient cells which will recognize the donor
HLA.

6. Acute rejection occurs due to

a. activation of recipient B cells by donor MHC on recipient Th2 cells.


b. activation of recipient Tc cells by donor MHC on donor APC.
c. activation of recipient Tc cells by donor MHC on recipient APC.

nolan x soju
d. activation of recipient Th1 cells by donor MHC on donor APC.
e. activation of recipient Th1 cells by donor minor H antigens on recipient APC.

7. All of the following statements about tissue matching are true EXCEPT

a. Many HLA antigens have not yet been identified.


b. Minor H antigens are often not encoded in the MHC.
c. The serological typing test can report a match when HLA antigens are different.
d. Tissue between HLA-identical siblings can sometimes be rejected.
e. Time for bone marrow typing is limited by the ability of the marrow cells to survive
outside the donor.

8. Xenografts of pig organs into people has so far failed because

a. high numbers of human T cells recognize pig MHC as foreign.


b. humans are allergic to pig tissue.
c. humans have preexisting antibodies to pig carbohydrates.
d. humans have preexisting antibodies to pig MHC.
e. pig organs are too small for most humans.

9. Long-term allograft survival is affected by all of the following EXCEPT


a. cause of the original organ failure.
b. CMV infections.
c. number of HLA antigens that are matched.
d. pre-formed antibodies to alloantigens.
e. toxicity of the anti-rejection drugs.

10. Possible immunotherapeutic strategy for treating autoimmunity would NOT include
administration of

a. adjuvants to tolerize APC.


b. antibodies to CD3.
c. azathioprine to block T cell division.
d. Cyclosporin A to block IL-2 production during T cell activation.
e. prednisone to reduce inflammation.

11. Bone marrow transplantation differs from organ transplantation because

a. activation of Tc cells by foreign HLA is the principal mechanism of acute rejection.


b. minor H antigens do not occur on marrow cells and do not contribute to rejection.
c. mature Tc cells in the graft can recognize and reject recipient tissue.
d. the donor is alive and can give permission for the transplant.
e. the recipient has no immune system and cannot reject the donor marrow cells..

12. The developing fetus is not rejected by the pregnant mother because

nolan x soju
a. the fetus has no antigens that are foreign to the mother's immune system.
b. the mother cannot make antibodies to paternal HLA antigens.
c. the mother's T cells are never exposed to fetal antigens.
d. trophoblast cells express MHC identical to that of the mother.
e. trophoblast cells secrete immunosuppressive cytokines.

13. Side effects of immune suppression include all of the following EXCEPT

a. cancer.
b. hair loss.
c. HAMA response.
d. infections.
e. weight gain.

14. Prednisone is a(n)

a. anti-inflammatory drug.
b. cholesterol drug.
c. cytotoxic drug.
d. drug that blocks T cell activation.
e. steroid inhibitor.
15. It is easier to treat transplant rejection than to treat autoimmunity because

a. HLA antigens are better characterized than autoantigens.


b. mechanisms for graft rejection are better understood than those for autoimmunity.
c. more drugs are available for controlling rejection than for controlling autoimmunity.
d. once an immune response is established it is harder to block.
e. the foreign antigens can be removed with a transplant but not with a self antigen.

1. Advantages of the oral polio vaccine compared with the killed polio vaccine are all of
the following EXCEPT that it

a. elicits IgA as well as IgG synthesis.


b. induces cellular as well as humoral immunity.
c. induces secretion of protective mucosal neutralizing antibodies.
d. is less expensive to administer.
e. is safer to give to immunosuppressed children.

2. To make a recombinant vaccine to a virus, the virus DNA is expressed

a. covalently linked with a polysaccharide.

nolan x soju
b. in a genetic variant of the same virus.
c. in a less virulent vector, which may be a virus or a bacterium.
d. in muscle cells of the immunized person.
e. on an ISCOM.

3. An advantage of giving passive immunization is that it provides ___________


immunity.

a. cellular
b. faster
c. longer lasting
d. more specific
e. safer

4. Bacterial polysaccharide vaccines are conjugated to proteins so that

a. the polysaccharide can act as an adjuvant to induce better immunity to the protein.
b. the polysaccharide looks less like blood typing antigens to the immune system.
c. the protein can act as an adjuvant to upregulate B7 expression on APC.
d. the protein can stimulate T cell help for polysaccharide-specific B cells.
e. none of the above.
5. Hepatitis B surface antigen vaccine, a recombinant vaccine produced in yeast cells,

a. cannot be given to immunosuppressed children.


b. employs the yeast cells as an adjuvant to increase its immunogenicity.
c. is less safe to administer than killed Hepatitis B virus vaccine.
d. is more immunogenic than the whole Hepatitis virus would be.
e. requires several boosters because it is also a subunit vaccine.

6. Passive immunization to a bacterial toxin would be recommended if

a. exposure to the toxin had already occurred in an unimmunized person.


b. the person who needed the immunization had been recently immunized with the
toxoid.
c. the toxin could not reproduce in the body.
d. the toxin was not immunogenic.
e. the toxoid was available only as part of a conjugate vaccine.

7. ISCOMs

a. are DNA vaccines.


b. are safe replicating vectors for recombinant vaccines.
c. stimulate APC to upregulate their B7 levels.
d. transport peptide vaccines into APC cytoplasm so they can be presented on Class I
MHC.

nolan x soju
d. transport killed viruses into T cell cytoplasm so they can induce cellular immunity.
1. Which cells in the body are involved in the antibody-mediated allergic reactions?
a. macrophage and NK cell
b. mast cells and basophile
c. Gamma/delta T cells
d. any phagocytic cell

2. Antibody effector functions include all of the following EXCEPT


a. activator of complement on the bacterial surface
b. activation of proteolysis of endocytosed proteins
c. blocking uptake of bacterial toxins
d. coating bacteria to promote their phagocytosis

3. Isotype switching resembles somatic recombination because both processes


a. are catalyzed by the products of RAG I and RAG 2
b. are regulated by helper T cell cytokines
c. occur in developing B cells in the bone marrow
d. result in the loss of DNA from the B cell

nolan x soju
4. Identify an important cytokine receptor interaction which initiates extensive cell division of the
primed T-cell
a. IL-2and a high affinity IL-2 receptor
b. IFN-gamma and the IFN gamma receptor
c. IL-7 and a high affinity IL-7 receptor
d. IL-7 and low affinity IL-7 receptor

5. Just before they enter the blood circulation from the lymphatic system, circulating lymphocytes
will be found in
a. A high endothelial venule
b. A lymph node
c. the thoracic duct
d. the bone marrow

6. Activation of mast cell by anaphylatoxins (C3a and C5a) causes


a. apoptosis of mast cell
b. presentation of the antigen to Th cells
c. secretion of IgE antibodies
d. secretion of allergic mediators
7. It was observed that milkmaids, infected with cowpox, were later immune to smallpox infections
This is an example of a(n)
a. innate immunity to a cross-reactive antigen
b. innate immunity of milkmaids to smallpox
c. memory response to a cross-reactive antigen
d. passive immunization from contact with cow’s milk antibodies

8. Inflammatory cytokines produced by macrophages activate all of the following EXCEPT


a. complement system via classical pathway
b. integrin on Leukocytes to bind more strongly to vascular CAMs
c. neutrophils to be more cytotoxic
d. NK cells to kill virus-infected cells

9. Natural killer cells


a. Are inhibited to kill infected host cells by MHC class I molecules
b. Are activated to kill infected host cells by MHC Class I molecules
c. Kill virus infected cells when the virus is acquired naturally but not by immunization
d. Secrete the complement MAC to lyse virus infected cells

nolan x soju
10. Engagement of the Fas receptor by Fas ligand (FasL) results in
a. apoptotic cell death
b. clonal proliferation
c. Cytotoxicity
d. somatic hypermutation

11. For the generation of TCR diversity T cells use


a. isotype switching
b. affinity maturation
c. N region addition of nucleotides
d. somatic hypermutation

12. Which of the following human cell types are NOT capable of presenting peptides using MHC
Class I?
a. fully mature lymphocytes
b. Dendritic cells
c. red blood cells
d. macrophages
13. what immune function is likely to be affected in case of mutation in the Fc gamma receptor
gene?
a. Opsonization
b. Lymphocyte adhesion
c. Antigen binding
d. Intracellular killing

14. Priming of naive T-helper cells by mature dendritic cells m the lymph node involves
a. CD40 on DC with CD40L on naive T-cell
b. CTLA-4 on T-cell with peptide presented by DC
c. CD56 on naive T-cell with CD28 on DC
d. CD8 on naive T-cell with MHC Class I on DC

nolan x soju
A CD3+ cell that secretes IL-2 when activated is a

A. Regulatory T-cell

B. Helper T-cell

C. Cytotoxic T-cell

D. Gamma/delta T-cell

Viruses cause tumors in “male mice” (which do not have thymus , because of a genetic defect ) but
not on normal mice. The BEST interpretation is that

A. Macrophages are required to reject poliomavirus-induced tumors

B. Natural killer cells can reject poliomavirus-induced tumors without help from T lymphocytes

C. T lymphocytes play an important role in rejection of poliomavirus-induced tumors

D. B-lymphocytes play no role in rejection of poliomavirus-induced tumors

Antibody production by B-lymphocytes is initiated contact between B-lymphocyte antigen receptor


and the antigen for which that receptor is specific, and which one of the following additional
requirements

A. Help from a T-helper cell that has been stimulated by MHC Class 1-pres…. Peptide

nolan x soju
B. Contact between the Fc portion of the antibody and an Fc receptor on the surface of a T-
helper2 (TH2) lymphocyte

C. Help from a macrophage that is presenting peptides using MHC Class ll molecules

D. Help from a T-helper 2(TH2) lymphocyte

A patient is admitted with multiple bacterial infections and is forced to have a complete absence of
CS. Which complement enhanced function would be mostly affected in such a patient

A. Lysis of pathogen

B. Generation of neutrophil chemotactic factors

C. Generation of anaphylatoxins

D. Opsonization of bacteria

Antibody mediated allergic reactions involve the attachment of one of the five isotypes to which one
of the following types of cells in the body ?

A. Macrophage ( via Fc receptors for IgG isotype)


B. Mast cells

C. NK cells

D. Any phagocytic cell

Regarding anaphylactic (type I) and immune complex (type III) hypersensitivities which of the
following is the MOST accurate?

A. IgE is involved in both anaphylactic and immune complex hypersensitivities

B. Complement is involved in both anaphylactic and immune complex hypersensitivities

C. Less antigen is typically needed to trigger anaphylactic reaction than in immune complex
reaction

D. Neutrophils play more important role in anaphylactic reaction than immune complex
reactions

In the absence of antibodies, phagocytes are not able to respond effectively against

A. Capsulated bacteria

B. Bacterial cells to which the phagocytes firmly attach

C. Bacteria that are present in the body but outside of the blood vessels

nolan x soju
D. Protozoa

The first line of defence against viruses is

A. IgA antibodies directed against viruses at mucus surfaces

B. IgG directed to an internal viral antigens

C. IgM directed to an external viral antigens

D. Interferon-gamma

Much more rapid appearance of antibody during a secondary response, compared to the primary
response, is due to which one of the following?

A. A much improved presentation of peptide antigen to T-helper 1 cells by B-lymphocytes


during the secondary response

B. Much higher number of B-lymphocytes at the beginning of the secondary response specific
for the antigen that stimulated both the primary and secondary antibody response
C. Improved binding of antibody molecules to Fc receptors on the surface of phagocytes which
leads to a higher rate of peptide presentation to B-lymphocytes

D. Each B-lymphocyte participating in a secondary antibody response is specific for a very large
number of different antigens, which each B-cell participating in the primary response is
specific for only one antigen

Identify the fundamental differences between the antigen recognition by H and T cells

A. Antigen must be presented in different ways

B. Clonal selection after antigen recognition

C. Heterogenicity of TCR and BCR from one lymphocyte to the next

D. Membrane location of antigen-specific receptors

Almost immediately after a naïve T-cell has been primed, it will begin to express an important
receptor on its surface that will respond to a cytokine that is secreted by the same recently primed
T-cell. This cytokine receptor interaction will initiate extensive cell division of that primed T-cell. This
important cytokine -receptor interaction in which one of the following?

A. IL- 1 and low affinity IL-1 receptor

B. IL-7 and a high affinity IL-7 receptor

nolan x soju
C. Interferon-gamma and the interferon-gamma receptor

D. IL-2 and a high affinity IL-2 receptor

Erythematosus is

A. Free from an immune complex disease component

B. Associated with antibodies against nucleic acid

C. A disease of red and white cells

D. An organ specific autoimmune disease

Interferon-gamma

A. Is synthesised by macrophages

B. Is synthesised by erythrocytes

C. Is released as a consequence of antigen- or mitogen induced of T-lymphocytes

D. Specifically binds to the antigen that induces its release


Which of the following direct consequence of antibody binding to antigen

A) Activation of B cells

B) Neutralization of antigen

C) Activation of complement

D) Phagocytosis of antigen

Which one of the following statements regarding various diseases is true

A) Tuberculoid leprosy is associated with a predominant Th2 profile

B) Epithelioid cells and giant cells are typical of immune complex diseases

C) Lepromatous leprosy is associated with a predominant Th1 profile

D) Crohns disease is an inflammatory disease in which granulomatous ??? penetrating fistulas


into other organs

Grafts between genetically identical individuals (i.e identical twins)

A) Are subject to acute rejection

B) Are subject to hyperacute rejection

C) Are not rejected even without immunosuppression

nolan x soju
D) Are not rejected if a kidney is grafted but skin grafts are rejected

Which class of antibodies is responsible for ??? a virus AND then contributes to the destruction of
that virus by phagocytosis

A) IgM

B) IgG

C) IgA

D) IgE

The following mechanisms may be involved in the clinical efficacy of injection therapy

A) Enhanced production of IgG, which binds allergens before it reaches mast cells

B) Enhanced production of IgE, which binds IgE receptor before it reaches antigen

C) Activation of mast cells that induces production of a large amount of anti-histamines

D) Increased local recruitment of eosinophils


Which of the following human cell types would not be capable of presenting peptides using MHC
class I

A) Macrophages

B) Fully mature red blood cells

C) Dendritic cells

D) B-lymphocytes

Which of the following statements as (are) characteristic of contact sensitivity

A) The best therapy is administration of the antigen

B) Patch testing with the allergens is useless for diagnosis

C) Sensitization can be passively transferred with serum from an allergic individual

D) Some chemicals acting haptens induce sensitivity by covalently binding to host proteins
acting as carriers

Patients with DiGeorge syndrome

A) Have increased number of suppressor T cells

B) Have selective IgA deficiency

nolan x soju
C) Have abnormal antigen presenting cells

D) Have a decreased number of helper T cells

Which of the following clinical diseases is most likely to involve a reaction to a hapten in its etiology

A) Goodpasture’s syndrome

B) Hemolytic anemia after treatment with penicillin

C) Rheumatoid arthritis

D) Serum sickness

What is the major consequence of the bone marrow transplantation in immunocompromised


patients

A) Potentially lethal graft versus host disease

B) High risk of T cell leukemia

C) Inability to use a live donor

D) Delayed hypersensitivity
Immunologic tolerance is defined as

A) The removal of microbes expressing the antigen, so that the host can tolerate the infection

B) Activation of only B cells and not T cells on exposure to an antigen

C) Unresponsiveness of the immune system to an antigen which is induced by previous


exposure to that antigen

D) The production of memory B cells and T cells on primary exposure to antigen

T cell mediated immune responses cannot result in

A) Formation of granulomas

B) Inflammation at the reaction site

C) Rejection of a kidney transplant

D) Production of antibodies

nolan x soju
387. Which category of hypersensitivity BEST describes hemolytic disease of the
newborn caused by Rh incompatibility?
A. atopic or anaphylactic
B. cytotoxic
C. immune complex
D. delayed

388. The principal difference between cytotoxic (type II) and immune complex
(type III) hypersensitivity is
A. the class (isotype) of antibody.
B. the site where antigen-antibody complexes are formed.
C. the participation of complement.
D. the participation of T cells.

389. A child stung by a bee experiences respiratory distress within minutes and
lapses into unconsciousness. This reaction is probably mediated by
A. IgE antibody.
B. IgG antibody.
C. sensitized T cells.
D. complement.
E. IgM antibody.

nolan x soju
390. A patient with rheumatic fever develops a sore throat from which beta-
hemolytic streptococci are cultured. The patient is started on treatment with
penicillin, and the sore throat resolves within several days. However, 7 days after
initiation of penicillin therapy the patient develops a fever of 103°F, a
generalized rash, and proteinuria. This MOST probably resulted from
A. recurrence of the rheumatic fever.
B. a different infectious disease.
C. an IgE response to penicillin.
D. an IgG-IgM response to penicillin.
E. a delayed hypersensitivity reaction to penicillin.

391. A kidney biopsy specimen taken from a patient with acute


glomerulonephritis and stained with fluorescein-conjugated anti-human IgG
antibody would probably show
A. no fluorescence.
B. uniform fluorescence of the glomerular basement membrane.
C. patchy, irregular fluorescence of the glomerular basement membrane.
D. fluorescent B cells.
E. fluorescent macrophages.
392. A patient with severe asthma gets no relief from antihistamines. The
symptoms are MOST likely to be caused by
A. interleukin-2.
B. slow-reacting substance A (leukotrienes).
C. serotonin.
D. bradykinin.

393. Hypersensitivity to penicillin and hypersensitivity to poison oak are both


A. mediated by IgE antibody.
B. mediated by IgG and IgM antibody.
C. initiated by haptens.
D. initiated by Th-2 cells.

394. A recipient of a 2-haplotype MHC-matched kidney from a relative still needs


immunosuppression to prevent graft rejection because
A. graft-versus-host disease is a problem.
B. minor histocompatibility antigens will not be matched.
C. minor histocompatibility antigens will not be matched.
D. complement components will not be matched.

nolan x soju
395. Bone marrow transplantation in immunocompromised patients presents
which major problem?
A. potentially lethal graft-versus-host disease
B. high risk of T cell leukemia
C. inability to use a live donor
D. delayed hypersensitivity

396. What is the role of class II MHC proteins on donor cells in graft rejection?
A. They are the receptors for interleukin-2, which is produced by macrophages
when they attack the donor cells.
B. They are recognized by helper T cells, which then activate cytotoxic T
cells to kill the donor cells.
C. They induce the production of blocking antibodies that protect the graft.
D. They induce IgE which mediates graft rejection.

397. Grafts between genetically identical individuals (i.e., identical twins)


A. are rejected slowly as a result of minor histocompatibility antigens.
B. are subject to hyperacute rejection.
C. are not rejected, even without immunosuppression.
D. are not rejected if a kidney is grafted, but skin grafts are rejected.
398. Penicillin is a hapten in both humans and mice. To explore the hapten-
carrier relationship, a mouse was injected with penicillin covalently bound to
bovine serum albumin and, at the same time, with egg albumin to which no
penicillin was bound. Of the following, which one will induce a secondary
response to penicillin when injected into the mouse 1 month later?
A. penicillin
B. penicillin bound to egg albumin
C. egg albumin
D. bovine serum albumin

399. AIDS is caused by a human retrovirus that kills


A. B lymphocytes.
B. lymphocyte stem cells.
C. CD4-positive T lymphocytes.
D. CD8-positive T lymphocytes.

400. Chemically-induced tumors have tumor-associated transplantation antigens


that
A. are always the same for a given carcinogen.
B. are different for two tumors of different histologic type even if
induced by the same carcinogen.
C. are very strong antigens.

nolan x soju
D. do not induce an immune response.

401. Polyomavirus (a DNA virus) causes tumors in "nude mice" (nude mice do
not have a thymus, because of a genetic defect) but not in normal mice. the
BEST interpretation is that
A. macrophages are required to reject polyomavirus-induced tumors.
B. natural killer cells can reject polyomavirus-induced tumors without help from T
lymphocytes.
C. T lymphocytes play an important role in the rejection of
polyomavirus-induced tumors.
D. B lymphocytes play no role in rejection of polyomavirus-induced tumors.

402. C3 is cleaved to form C3a and C3b by C3 convertase. C3b is involved in all
of the following EXCEPT
A. altering vascular permeability.
B. promoting phagocytosis.
C. forming alternative-pathway C3 convertase.
D. forming C5 convertase.

403. After binding to its specific antigen, a B lymphocyte may switch its
A. immunoglobulin light-chain isotype.
B. immunoglobulin heavy-chain class.
C. variable region of the immunoglobulin heavy chain.
D. constant region of the immunoglobulin light chain.

404. Diversity is an important feature of the immune system. Which one of the
following statements about it is INCORRECT?
A. Humans can make antibodies with about 108 different VH X VL combinations.
B. A single cell can synthesize IgM antibody, then switch to IgA antibody.
C. The hematopoietic stem cell carries the genetic potential to create more than
104 immunoglobulin genes.
D. A single B lymphocyte can produce antibodies of many different
specificities, but a plasma cell is monospecific.

405. C3a and C5a can cause


A. bacterial lysis.
B. vascular permeability.
C. phagocytosis of IgE-coated bacteria.
D. aggregation of C4 and C2.

406. Neutrophils are attracted to an infected area by


A. IgM.
B. vascular permeability.
C. phagocytosis of IgE-coated bacteria.

nolan x soju
D. aggregation of C4 and C2.

407. Complement fixation refers to


A. the ingestion of C3b-coated bacteria by macrophages.
B. the destruction of complement in serum by heating at 56°C for 30 minutes.
C. the binding of complement components by antigen-antibody
complexes.
D. the interaction of C3b with mast cells.

408. The classic complement pathway is initiated by interaction of C1 with


A. antigen.
B. factor B.
C. antigen-IgG complexes.
D. bacterial lipopolysaccharides.

409. Patients with severely reduced C3 levels tend to have


A. increased numbers of severe viral infections.
B. increased numbers of severe bacterial infections.
C. low gamma globulin levels.
D. frequent episodes of hemolyticanemia.
410. Individuals with a genetic deficiency of C6 have
A. decreased resistance to viral infections.
B. increased hypersensitivity reactions.
C. increased frequency of cancer.
D. decreased resistance to Neisseria bacteremia.

411. Natural killer cells are


A. B cells that can kill without complement.
B. cytotoxic T cells.
C. increased by immunization.
D. able to kill virus-infected cells without prior sensitization.

412. A positive tuberculin skin test (a delayed hypersensitivity reaction) indicates


that
A. a humoral immune response has occurred.
B. a cell-mediated immune response has occurred.
C. both the T and B cell systems are functional.
D. only the B cell system is functional.

413. Reaction to poison ivy or poison oak is


A. an IgG-mediated response.
B. an IgE-mediated response.
C. a cell-mediated response.
D. an Arthus reaction.

nolan x soju
414. A child disturbs a wasp nest, is stung repeatedly, and goes into shock within
minutes, manifesting respiratory failure and vascular collapse. This is MOST likely
to be due to
A. systemic anaphylaxis.
B. serum sickness.
C. an Arthus reaction.
D. cytotoxic hypersensitivity.

415. "Isotype switching" of immunoglobulin classes by B cells involves


A. simultaneous insertion of VH genes adjacent to each CH gene.
B. successive insertion of a single VH gene adjacent to different CH
genes.
C. activation of homologous genes on chromosome 6.
D. switching of light-chain types (kappa and lambda).

416. Which one of the following pairs of genes is linked on a single chromosome?
A. V gene for lambda chain and C gene for kappa chain
B. C gene for gamma chain and C gene for kappa chain
C. V gene for lambda chain and V gene for heavy chain
D. C gene for gamma chain and C gene for alpha chain
417. Idiotypic determinants are located within
A. hypervariable regions of heavy and light chains.
B. constant regions of light chains.
C. constant regions of heavy chains.
D. the hinge region.

418. A primary immune response in an adult human requires approximately how


much time to produce detectable antibody levels in the blood?
A. 12 hours
B. 3 days
C. 1 week
D. 3 weeks

419. The membrane IgM and IgD on the surface of an individual B cell
A. have identical heavy chains but different light chains
B. are identical except for their CH regions
C. are identical except for their VH regions
D. have different VH and VL regions

nolan x soju
420. During the maturation of a B lymphocyte, the first immunoglobulin heavy
chain synthesized is the
A. Mu chain.
B. gamma chain.
C. epsilon chain.
D. alpha chain.

421. In the immune response to a hapten-protein conjugate, in order to get anti-


hapten antibodies it is essential that
A. the hapten be recognized by helper T cells.
B. the protein be recognized by helper T cells.
C. the protein be recognized by B cells.
D. the haptenbe recognized by suppressor T cells.

422. In the determination of serum insulin levels by radioimmunoassay, which


one of the following is NOT needed?
A. isotope-labeled insulin
B. anti-insulin antibody made in goats
C. anti-goat gamma globulin made in rabbits
D. isotope-labeled anti-insulin antibody made in goats
423. Which one of the following sequences is appropriate for testing a patient for
antibody against the AIDS virus with the ELISA procedure? (The assay is carried
out in a plastic plate with an incubation and a wash step after each addition
except the final one.)
A. patient's serum/enzyme substrate/HIV antigen/enzyme-labeled antibody
against HIV
B. HIV antigen/patient's serum/enzyme-labeled antibody against human
gamma globulin/enzyme substrate
C. enzyme-labeled antibody against human gamma globulin/patient's serum/HIV
antigen/enzyme substrate
D. enzyme-labeled antibody against HIV/HIV antigen/patient's serum/enzyme
substrate

424. The BEST method to demonstrate IgG on the glomerular basement


membrane in a kidney tissue section is the
A. precipitin test.
B. complement fixation test.
C. agglutination test.
D. indirect fluorescent-antibody test.

425. A woman had a high fever, hypotension, and a diffuse macular rash. When
all cultures showed no bacterial growth, a diagnosis of toxic shock syndrome was

nolan x soju
made. Regarding the mechanism by which the toxin causes this disease, which
one of the following is LEAST accurate?
A. The toxin is not processed within the macrophage.
B. The toxin binds to both the class II MHC protein and the T cell receptor.
C. The toxin activates many CD4-positive T cells, and large amounts of
interleukins are released.
D. The toxin has an A-B subunit structure--the B subunit binds to a
receptor, and the A subunit enters the cells and activates them.

426. A patient with a central nervous system disorder is maintained on the drug
methyldopa. Hemolyticanemia develops, which resolves shortly after the drug is
withdrawn. This is MOST probably an example of
A. atopic hypersensitivity.
B. cytotoxic hypersensitivity.
C. immune-complex hypersensitivity.
D. cell-mediated hypersensitivity.

427. Which one of the following substances is NOT released by activated helper T
cells?
A. interleukin-1
B. gamma interferon
C. interleukin-2
D. interleukin-4

428. A delayed hypersensitivity reaction is characterized by


A. edema without a cellular infiltrate.
B. an infiltrate composed of neutrophils.
C. an infiltrate composed of helper T cells and macrophages
D. an infiltrate composed of eosinophils.

429. Two dissimilar inbred strains of mice, A and B, are crossed to yield an F1
hybrid strain, AB. If a large dose of spleen cells from an adult A mouse is injected
into an adult AB mouse, which one of the following is MOST likely to occur? An
explanation of the answer to this question is given on p. 24.
A. The spleen cells will be destroyed.
B. The spleen cells will survive and will have no effect in the recipient.
C. The spleen cells will induce a graft-versus-host reaction in the
recipient.
D. The spleen cells will survive and induce tolerance of strain A grafts in the
recipient.

nolan x soju
430. This question is based on the same strains of mice described in the previous
question. If adult AB spleen cells are injected into a newborn B mouse, which one
of the following is MOST likely to occur? An explanation of the answer to this
question is given on p. 24.
A. The spleen cells will be destroyed.
B. The spleen cells will survive without any effect on the recipient.
C. The spleen cells will induce a graft-versus-host reaction in the recipient.
D. The spleen cells will survive and induce tolerance of strain A grafts in
the recipient.

431. The minor histocompatibility antigens on cells


A. are detected by reaction with antibodies and complement.
B. are controlled by several genes in the major histocompatibility complex.
C. are unimportant in human transplantation.
D. induce reactions that can cumulatively lead to a strong rejection
response.

432. Which one of the following is NOT true of class I MHC antigens?
A. They can be assayed by a cytotoxic test that uses antibody and complement.
B. They can usually be identified in the laboratory in a few hours.
C. They are controlled by at least three gene loci in the major histocompatibility
complex.
D. They are found mainly on B cells, macrophages, and activated T
cells.

433. An antigen found in relatively high concentration in the plasma of normal


fetuses and a high proportion of patients with progressive carcinoma of the colon
is
A. viral antigen.
B. carcinoembryonic antigen.
C. alpha-fetoprotein.
D. heterophil

434. An antibody directed against the idiotypic determinants of a human IgG


antibody would react with
A. the Fc part of the IgG.
B. an IgM antibody produced by the same plasma cell that produced the
IgG.
C. all human kappa chains.
D. all human gamma chains.

435. Which one of the following is NOT true of the gene segments that combine
to make up a heavy-chain gene?
A. Many V region segments are available.

nolan x soju
B. Several J segments and several D segments are available.
C. V, D, and J segments combine to encode the antigen-binding site.
D. A V segment and a J segment are preselected by an antigen to make
up the variable-region portion of the gene.

436. When immune complexes from the serum are deposited on glomerular
basement membrane, damage to the membrane is caused mainly by
A. gamma interferon.
B. phagocytosis.
C. cytotoxic T cells.
D. enzymes released by polymorphonuclear cells.

437. If an individual was genetically unable to make J chains, which


immunoglobulin(s) would be affected?
A. IgG
B. IgM
C. IgA
D. IgG and IgM
E. IgM and IgA
438. The antibody-binding site is formed primarily by
A. the constant regions of H and L chains.
B. the hypervariable regions of H and L chains.
C. the hypervariable regions of H chains.
D. the variable regions of H chains.
E. the variable regions of L chains.

439. The class of immunoglobulin present in highest concentration in the blood


of a human newborn is
A. IgG.
B. IgM.
C. IgA.
D. IgD.
E. IgE.

440. Individuals of blood group type AB


A. are Rh(d)-negative.
B. are "universal recipients" of transfusions.
C. have circulating anti-A and anti-B antibodies.
D. have the same haplotype.

nolan x soju
441. Cytotoxic T cells induced by infection with virus A will kill target cells
A. from the same host infected with any virus.
B. infected by virus A and identical at class I MHC loci of the cytotoxic T
cells.
C. infected by virus A and identical at class II MHC loci of the cytotoxic T cells.
D. infected with a different virus and identical at class I MHC loci of the cytotoxic
cells.
E. infected with a different virus and identical at class II MHC loci of the cytotoxic
cells.

442. Antigen-presenting cells that activate helper T cells must express which one
of the following on their surfaces?
A. IgE
B. gamma interferon
C. class I MHC antigens
D. class II MHC antigens

443. Which one of the following does NOT contain C3b?


A. classic-pathway C5 convertase
B. alternative-pathway C5 convertase
C. classic-pathway C3 convertase
D. alternative-pathway C3 convertase

444. Which one of the following is NOT true regarding the alternative
complement pathway?
A. It can be triggered by infectious agents in absence of antibody.
B. It does not require C1, C2, or C4.
C. It cannot be initiated unless C3b fragments are already present.
D. It has the same terminal sequence of events as the classic pathway.

445. In setting up a complement fixation test for antibody, the reactants should
be added in what sequence? (Ag = antigen; Ab = antibody; C = complement; EA
= antibody-coated indicator erythrocytes.)
A. Ag + EA + C/wait/ + patient's serum
B. C + patient's serum + EA/wait/ + Ag
C. Ag + patient's serum + EA/wait/ + C
D. Ag + patient's serum + C/wait/ + EA

446. Proteins from two samples of animal blood, A and B, were tested by the
double-diffusion (Ouchterlony) test in agar against antibody to bovine albumin.

nolan x soju
Which sample(s) contain horse blood? An explanation of the answer to this
question is given on p[dse2] . 24.
A. sample A
B. sample B
C. both samples
D. neither sample

447. Complement lyses cells by


A. enzymatic digestion of the cell membrane.
B. activation of adenylatecyclase.
C. insertion of complement proteins into the cell membrane.
D. inhibition of elongation factor 2.

448. Graft and tumor rejection are mediated primarily by


A. non-complement-fixing antibodies.
B. phagocytic cells.
C. helper T cells.
D. cytotoxic T cells.

449. Which one of the following properties of antibodies is NOT dependent on the
structure of the heavy-chain constant region?
A. ability to cross the placenta
B. isotype (class)
C. ability to fix complement
D. affinity for antigen

450. In which one of the following situations would a graft-versus-host reaction


be MOST likely to occur? (Mouse strains A and B are highly inbred; AB is an F1
hybrid between strain A and strain B.)
A. newborn strain A spleen cells injected into a strain B adult
B. x-irradiated adult strain A spleen cells injected into a strain B adult
C. adult strain A spleen cells injected into an x-irradiated strain AB
adult
D. adult strain AB spleen cells injected into a strain A newborn

451. In a mixed-lymphocyte culture, lymphocytes from person X, who is


homozygous for the HLA-Dw7 allele, are irradiated and then cultured with
lymphocytes from person Z. It is found that DNA synthesis is NOT stimulated.
The proper conclusion to be drawn is that
A. person Z is homozygous for HLA-Dw7.
B. person Z is homozygous or heterozygous for HLA-Dw7.
C. person Z is heterozygous for HLA-Dw7.
D. person Z does not carry the HLA-Dw7 allele.

nolan x soju
452. A patient skin-tested with purified protein derivative (PPD) to determine
previous exposure to Mycobacterium tuberculosis develops induration at the skin
test site 48 hours later. Histologically, the reaction site would MOST probably
show
A. eosinophils.
B. neutrophils.
C. helper T cells and macrophages.
D. B cells.

453. Hemolytic disease of the newborn caused by Rh blood group incompatibility


requires maternal antibody to enter the fetal bloodstream. Therefore, the
mediator of this disease is
A. IgE antibody.
B. IgG antibody.
C. IgM antibody.
D. IgA antibody.

454. An Rh-negative woman married to a heterozygous Rh-positive man has


three children. The probability that all three of their children are Rh-positive is
A. 1:2.
B. 1:4.
C. 1:8.
D. zero.

455. Which one of the following statements BEST explains the relationship
between inflammation of the heart (carditis) and infection with group A beta-
hemolytic streptococci?
A. Streptococcal antigens induce antibodies cross-reactive with heart
tissue.
B. Streptococci are polyclonal activators of B cells.
C. Streptococcal antigens bind to IgE on the surface of heart tissue and
histamine is released.
D. Streptococci are ingested by neutrophils that release proteases that damage
heart tissue.

456. Your patient became ill 10 days ago with a viral disease. Laboratory
examination reveals that the patient's antibodies against this virus have a high
ratio of IgM to IgG. What is your conclusion?
A. It is unlikely that the patient has encountered this organism
previously.
B. The patient is predisposed to IgE-mediated hypersensitivity reactions.

nolan x soju
C. The information given is irrelevant to previous antigen exposure.
D. It is likely that the patient has an autoimmune disease.

457. If you measure the ability of cytotoxic T cells from an HLA-B27 person to kill
virus X-infected target cells, which one of the following statements is CORRECT?
A. Any virus X-infected target cell will be killed.
B. Only virus X-infected cells of HLA-B27 type will be killed.
C. Any HLA-B27 cell will be killed.
D. No HLA-B27 cell will be killed.

458. You have a patient who makes autoantibodies against his own red blood
cells, leading to hemolysis. Which one of the following mechanisms is MOST
likely to explain the hemolysis?
A. Perforins from cytotoxic T cells lyse the red cells.
B. Neutrophils release proteases that lyse the red cells.
C. Interleukin-2 binds to its receptor on the red cells, which results in lysis of the
red cells.
D. Complement is activated, and membrane attack complexes lyse the
red cells.
459. Your patient is a child who has no detectable T or B cells. This
immunodeficiency is most probably the result of a defect in
A. the thymus.
B. the bursal equivalent.
C. T cell-B cell interaction.
D. stem cells originating in the bone marrow.

460. The role of the macrophage during an antibody response is to


A. make antibody.
B. lyse virus-infected target cells.
C. activate cytotoxic T cells.
D. process antigen and present it.

461. The structural basis of blood group A and B antigen specificity is


A. a single terminal sugar residue.
B. a single terminal amino acid.
C. multiple differences in the carbohydrate portion.
D. multiple differences in the protein portion.

462. Complement can enhance phagocytosis because of the presence on


macrophages and neutrophils of receptors for

nolan x soju
A. factor D.
B. C3b.
C. C6.
D. properdin.

463. The main advantage of passive immunization over active immunization is


that
A. it can be administered orally.
B. it provides antibody more rapidly.
C. antibody persists for a longer period.
D. it contains primarily IgM.

464. On January 15, a patient developed an illness suggestive of influenza, which


lasted 1 week. On February 20, she had a similar illness. She had no influenza
immunization during this period. Her hemagglutination inhibition titer to
influenza A virus was 10 on January 18, 40 on January 30, and 320 on February
20. Which one of the following is the MOST appropriate interpretation?
A. The patient was ill with influenza A on January 15.
B. The patient was ill with influenza A on February 20.
C. The patient was not infected with influenza virus.
D. The patient has an autoimmune disease.
465. An individual who is heterozygous for Gmallotypes contains two allelic forms
of IgG in serum, but individual lymphocytes produce only one of the two forms.
This phenomenon, known as "allelic exclusion," is consistent with
A. a rearrangement of a heavy-chain gene on only one chromosome in a
lymphocyte.
B. rearrangements of heavy-chain genes on both chromosomes in a lymphocyte.

C. a rearrangement of a light-chain gene on only one chromosome in a


lymphocyte.
D. rearrangements of light-chain genes on both chromosomes in a lymphocyte.

466. Each of the following statements concerning class I MHC proteins is correct
EXCEPT:
A. They are cell surface proteins on virtually all cells.
B. They are recognition elements for cytotoxic T cells.
C. They are codominantly expressed.
D. They are important in the skin test response to Mycobacterium
tuberculosis.

nolan x soju
467. Which one of the following is the BEST method of reducing the effect of
graft-versus-host disease in a bone marrow recipient?
A. matching the complement components of donor and recipient
B. administering alpha interferon
C. removing mature T cells from the graft
D. removing pre-B cells from the graft

468. Regarding Th-1 and Th-2 cells, which one of the following is LEAST
accurate?
A. Th-1 cells produce gamma interferon and promote cell-mediated immunity.
B. Th-2 cells produce interleukin-4 and -5 and promote antibody-mediated
immunity.
C. Both Th-1 and Th-2 cells have both CD3 and CD4 proteins on their outer cell
membrane.
D. Before naïve Th cells differentiate into Th-1 or Th-2 cells, they are
double-positives; i.e., they roduce both gamma interferon and
interleukin-4.

469. Each of the following statements concerning the variable regions of heavy
chains and the variable regions of light chains in a given antibody molecule is
correct EXCEPT:
A. They have the same amino acid sequence.
B. They define the specificity for antigen.
C. They are encoded on different chromosomes.
D. They contain the hypervariable regions.

470. Each of the following statements concerning class II MHC proteins is correct
EXCEPT:
A. They are found on the surface of both B and T cells.
B. They have a high degree of polymorphism.
C. They are involved in the presentation of antigen by macrophages.
D. They have a binding site for CD4 proteins.

471. Which one of the following statements concerning immunoglobulin allotypes


is CORRECT?
A. Allotypes are found only on heavy chains.
B. Allotypes are determined by class I MHC genes.
C. Allotypes are confined to the variable regions.
D. Allotypes are due to genetic polymorphism within a species.

472. Each of the following statements concerning immunologic tolerance is


correct EXCEPT:

nolan x soju
A. Tolerance is not antigen-specific; i.e., paralysis of the immune cells
results in a failure to produce a response against many antigens.
B. Tolerance is more easily induced in T cells than in B cells.
C. Tolerance is more easily induced in neonates than in adults.
D. Tolerance is more easily induced by simple molecules than by complex ones.

473. Each of the following statements concerning a hybridoma cell is correct


EXCEPT:
A. The spleen cell component provides the ability to form antibody.
B. The myeloma cell component provides the ability to grow indefinitely.
C. The antibody produced by a hybridoma cell is IgM, because heavy-
chain switching does not occur.
D. The antibody produced by a hybridoma cell is homogeneous; i.e., it is directed
against a single pitope.

474. Each of the following statements concerning haptens is correct EXCEPT:


A. A hapten can combine with (bind to) an antibody.
B. A hapten cannot induce an antibody by itself; rather, it must be bound to a
carrier protein to be ble to induce antibody.
C. In both penicillin-induced anaphylaxis and poison ivy, the allergens are
haptens.
D. Haptens must be processed by CD8+ cells to become immunogenic.

Answers (Questions 387-474):

387.D

388.B

389.A

390.D

391.C

392.B

393.C

394.C

395.A

396.B

nolan x soju
397.C

398.D

399.C

400.B

401.C

402.A

403.B

404.D

405.B

406.C

407.C
408.C

409.B

410.D

411.D

412.B

413.C

414.A

415.B

416.D

417.A

418.C

419.B

nolan x soju
420.A

421.B

422.D

423.B

424.D

425.D

426.B

427.A

428.C

429.C

430.D
431.D

432.D

433.B

434.B

435.D

436.D

437.E

438.B

439.A

440.B

441.B

442.D

443.C

nolan x soju
444.C

445.D

446.B

447.C

448.D

449.D

450.C

451.B

452.C

453.B

454.C
455.A

456.A

457.B

458.D

459.D

460.D

461.A

462.B

463.B

464.A

465.A

466.D

nolan x soju
467.C

468.D

469.A

470.A

471.D

472.A

473.C

474. D
1. Select INCORRECT answer. Complement that has been activated by bacterial cells can contribute
to the destruction or elimination of these cells
a. By destroying the bacterial cell membrane
b. By assisting phagocytes to attach to bacterial cells
c. By attracting more phagocytes to the area of complement activation
d. By assisting phagocytes to produce memory cells

2. To elicit the best antibodies to rat self antigen, you should inject rat cells into
a. An animal of different species
b. A rat of the same strain
c. A rat of a different strain
d. The rat you isolated it from

3. Complement receptors (CRs)


a. Activate complement system on the surface of pathogens
b. Bind only activated complement proteins
c. Are involved in elimination of immune complexes
d. On macrophages signal host cell to make opsonin

4. What is the role of the macrophage during antibody formation?

nolan x soju
a. Development of allergic reactions
b. Lysis of virus infected cells
c. Antigen presentation
d. Synthesis of antibodies

5. Identify the fundamental difference between the antigen recognition by B and T cells
a. Antigen must be prepared in different ways
b. Clonal selection
c. Heterogeneity of TCR and BCR from one lymphocyte to the next
d. Membrane location of antigen specific receptors

6. Antibody-dependent cell mediated cytotoxicity (ADCC) is a process in which antibody-coated


cells are killed by
a. Cells with Fc receptors for IgM
b. Cells with Fc receptors for complement components
c. Cells with Fc receptors for secretory IgA
d. Cells with Fc receptors for IgG
7. An antibody fab fragment
a. Contains complementarity determining regions
b. H and L chain constant regions
c. Different antigen binding regions
d. Binds to Fc receptors

8. Inflammation does NOT involve


a. Cytokine production by macrophages
b. Migration of leukocytes out of the circulation
c. Swelling of tissues and pain
d. Production and maturation of lymphocytes

9. If pathogens block the fusion of lysosomes with the phagocytic vesicle, they can be killed
through the effector function of
a. Memory B cells
b. Complement
c. Cytotoxic T cells
d. Opsonizing antibody

10. Which of the following has the MAJOR influence on the antibody response carried out by a B-

nolan x soju
lymphocyte?
a. A t-cytotoxic cell surface molecule
b. Cytokines released by a T-helper cell
c. Anaphylatoxins
d. Phagocytes presenting peptides to T-helper cells

11. T cells are MHC-restricted in their ability to respond to antigen because


a. During an infection, all cells in the body present antigen on MHC Class I
b. MHC binds antigen more specifically than TCR does
c. TCR must recognize both antigen and MHC molecules
d. The T cells should not respond to antigen on allogenic cells

12. Examples of primary T-cell deficiency disease include


a. Hereditary angioneurotic edema
b. Chronic granulomatous disease
c. Wiskott-Aldrich syndrome
d. Leukocyte adhesion deficiency
13. Type IV hypersensitivity reactions result in
a. Formation of granulomas
b. Development of anaphylactic reaction
c. Complement mediated lysis of infected cells
d. Serum sickness

14. Which of the following is NOT correct regarding autoimmune disease?


a. Genetic factors play a role in development of autoimmune diseases
b. Abrogation of CD25+ cells leads to downregulation of the autoimmune response
c. An individual can have more than one autoimmune disease
d. Autoantibody production alone does not equal autoimmune disease

15. Type II hypersensitivity is


a. Antibody independent
b. Complement independent
c. Caused by antibodies against soluble antigen
d. Caused by antibodies against cell surface antigen

16. Hemolytic disease of the newborn caused by Rh blood group incompatibility requires maternal
antibody to enter the fetal bloodstream. Therefore, the mediator of this disease is

nolan x soju
a. IgE antibody
b. IgG antibody
c. IgM antibody
d. IgA antibody

17. When cells die by apoptosis they do not induce inflammation and damage of the surrounding
tissue because they
a. Do not release the cytoplasmic contents
b. Are not subsequently phagocytized
c. Migrate into the blood stream prior to death
d. Are rarely malignant or infected by virus

18. Which statement about antigen epitopes is FALSE?


a. An epitope may be shared by two different antigens
b. A protein molecule usually contains multiple epitopes
c. Epitope is a part of an antibody which recognizes and binds to an antigen
d. Some epitopes are more immunogenic than others
19. Which one of the following types of cells is NOT capable of phagocytosis?
a. Dendritic cell
b. NKT cell
c. Macrophage
d. Neutrophil

20. Which of the following immune effector mechanism is the most important in completely
clearing of infection caused by Streptococcus pneumoniae?
a. Antibody dependent cell cytotoxicity
b. Complement mediated opsonization
c. Cytotoxic T cell lymphocytes
d. Natural killer cells

21. Which of the following is NOT correct regarding immunity to cancer?


a. Despite the immune response tumors continue to grow
b. Abrogation of CD25+ cells leads to protective tumor immunity
c. Vaccination can be used to treat cancer
d. Patients with cancer have successful tumor immunity

22. Positive selection induces apoptosis in developing T cell that bind to

nolan x soju
a. Both MHC I and II
b. Neither MHC I nor II
c. Self-antigen
d. Foreign antigen

23. All of the following are true for Immune hemolytic anemia EXCEPT
a. It is the clinical condition in which IgG antibodies bind to red cell surface antigens
b. It is the clinical condition in which IgM antibodies bind to red cell surface antigens
c. Antibodies initiate red cell destruction via the complement system and phagocytes
d. Red cell destruction occurs by T cytotoxic and NK cells

24. Positive skin tests showing delayed type hypersensitivity, such as for………. Or tuberculosis,
indicate that
a. A humoral immune response has occurred
b. A cell-mediated immune response has occurred
c. Only the B-cell system is functional
d. The patient has an immune deficiency
25. A transplant of tissue between individuals of different species is called a(n)
a. Isograft
b. Allograft
c. Autograft
d. xenograft

26. which of the following statements can correctly describe type III hypersensitivity?
a. It occurs within 72 hours of re-exposure to antigen
b. It is not mediated by complement fixing IgG
c. Complement is an important mediator
d. Desensitization is used for its control

27. Priming of Naïve T-helper cells by mature dendritic cells in the lymph node involves
a. CD40 on DC with CD40L on naive T-cell
b. CTLA -4 on T cell with peptide presented by DC
c. CD56 on naïve T-cell with CD28 on DC
d. CD8 on naïve T-cell with MHC Class I on DC

28. The specificity of any lymphocyte is determined by


a. The type of CD molecules on that cell

nolan x soju
b. The specificity of the antigen receptors on that cells
c. MHC Class I molecules
d. PAMP recognition receptors

29. Which is NOT characteristic for both – Type II and Type III hypersensitivity reactions?
a. Involves attack by Tc cells
b. Involves IgG antibodies
c. Is likely to result in kidney damage
d. Can be evoked by exposure to a pathogen

30. The inflammation that results from a mosquito bite is different from the inflammation that
results from a positive TB skin test in that the reaction to the mosquito bite
a. Does not require prior sensitization
b. Takes more time to appear
c. Results primarily from antibody antigen interactions
d. Involves basophils, mast cells and Tc cells
31. The naïve lymphocytes encounter the antigens for which they are specific during the priming
phase
a. In the primary lymphoid organs
b. In the thoracic duct
c. In the secondary lymphoid organs
d. In the blood

32. In the thymus, T cells that recognize MHC class II molecules differentiate to become?
a. CD8 lymphocyte
b. Gamma/delta T cell
c. NK cell
d. CD4 lymphocyte

33. Effects in neutrophil NADPH oxidase system produce


a. Chronic granulomatous disease
b. Chediak-Higashi disease
c. Leukocyte adhesion deficiency
d. Hashimoto’s disease

34. Identify the fundamental difference between antigen recognition by B and T cells

nolan x soju
a. Antigen most presenting in different way
b. Local selection after antigen recognition heterogenicity
c. Heterogenicity of both T cells and B cell from on lymphocyte
d. Membrane location of antigen specific epitope

35. Staphylococci resistant to penicillin G produce


a. beta catalase
b. Beta lactase
c. Gamma catalase
d. Gamma lactamase

36. In a person with HIV infection potentially fluids include all of the following except
a. Blood
b. Genital secretion
c. Urine
d. Amniotic fluid

37. Non photosynthetic protists growing as a mass of branching filaments are called
a. Fungi
b. Protozoa
c. Algae
d. Chlamydia

38. Lymphocytes are stored antigen binding


a. Thymus
b. Blood stream
c. Spleen
d. Liver

39. The name given to the amino acids that actually form bonds between…. antigen for which that
antibody molecule is very strongly specific are called the
a. Framework amino acids
b. Variable amino acids
c. Hypervariable amino acids
d. Domain amino acids

40. All of the following are associated with the expression of class I MHC molecules except
a. Peptide presentation via class I MHC to T cytotoxic cells

nolan x soju
b. Acute and chronic graft rejection
c. Lysis of virus infected cells
d. Affinity maturation of antibodies

41. Amoxicillin is a small molecule that does not induce antibody formation. However, it binds to
serum protein and forms a complex that in several people induces allergic reaction.
a. An antigen
b. A hapten
c. An immunogen
d. A super antigen

42. Cell located at the bacteria


a. Helper T cell
b. Cytotoxic T cell

43. T cells that recognize MHC class I molecule differentiate in the thymus to become
a. CD8 cytotoxic lymphocyte
b. Nature killer cell
c. CD4 helper cell
d. Neutrophils

44. Identify the cytokine that promotes the development of the Th2 response
a. Interleukin 17
b. Interleukin 4
c. Interleukin 12
d. Interleukin 5

45. Which antibodies are typically involved in both type II and type III hypersensitivity
a. IgA
b. IgG
c. IgD
d. IgE

46. Plasma cell secreting IgA antibodies would be most likely to associate with
a. Tumors
b. Lymph nodes
c. MALT
d. Bone marrow

nolan x soju
2nd set 14,35

1. Humoral immunity can be acquired passively by

a. catching a virus from a friend by shaking hands.


b. receiving a vaccine of influenza virus grown in eggs.
c. receiving serum from someone who has recovered from an infection.
d. receiving leukocytes from an immune family member.
e. sharing a soda with someone who has a cold.

2. Inflammation does NOT involve

a. cytokine production by macrophages.


b. migration of leukocytes out of the circulation.
c. pain.
d. secretion of antibodies.
e. swelling at the site of infection.

3. Innate immune responses are most effective against

a. antigens resembling self antigens.


b. common antigens on bacteria.
c. genetically engineered antigens.
d. viruses.
e. viruses that have previously caused infection.

4. Lymphocytes acquire their antigen specificity

nolan x soju
a. as they enter the tissues from the circulation.
b. before they encounter antigen.
c. depending on which antigens are present.
d. from contact with self antigen.
e. in the secondary lymphoid organs.

5. A secondary immune response is NOT

a. faster than a primary response.


b. larger than a primary response.
c. longer lasting than a primary response.
d. more likely to result in increased adaptive immunity than a primary response.
e. preceded by a longer lag period than a primary response.

6. Antibody effector functions include all of the following EXCEPT

a. activating complement on bacterial surfaces to promote phagocytosis by neutrophils.


b. binding extracellular viruses to block their entry into host cells.
c. binding intracellular viruses to initiate cytotoxicity.
d. blocking uptake of bacterial toxins by host cells.
e. coating bacteria to promote their phagocytosis by neutrophils.

7. Effector functions of complement include all of the following EXCEPT


a. attracting phagocytes to the site of infection.
b. facilitating phagocytosis of complement-coated bacteria.
c. increasing blood vessel permeability to plasma proteins.
d. lysing bacterial cells.
e. presenting antigen to B cells.

8. Jenner observed that milkmaids who were infected with cowpox were later immune to smallpox
infections. This is an example of a(n)

a. acquired immunity of barrier skin cells.


b. active immunization with a non-related organism that causes similar symptoms.
c. innate immunity of milkmaids to smallpox.
d. memory response to a cross-reactive antigen.
e. passive immunization from contact with cow's milk antibodies.

9. Macrophages generally kill the bacteria they phagocytose by fusing lysosomes containing digestive
enzymes with the phagocytic vesicle. In the case of pathogens which block this fusion, pathogen killing
can still be achieved through the effector function of

a. B cells.
b. complement.
c. cytotoxic T cells.
d. opsonizing antibody.
e. Th1 cells.

nolan x soju
10. Phagocytosis

a. can be stimulated by antigen binding to complement or antibody.


b. is an antigen-specific process.
c. must be preceded by antigen processing.
d. rids the body of virus-infected cells.
e. only occurs after plasma cells begin secreting antibody.

11. Several friends who went on a picnic together developed vomiting and diarrhea from eating potato
salad contaminated with Staphylococcus aureus enterotoxin. Effects of the toxin could best be
counteracted by

a. antibody binding and neutralization of the toxin.


b. antibody opsonization and phagocytosis of S. aureus.
c. antibody opsonization and phagocytosis of the toxin.
d. B cell binding to S. aureus.
e. cytotoxic T cell binding and lysis of S. aureus.

12. Which of the following statements is FALSE?

a. An example of passive humoral immunity is treatment with horse anti-snake venin.


b. Antigen recognized by helper T cells must be associated with Class II MHC molecules on the surface
of professional APC .
c. Each lymphocyte has many antigen binding receptors, each receptor capable of binding the same
antigen.
d. Recognition and killing of virus-infected cells by cytotoxic T cells is an example of adaptive immunity.
e. The innate immune system does not deal with endogenous antigen.

13. The ability of an antigen to induce an immune response does NOT depend on the antigen's

a. ability to enter the thyroid.


b. degree of aggregation.
c. dose.
d. size.
e. usual presence in the body.

14.Cytokines are NOT

a. able to inhibit the function of other cytokines.


b. able to stimulate the synthesis of other cytokines.
c. produced by more than one cell type.
d. small protein molecules.
e. stored in the cell for quick release.

nolan x soju
15. If a person is born without C2 and C4,

a. C5 can still be cleaved by the classical pathway.


b. C3b will not be able to bind to bacteria.
c. C9 will polymerize inappropriately and lyse host cells.
d. the classical pathway will be changed into the alternative pathway.
e. the amount of C3b produced during bacterial infections will be reduced.

16. In the membrane attack phase of the classical complement pathway, the role of C5b is to

a. activate the C5 convertase activity.


b. attract neutrophils to lyse the pathogen.
c. initiate formation of the MAC.
d. polymerize into a membrane-spanning channel.
e. All of these are activities of C5b.

17. An antibody Fab contains

a. complementarity determining regions.


b. H and L chain variable regions.
c. one antigen binding region.
d. one H-L interchain disulfide bond.
e. all of the above.
18. The regions of the antibody molecule which contribute MOST to the affinity of the antibody for antigen
are the

a. CDR.
b. Fab regions.
c. Fc regions.
d. framework regions.
e. hinge regions.

19. Antibody Fc fragments contain

a. antigen-binding sites.
b. CDR.
c. complement-binding sites.
d. framework residues.
e. light chain variable domains.

20. The immunoglobulin isotype is determined by the

a. antigen specificity.
b. H chain constant region.
c. L chain variable region.
d. number of antigen-binding sites.
e. number of VH domains.

21.Antibody affinity for antigen depends on

nolan x soju
a. the antibody isotype.
b. the complementary shape and charge of each antibody V region for its antigen epitope.
c. the number of Fab regions in each antibody molecule.
d. whether the antibody is in the serum or on the cell surface.
e. whether the light chains are kappa or lambda.

22. Allotypic determinants are

a. constant region determinants that distinguish each Ig class and subclass within a species.
b. expressed only from the paternal chromosome.
c. generated by the conformation of antigen-specific VH and VL sequences.
d. Not immunogenic in individuals who do not have that allotype.
e. amino acid differences encoded by different alleles for the same H or L chain locus.

23. Which of the following is NOT a characteristic of IgG?

a. It contains 2 and 2 L chains


b. It crosses the placenta.
c. It is the predominant immunoglobulin in blood, lymph, and peritoneal fluid.
d. It is the largest of all the Igs.
e. Its L chains are either  or.

24. T cells use all of the following for generating antigen-recognition diversity on the TCR, except

a. combinatorial association of chains.


b. combinatorial association of segments.
c. large germline pool of gene sequences.
d. N region addition of nucleotides.
e. somatic hypermutation.

25. Endogenous antigen presentation requires delivery of antigen peptides to the endoplasmic reticulum
by (http://www.microvet.arizona.edu/Courses/MIC419/Tutorials/MHC.html)

a. Class I MHC and invariant chain.


b. calnexin and tapasin.
c. HLA-DM.
d. leader sequence.
e. TAP-1 and TAP-2.

26. Exogenous antigen is processed

a. after presentation by antigen presenting cells.


b. by nearly every nucleated cell.
c. by the cytosolic processing pathway.
d. in the presence of 2-microglobulin.
e. in acidified endosomes.

27.Class II MHC does not efficiently present endogenous antigen because

a. antigen synthesized inside the cell never makes it to the endosomal compartment.
b. endogenous antigen cannot be processed into peptides small enough.
c. HLA DM transports Class II to the surface before it can bind endogenous peptide.
d. invariant chain blocks binding of endogenous peptide in the ER.
e. phagocytosed antigen binds Class II as rapidly as Class II is synthesized.

nolan x soju
28. Human Class II MHC molecules

a. are encoded by the genes HLA-A, B, and C.


b. are found on all nucleated cells.
c. have an antigen binding site formed from regions of two polypeptide chains.
d. must be associated with 2-microglobulin molecules to bind peptide.
e. present antigen to CD8 cytotoxic T cells.

29. A ligand is

a. a cytokine.
b. a molecule that specifically binds a receptor.
c. an antigen.
d. an enzyme.
e. all of the above are ligands.

http://www.microvet.arizona.edu/Courses/MIC419/Tutorials/receptorsignaling.html

30. The ligand for TCR is


a. BCR.
b. MHC
c. MHC + peptide.
d. peptide.
e. TCR ligand.

31.which of the following is a major interleukin produced by CD4+ T helper 1(TH1) lymphocytes?

Ans: IL 2

32.Antigen processed by exogeneous antigen presentation

Ans:TCR

33.What is the role of macrophage during antibody formation?

A. Activation of cytotoxic CD8 T cells

B. Delayed hypersensitivity reaction

C. Lysis of virus-infected cells

D. Processing antigen and presenting it to T helper CD4 cells

E. Synthesis of immunoglobulin

nolan x soju
34.T cell

ans: CD3

35.

36.NK cell marker :

Ans: CD16 and CD 56

37. T cell that recognize MHC class II

Ans: T helper cell

38. which one of the following cells is generally the first one to encounter a bacterial infections in the tissues.it is
usually capable of phagocytozing some of those bacteria and sectretes a wide range of cytokines and chemokines
that effect nearby blood vessels and recruit and activate other cells that can also participate in eliminating the
infection?

Ans: neutrophils
39. Ig E

40. B or C

nolan x soju
3rd set
1. Alum is an effective adjuvant because it

a. disaggregates the antigen.


b. is immunogenic for stem cells
c. is immunogenic for T cells.
d. slows the release of antigen.
e. transports antigen into the cytoplasm of antigen-presenting cells.

2. Antibody cross-reactivity is demonstrated by antibody binding to

a. a cell surface marker.


b. a hapten.
c. a hapten-carrier complex.
d. an antigen that is structurally similar to the immunogen
e. the immunogen.

5. The antibiotic penicillin is a small molecule that does not induce antibody
formation. However, penicillin binds to serum proteins and forms a complex
that in some people induces antibody formation resulting in an allergic

nolan x soju
reaction. Penicillin is therefore

a. an antigen.
b. a hapten.
c. an immunogen.
d. both an antigen and a hapten.
e. both an antigen and an immunogen.

4. Antigen entering the body in a subcutaneous injection activates its specific


lymphocytes in the

a. blood circulation.
b. draining lymph nodes.
c. MALT.
d. skin.
e. spleen.

5. To detect a humoral immune response to influenza virus, you would


measure
a. cytotoxicity of virus-infected cells in the lung.
b. cytotoxicity of virus-infected cells in tissue culture.
c. dividing T cells in the draining lymph nodes.
d. plasma cytokine levels.
e. serum antibody titer.

6. During the lag period between antigen contact and detection of adaptive
immunity,

a. antigen is hidden from the immune system in macrophages.


b. cellular immunity can be detected but antibodies cannot.
c. innate immune effectors are eliminating antigen.
d. innate immunity blocks the activation of adaptive immune effector cells.
e. new B and T cells with the appropriate antigen specificity must be produced
in the bone marrow.

7. To elicit the best antibodies to mouse MHC I, you should inject it into

a. a goat.
b. a mouse of the same genetic background (strain).
c. a mouse of a different strain.
d. a rat.

nolan x soju
e. the mouse you isolated it from.

8. For specific antigen recognition by T cells,

a. antigen is bound by a T cell membrane antibody.


b. denaturation of antigen does not reduce epitope recognition.
c. MHC molecules are not required.
d. soluble antigen is bound directly without processing.
e. antigen exposure during T cell maturation is required.

9. The immune response to a booster vaccine is called a(n)

a. cellular response.
b. humoral response.
c. innate response.
d. primary response.
e. secondary response

10.. Cytokines are NOT


a. antigen specific.
b. capable of activating more than one cell type.
c. made by lymphocytes.
d. small protein molecules.
e. synthesized de novo in response to antigen or other cytokines.

11. Lymphocytes are activated by antigen in the

a. blood stream.
b. bone marrow.
c. liver.
d. lymph nodes.
e. skin.

12. A molecule that can be covalently linked to a non-immunogenic antigen to


make it an immunogen is called a(n)

a. adjuvant.
b. carrier.
c. hapten.
d. mitogen.
e. superantigen.

nolan x soju
13. A polyclonal antibody response

a. is not antigen-specific.
b. is produced only in response to polymeric antigens.
c. is produced by several B cells recognizing different epitopes on the
same antigen.
d. occurs during the lag phase of the immune response.
e. violates clonal selection.

14. Which statement about antigen epitopes is FALSE?

a. An epitope may be shared by two different antigens.


b. A protein molecule usually contains multiple epitopes.
c. B cells bind only processed antigen epitopes.
d. Epitopes may be linear or assembled.
e. Some epitopes are more immunogenic than others

15. Activated CTL can regulate immune responses by signaling activated


lymphocytes to undergo
a. apoptosis.
b. clonal deletion.
c. clonal proliferation.
d. cytotoxicity.
e. somatic hypermutation.

16.Complement receptors (CR)

a. activate complement on the surface of pathogens.


b. bind only activated complement proteins.
c. inhibit complement activation on the surface of host cells.
d. on erythrocytes remove immune complexes from the circulation.
e. on macrophages signal host cells to make opsonins.

17 . ans :C3B

18. A deficiency in complement proteins or in their regulators can result in

a. decreased levels of certain complement proteins in the circulation.


b. immune complex disease.
c. increased numbers of infections.
d. All of the above can result from complement deficiencies.

nolan x soju
19. The Ig isotype which would be most important for neutralizing polio virus before it
could infect intestinal cells would be
a. secretory IgA.
b. serum IgA.
c. serum IgD.
d. serum IgG.
e. membrane IgM.
20. IgA can be secreted from the body because it
a. binds poly-Ig receptor on mucosal epithelial cells.
b. has a specialized H chain called secretory chain.
c. has a special secretory idiotype.
d. is small enough to pass between mucosal epithelial cells and leave the body.
e. is synthesized by mucosal epithelial cells and secreted directly into the intestinal
lumen.
21. Allergy symptoms are produced when antigen binds to IgE on FcR on
a. A cells.
b. macrophages.
c. mast cells.
d. neutrophils.
e. Th1 cells.
22. One amino acid difference in the Fc region of different human  chains is
the epitope recognized by anti-
a. allotype.
b. idiotype.
c. isotype.
d. IgG.
e.  chain
23. Genes for immunoglobulins are unlike other human genes in that
a. each polypeptide chain is encoded by several exons.
b. Ig genes are composed of introns and exons
c. somatic recombination occurs before mRNA is transcribed
d. there is less Ig genetic material in mature B cells than in other somatic cells
e. both c and d are true.
24. Somatic recombination occurs
a. in the bone marrow stem cell.
b. in the progenitor cell as it is becoming a B cell.
c. in the mature B cell following antigen contact.
d. in the plasma cell after antigen contact.
e. in the plasma cell after antibody secretion

nolan x soju
25. Isotype switching
a. changes the leader sequence exon so the antibody is secreted.
b. improves the antigen binding specificity of an Ig molecule.
c. increases the affinity of antibodies in a process called affinity maturation.
d. increases the functional diversity of Ig molecules.
e. occurs randomly between switch regions.
26. Isotype switching resembles somatic recombination because both processes
a. are catalyzed by the products of RAG1 and RAG2
b. are regulated by helper T cell cytokines.
c. can result in stop codons in coding sequences.
d. occur in developing B cells in the bone marrow.
e. result in the irreversible loss of DNA from the B cell.
27. Somatic hypermutation does NOT
a. occur by somatic recombination.
b. occur during B cell proliferation.
c. occur in the B cell following antigen stimulation.
d. result in increased affinity of antibodies secreted later in immune responses.
e. result in the death of some B cells which no longer bind antigen.
28. CD8 is a co-receptor on T cells that binds
a. CD3.
b. endogenous antigen peptide.
c. the constant region of Class I MHC.
d. the constant region of TCR.
e. the variable region of Class I MHC.
29. All of the following are true for antigen receptors on both B cells and T cells
EXCEPT
a. associated with signal transduction molecules in the membrane.
b. generated by somatic recombination during lymphocyte development.
c. members of the Ig gene superfamily.
d. MHC-restricted in their ability to bind antigen.
e. specific for a single antigen epitope.
30. Both Class I and Class II MHC molecules are
a. composed of  and  chains with variable and constant regions.
b. expressed constitutively on all nucleated cells.
c. expressed on the B cell membrane.
d. part of the T cell receptor for antigen.
e. synthesized in response to antigen processing.
31. The ligand for TCR is
a. BCR.
b. MHC
c. MHC + peptide.

nolan x soju
d. peptide.
e. TCR ligand.
32. T cells are MHC-restricted in their ability to respond to antigen because
a. all antigen must be processed and presented to activate lymphocytes.
b. during an infection, all cells in the body present antigen on MHC Class I.
c. MHC binds antigen more specifically than TCR does.
d. TCR must recognize both antigen and MHC molecules.
e. the T cells should not respond to antigen on allogeneic cells
33. All of the following are associated with the expression of Class I MHC molecules
EXCEPT
a. antigen peptide presentation on membrane Class I MHC to Tc.
b. graft rejection.
c. increased risk of certain autoimmune diseases.
d. lysis of virus-infected cells.
e. stimulation of antibody production.
34. Human Class II MHC molecules
a. are encoded by the genes HLA-A, B, and C.
b. are found on all nucleated cells.
c. have an antigen binding site formed from regions of two polypeptide chains.
d. must be associated with 2-microglobulin molecules to bind peptide.
e. present antigen to CD8 cytotoxic T cells.
35. The signal transduction molecules associated with TCR are
a. CD1.
b. CD3.
c. CD4.
d. CD8.
e. CD22.
36. The signal transduction molecules associated with BCR are
a. CD21 and CD81.
b. Ig and Ig
c. IgD and IgM.
d. ITAMs and ITIMs.
e. RAG-1 and RAG-2.
37. Antibody-dependent cell-mediated cytotoxicity (ADCC) is a process in which
antibody-coated cells are killed by
a. the antibodies.
b. complement.
c. cytotoxic T cells.
d. cells with Fc receptors for IgG3.
e. cells with Fc receptors for IgE.

nolan x soju
1- Takes immune complex for removal

Ans- RBC

2- Same species

Ans- syngenic

3- C9 deficiency

Ans- decreased resistance to neisseria

4- Repid and non specific responses

Ans- macrophages & neutrophils

5- Anti b antibodies

Ans- A & O

6- Wiskott aldrick

Ans- thrombocytopenia

7- CD4THI - VIRAL INFECTION

8- HIV - CCR5

9- AIDS effect - Macrophages

10- Microbacterium - chronic graculoma

11- Messive histamine release - Wheezing

12- Stephylococcus aureus - any bacteria option or NADPH OXIDASE

13- Type4 hypersensitive - delayed hypersensitivity

nolan x soju
14- Girl , x linked desire- Bruton disease

15- Germinal centre - B lymphocytes

16- Double negative precursor of T lymphocytes found in upper cortex of Thymus

17- Deficiency of C2, C6,C9 - ?

18- chemotaxis- C5a

19- B lymphocyte chain , first to synthesise

Ans- mui chain

20- defect in neutrophils-?

21- primary immune deficiency - infections

22- CD56 is marker of NK cells

23- leprosy - Type4

24- Deficiency in chronic granuloma desease

Ans- NADPH OXYDASE

25- prick test - Type 1

26- Naive b cells- spleen

27- NAIVE cells helps T cells by secreating IFN GAMA

Ans- Nk cells

28- Idiotype - d option is answer

29- Patch test - Type 1

30- No effect on j chain - Iga & Igm

31- Girl have chicken pox immunity- Igg

32- ans - none of the above (there is only 1 question with this option )

32- ans- all of the above ( d option ) ( when n there is both all of above & none of these ans is all
of above)

33- kideney transplant - AB GROUP

35- Bee sting - systemic anaphylacsis

36- nurse given vaccine- deficient active immunity

37-graves disease - (a option ) prevention of activation of self reactive lymphocytes

nolan x soju
1. PRICK TEST IS DONE FOR – TYPE I HYPERSENSITIVITY
2. MOLECULARY MIMCRY DONE BY- MICROBES
3. CYTOKINES- CYTOKINES ARE SMALL PROTEINS PARTICIPATE IN IMMUNOLOGIC
RESPONSE
4. CASE ABOUT NEISSERIA GONORROHEA- C6 DEFICIENCY
5. ALL ARE ANTIBODY FUNCTION EXCEPT- IT ATTACHES TO ANTIGEN AND DIRECTLY KILLS
ANTIGEN
6. DIAGNOSTIC MARK OF MYCOBACTERIUM- GRANULOMA
7. IMMUNOGLOBULIN THAT HAS 2 ISOFORMS- IG-A
8. CD MARKER IN NK CELLS- CD56
9. KIDNEY TRANSPLANT QUESTION- MHC 2 PRESENTING RECEPIENT ANTIGENS TO
RECIPIENT HELPER CELLS
10. NON ANTIGEN SPECIFIC RESPONSE- NEUTROPHILS AND MACROPHAGES
11. CASE ABOUT NO SERUM IG AND X LINKED DISORDER- BRUTON’S
IMMUNOGLOBULINEMIA
12. DEFICIENCY OF C9- DISEASE RESISTANCE TO NEISSERIA
13. QUESTION ABOUT CHICKEN POX VIRUS-IG-G
14. WHICH HELPS PHAGOCYTES ATTACH TO BACTERIAL SURFACE- C3B

nolan x soju
15. NEUTROPHILS CANT PHAGOCYTOZE MICROBES- NADH OXIDASE ENZYME DEFICIENCY
16. WISKOT ALDRICH SYNDOME-THROMBOCYTOPENIA
17. PERSON WITH PRIMARY IMMUNODEFICIENCY ARE PRONE TO – INFECTIONS
18. CASE ABOUT ENZYME DEFICIENCY- CHRONIC GRANULOMATOUS DISEASE
19. HYPERACUTE RESPONSE- PREEXISTED ANTIBODIES
20. H3N1 INFLUENZA VACCINES- ARTIFICIAL ACTIVE
21. J CHAIN IS FOUND IN – IG – M&A
22. TRANSPORT OF IMMUNE COMPLEX TO SITE OF REMOVAL- MACROPHAGES
23. GRAVE’S DISEASE- SELF REACTIVE LYMPHOCYTES
24. MICROBES EVADE IMMUNE RESPONSE- DECREASED PRODUCTION OF MHC 1 IN
ENDOPLASMIC RETICIULUM
25. INHIBITORY SIGNAL FOR T CELL – CD28
26. SHOCK,RESPIRATORY DISEASES- SYSTEMIC ANAPHYLAXIS
27. ANTIBODY IDIOTYPE- HYPERVARIABLE HEAVY AND LIGHT CHAINS
28. DURATION MATURATION OF B CELLS WHICH CHAIN ACTIVATES- µ CHAIN
29. TYPE I HYPERSENSITIVITY LEADS TO WHEEZING AND HYPOTENSION
30. MHC 2 INVOLVES ALL EXCEPT- RED BLOOD CELLS
31. GRAFT BETWEEN TWO IDENTICAL ANIMALS- SYNGENIC
32. INNATE IMMUNITY – DOES NOT INDUCE MEMORY
33. PATIENT WITH THIRD DEGREE BURNS AND GRAFT IS REQUIRED FROM WHICH IS LESS
REJECTED-PATIENT
34. WHICH BLOOD SERUM HAS ANTI- B ANTIBODIES- BLOOD GROUP A AND O

nolan x soju
Typical tests in immunology

1. Which of the following biomolecules is present in T cells?


MH C I Complex with class molecules?
1. Viral proteins
2. Lipids localized in the cytoplasm
3. Bacterial proteins degraded in phagolysosomes
4. Carbohydrates localized on the cell surface

2. Are memory cells


1. Neutrophils
2. T- and B- Lymphocytes
3. Macrophages
4. Antigen-presenting cells

3. Which of the surface molecules of helper T cells is involved IL-2- He in the


cessation of synthesis
1. B-7
2. C D 28
3. CT LA-4
4. C D4

4. Biologically active fragments are formed as a result of complement activation. Which of the

nolan x soju
following enhances phagocytosis?
( Which is opsonin)?
1. C3
2. C 3- Convertase
3. C 3A

4. C3b

5. Participates in the determination of the antibody class (isotype switching) synthesized by


plasma cells
1. C D40- C D40L Feedback
2. Antigen type
3. Cytokines produced by T-lymphocytes
4. All answers are correct

6. A somatic mutation occurs


1. Ripe B- In lymphocytes
2. In the first lymphatic organs
3. In mature T-lymphocytes
4. All answers are correct



















7. What is the specific term for a bacterial or other foreign protein that causes the body to
produce antibodies?
1. Peptide
2. MH C II Molecule
3. Complement
4. Antigen

8. Which of the following plays a leading role in tumor cell attack and death?

1. Red blood cells


2. Cytotoxic T-cells
3. Platelets
4. Porous cells

9. Which term is used to denote the outflow of leukocytes between the endothelial cells
of the vascular epithelium in the direction of the site of infection?

1. Diapedesis
2. Chemotaxis
3. Phagocytosis
4. Enucleation

10. Bacterial absorption and intracellular digestion by neutrophils and macrophages

nolan x soju
1. Halitosis
2. Chemotaxis
3. Botulism
4. Phagocytosis

11. Are antibody-producing cells


1. NK- Cell
2. Plasma cell
3. Red blood cells
4. Dendritic cells

12. There are opsons

1. C 3 b and antibodies
2. C 3 a and C 4 a
3. C 5 a and C 4 a

4. C 6, C 7, C 8 And C 9

13. NK- Cells are involved


1. In antigen presentation
2. In complement activation























3. In phagocytosis
4. In a virus-infected cell trail

14. NK- Expressed on the cell KI The R receptor guesses


1. Irreplaceable MH C I Molecule
2. Irreplaceable MH C II Molecule
3. C D8 Receptor
4. C D4 Receptor

15. Follicular-dendritic cells bind antigen


1. MH C I Through molecules
2. MH C II Through molecules
3. Through the complement receptor
4. F C •• By receptor

16. Neutrophils migrate


1. From the bone marrow to the site of inflammation

2. From the bone marrow to the secondary lymph nodes


3. From the spleen to the bone marrow

4. The secondary lymphatic organs are resident cells

17. Follicular-dendritic cells


1. Secondary lymphatic organs B- The zone is resident cells

nolan x soju
2. Migrate from the site of inflammation to the secondary lymph nodes
3. Tissues are transformed into macrophages
4. Produce antibodies

18. Belongs to secondary lymph nodes


1. Spleen and thymus
2. Spleen and lymph nodes
3. Lymphatic tissue associated with mucosa and thymus
4. Bone marrow and spleen

19. Provides naive (inactivated) lymphocyte recirculation

1. Vascular addressees expressing on high endothelial venules and lymphocyte homing


receptors
2. Expressed on capillary endothelium MH C II Class molecules and lymphocyte
T-cell receptor
3. B7- C D28 Feedback
4. C D40 - C D40L Feedback

20. Complement proteins are not involved


1. In offspring




















2. In the killing of bacteria


3. Antigen presentation to T cells in porous cell
4. degranulation

21. Inflammation is characterized

1. By increasing capillary permeability


2. By cell migration
3. By enhancing the blood supply
4. All answers are correct

22. C 5 b, C 6- C 9 The assembly of a membrane-attacking complex from proteins takes place


1. C 1 In case of inhibitor action
2. In the last phase of phagocytosis
3. In the last phase of complement activation
4. During apoptosis

23. Anaphylatoxins
1. C 3 a and C 5 a

2. C3b
3. C 3- Convertase
4. C 5- Convertase

24. In order to activate resting cells, the antigen present on the cell surface must be expressed

nolan x soju
1. MH C I Molecules
2. MH C II Molecules
3. Antigen-presenting molecules and adhesive molecules simultaneously

4. C D1 Molecules

25. The T-cell receptor recognizes the antigen


1. In a natural way
2. MH In complex with the C-molecule
3. In the complex with the antibody
4. After binding to complement proteins

26. Antigens are characterized by cross-reactivity if


1. They have a common antigenic determinant
2. They are polysaccharides
3. Their carrier microbes invade the body simultaneously
4. They are lipids

27. Adjuvant function no Represents


1. Slow and prolonged release of antigen























2. Antigen absorption, processing and presentation by the macrophage


Stimulation
3. Expression of co-stimulated receptors
4. Activation of the complement

28. High-endothelial venules are found


1. In secondary lymph nodes and foci of chronic inflammation
2. In the liver
3. In the focus of acute inflammation

4. In the kidneys

29. Superantigens
1. Are proteins with low molecular weight
2. Of the TCR V • Domain-specific non-specific activation of lymphocytes

3. C D1 Are glycolipids presented by molecules


4. Are polysaccharides

30. C D8 + Carried by T-lymphocytes


1. Antibody-dependent cytotoxic reactions
2. Examination of target cells by apoptosis
3. Antibody synthesis
4. All answers are correct

nolan x soju
31. C 3- That deficiency will be detected clinically
1. By increasing susceptibility to viral infections
2. By increasing susceptibility to fungal infections
3. Increased susceptibility to bacterial infections and damage to blood vessels by immune
complexes
4. Increased susceptibility to viral infections and thrombocytopenia

32. Steptococcus pyogenes M Proteins produced against the protein


1. They block binding to the bacterial cell host membrane
2. Block surface proteins involved in the transport of nutrients into the bacterial cell

3. Increases the antibacterial activity of natural killers


4. They do S. pyogenes Opsonization

33. Characteristic of chronic granuloma


1. C 3 b Deficiency of receptors
2. Deficiency of microbicidal substances (hydrogen peroxide and superoxide
radicals)
3. Defect in microbial phagocyte binding























4. Microbial absorption defect

34. Which provision is correct for hypersensitivity I About the type?


1. It is also called slow-type hypersensitivity
2. It does not require I g E- Sensitized fat cells
3. It takes place when I g E- The answer is directed directly against the dust grains

4. It involves the degranulation of polymorphonuclear eosinophils

35. Which provision is correct for hypersensitivity II About the type?


1. It is antibody-independent
2. He is complement-independent
3. Does not participate in it NK- Cells
4. None of this is right

36. Severe combined immunodeficiencies develop when they occur


1. X- Agammaglobulinemia associated with chromosomes
2. T and B Lack of formation of lymphocyte antigen-receptors

3. Inability of adhesion by leukocytes


4. IL-12- Heterodimer P40 Subunit defect

37. Which enzyme ensures that the human immunodeficiency virus is included in the DNA of

nolan x soju
the host provirus?
1. Transcriptase
2. Integration
3. Protease
4. Reverse transcriptase

38. Why is the human immunodeficiency virus difficult to treat with a single drug?

1. Has the ability to break down drugs


2. Has an inaccessible capsule for medication
3. Has the ability to mutate
4. All answers are correct

39. In human immunodeficiency virus infection, the enzyme provides reverse


transcriptase
1. Conversion of viral DNA to RNA
2. Conversion of viral RNA to DNA
3. Conversion of proteins into viruses
4. Involvement of viral RNA in the host chromosome





40. Which membrane cell structure of the host cell is attached to the human immunodeficiency
virus?
1. Ion channels
2. Nucleus pores
3. Ribosomes
4. C D4 + Receptors

41. Participates in virus neutralization (I think should not participate)


1. Phagocytes
2. Complement
3. MH C I Class antigen-presenting molecules
4. Specific antibodies

42. For capsule bacterial cell phagocytosis, a specific antibody to the capsule antigen attaches
to the surface of the macrophage
1. An antigen that is identical to the capsule antigen
2. MH C II Molecule
3. Complement receptor
4. F C-receptor

43. In the absence of antibodies, against which phagocytes can usually carry out an
effective response
1. Of viruses

nolan x soju
2. Exotoxins secreted by bacteria
3. Bacteria that are strongly bound to phagocytes
4. Worms

44. In the development of antibody-induced allergic reactions, one isotype of the antibody binds to
the following cell
1. Macrophages

2. Pocher cell
3. NK- Cell
4. B- Lymphocytes

45. One of the mechanisms by which natural killers ( NK) Tumor cell destruction is mediated by
antibody-dependent cellular cytotoxicity. Which antibodies are involved in carrying out this
process?

1. I g E, Which is specific NK- Towards foreign antigens expressed on the cell surface

2. I g E, Which is specific for foreign antigens expressed on the surface of tumor cells

3. I g G, Which is specific NK- Towards foreign antigens expressed on the cell surface

4. I g G, Which is specific for foreign antigens expressed on the surface of tumor cells

46. Which isotype of the antibody is responsible for the release of large amounts of
histamine, which leads to the development of allergic reactions?

1. I g M
2. IgD
3. I g G
4. I g E

47. T-cell immune response may be the result


1. Granuloma formation
2. Removal of a heart transplant
3. Development of eczema on the skin at the site of prolonged contact with rubber
4. All answers are correct

48. HLA-compatible individual transplant without immunosuppression


1. If the transplant is in the heart and lungs, it will not be removed, and in the case of the skin, it will be removed

2. Will inevitably cause a transplant against the host


3. Despite the presence of minor histocompatibility antigens, it is not removed

nolan x soju
4. Removes slowly due to the presence of histocompatibility minor antigens

49. Patients with Di-George syndrome have the ability


1. Get rid of a bone marrow transplant
2. Fight intracellular parasites
3. Produce antibodies to T-dependent antigens
4. All answers are incorrect

50. Which of the following components is involved in the development of a positive skin
reaction during a slow-type hypersensitivity?
1. Antigen, complement, and lymphokines
2. Antigen, antigen-specific lymphocytes, and macrophages
3. Antigen-antibody complex, complement, and neutrophils
4. I g E Antibody, antigen, and proliferative cells

51. Cellular immunity is not conditioned


1. Removal of a liver transplant
2. Serum sickness
3. Tuberculin reaction
4. Immunity against chickenpox

52. The following diseases are caused by immune complexes


1. Systemic lupus erythematosus
2. Graves' disease
3. Goodpasture Syndrome
4. All answers are incorrect

53. Characterized by Artius's reaction


1. Lymphocyte infiltration around the veins
2. Formation of antigen-antibody precipitate on blood vessel walls
3. I g E- He cross-linked
4. no one

54. It is characteristic of serum sickness


1. Accumulation of immune complexes in the walls of blood vessels
2. Phagocytosis of complexes by granulocytes
3. Complement participation
4. All answers are correct

55. Which of the following diseases is involved in the pathogenesis of immune complexes

1. Poststreptococcal glomerulonephritis
2. Lungs of pigeon lovers
3. Serum sickness

nolan x soju
4. All answers are correct

56. Serum sickness develops when it occurs


1. Extreme abundance of antibodies
2. I g E- S products
3. In the presence of soluble immune complexes
4. Neutrophil deficiency

57. Antigen interactions I g E- With takes place


1. During the Proustnitz-Kustner reaction
2. During rhinitis caused by ragweed dust
3. During eosonophilia
4. All right

58. Antihistamines
1. Binds to the histamine receptor and blocks the pharmacological effect of
histamine
2. Does not affect the activity of leukotrienes
3. Do not affect I g E- S to connect with porous cells
4. All answers are correct

59. Which statement is not correct about tumor immunotherapy


1. Activation of specific cytotoxic cells against tumor antigens

2. Activation of non-specific anti-tumor effector cells

3. Prevention of immunosuppression
4. Stimulation of tumor-associated macrophages

60. Immediate hypersensitivity skin reactions


1. Usually occurs from contact with antigen 45 Minutes later
2. Characterized by the development of ulcers caused by mononuclear cell infiltration

3. Characterized by redness developed as a result of vasodilation


4. Caused by monovalent haptens

nolan x soju

1. Central (primary) lymphoid organs

a. are efficient in exposing T cells to foreign antigen.


b. are the primary site of antibody synthesis and release.
c. filter blood and trap blood-borne antigens.
d. provide the microenvironment for maturation of T and B cells.
e. line the mucosal surfaces of the body for efficient antigen contact.

2. Hematopoietic stem cells are pluripotent, which means that they are
a. antigen-specific cells.
b. capable of developing into any blood cells.
c. committed to produce cells of a single lineage.
d. not self-renewing.
e. T and B lymphocytes of many different antigen specificities.

3. Lymphocytes continually recirculate through peripheral lymphoid tissue in


order to
a. be killed before they cause autoimmunity.
b. efficiently encounter antigen.
c. mature from stem cells into lymphocytes.
d. phagocytose antigen and kill it.
e. go where no cell has gone before.

nolan x soju
4. Peripheral lymphoid organs
a. are centrally located in the abdomen to protect their vital functions.
b. are designed to maximize contact between antigen and lymphocytes.
c. produce antigen-specific lymphocytes from stem cells in response to antigen.
d. sequester antigen to minimize its damage to the body.
e. store large numbers of activated effector cells for a rapid response to antigen.

5. The PRIMARY purpose of the adaptive immune system is to


a. block all pathogens from entering the body.
b. cure allergic reactions.
c. kill tumor cells.
d. protect from disease upon re-infection with a specific pathogen.
e. reject foreign transplants.

6. Rapid but non-antigen specific immune responses are produced by the


a. adaptive immune system.
b. innate immune system.
c. leukocytes.
d. lymphatic system.
e. memory response.
7. Vaccination protects us from infectious disease by generating memory
a. antigen.
b. lymphocytes.
c. macrophages.
d. PMNs.
e. stem cells.

8. Which situation below describes an example of innate immunity?


a. antibody production by plasma cells.
b. antigen removal by cilia in the respiratory tract.
c. complement activation by antibody bound to the surface of a bacterium.
d. memory response to influenza virus
e. recognition and killing of virus-infected cells by cytotoxic T cells.

9. The antigen specificity of an adaptive immune response is due to


a. activation of antigen-specific lymphocytes.
b. folding of antibody to fit the pathogen.
c. lysis of only certain pathogens by neutrophils.
d. phagocytosis of only certain pathogens by macrophages.
e. production of cytokines by antigen-specific macrophages

10. Clonal selection


a. begins with inflammation.
b. occurs for all leukocytes.

nolan x soju
c. occurs in response to self antigens.
d. results in innate immunity.
e. results in proliferation of antigen-specific lymphocytes.

11. Cytokines are NOT


a. able to induce increased blood vessel permeability.
b. antigen-specific.
c. made in response to bacterial antigens.
d. signals from one cell that affects the behavior of another cell.
e. secreted by macrophages.

12. A fundamental difference between the antigen receptors on B cells (BCR) and
on T cells (TCR) is their
a. different requirements for antigen presentation.
b. function following antigen binding.
c. heterogeneity from one lymphocyte to the next.
d. heterogeneity on each lymphocyte.
e. membrane location.
13. Genes for immunoglobulins (antibodies) are unlike other human genes in that

a. antibody genes are composed of introns and exons.


b. DNA for antibody molecules is inherited from only one parent.
c. gene segments must be spliced together to make each unique antibody
molecule.
d. several exons encode each antibody molecule.
e. none of the above is true.

14. Humoral immunity can be acquired passively by


a. catching a virus from a friend by shaking hands.
b. receiving a vaccine of influenza virus grown in eggs.
c. receiving serum from someone who has recovered from an infection.
d. receiving leukocytes from an immune family member.
e. sharing a soda with someone who has a cold.

15. Inflammation does NOT involve


a. cytokine production by macrophages.
b. migration of leukocytes out of the circulation.
c. pain.
d. secretion of antibodies.
e. swelling at the site of infection.

16. Innate immune responses are most effective against


a. antigens resembling self antigens.
b. common antigens on bacteria.

nolan x soju
c. genetically engineered antigens.
d. viruses.
e. viruses that have previously caused infection.

17. Lymphocytes acquire their antigen specificity


a. as they enter the tissues from the circulation.
b. before they encounter antigen.
c. depending on which antigens are present.
d. from contact with self antigen.
e. in the secondary lymphoid organs.

18. A secondary immune response is NOT


a. faster than a primary response.
b. larger than a primary response.
c. longer lasting than a primary response.
d. more likely to result in increased adaptive immunity than a primary response.
e. preceded by a longer lag period than a primary response.

19. Antibody effector functions include all of the following EXCEPT


a. activating complement on bacterial surfaces to promote phagocytosis by
neutrophils.
b. binding extracellular viruses to block their entry into host cells.
c. binding intracellular viruses to initiate cytotoxicity.
d. blocking uptake of bacterial toxins by host cells.
e. coating bacteria to promote their phagocytosis by neutrophils.

20. Effector functions of complement include all of the following EXCEPT


a. attracting phagocytes to the site of infection.
b. facilitating phagocytosis of complement-coated bacteria.
c. increasing blood vessel permeability to plasma proteins.
d. lysing bacterial cells.
e. presenting antigen to B cells.

21. Jenner observed that milkmaids who were infected with cowpox were later
immune to smallpox infections. This is an example of a(n)
a. acquired immunity of barrier skin cells.
b. active immunization with a non-related organism that causes similar
symptoms.
c. innate immunity of milkmaids to smallpox.
d. memory response to a cross-reactive antigen.
e. passive immunization from contact with cow's milk antibodies.

22. Macrophages generally kill the bacteria they phagocytose by fusing

nolan x soju
lysosomes containing digestive enzymes with the phagocytic vesicle. In the case
of pathogens which block this fusion, pathogen killing can still be achieved
through the effector function of
a. B cells.
b. complement.
c. cytotoxic T cells.
d. opsonizing antibody.
e. Th1 cells.

23. Phagocytosis
a. can be stimulated by antigen binding to complement or antibody.
b. is an antigen-specific process.
c. must be preceded by antigen processing.
d. rids the body of virus-infected cells.
e. only occurs after plasma cells begin secreting antibody.

24. Several friends who went on a picnic together developed vomiting and
diarrhea from eating potato salad contaminated with Staphylococcus aureus
enterotoxin. Effects of the toxin could best be counteracted by

a. antibody binding and neutralization of the toxin.


b. antibody opsonization and phagocytosis of S. aureus.
c. antibody opsonization and phagocytosis of the toxin.
d. B cell binding to S. aureus.
e. cytotoxic T cell binding and lysis of S. aureus.

25. Which of the following statements is FALSE?


a. An example of passive humoral immunity is treatment with horse anti-snake
venin.
b. Antigen recognized by helper T cells must be associated with Class II MHC
molecules on the surface of professional APC .
c. Each lymphocyte has many antigen binding receptors, each receptor capable
of binding the same antigen.
d. Recognition and killing of virus-infected cells by cytotoxic T cells is an example
of adaptive immunity.
e. The innate immune system does not deal with endogenous antigen.

part-2

1. The ability of an antigen to induce an immune response does NOT depend on


the antigen's

nolan x soju
a. ability to enter the thyroid.
b. degree of aggregation.
c. dose.
d. size.
e. usual presence in the body.

2. Alum is an effective adjuvant because it


a. disaggregates the antigen.
b. is immunogenic for stem cells
c. is immunogenic for T cells.
d. slows the release of antigen.
e. transports antigen into the cytoplasm of antigen-presenting cells.

3. Antibody cross-reactivity is demonstrated by antibody binding to


a. a cell surface marker.
b. a hapten.
c. a hapten-carrier complex.
d. an antigen that is structurally similar to the immunogen
e. the immunogen.

4. The antibiotic penicillin is a small molecule that does not induce antibody
formation. However, penicillin binds to serum proteins and forms a complex that
in some people induces antibody formation resulting in an allergic reaction.
Penicillin is therefore
a. an antigen.
b. a hapten.
c. an immunogen.
d. both an antigen and a hapten.
e. both an antigen and an immunogen.

5. Antigen entering the body in a subcutaneous injection activates its specific


lymphocytes in the
a. blood circulation.
b. draining lymph nodes.
c. MALT.
d. skin.
e. spleen.

6. To detect a humoral immune response to influenza virus, you would measure


a. cytotoxicity of virus-infected cells in the lung.
b. cytotoxicity of virus-infected cells in tissue culture.
c. dividing T cells in the draining lymph nodes.
d. plasma cytokine levels.
e. serum antibody titer.

7. During the lag period between antigen contact and detection of adaptive
immunity,

nolan x soju
a. antigen is hidden from the immune system in macrophages.
b. cellular immunity can be detected but antibodies cannot.
c. innate immune effectors are eliminating antigen.
d. innate immunity blocks the activation of adaptive immune effector cells.
e. new B and T cells with the appropriate antigen specificity must be produced in
the bone marrow.

8. To elicit the best antibodies to mouse MHC I, you should inject it into
a. a goat.
b. a mouse of the same genetic background (strain).
c. a mouse of a different strain.
d. a rat.
e. the mouse you isolated it from.

9. For specific antigen recognition by T cells,


a. antigen is bound by a T cell membrane antibody.
b. denaturation of antigen does not reduce epitope recognition.
c. MHC molecules are not required.
d. soluble antigen is bound directly without processing.
e. antigen exposure during T cell maturation is required.
10. The immune response to a booster vaccine is called a(n)
a. cellular response.
b. humoral response.
c. innate response.
d. primary response.
e. secondary response.

11. Immunogenicity
a. depends on the ability of the native antigen to be presented by MHC.
b. is usually a property of "self" antigens such as eye tissue.
c. is not a property of antibodies.
d. is not a property of haptens.
e. only applies to antigens that are composed of proteins.

12. Lymphocytes are activated by antigen in the


a. blood stream.
b. bone marrow.
c. liver.
d. lymph nodes.
e. skin.

13. A molecule that can be covalently linked to a non-immunogenic antigen to


make it an immunogen is called a(n)
a. adjuvant.
b. carrier.

nolan x soju
c. hapten.
d. mitogen.
e. superantigen.

14. A polyclonal antibody response


a. is not antigen-specific.
b. is produced only in response to polymeric antigens.
c. is produced by several B cells recognizing different epitopes on the same
antigen.
d. occurs during the lag phase of the immune response.
e. violates clonal selection.

15. Very low doses of antigen may induce


a. a secondary response.
b. hypersensitivity.
c. immunological ignorance.
d. low zone tolerance.
e. low zone immunity.
16. A virus vaccine that can activate cytotoxic T cells MUST contain
a. a high dose of virus particles.
b. an adjuvant to stimulate T cell division.
c. foreign MHC.
d. live virus.
e. virus peptides.

17. Which statement about antigen epitopes is FALSE?


a. An epitope may be shared by two different antigens.
b. A protein molecule usually contains multiple epitopes.
c. B cells bind only processed antigen epitopes.
d. Epitopes may be linear or assembled.
e. Some epitopes are more immunogenic than others

18. CD antigens
a. allow leukocytes to recognize antigen.
b. are each expressed on only one cell type.
c. are expressed on immune cells by immunologists to "mark" them for
separation.
d. are found only on leukocytes.
e. function as receptors for cytokine and CAMs.

19. A patient desperately needs a bone marrow transplant, and a perfect match
cannot be found. The rejection response in unmatched marrow is primarily due to
the presence of mature T cells that recognize the recipient's cells as foreign. To

nolan x soju
minimize this rejection response, the marrow can be treated before transfusion
into the recipient with complement plus antibody to human
a. CD3.
b. CD4.
c. CD8.
d. CD28.
e. CD154.

20. Antibody to membrane receptors sometimes inhibits receptor function and


sometimes mimics the action of the normal receptor ligand. (For example, some
antibodies to insulin receptor block the action of insulin and some mimic the
action of insulin.) An antibody which should NOT either block or stimulate B cell
function would be anti-
a. CD21.
b. CD56.
c. CD80.
d. Iga.
e. m chain.
part-3

1. Cytokines may exhibit __________ action, signaling the cells that produce them.

a. antagonistic
b. autocrine
c. endocrine
d. paracrine.
e. synergistic

2. Cytokines are NOT


a. antigen specific.
b. capable of activating more than one cell type.
c. made by lymphocytes.
d. small protein molecules.
e. synthesized de novo in response to antigen or other cytokines.

3. Several cytokines may have the same effect on the cells they bind. This is an
example of
a. a cascade.
b. antagonism.
c. pleiotropism.
d. redundancy.
e. synergy.

nolan x soju
4. Characterization of cytokine activities is NOT made more difficult by their
a. gene structure.
b. pleiotropism.
c. redundancy.
d. secretion close to target cell membranes.
e. short half-lives.

5. Interferons
a. activate B cells to make virus-specific antibodies.
b. are Th2 cytokines.
c. are virus proteins that interfere with activation of cytotoxic T cells.
d. block virus infection of host cells.
e. inhibit virus replication by infected cells.

6. A cytokine can do all of the following EXCEPT


a. bind to receptors which do not share cytokine-binding subunits.
b. bind to its specific receptor on the same cell that produced it.
c. bind to receptor antagonists produced by pathogenic viruses.
d. compete with other cytokines whose receptors share signal-transducing
subunits
e. upregulate (increase) synthesis of high affinity subunits for its receptor.
7. Members of a cytokine receptor family
a. all bind the same cytokines.
b. are grouped together because they share antigen specificity
c. are often found on the same cells
d. are similar in protein structure and sometimes in regions of amino acid
sequence.
e. are specific for cytokines produced by a single cell type

8. The ability of a cytokine to change gene expression in the target cell is


influenced by all of the following EXCEPT
a. presence of high-affinity receptors on the target cell.
b. presence of soluble cytokine receptors.
c. proximity of the producing and target cells.
d. rate of transport of cytokine-receptor complexes into the cytoplasm.
e. simultaneous production of another cytokine whose receptor uses the same
signal transducing subunit.

9. Cytokines are NOT


a. able to inhibit the function of other cytokines.
b. able to stimulate the synthesis of other cytokines.
c. produced by more than one cell type.
d. small protein molecules.

nolan x soju
e. stored in the cell for quick release.

10. The IL-2R subfamily consists of receptors for IL-2, IL-4, IL-7, IL-9, and IL-15.
This group of cytokine receptors
a. bind all five cytokines to promote synergistic action on target cells.
b. bind cytokines which are produced by the same cell.
c. each has a unique high affinity cytokine-specific a chain.
d. shift the immune response towards cellular immunity.
e. each has a unique signal-transducing g chain.

11. An antagonist for cytokine X may NOT be


a. cytokine A competing for a shared receptor subunit.
b. cytokine B which acts synergistically with cytokine X.
c. cytokine C which inhibits the activation of the cell that produces cytokine X.
d. made by microorganisms.
e. soluble cytokine X receptors.

12. A knock-out mouse for a particular cytokine allows immunologists to


characterize cytokine function
a. by doing a dose-response study with competing cytokines.
b. in the absence of all other cytokines.
c. on all cell types simultaneously.
d. under controlled conditions of local cytokine concentrations.
e. with defined cell populations.

13. Activated Tc can regulate immune responses by signaling activated


lymphocytes to undergo
a. apoptosis.
b. clonal deletion.
c. clonal proliferation.
d. cytotoxicity.
e. somatic hypermutation.

part-4

1. Complement
a. is a group of active proteolytic enzymes found in serum.
b. is secreted by macrophages and hepatocytes in response to antigen binding.
c. participates in both innate and adaptive immune responses.
d. prevents lysis of virus-infected cells.
e. All of the above statements about complement are true.

2. Complement is involved in all of the following except


a. attraction of neutrophils to an infection site.
b. increased presence of serum proteins in the infected tissues.
c. lysis of bacteria in the absence of specific antibodies.

nolan x soju
d. opsonization of microorganisms for phagocytosis.
e. sensitization of T cells to antigen

3. Complement is
a. activated by binding to specific complement receptors.
b. antigen-specific.
c. a potent promoter of virus entry into host cells.
d. a series of intracellular proteins which work with antibody to eliminate
endogenous antigen.
e. present in the circulation in an inactive form.

4. The alternative pathway of complement activation


a. causes tissue damage in the absence of C1INH.
b. occurs after the classic pathway is activated.
c. occurs only if the classical pathway is ineffective in pathogen clearance.
d. requires C3.
e. requires C4.

5. If a person is born without C2 and C4,


a. C5 can still be cleaved by the classical pathway.
b. C3b will not be able to bind to bacteria.
c. C9 will polymerize inappropriately and lyse host cells.
d. the classical pathway will be changed into the alternative pathway.
e. the amount of C3b produced during bacterial infections will be reduced.

6. Which of the following are least sensitive to complement-mediated lysis?


a. Enveloped viruses
b. Erythrocytes
c. Gram negative bacteria
d. Gram positive bacteria
e. Leukocytes

7. In the membrane attack phase of the classical complement pathway, the role
of C5b is to
a. activate the C5 convertase activity.
b. attract neutrophils to lyse the pathogen.
c. initiate formation of the MAC.
d. polymerize into a membrane-spanning channel.
e. All of these are activities of C5b.

8. Complement receptors (CR)


a. activate complement on the surface of pathogens.
b. bind only activated complement proteins.
c. inhibit complement activation on the surface of host cells.
d. on erythrocytes remove immune complexes from the circulation.
e. on macrophages signal host cells to make opsonins.

nolan x soju
9. As complement is activated by complexes of antibody-coated bacteria,
bystander lysis of nearby host cells is prevented by
a. a long-lived thioester bond on active complement proteins.
b. covalent attachment of all active complement proteins to the pathogen
surface.
c. plasma proteins that inactivate the anaphylatoxins.
d. proteins on host cell membranes that inhibit MAC formation.
e. the slow catalytic rates of complement proteases.

10. Complement activity is restricted by all of the following EXCEPT


a. dissociation of C3 and C5 convertases.
b. Gram positive cell walls that are resistant to MAC polymerization.
c. host cell plasma proteins that inactivate C3a, C4a, and C5a activity.
d. LPS in the outer membrane of Gram negative bacteria that inactivates C3b.
e. proteolytic cleavage of complement proteins into smaller fragments.

11. A deficiency in complement proteins or in their regulators can result in


a. blood in the urine from erythrocyte lysis.
b. decreased levels of certain complement proteins in the circulation.
c. immune complex disease.
d. increased numbers of infections.
e. All of the above can result from complement deficiencies.
part-5

1. Phagocytosis must be preceded by


a. antigen binding to the phagocyte.
b. chemotaxis.
c. extravasation.
d. integrin binding to Ig superfamily CAMs.
e. oxidative burst.

2. Phagocytes bind antigen using receptors for


a. C5a.
b. chemokines.
c. glucose.
d. LPS.
e. selectins.

3. Pathogens engulfed by macrophages


a. are completely degraded by hydrolytic enzymes into their component amino
acids and sugars.
b. are degraded to small peptides and carbohydrates which are presented on
Class I MHC to Tc.
c. may survive and replicate in the macrophage phagocytic vesicles.

nolan x soju
d. stimulate macrophages to adhere to B cells.
e. stimulate vascular endothelium to upregulateselectin expression..

4. An inflammatory response
a. is characterized by a decrease in vascular permeability.
b. is stimulated by cytokines produced by neutrophils.
c. occurs only during a secondary response.
d. recruits phagocytes to the infection site.
e. usually lasts for many weeks to ensure antigen is completely removed

5. Natural Killer cells


a. are stimulated to kill infected host cells via carbohydrate-binding receptors.
b. kill normal host cells with high levels of membrane MHC Class I.
c. kill virus-infected cells when the virus is acquired naturally but not by
immunization.
d .recognize virus-infected cells by the presence of viral peptide on MHC Class II.
e. secrete the complement MAC to lyse virus-infected cells.

6. Interferonsa and b do NOT


a. activate NK cells to kill virus-infected cells.
b. get synthesized by virus-infected cells in response to infection.
c. induce macrophages to increase expression of Class II MHC.
d. inhibit virus replication in infected cells.
e. stimulate expression of molecules required for Class I MHC presentation of
viral proteins.

7. Immune system cell adhesion molecules do NOT


a. allow macrophages to leave the circulation.
b. allow T cells to home specifically to peripheral or mucosal lymphoid tissue.
c. attract leukocytes to an infection site.
d. help cytotoxic T cells to bind to their targets.
e. signal neutrophils that they have arrived at an infection site.

8. Early induced immune responses are like adaptive immunity in that they
a. are antigen-specific
b. demonstrate immune memory.
c. involve macrophages and complement.
d. involve T and B lymphocytes
e. use pre-synthesized proteins which can be released quickly upon cell
activation.

9. Selectins
a. are present on both leukocytes and vascular endothelial cells.
b. bind Ig-like vascular addressins.
c. include ICAM, VCAM, and MAdCAM.

nolan x soju
d. select antigen-specific lymphocytes to extravasate into the infection site.
e. select antigen-specific macrophages to extravasate into the infection site.

10. Lymphocyte recirculation


a. activates inflammatory cytokines to promote antigen presentation to T cells.
b. allows B cells to go to the site of infection to produce antibody.
c. circulates lymphokines efficiently throughout the body.
d. occurs for both naïve and effector lymphocytes
e. only occurs during an infection.

11. Phagocytes kill bacteria using all of the following EXCEPT


a. H2O2.
b. hydrolytic enzymes.
c. low pH
d. lysozyme.
e. strong reducing agents.

12. For a circulating neutrophil to reach the site of inflammation, it must bind to
blood vessel endothelial cell and then pass between the endothelial cells in a
process called
a. addressinazition.
b. chemotaxis.
c. extravasation.
d. marginalization.
e. opsonization.

13. Macrophages are attracted to the site of infection by all of the following
EXCEPT
a. bacterial peptides.
b. chemokines.
c. C5a.
d. IL-8.
e. MAdCAM.

14. Inflammatory cytokines produced by macrophages activate all of the


following EXCEPT
a. B cells to secrete acute phase proteins.
b. integrin on leukocytes to bind more strongly to vascular CAMs.
c. neutrophils to be more cytotoxic.
d. NK cells to kill virus-infected cells.
e. vascular endothelium to increase expression of CAMs.

part-6

1. An antibody Fab contains

nolan x soju
a. complementarity determining regions.
b. H and L chain variable regions.
c. one antigen binding region.
d. one H-L interchaindisulfide bond.
e. all of the above.

2. Myeloma proteins are


a. abnormally formed antibodies secreted from cancerous plasma cells.
b. cancerous plasma cells that divide without requiring antigen activation.
c. cell lines that secrete specific antibodies for a short time, then die.
d. homogeneous antibody molecules secreted by plasma cell tumors.
e. protein signaling molecules that make a plasma cell become a multiple
myeloma.

3. The regions of the antibody molecule which contribute MOST to the affinity of
the antibody for antigen are the
a. CDR.
b. Fab regions.
c. Fc regions.
d. framework regions.
e. hinge regions.
4. Antibody Fc fragments contain
a. antigen-binding sites.
b. CDR.
c. complement-binding sites.
d. framework residues.
e. light chain variable domains.

5. The immunoglobulin isotype is determined by the


a. antigen specificity.
b. H chain constant region.
c. L chain variable region.
d. number of antigen-binding sites.
e. number of VH domains.

6. Which statement about antigen epitopes is FALSE?


a. An epitope may be shared by two different antigens.
b. A protein molecule usually contains multiple epitopes.
c. B cells bind only processed antigen epitopes.
d. Epitopes may be linear (composed of sequential amino acids) or assembled by
protein folding from amino acids far apart in the protein primary amino acid
sequence.
e. Some epitopes are more immunogenic than others.

7. An example of an antigen epitope from an infectious organism would be


a. a bacterial endotoxin (LPS) molecule.

nolan x soju
b. a fungal cell wall protein.
c. a peptide on the surface of a virus capsid protein.
d. a whole virus.
e. All of the above are antigen epitopes.

8. Antibody affinity for antigen depends on


a. the antibody isotype.
b. the complementary shape and charge of each antibody V region for its antigen
epitope.
c. the number of Fab regions in each antibody molecule.
d. whether the antibody is in the serum or on the cell surface.
e. whether the light chains are kappa or lambda.

9. Avidity
a. is a pathogenic agent, causing a very serious disease.
b. occurs when the ratio of antibody to antigen is optimal.
c. refers to the strength of interactions between a multivalent antibody and a
multivalent antigen.
d. results in a loss of antibody reactivity.
e. results in cross-reactivity when antibody binds two different antigens.

10. A colleague sends you an antibody to polio virus capsid protein. You perform
equilibrium dialysis on the antibody to measure its affinity. Plotting r/c versus r
gives you a curved line with K= 2.5 X 108 L/mole and an r intercept of 4. From
these results, you conclude that the antibody is probably
a. a cross-reactive antibody.
b. a monoclonal anti-polio virus antibody.
c. a polyclonal IgG antibody.
d. IgA anti-polio virus.
e. not specific for polio virus.

11. Allotypic determinants are


a. constant region determinants that distinguish each Ig class and subclass
within a species.
b. expressed only from the paternal chromosome.
c. generated by the conformation of antigen-specific VH and VL sequences.
d. Not immunogenic in individuals who do not have that allotype.
e. amino acid differences encoded by different alleles for the same H or L chain
locus.

12. Which of the following is NOT a characteristic of IgG?


a. It contains 2 g and 2 L chains
b. It crosses the placenta.
c. It is the predominant immunoglobulin in blood, lymph, and peritoneal fluid.
d. It is the largest of all the Igs.
e. Its L chains are either k or l.

nolan x soju
13. Human serum IgA is isolated and injected into a rabbit. The rabbit anti-IgA
antibodies will react against all of the following EXCEPT human
a. a chain.
b. IgG.
c. k chain.
d. l chain.
e. secretory component.

14. You have purified some Fab from an IgG myeloma protein. Under appropriate
conditions, you could use this Fab to generate antibodies to
a. both k and l chain.
b. g chain hinge region.
c. J chain.
d. g chain allotypic determinants.
e. the idiotype of this myeloma.

15. The Igisotype which would be most important for neutralizing polio virus
before it could infect intestinal cells would be
a. secretory IgA.
b. serum IgA.
c. serum IgD.
d. serum IgG.
e. membrane IgM.

16. Which of the following changes to a serum IgM antibody molecule would
definitely DECREASE its avidity?
a. Increase noncovalent antigen-antibody interactions in the CDR.
b. Remove the secretory component.
c. Replace the Fc portion of the mu chains with the Fc portion of alpha chains.
d. Replace VH and VL framework regions with those from a different antibody.
e. Use limited enzyme digestion to make Fab fragments.

17. IgA can be secreted from the body because it


a. binds poly-Ig receptor on mucosal epithelial cells.
b. has a specialized H chain called secretory chain.
c. has a special secretory idiotype.
d. is small enough to pass between mucosal epithelial cells and leave the body.
e. is synthesized by mucosal epithelial cells and secreted directly into the
intestinal lumen.

18. The ability to make antibody with the same antigen specificity but different
Fc regions
a. causes allelic exclusion of Ig molecules.
b. does not occur against bacterial antigens.
c. improves the antigen binding specificity of an Ig molecule.
d. increases the effector functions of Ig molecules.
e. requires clonal elimination.

nolan x soju
19. Allergy symptoms are produced when antigen binds to IgE on FcR on
a. A cells.
b. macrophages.
c. mast cells.
d. neutrophils.
e. Th1 cells.

20. One amino acid difference in the Fc region of different human g chains is the
epitope recognized by anti-
a. allotype.
b. idiotype.
c. isotype.
d. IgG.
e. g chain.

part-7

1. Genes for immunoglobulins are unlike other human genes in that


a. each polypeptide chain is encoded by several exons.
b. Ig genes are composed of introns and exons
c. somatic recombination occurs before mRNA is transcribed
d. there is less Ig genetic material in mature B cells than in other somatic cells
e. both c and d are true.

2. The gene segments needed to encode the variable region of a k chain are
a. one Jk plus one Dk.
b. one Jk plus one Ck.
c. one Vk plus one Dk.
d. one Vk plus one Jk.
e. one Vk plus one Jk plus one Dk.

3. Pseudogenes are DNA sequences which look very similar to functional genes
except for the presence of a(n)
a. intron.
b. leader sequence.
c. promoter codon.
d. signal sequence.
e. stop codon.

4. Combinatorial diversity says that by random combination of 40 functional Vk


segments with five Jk segments, the number of possible different k chains that
could be made are
a. 40.
b. 45.
c. 70.
d. 200.

nolan x soju
e. 1200.

5. Which does NOT contribute to Ig antigen-binding diversity


a. Any L chain can combine with any H chain to form a functional antibody.
b. Any Vk can be joined to any Jk to encode the light chain V region.
c. Many CH genes are present in the germline DNA.
d. Random numbers of N nucleotides can be added during somatic
recombination.
e. VJL and VDJH joining is imprecise.

6. The proper joining of one VL to one JL is regulated by


a. heptamer and nonamer sequences.
b. leader sequences.
c. P-nucleotide addition sites.
d. 12 and 23 nucleotide spacers between heptamer and nonamer sequences.
e. TdT binding site for DNA.

7. Since each B cell productively rearranges a single H and L chain allele, it


exhibits
a. affinity.
b. allelic exclusion
c. antibody restriction.
d. antigen-binding diversity.
e. cross-reactivity

8. Primary mRNA for H chain encodes


a. one VH, one DH, and one JH segment.
b. one VH, one DH, and multiple JH segments.
c. multiple VH, one DH, and one JH segments.
d. multiple VH, one DH, and multiple JH segments.
e. multiple VH, DH, and JH segments.

9. Somatic recombination occurs


a. in the bone marrow stem cell.
b. in the progenitor cell as it is becoming a B cell.
c. in the mature B cell following antigen contact.
d. in the plasma cell after antigen contact.
e. in the plasma cell after antibody secretion.

10. Junctional diversity affects primarily the amino acid sequence in


a. all CDR equally.
b. CDR1.
c. CDR2.
d. CDR3.
e. FR3.

11. Isotype switching

nolan x soju
a. changes the leader sequence exon so the antibody is secreted.
b. improves the antigen binding specificity of an Ig molecule.
c. increases the affinity of antibodies in a process called affinity maturation.
d. increases the functional diversity of Ig molecules.
e. occurs randomly between switch regions.

12. Isotype switching resembles somatic recombination because both processes


a. are catalyzed by the products of RAG1 and RAG2
b. are regulated by helper T cell cytokines.
c. can result in stop codons in coding sequences.
d. occur in developing B cells in the bone marrow.
e. result in the irreversible loss of DNA from the B cell.

13. Alternative mRNA splicing


a. allows the B cell to improve its antigen-binding fit after antigen contact.
b. allows the B cell to make membrane IgM from the mature mRNA for secreted
IgD.
c. can be used for the simultaneous production of any two Igisotypes.
d. is a process by which a B cell can simultaneously synthesize m and d chains.
e. occurs in response to T cell cytokines.

14. Because of the order of the CH gene segments (Cm, Cd, Cg3, Cg1,
pseudogeneCe, Ca1, Cg2, Cg4, Ce, and Ca2), a human B cell which undergoes
isotype switching from IgM to IgG1 can never in the future secrete
a. IgA.
b. IgE.
c. IgG2.
d. IgG3.
e. IgG4.

15. Isotype switching is always productive because


a. B cells produce all isotypes simultaneously.
b. isotype switching does not involve recombination of DNA gene segments.
c. no DNA is deleted from the chromosome in isotype switching.
d. no effector diversity results from isotype switching.
e. recombination between switch sites occurs in introns so it cannot introduce
stop codons into coding regions.

16. Somatic hypermutation does NOT


a. occur by somatic recombination.
b. occur during B cell proliferation.
c. occur in the B cell following antigen stimulation.
d. result in increased affinity of antibodies secreted later in immune responses.
e. result in the death of some B cells which no longer bind antigen.

nolan x soju
part-8
1. Which of the following is NOT True about TCR?
a. All TCRs on a particular T cell have identical idiotypes.
b. CDR3 of TCR has the most sequence variability from molecule to molecule.
c. TCR has binding sites for both antigen and self MHC.
d. TCR is a disulfide-bonded heterodimer.
e. The ab or gdisotype of TCR determines the biological function of its secreted
form.

2. The antigen-binding region of TCR is formed by the folding of


a. Va and Vb chains.
b. Va, Vb, and CD3 chains.
c. Va and Vb2-microglobulin chains.
d. Vg and Va chains.
e. VL and VH chains.

3. Which of the following properties are NOT shared by TCR and BCR?
a. Antigen-binding avidity is increased by the presence of two antigen binding
regions on each receptor.
b. Antigen-binding diversity is generated through gene rearrangement.
c. Folding of protein domains is maintained by intrachaindisulfide bonds.
d. Membrane expression and lymphocyte activation by antigen require receptors
to be associated with signal transduction molecules.
e. Receptor antigen-binding sites are formed from two polypeptide chains.

4. TCR most closely resembles


a. Class I MHC.
b. Class II MHC.
c. Fab region of immunoglobulin.
d. Fc region of immunoglobulin.
e. light chain of immunoglobulin.

5. Rearrangement of both TCR and BCR gene segments does NOT


a. generate diversity of antigen binding by recombination of a large pool of
germline V, D, and J segments.
b. lead to CDR3 being the most hypervariable region in the receptor chains.
c. require RAG-1, RAG-2, and TdT expression.
d. result in allelic exclusion of membrane receptors.
e. result in isotype switching after antigen stimulation of the mature
lymphocytes.

6. The amount of diversity in TCR generated within one individual by somatic


recombination
a. is higher than BCR diversity.
b. is about the same as for BCR diversity.
c. is lower than BCR diversity.
d. is lower than Class I MHC diversity.

nolan x soju
e. is lower than Class II MHC diversity.

7. T cells use all of the following for generating antigen-recognition diversity on


the TCR, except
a. combinatorial association of chains.
b. combinatorial association of segments.
c. large germline pool of gene sequences.
d. N region addition of nucleotides.
e. somatic hypermutation.

8. CD8 is a co-receptor on T cells that binds


a. CD3.
b. endogenous antigen peptide.
c. the constant region of Class I MHC.
d. the constant region of TCR.
e. the variable region of Class I MHC.

9. All of the following are true for antigen receptors on both B cells and T cells
EXCEPT
a. associated with signal transduction molecules in the membrane.
b. generated by somatic recombination during lymphocyte development.
c. members of the Ig gene superfamily.
d. MHC-restricted in their ability to bind antigen.
e. specific for a single antigen epitope.

10. Which of the following statements is FALSE?


a. TCR is allelically excluded on individual T cells.
b. CD4 and CD8 co-receptors are also signal transducing molecules for T cell
activation.
c. The arrangement of a chain gene segments most closely resembles that of k
chain.
d. The gene segments for the d chain are interspersed with those for the g chain.

e. The T cells that are most likely to react against allogeneic kidney cells are
CD8+ cytotoxic T cells.

part-9

1. Exogenous antigen includes all of the following EXCEPT


a. bacterial toxins.
b. extracellular protozoan parasites.
c. most bacteria.
d. ragweed pollen.
e. viruses.

nolan x soju
2. Human Class I MHC a chain molecules are
a. b2-microglobulin.
b. H-2 D, K, and L.
c. H-2 IA and IE
d. HLA-A. -B, and -C.
e. HLA-DR, -DP, and -DQ.

3. Cells which have MHC Class II are _________________, which present


_____________ antigen to Th cells.
a. antigen presenting cells, endogenous
b. antigen presenting cells, exogenous
c. infected cells, inflammatory
d. target cells, endogenous
e. target cells, exogenous

4. Signaling to a cytotoxic T cell that a liver cell is infected with hepatitis virus
depends on
a. binding of Ii to Class I MHC until the peptide is loaded.
b. binding of TCR on the cytotoxic T cell to Class II MHC on the infected cell.
c. binding of processed antigen to liver cell Class I MHC.
d. processing the hepatitis virus peptides to the correct size and anchor residues
in the endosomal pathway.
e. both c and d are correct.

5. Endogenous antigen presentation requires delivery of antigen peptides to the


endoplasmic reticulum by
a. Class I MHC and invariant chain.
b. calnexin and tapasin.
c. HLA-DM.
d. leader sequence.
e. TAP-1 and TAP-2.

6. Following virus infection, peptides produced from the proteasome are more
likely to be presented on the surface of the target cell because
a. MHC Class I is synthesized in response to virus infection.
b. proteasomal enzymes which produce shorter peptides are synthesized in
response to virus infection.
c. TAP-1 and TAP-2 specifically bind virus peptides.
d. virus amino and carboxyl terminal amino acids bind better to Class I MHC than
peptides from self proteins.
e. virus infection induces expression of proteases which cut proteins at sites
which bind best to TAP-1 and TAP-2.

7. Exogenous antigen is processed


a. after presentation by antigen presenting cells.
b. by nearly every nucleated cell.
c. by the cytosolic processing pathway.

nolan x soju
d. in the presence of b2-microglobulin.
e. in acidified endosomes.

8. Class II MHC does not efficiently present endogenous antigen because


a. antigen synthesized inside the cell never makes it to the endosomal
compartment.
b. endogenous antigen cannot be processed into peptides small enough.
c. HLA DM transports Class II to the surface before it can bind endogenous
peptide.
d. invariant chain blocks binding of endogenous peptide in the ER.
e. phagocytosed antigen binds Class II as rapidly as Class II is synthesized.

9. MIIC is a specialized intracellular compartment where


a. HLA DM promotes the release of CLIP and peptide binding to Class II MHC.
b. invariant chain binds to Class II MHC a and b chains.
c. peptides are transported into the ER for binding to Class II.
d. proteins are broken down into peptides by proteasomes.
e. some pathogens live protected from lysosomal enzymes.

10. In order to have pathogen peptide plus Class II MHC molecules expressed on
the membrane of host cells, all of the following are required EXCEPT
a. b2-microglobulin.
b. CLIP.
c. HLA-DM.
d. HLA-DR, -DP, and -DQ alpha chains.
e. Ii .

11. Invariant chain (Ii)


a. inhibits binding of endogenous peptide to Class I MHC.
b. is degraded in the MIIC compartment to CLIP.
c. is released from Class II upon binding of b2-microglobulin.
d. is the constant region of Class I peptide binding site.
e. prevents exogenous peptide binding to Class II MHC in the ER.

12. Antigen binding by Class I MHC molecules


a. accommodates many different peptides.
b. preferentially occurs for peptides 13-18 amino acids in length.
c. occurs at a site on Class I MHC formed by folding of a1 and b2-microglobulin
domains.
d. occurs only on antigen presenting cells.
e. takes place at the plasma membrane of the infected cell.

13. Both Class I and Class II MHC molecules are


a. composed of a and b chains with variable and constant regions.
b. expressed constitutively on all nucleated cells.
c. expressed on the B cell membrane.
d. part of the T cell receptor for antigen.

nolan x soju
e. synthesized in response to antigen processing.

14. The major histocompatibility complex has


a. dozens of loci for Class I and Class II proteins.
b. genes that encode proteins associated with antigen processing.
c. only genes encoding Class I and Class II molecules.
d. single loci for Class I and Class II proteins.
e. three regions encoding Class I, Class II, and Class III receptors.

15. MHC polymorphism


a. is generated by recombination of HLA A, B, and C gene segments.
b. is present primarily in the peptide-binding regions of MHC proteins.
c. is the result of random association of many alpha and beta genes.
d. restricts the ability of B cells to bind antigen.
e. results in expression of dozens of MHC alleles on each APC.

16. T cells are MHC-restricted in their ability to respond to antigen because


a. all antigen must be processed and presented to activate lymphocytes.
b. during an infection, all cells in the body present antigen on MHC Class I.
c. MHC binds antigen more specifically than TCR does.
d. TCR must recognize both antigen and MHC molecules.
e. the T cells should not respond to antigen on allogeneic cells.
17. Linkage of a disease to an HLA allele means that
a. everyone with that allele will eventually get the disease.
b. people with that allele have a higher risk for the disease.
c. the MHC protein encoded by that allele is defective.
d. the allele will eventually disappear from the population.
e. None of the above is true.

18. All of the following are associated with the expression of Class I MHC
molecules EXCEPT
a. antigen peptide presentation on membrane Class I MHC to Tc.
b. graft rejection.
c. increased risk of certain autoimmune diseases.
d. lysis of virus-infected cells.
e. stimulation of antibody production.

19. Human Class II MHC molecules


a. are encoded by the genes HLA-A, B, and C.
b. are found on all nucleated cells.
c. have an antigen binding site formed from regions of two polypeptide chains.
d. must be associated with b2-microglobulin molecules to bind peptide.
e. present antigen to CD8 cytotoxic T cells.

20. Humans inherit from each of their parents


a. a random set of MHC Class I, Class II, and Class III genes.

nolan x soju
b. enough diversity in MHC to present epitopes from most pathogens.
c. enough diversity in MHC to present every possible antigen epitope.
d. genes for a and b chains that can be recombined to increase their diversity.
e. the same Class I and Class II MHC genes as their siblings.

21. The a chain of HLA-DR


a. can be expressed with the b chain of any MHC molecule.
b. can be expressed with the b chain of any Class II MHC molecule.
c. can be expressed with the b chain of any Class II DR molecule.
d. must be expressed with b2-microglobulin.
e. must be expressed with the b chain of Class II DR from the same chromosome.

22. Which of the following statements is TRUE?


a. Each individual expresses all the diversity of MHC protein structure.
b. If a family has four children, no two of them will have the same MHC genotype.

c. Someone with bare lymphocyte syndrome who expressed no MHC proteins


would die in infancy.
d. TCR on Tc cells binds a1 and b2 domains of Class I MHC protein.
e. The chances of finding a tissue match are much higher between children and
their parents than between siblings.
23. Which of the following statements is FALSE?
a. All MHC alleles in the population have been counted.
b. CD4 T cells see antigen on self Class II MHC but not on self Class I MHC.
c. Human Class II MHC proteins are called HLA DP, HLA DQ, and HLA DR.
d. Class I and Class II MHC are less antigen-specific than Ig.
e. Peptides presented by Class I MHC must be 8-10 amino acids long.

24. Which of the following statements is FALSE?


a. A peptide binding to Class I must have certain amino- and carboxyl-terminal
amino acids to bind tightly to the ends of the Class I binding cleft.
b. A transplant is most likely to be successful between people who share the
same alleles at all Class I and Class II MHC loci.
c. Identical twins share all their Class I and Class II MHC alleles.
d. Peptide binding to TCR is influenced by both its own conformation and the
conformation of the MHC protein to which it is bound.
e. The gene for b2-microglobulin is in the Class I region of the MHC.

part-10

1. An antigen binding signal at the membrane results in the mature B


lymphocyte changing its
a. antigen-binding specificity.
b. color.
c. Ig V-D-J gene rearrangement.

nolan x soju
d. gene expression.
e. signal transduction molecules.

2. Signal transduction is the process of converting


a. a B cell to a T cell.
b. a binding signal to a chemical signal.
c. a hapten to an antigen.
d. IgA to secretory IgA.
e. a kinase to a phosphatase.

3. A ligand is
a. a cytokine.
b. a molecule that specifically binds a receptor.
c. an antigen.
d. an enzyme.
e. all of the above are ligands.

4. A tyrosine kinase which is activated by antigen binding is found in the


__________ of the BCr or TCR complex.
a. cytoplasmic domain
b. extracellular domain.
c. Ig superfamily domain.
d. transmembrane domain.
e. variable domain.

5. The ligand for TCR is


a. BCR.
b. MHC
c. MHC + peptide.
d. peptide.
e. TCR ligand.

6. An oncogene is a gene that is associated with


a. apoptosis.
b. cancer.
c. ITIMs.
d. TCR and BCR signal transduction.
e. viruses.

7. Antigen binding to B cells is most effective at sending an activation signal to


the B cell if it causes
a. antigen processing and presentation on Class II MHC.
b. BCR clustering.
c. BCR internalization.
d. inflammation.
e. opsonization.

nolan x soju
8. An enzyme which puts a phosphate group on a protein molecule is called a
a. co-receptor.
b. ITAM.
c. kinase.
d. phosphatase.
e. receptor.

9. Gene expression does NOT necessarily involve


a. changes in a cell's activities (phenotype).
b. mRNA synthesis.
c. protein synthesis.
d. DNA synthesis.
e. transcription factors.

10. The signal transduction molecules associated with TCR are


a. CD1.
b. CD3.
c. CD4.
d. CD8.
e. CD22.

11. The signal transduction molecules associated with BCR are


a. CD21 and CD81.
b. Iga and Igb
c. IgD and IgM.
d. ITAMs and ITIMs.
e. RAG-1 and RAG-2.

12. The second messenger IP3 increases the cytoplasmic concentration of


a. antigen.
b. calcium.
c. Class I MHC.
d. phosphate.
e. sodium.

13. DAG and IP3 are released from PIP2 by the action of
a. adaptor protein.
b. phospholipase C (PLC).
c. protein kinase C (PKC).
d. small G protein.
e. TdT.

14. Small G proteins (like Ras) convert GTP to GDP by their ___________ activity.
a. GEF.
b. kinase.

nolan x soju
c. phosphatase.
d. polymerase.
e. protease.

15. Transcription factors


a. increase synthesis of mRNA.
b. increase synthesis of DNA.
c. inhibit synthesis of mRNA.
d. promote DNA phosphorylation.
e. synthesize mRNA.

16. An enzyme cascade is a


a. case where the enzyme catalyzes its own inactivation, like small G proteins.
b. pair of enzymatic reactions that have opposite effects, like kinases and
phosphatases.
c. series of enzymatic reactions that result in cancer.
d. series of enzymatic reactions where the product of one reaction catalyzes the
next reaction.
e. small waterfall.

17. Signal transduction complex associates with TCR in the membrane through
a. agonist peptides.
b. covalent bonds.
c. enzyme cascades.
d. reverse phosphorylation.
e. salt bridges.

18. If IgaIgb cannot be made, B cells


a. cannot express BCR.
b. cannot express Class II MHC.
c. express 1,000-fold less BCR than usual
d. synthesize CD3 and become T cells.
e. require 1,000-fold more antigen to be activated.

19. The immune system of a person who had a mutation in CD3 could NOT fight
a viral hepatitis A infection by
a. blocking Hepatitis A virus from infecting liver cells with neutralizing IgG
antibodies.
b. generating cytotoxic T cells to lyse infected liver cells
c. lysing virus-infected cells with NK cells.
d. phagocytosing complement-opsonized Hepatitis A virus.
e. Both 1 and 2 are correct.

20. Amino acid sequences in lymphocyte signal transduction complexes which


are phosphorylated following antigen binding are called
a. ITAMs.
b. ITIMs.

nolan x soju
c. MAPs.
d. PTKs.
e. syks.

21. An immune deficiency resulting from a defective PTK in the activation


cascade in B cells would probably be characterized by
a. high numbers of circulating B cells.
b. high numbers of circulating lymphocytes.
c. high concentrations of plasma immunoglobulins.
d. low concentrations of plasma immunoglobulins.
e. low numbers of circulating T cells.

22. B cell co-receptor complex CD19, CD22, and CD81


a. allows B cells to be activated with 1,000-fold less complement-coated antigen.

b. allows B cells to be activated with 1,000-fold more complement-coated


antigen.
c. decreases B cell expression of BCR.
d. increases B cell expression of BCR.
e. prevents B cell activation by self antigen.

23. The anti-rejection drugs cyclosporin A and FK506 block rejection of


transplanted organs by interfering with
a. activation of a T cell transcription factor required for T cell activation.
b. antibody synthesis required for ADCC of transplanted cells.
c. CD3 expression.
d. MHC Class I expression.
e. processing of graft peptides and presentation on Class I MHC.

24. Antagonist peptides


a. fail to bind to T cells.
b. fully activate T cells.
c. interfere with T cell activation by agonist peptides.
d. partially activate T cells.
e. require partial agonist peptides to fully activate T cells.

25. Antibody-dependent cell-mediated cytotoxicity (ADCC) is a process in which


antibody-coated cells are killed by
a. the antibodies.
b. complement.
c. cytotoxic T cells.
d. cells with Fc receptors for IgG3.
e. cells with Fc receptors for IgE.

26. When IgE on mast cell FceR is cross-linked by, antigen, the mast cell

nolan x soju
responds by
a. apoptosis.
b. presenting the antigen to Th cells.
c. secreting IgE.
d. secreting histamine and other allergic mediators.
e. stimulating macrophage and neutrophil phagocytosis of the coated antigen.

27. Homeostasis is
a. macrophage activation by bacterial antigens.
b. programmed cell death.
c. the normal process of signal transduction.
d. the synthesis from all leukocytes from bone marrow stem cells.
e. the regulation of biological systems within normal limits.

28. STAT proteins are NOT


a. cytosolic proteins.
b. involved in cytokine signaling.
c. JAK kinases.
d. signal transducers.
e. transcription activators.

29. Cells receive a death signal through


a. bcl-2 receptor.
b. death receptor.
c. Fas.
d. Fas ligand.
e. STAT ligand.

30. The most important receptor through which lymphocytes receive life and
death signals is
a. antigen receptor.
b. bcl-2 receptor.
c. Fas receptor.
d. FcR.
e. growth factor receptor.

nolan x soju
1) Exogenous antigen is processed by
a) by every nucleated cell
b) by the cytosolic processing pathway
c) after degradation inside proteosome
d) after fusion of phagosomes with lysosomes

2) Th17 cytokine that promotes killing of extracellular pathogen is a


a) interferon gamma
b) interleukin 17
c) interleukin 10
d) interleukin 4

3) identify the cytokine that promotes he development of the Th2 response


a) interleukin 17
b) interleukin 4
c) interleukin 12
d) interleukin 5

4) positive selection induces apoptosis in developing T cells that bind to


a) Both MHC 1 and 2
b) Neither MHC 1 and 2
c) Self antigen
d) Foreign antigen

nolan x soju
5) somatic recombination occurs
a) After affinity maturation
b) In the progenitor cells as it is becoming a B cell
c) After isotype switching
d) In the plasma cell after antibody secretion

6) the isotypes of antigen specific receptors found in fully mature B lymphocytes are
a) IgM and IgD
b) secretory IgA and IgG
c) IgG and IgD
d) IgD and secretory IgA

7) interferon -gamma
a) Is released as a consequence of antigen induced activation of Th2 lymphocytes
b) Is released as a consequence of antigen induced activation ofTh1 lymphocytes
c) Is released as a consequence of antigen induced activation of Th17 lymphocytes
d) Is released as a consequence of antigen induced activation of treg lymphocytes

8)Positive selection of T lymphocytes


a) Takes place In the secondary lymphoid organs
b) ) Ensures the development of Th and Tc cells
c) Takes place in the bone marrow
d) Ensures the development of NK cells

9) phagocytes bind the pathogen using receptor for

a) C5a
b) Chemokines
c) Peptidoglycan
d) Selectins

10) Which antibodies are typically involved in both type 2 and type 3
hypersensitivity reaction?
a) IgA
b) IgG
c) IgD
d) IgE

11) The bacterial components that bind directly to the variable regions of beta chains of

nolan x soju
TCR and cross link them to the MHC molecules on APCs usually outside the normal
antigen binding groove are
a) Adjuvants
b) Carries
c) Mitogens
d) superantigens

12) Human class 2 MHC molecules


a) Are encoded by the genes HLA A,B and C
b) Are found on all nucleated cells
c) Are encoded by the genes HLA –DP,DR and DQ
d) Present antigen to CD8 cytotoxic T cells

13) lymphocytes are activated by antigen in the


a) Blood stream
b) Bone marrow
c) Liver
d) Spleen

14) the antigen specificity of an adaptive immune response is due to


a) Activation of antigen specific lymphocytes
b) Activation of macrophages
c) Lysis of only certain pathogens by neutrophils
d) Phagocytosis of pathogens by macrophages
15) a 59 year old women was treated with antibiotic for acute respiratory infection, ten
minutes after the ingestion patient developed pruritus, generalized urticaria ,
wheezing and edema of larynx. This is most likely to be due to
a) Systematic anaphylaxix
b) Serum sickness
c) Deposition of immune complex
d) Cytotoxic hypersensitivity

16) which cells are involved in DTH


a) TH1 and macrophages
b) NKT cells and mast cells
c) NK and B cells
d) B cells and eosinophils

17) which of the following statements nest explain the relationship between inflammation
of the heart(carditis) and infection with group A beta hemolytic streptococci ?
a) Streptococcal antigens induce antibodies cross reactive with heart tissue
b) Streptococcal are polyclonal activators of B cells
c) Streptococcal antigens bind to IgE on the surface of heart tissue and histamine

nolan x soju
is released
d) Streptococci are ingested by neutrophils that release proteases that damage
heart tissue

18) Bacteria growing within macrophages are killed by


a) Reactive oxygen and nitrogen intermediates
b) Complement components
c) Cytotoxic T cells
d) Antibody dependent cell cytotoxicity

19) removal of immune complexes are correctly described by which of the


following statement
a) It is independent of c3 for particulate complexes
b) It occurs primarily in the spleen and liver
c) It is independent of size of the immune complexes
d) It occurs primarily in the thymus and spleen

20)which experiment suggests that tumour cells express tumor specific transplantation
antigens?
a) When a syngeneic mouse is injected with the tumor cells, accepts the tumour
and dies
b) When a syngeneic mouse is injected with the tumour cells ,reject the
tumour and survives
c) When a allogenic mouse is injected with the tumour cells, rejects the tumour
and survives
d) When a allogenic mouse is injected with the tumour cells ,accepts the tumour
and dies

21) which category of hypersensitivity best describes transfusion reactions when a recipient
has antibodies against donor erythrocytes?
a) Anaphylactic
b) Cytotoxic
c) Immune complex
d) Delayed

22) complement activated ny pathogenic bacteria can contribute to the destruction of this
bacteria
a) By solubilization of immune complexes
b) By assisting phagocytes to attack to baterical cells
c) By attracting more treg cells to the site of infection
d) By deposition of immune complexes insite blood vessels

23) infantile X linked agammaglobulinemia is associated with excessive infection of the nolan x soju
following type
a) Intracellular bacterial
b) Capsulated bacteria
c) Viral
d) Fungal

24) which of the following statements about type 1 hypersensitivity is true


a) It is also known is delayed type hypersensitivity
b) Cytokines involved in this type of reaction are IL-4 ,IL-13,IL-6
c) Cytokines involved in this type of reactions are IFN gamma ,IL-17,IL-7
d) It is independent from pharmacologic mediators

25) viruses cause tumors in nude mice(which do not have a thymus because of genetic
defect) interpretation is that
a) Microphages are required to reject polyomavirus induced tumors
b) Natural killer cells can reject polyomavirus induced tumurs without help frm T
lymphocytes
c) T lymphocytes play an important role in rejection of polyomavirus induced tumours
d) B lymphocytes play no role in rejection of polyomavirus induced tumours

26) immune complexes are not involved in the pathogenesis of


a) Pigeon breeders disease
b) Serum sickness
c) Chrons disease
d) Post streptococcal glomerulonephritis

27) the first line of defense against virus is


a) Antibodies directed against external viral antigens
b) IgG directed to an internal viral antigen
c) IgM directed to an external viral antigen
d) Interferon produces by infected cells

28) formation of MAC in initiated by


a) C5a
b) C3a
c) C5b
d) C3b

29) CD8 is a co receptor in T cells that binds


a) Exogenous antigen peptide

nolan x soju
b) Endogenous antigen peptide
c) Alpha chain of class 1 MHC molecule
d) Beta 2 microglobulin of class 1 MHC molecule

30) lymphocyte recirculation


a) Activates inflammatory cytokines to promote antigen presentation of T cells
b) Allows lymphocytes to encounter an approptaite antigen
c) Circulates lymphokines efficiently throughout the body
d) Circulates chemokines efficiently throughout the body
31) type 2 hypersensitivity differs from type 3 in that
a) Mast cells trigger 2 , neutrophils trigger 3
b) In 2 phagocytes attack cell with surface antigen- antibody,in 3 they engulf immune
complexes
c) Penicillin has not been identified as a trigger of 2 ,but it does trigger 3
d) 2 puts a person at risk for kidney damage,3 does not

32) signal transduction is the process of converting


a) A B cell to a T cell
b) A binding signal to a chemical signal
c) A hapten to an antigen
d) IgA to secretory IgA
33) which one of the following on the surface of the macrophages initiates phagocytosis of
capsulated bacteria?
a) An antigen on the macrophage surface that is identical to the antigen on the capsule
surface
b) MHC class 2 molecules on the macrophage surface
c) A receptor for a fragment of activated complement
d) an Fc receptor on the macrophage surface

34 ) lymphocyte recirculation
a) Activates inflammatory cytokines to promote antigen presentation to T cells
b) Allows lymphocytes to encounter an appropriate antigen
c) Circulates lymphocytes efficiently throughout the body
d) Circulates chemokines efficiently throughout the body

35 ) pathological consequences of immune responses could be


a) Elimination of immune complexes

nolan x soju
b) Endotoxic shock
c) Artificial passive immunity
d) Natural active immunity

36 ) identify the part of antibody molecule that bind to the receptors on the macrophage
a) The constant portion of light chain
b) The variable domain of heavy chain
c) The Fc portion of antibody molecule
d) The Fab portion of antibody molecule

37 ) immune complex formation can result from all except


a) Perisistent infection
b) Inhalation of antigenic material
c) Autoimmune disease
d) Intradermal antigen

38) a diagnostic marker for tumours of the colon is


a) Alpha –fetoprotein
b) Common acute hyphoblastic leukemia antigen (CALLA )
c) EBV related antigen
d) Carcinoembryonic antigen (CEA)
39) plasma cell secreting IgA antibodies would be most likely to associate with
a) Tumours
b) Lymph nodes
c) MALT
d) Bone marrow

40) Naive Treg cells unlike naive Th cells express


a) CD4
b) CD25
c) FOXp3
d) CD3

41) which of the following is correct regarding the antibodies which provide protection
against bacterial exotoxins?
a) Activate complement that breaks down the toxins molecules and bacterium
fragments
b) Enable macrophages to present toxin antigen to T lymphocytes
c) Bound to the toxin molecules and prevent the toxin from bindind to the the body cell
surfaces
d) Causes blood vessels to become slightly leaky allowing cells on the blood stream to
enter the tissue and destroy toxic molecules

nolan x soju
42) for specific antigen recognition by T cells
a) CD 25 molecules are required
b) HLA molecules are required
c) Soluble antigen is bound directly without processing
d) Anti gen exposure during T cell maturation is required
43 ) all of the following are associated with the expression of class 1 MHC molecules except
a) Peptide presentation via class 1 MHC to T cytotoxic cells
b) Acute and chronic graft rejection
c) Lysis of virus infected cells
d) Affinity maturation of antibodies

45) amoxicillin is a small molecule that does not induce antibody formation. However it
binds to serum protein and forms a complex that in several people induces allergic reaction.
Amoxicillin is therefore
a) An antigen
b) A hapten
c) An immunogen
d) A super antigen

46) which statement about antigen epitope is FALSE


a) An epitope may be shared by two different antigens
b) A protein molecule usually contains multiple epitopes
c) Epitope is a part of an antibody which recognizes and binds to an antigen
d) Some epitopes are more immunogenic than others

47) identify the isotypes of antibodies involved in providing protection in a new born infant
a) IgM and IgE
b) IgD and IgF
c) IgA and IgM
d) IgG and IgA

48) the immunoglobulin isotype is determined by the

a) Antigen specificity
b) H chain constant region
c) L chain variable region
d) Number of antigen binding sites

49) what naive T cytotoxic cells will do if they encounter the antigen without any co-
stimulation?
a) Undergo significant cell division
b) Die(via apoptosis )
c) Become anergic
d) Express CTLA-4 in high concentration on its surface

50) Humeral immunity can be acquired passively by

nolan x soju
a) Receiving complement factors from someone who had recovered from infection
b) Receiving a vaccine of influenza virus grown in eggs
c) Receiving serum from someone who had recovered from an infection
d) Receiving leukocytes from family member

51) What is the role of macrophage in antibody formation?


a) Development of allergic reaction
b) Lysis of virus infected cells
c) Antigen presentation
d) Synthesis of antibodies

52) which of the following cell surface antigen markers are used to identify the T cells in the
blood sample?
a) CD3
b) CD8
c) CD20
d) CD56

53) Rapid but non antigen specific immune response are produced by the
a) Adaptive immunity system
b) Innate immunity system
c) Lymphocytes
d) Lymphatic system
54) the naive lymphocytes encounter the antigens for which they are specific during the
priming phase
a) In the primary lymphoid organs
b) In the thoracic duct
c) In the secondary lymphoid organs
d) In the blood

55) destruction of virus infected cells by T cytotoxic cells is initiated by interactions between

a) CD4 on T cytotoxic cell and MHC class 1on virus infected cell
b) CD8 on virus infected cell and MHC class 1 on T cytotoxic cell
c) MHC class 2 on virus infected cell and CD4 on T cytotoxic cell
d) CD8 on T cytotoxic cell and MHC class 1 on virus infected cell

56) the signal transduction molecules associated with TCR are


a) CD1
b) CD3
c) CD4
d) CD8

57) Select incorrect answer. Complement that has been activated by bacterial cells can
contribute to the destruction and elimination of these cells
a) by attracting more phagocytes to the area of complement activation
b) by stimulating somatic hypermutuation in the phagocytes that are already
actively engaged in phagocytizing the bacterial cells

nolan x soju
c) by assisting phagocytes to attach the bacterial cell
d) By destructing bacterial cell membrane

58) Which on the following statement regarding various disease is true?


a) Tuberculoid leprosy associated predominantly Th2 profile
b) Epitheloid cell and giant cell are typically of immune complex disease
c) Lepramatous leprosy leprosy associated predominantly Th1 profile
d) Crohns disease is inflammatory disease in which granulomatous reaction cause
strurcture and

59) what is the major consequences of the bone marrow transplantation in


immunocompromised patients? Potentially lethal graft versus host disease
60) Delayed hypersensitivity reaction characterized by
a) Edema without a cellular infiltrate
b) Ipnfiltrate composed of neutrophils
c) Differerentiation of macrophages into epitheloid cell
d) Infiltrate composed of eosinophils

61 ) In the thymus, T cells that recognize MHC class 2 molecules differentiate to become ?
a) CD8 lymphocyte
b) Gamma/delta T cell
c) NK cell
d) CD4 lymphocyte
62) a secondary Immune response may develop
a) Result from activation of naïve B cells
b) Increase adaptive immunity than primary immunity
c) Increase primary response than adaptive immunity
d)

63) strongly immunogenic tumours appear


a) In all cancers
b) In immune suppressed patients
c) Only lymphoma and leukaemia
d) In case liver caner

64) Identify the fundamental difference between antigen recognition by B and T cells
a) Antigen most presenting in different way
b) Clocal selection after antigen recognition herterogenisity
c) Herterogenesity of Both T cell and B cell from one lymphocyte
d) Membrane location of antigen specific epitope

65 ) T cells that recognize MHC class 1 molecule differentiate in the thymus to become
a) CD8 cytotoxic lymphocyte
b) Nature killer cell
c) CD4 helper cell

nolan x soju
d) Neutrophils

66) the major mechanism of host resistance to M.Leprae

a) Humurol antibody type


b) Type 4 hypersensitivty
c) High level of calcium in serum
d) Type 1 hypersensitivty

67) Mast cell release mediators after


a) Histamine stimulation activate toll ike receptor
b) Corss linking og Fc receptor on the surface
c) IgE binding to Fc receptor
d) Histamine stimulation

68) type 2 hypersensitivity is


a) Antibody independent
b) Complement independent
c) Cause by antibody against soluble antigen
d) Cause by antibody against cell surface antigen
69) Positive tuberculin skin test is performed by injecting 0.1 ml of tuberculin purified
protein derivative (PPD) into the inner surface of the forearm .histologically the reaction site
would most probably show
a) Mast cell
b) Neutrophils
c) Th cell and macrophage
d) T and B cells

70) when cell die by apoptosis as compared to by necrosis they potent to induce
inflammation and damage of the surrounding tissue because they
a) Don’t release cytoplasmic content
b) Are not subsequently phagocytosis
c) Migrate into the blood stream prior to death
d) Rarely malignant or infected by virus

71) Against which pathogen IgE antibody can provide immunity ?


a) Human hepatitis c virus
b) Schistosoma mansoni
c) Staphylococcus aureus
d) Salmonella species

nolan x soju
72) examples of primary T cell deficiency disease includes
a) Leukocyte adhesion deficiency
b) Wiskiit Aldrich syndrome
c) Chronic granuloma disease
d) Hereditary angio neurotic edema

73) patient with di George syndrome have


a) Increase number of T cells
b) Selective IgA deficiency
c) Abnormal antigen presenting cells
d) Decrease number of T cells

74) non photosynthetic protists growing as a mass of branching filaments are called
a) Fungi
b) Pr 1otozoa
c) Algae
d) Chlamydie
75) in a person with HIV infection potentially fluids include all of the following except
a) Blood
b) Genital secretion
c) Urine
d) Amniotic fluid

76) staphylococci resistant to penicillin G produce


a) Beta catalase
b) Beta lactase
c) Gamma catalase
d) Gamma lactamase

77) which mechanism is most likely to explain hemolysis in patient with auto antibodies
a) Pirforins from cytotoxic T cells lyse red cell
b) IL-2 binds to receptor to cell which auto lysis of red cell
c) Complement is activated and the complete lysis the red cell
d) Neutrophils that use protease lyse the red cells

78) antigens normally expressed only in embryonic cells but also found in a certain tumor
cells known as
a) cryptic antigen
b) Oncofetal antigen
c) Super antigen

nolan x soju
d) HTLV-1

79) The specificity of any lymphocyte is determined by


a) The type of CD molecule binds to it
b) The specificity of the gene receptor
c) MHC class 1 molecule
d) recognition receptor

80) Somatic hypermutation occur by


a) Somatic recombination
b) T cell progression
c) B cell antigen
d) Result from specific antibodies

81) antibody cross reactivity is administered b


a) Antibody binding to CD3 T cells surface molecule
b) Antigen –antibody complex
c) Antibody structure
d)

82) the region of antibody molecule with the highest affinity to antigens are?
a) Hypervariable region
b) Fc region
c) Framework region
d) FAB region

83) T cell located at the bacteria


a) Helper T cell
b) Cytotoxic T cell
c)
d)

84) Which of the following immune factor mechanism is the most important in completely
of infection caused by streptococcus pneumonia
a) Antibody dependent cell cytotoxicity
b) Complement mediated opsoization
c) Cytotoxic t cell lymphocyte
d) NK cells

85) both class 1 and class 2 MHC molecules are?


a) Express constantly on nucleated cells
b) Express on surface of antigen presenting cells(APC)
c) Parts of T cell receptor
d)
86) which cell is the first to encounter a bacterial infection in the tissues and is capable of
phagocytosing bacteria followed by the cytokine secretion?

nolan x soju
a) NKT cell
b) Dendritic cell
c) Gamma/delta t cell
d) Resident macrophage

87) a cell producing cytotoxic compounds following TH1 cell activation is a(n)?
a) mature dendritic cells
b) Macrophage
c) basophils
d) Th1 cells activation

88) indicate vaccine target that might prove useful in burkitt lymphoma ?
a) HTLV-1
b) Human papilloma virus
c) Epstein barr virus
d) Cytomegalo virus

89) all of the following are true for immune hemolytic anemia except ?
a) It is the clinical condition in which IgG antibodies bind to red cell surface antigens
b) Is is the clinical condition in which IgM antibodies bind to red cell surface
antigens
c) Antibodies initiate red cell destruction via the complement system and
phagocytes
d) Red cell destruction occurs by T cytotoxic and NK cells.

90) Type 4 hypersensitivity reaction results in?


a) Formation of granuloma
b) Development of anaphylatic reaction
c) Serum sickness
d) Complement mediated lysis of infected cells

nolan x soju
1. what is the source of all undifferentiated stem cells that can produce any of the cells
involved in immune response ?
a) The spleen
b) The bone marrow
c) Any lymph node
d) The thoracic duct

2. somatic recombination occurs


a) After affinity maturation
b) In the progenitor cell as it is becoming a B cell
c) After isotype switching
d) In the plasma cell afte antibody secretion
3. antibody effector function include all of the following except
a) activation of complement on the bacterial surface
b) activation of proteolysis of endocytosed proteins

c) blocking uptake of bacterial toxins


d) coating bacteria to promote their phagocytosis

4. vaccine stimulate
a) Natural passive immunity
b) Artificial active immunity
c) Artificial passive immunity

nolan x soju
d) Natural active immunity

5. the antigen specificity of an adaptive immune response is due to


a) Activation of antigen –specific lymphocytes
b) Activation of macrophages
c) Lysis of only certain pathogens by neutrophils
d) Phagocytosis of pathogens by macrophages

6. which of the following human cell types are not capable of presenting peptides using MHC
class 1?
a) Fully mature lymphocytes
b) Dendritic cells
c) Red blood cels
d) Macrophages

7. Isotype switching resembles somatic recombination because both processes


a) Are catalysed by the products of RAG1 and RAG2
b) Are regulated by helper T cell cytokines
c) Occur in developing B cells in the bone marrow
d) Result in loss of DNA from the B cell

8. engagement of Fas receptor by Fas ligand (FasL) results in


a) Apoptotic cell death
b) Clonal proliferation
c) Cytotoxicity
d) Somatic hypermutuation

9. for the generation of TCR diversity T cells use


a) Isotype switching
b) Affinity maturation
c) N region addition of nucleostides
d) Somatic hypermutation

10. antibody Fc fragments


a) Contain antigen binding sites
b) Contain complementarity determining regions
c) Bind to complement component
d) Binds to toll like receptors

11. activation of mast cells by anaphylaatoxins (C3a and C5a) causes


a) Apoptosis of mast cell
b) Presentation of the antigen to Th cells

c) Secretion of IgE antibodies


d) Secretion of allergic mediators

12. TCR most closely resembles


a) Class 1 MHC
b) Class 2 MHC
c) Fab region of immunoglobulin
d) Fc region of immunoglobulin

13. priming of naïve T helper cells by mature dendritic cells in the lymph node involves
a) CD40 on DC with CD40L on naïve T cell

nolan x soju
b) CTLA-4 on T cell with peptide presented by DC
c) CD56 on naïve T cell with CD 29 on DC
d) CD8 on naïve T cell with MHC class 1 on DC

14. identify the cytokine that promotes the development of the Th2 response
a) Interleukin 17
b) Interleukin 4
c) Interleukin 12
d) Interleukin 5

15. The IgG isotype is often used as an example of the typical immunoglobulin .identify the
physical or biological characteristics of one antibody molecule that has the IgG isotype?
a) Has one heavy chain and two identical light chains
b) Is able to bind two different antigens/epitopes
c) Papain treatment will produce two identical Fab fragments and two identical Fc
fragments
d) Is able to bind to specific antigen

16. human class 2 MHC molecule


a) Are encoded by the genes HLA-A,B and C
b) Are found on all nucleated cells
c) Are encoded by the genes HLA-DP,DR and DQ
d) Present antigen to CD 8 cytotoxic T cells

17. what immune function is likely to be affected in case of mutuation in the Fc gamma
receptor gene?
a) Opsonisation
b) Lymphocyte adhesion
c) Antigen binding
d) Intracellular killing

18. Signal transduction is the process of converting


a) A b cell to a T cell
b) A binding signal to a chemical signal
c) A hapten to an antigen
d) IgA to secretory IgA

19.a cell which secretes large quantities of antibodies but does not express surface
immunoglobulin is a
a) Naïve mature B cell
b) Memory B cell
c) B 1 cell
d) Plasma cell

20. identify an important cytokine receptor interaction which initiates extensive cell division
of the primed T cell
a) IL 2 and high affinity IL 2 receptor
b) IFN gamma and the IFN gamma receptor
c) IL 7 and high affinity IL 7 receptor
d) IL 7 and low affinity IL 7 receptor

nolan x soju
21. natural killer cells

a) Are inhibited to kill infected host cells by MHC class 1 molecules


b) Are activated to kill infected host cells by MHC class 1 molecules
c) Kill virus infected cells when the virus is acquired normally but not by immunization
d) Secrete the complement MAC to lyse virus infected cells
22. besides the direct contact between the B lymphocytes and the antigen ,antibody production
by B lymphocytes requires
a) Help from a T helper cell
b) Help from a macrophage
c) Help from a NKT cell
d) Help from a T cytotoxic cell

23. Th17 cytokines that promotes killing of extracellular pathogen is


a) Interferon gamma
b) Interleukin 17
c) Interleukin 10
d) Interleukin 4

24. just before they enter blood circulation from the lymphatic system circulating lymphocytes
will be found in
a) A high endothelial venule
b) A lymph node
c) The thoracic duct
d) The bone marrow

25. inflammatory cytokines produced by macrophage activate all of the following except
a) Complement system via classical pathway
b) Integrin on leukocytes to bind more strongly to vascular CAMs
c) Neutrophils to be more cytotoxic
d) NK cells to kill virus infected cells

26. genes for immunoglobulin are unlike other human genes in that
a) Antibody genes are composed of introns and exons

b) DNA for antibody moecules in inherited from only one parent


c) Gene segments must be spliced together to make each unique antibody molecule
d) Several introns encode each antibody molecule

27. which cells in the body are involved in the antibody mediated allergic reactions?
a) Macrophage and NK cells
b) Mast cells and basophil
c) Gamma/delta T cells
d) Any phagocytic cell

28. it was observed that milkmaids infected with cowpox were later immune to smallpox
infections . this is an example of an
a) Innate immunity to a cross reactive antigens
b) Innate immunity of milkmaids to smallpox
c) Memory response to a cross reactive antigen
d) Passive immunization from contact with cows milk antibodies

29. which of the following is the neutralizing antibody

nolan x soju
a) IgM
b) IgG
c) IgD
d) IgE

30. all of the following are associated with the secretion of proinflammatory cytokines except
a) Failure to make immune response following stimulation by antigen
b) Upregulation to adhesive molecules on the vascular endothelium
c) Migatrion of inflammatory cells into the damaged area
d) Increased permeability of vascular vessels

31. isotype switching


a) Decreases the functional diversity of Ig molecues
b) Improves the antigen binding specificity of an Ig molecule
c) Increases the affinity of antibody in a process called affinity maturation
d) Increases the functional diversity of Ig molecules

32. phagocytes that encounter bacterial cells in the extra vascular tissues are stimulated to
release cytokines and chemokines. Choose the event occurring as a result of the release of
these cytokines and chemokines by the phagocytes?
a) The spaces between the cells that form the blood vessel do not increasd
b) Some of the phagocytes are stimulated to secrete specific antibodies
c) Some phagocytic cell in the nearby blood stream are actually attracted to the site of
the infection
d) T lymphocytes leave the blood circulation at the site of the infection to produce
specific antibodies

33. an antibody Fab fragment


a) Contains complementarity determining regions
b) H and L chain constant regions
c) Different antigen binding regions
d) Binds to Fc receptors

34. peripheral lymphoid organs


a) Are located in the abdomen to protect their viral functions
b) Maximise contact between antigen and lymphocytes
c) Produce antigen specific lymphocytes from stems cells
d) Ssequester antigen to minimize its damage to the body

35. lymphocytes completely recirculate through peripheral lymphoid tissue in order to


a) Be killed before they cause autoimmunity
b) Efficiently encounter antigen
c) Mature from stem cells into lymphocytes
d) Phagocytose antigen and kill it

36. cytokines are


a) Not able to increase blood vessel permeability
b) Not able to induce antigen specific response
c) Able to induce antigen specific response
d) Able to activate complement system

37. which of the following is not a characteristic of IgG?


a) It crosses the placenta

nolan x soju
b) It neutralizes tooxins
c) It is aa major antibody of saliva
d) It activates complement

38. identify the part of the antibody molecule in which of the following in which the amino
acids form the most specific bonds with an antigen
a) The framework
b) The variable
c) The hypervariable
d) The constant domain

39. in case of C2 and C4 deficiencies


a) C5 can still be cleaved by the classical pathway
b) C3b will not be able to bind to bacteria
c) C9 will not polymerize and lyse host cells
d) C3b production will be reduced

40. effects of bacterial toxins could best be counteracted by


a) Antibody binding and neutralization of the toxin
b) Antibody opsonisation and phagocytosis of toxin producing bacteria
c) B cell binding to Toxin and producing bacteria
d) Cytotoxic T cell binding and lysis of toxin producing bacteria

41. which statement about Treg cells is false?


a) Natural Treg cells differentiate in the thymus
b) iTreg cells differentiate in the periphery

c) Tregs secrete immunosuppressive cytokines


d) Tregs secrete proinflammatory cytokines

42.a cell expressing CD3+ ,CD25+ and FoxP3+ is a


a) Gamma/delta T cell
b) Cytotoxic T cell
c) Natural killer cell
d) Regulatory cell

43. clonal selection


a) Begins with inflammation
b) Occurs for all leukocytes
c) Occurs in response to self antigen
d) Occurs for all lymphocytes

44. phagocytes can not attach to the bacterial cell wall surface by which of the following ways
a) Directly attaching to the bacterial cell wall lipopolysaccharides
b) Attaching to molecules on the bacterial cell surface using MHC class 2 molcules on
the phagocyte
c) Using an antibody molecule that is specific for an antigen on the surface of the
bacterial cell
d) Using a fragment resulting from complement activation by the bacterial cell

45. the ability of an antigen to induce an immune response depends on

nolan x soju
a) The antigens ablity to enter the thyroid
b) The antigens dose and size
c) Antigen processing mechanism
d) Antigen receptor gene rearrangement

46. antibody dependent cellular cytotoxicity (ADCC) requires the involvement of


a) Dendritic cell and infected cel
b) Macrophage and cancer cell
c) Erythrocyte and antibodies
d) Antibodies and follicular dendritic cell

47. an example of innate immunity


a) Antibody production by plasma cels
b) Pathogen recognition by dendritic cells
c) Distribution of infected cell by cutotoxix lymphocytes
d) Memory response to influenza virus

48. secondary immune response compare to primary contains


a) Low level of specific of IgG antibodies
b) High level of of specific of IgM antibodies
c) high level of specific of IgG antibodies
d) high level of specific of IgD antibodies

49. what is likely to be elevated in patient infected by Trichinella spiralis?


a) Basophils
b) Neutrophils
c) Lymphocytes
d) Eosinophils

50.A CD3+ cell that secretes perforin and granzyme is a


a) gamma/delta T cell
b) cytotoxic T cell
c) helper T cell
d) regulatory T cell

51. autoimmunity results from


a) antigen driven self reactive lymphocytes
b) accelerating clearance of apoptotic cells
c) elimination of extracellular pathogens
d) activation of Treg cells

52. Complement activated by pathogenic bacteria can contribute to the destruction of this
bacteria
a) by solubilisation of immune complex
b) by assisting phagocytes to attach to the bacterial cells
c) by attracting more Treg cells to the site of infection
d) by deposition of immune complexis insite blood vessels

53. which category of hypersensitivity best describes transfusion reactions when a recipient
has antibodies against donor erythrocytes?
a) Anaphylactic
b) Cytotoxic

nolan x soju
c) Immune complex
d) Delayed

54. Respiratory distress and unconsciousness developed wothin minutes after pencillin
injection is probably medicated by
a) IgE antibody
b) IgG antibody
c) Sensitized T cells
d) IgM antibody

55.a Tc cell can mount a cytotoxic attack on a viral infected cell if it binds to a cell
displaying
a) Insufficient MHC 1
b) Insufficient MHC 2
c) MHC 1 bearing foreign antigen
d) MHC 2 bearing foreign antigen

56. the inflammation that results from a mosquito bite is different from the inflammation
that results from a positive TB skinn test in that the reaction to the mosquito bite
a) Does not require prior sensitization
b) Takes more time to appear
c) Results primarily from antibody antigen interactions
d) Involves basophils,mast cells and Tc cells
57. Patients with wiskott Aldrich syndrome have
a) An increased number of B cells
b) An increased number of T cells
c) Low platelet level in blood
d) High platelet level in blood

58. immune complex formation can result from all except


a) Persistent infection
b) Inhalation of antigenic material
c) Autoimmune disease
d) Intradermal antigen

59. Opsonin treated bacteria are more readily engulfed by phagocytes than are untreated
bacteria because
a) The capsule is removed by opsonin
b) Opsonin digest the wall component
c) Opsonin induces lysosomal enzymes
d) Phagocyte contains receptors that bind antibodies

60. lupus erythematosis is


a) Free from an immune complex disease component
b) Associated with antibodies against nucleic acid
c) A disease of red and white cells
d) An organ specific autoimmune disease

61. Di George syndrome results from a defect in

nolan x soju
a) Purine nucleoside phosphorylase
b) WASP
c) Thymic development
d) DNA repair

62. IgE production depends in


a) Th1 cells
b) Th2 cells
c) Th17 cells
d) NK cells

63.a patient is found to have complete absence of C1 inhibitor. The most likely consequence
of this deficiency is
a) hereditary angioedema
b) immune complex disease
c) granuloma formation
d) deposition of immune complexes

64. The lesion in immune complex induced glomerulonephritis are dependent on


a) Erythroocytes and complement
b) The production of urine
c) Complement and neutrophils
d) The presence of macrophages
65. certain HLA genes are linked to autoimmune diseases because
a) It may be the result of a single gentic defect
b) It may be the result of the decreased number of helper T cells
c) It may be result of cross reactivity between self antigen and infection agent
d) It carries no increased risk for a specific disease for those individuals with the gene

66. Th2 cells are responsibe for fighting against


a) Tuberculosis bacteria
b) Flu virus
c) Tumor cells
d) Worms

67. Granulomatous hypersensitivity is correctly described by which of the following


statements?
a) Granuloma contains mast cells and neutrophils
b) IgG has a major role in granuloma formation
c) IFN gamma is required for granuloma formation
d) IL-4 is required for granuloma formation

68.a common strategy by which microbes survive their hosts immune response involves

nolan x soju
changing the structures of the molecules they produce so that they no longer recognized by
the hosts immune system. This strategy called antigenic variations is most likely to allow
evasion of which type of immune recognition?
a) Toll like receptor dependent recognition
b) Mannose receptop dependent recognition
c) Antibody recognition of microbial cell surface molecules
d) Natural killer cell inhibitory receptor recognition of class 1 major histocompatabiity
complex(MHC)molecules on infected cells

69. the most important receptors that act as coreceptors for HIV are
a) CXCR5 and CCR6
b) CXCR4 and CCR5
c) CXCR7 and CCR5
d) CXCR6 and CCR4

70. infantile X linked agammaglobulinemia is associated with excessive infections of the


following type
a) Intracellular bacterial
b) Capsulated bacteris
c) Viral
d) Fungal

71. bacteria growing within macrophages are killed by


a) Reactive oxygen and nitrogen intermediates
b) Complement components
c) Cytotoxic t cells
d) Antibody dependent cell cytotoxicity

72. which of the following is a common cause of hypersensitivity disease


a) Failure of lymphocyte maturation
b) Failure of self tolerances
c) Treatment with corticosteroids
d) Deseminated cancer

73. positive skin test showing delayed type hypersensitivity such as for mumps or
tuberculosis indicate that
a) humoral immune response has occurred
b) a cell mediated immune response
c) only B cell system is functional
d) the patient has immune deficiency

nolan x soju
1. Which of the following statement are correctly describe type 3
hypersensitivity? a.it is occur within 72 hrs of pre-exposure to antigen
b.it is not mediated by compliment
c. fixing igG compliment is an imp mediator
desensitization is used for its control

2. Identify a pathogen against which IgE antibody is the most important to provide
immunity?
a.capsulated bacteria
b Parasite worm
c.macrobacterial tuberculosis
d.heptitis c

3. Effector function of complement include all of the following except

a.attracting phagocytes to the site of infection


b.facilitating phagocytosis of compliment coated bacteria
c.lysing bacterial uninfected cells
d.blocking uptake of bacterial toxins

nolan x soju
4. Which statement about the deficiency of complement protein is false ?
5. Administration of the rabies vaccine would stimulate which of the following types of
immunity ?
a.natural passive

b.artifical active
c.artifical passive
d.natural active
6. To provide protection from the bacterial exotoxins ,antibodies
following mechanim
7. The ligand of BCR is
a.t cell receptor (tcr)
b.MHC class 1 molecule
c.antigen determinant
d.mhc class 2 molecule

8. A typical IgM isotype antibody molecule does not consists of ?


9. the cytokine secreted by Th1 cells which promotes the phagocytosis is . inf gamma
and IL 2
10. signalling to a cytotoxic t cell that the cell is infected with virus depends on the CD8+
MHC2
11. which one of the following mechanism may be involved in the clinical efficacy of
immunotherapy with allerger extracts(hyposensitization)
12. Laboratory examination results IgG antibodies specific to cytomegalovirus (CMV) in
the serum of pregnant woman.what is your conclusion? Acute active infection
13. the maximal reaction time for tuberculin type hypersensitivity is 21-28 hrs

14. Adjuvant causes Ans : release of cytokine

nolan x soju
1) Hemolytic disease of the caused by RH blood group complementary response
maternal antibody to fetal bloof stream therefore the nuclease of the disease is
a) IgE antibody
b) IgG antibody
c) IgM antibody
d) IgE antibody

2)which of the following is not correct regarding immunity to cancer?


a) Despite the immune response tumour continuous to grow
b) Activation of CD25+ cell lead to protective tumour immunity
c) Vaccination can be used to treat cancer
d) Patient with cancer have successful tumour immunity

2) An example if bacterial carcinogenic virus


a) Herpes simplex virus
b) HIV 2
c)
d) Epstein Barr virus

nolan x soju
3) Which components of MICs transplant can initiate graft rejection
a) HLA DP,DR,DQ that are also found on the body of recepients
b) Histocompatabilty antigen are found on the body of recipient
c) Histocompatabilty antigen that are also found on the body reception
d) Histocompatabilty antigen that can induce donor specific tolerance

4) Which one the following mechanism may be achieved on the clinical efficacy of immune
therapy with cause (hypersensitivity )
a. inhanced the production of igG which boundenlarge before it reach mast cell
b.inhanced production of igG which bound igG receptor before it reach antigen.
c.activation of mast cell that inducethe production of large of amount of anti histamine.
d.increase local recurrument of the esinophiles

5) Which of the following can activate complement system?


a.igM and igG
b. igD and ig G
c. igA and igE
D.igE and igA
15 ) a patient with severe asthemia get no relief from anti histamine these symptoms are
most likely to be caused by
a) IL-2
b) leukotrienes
c) Serotonin
d) bradykinin

16 ) identify molecule on the surface of an infected cells that are responsible for recognition
by host CD8+ cell?
a) CD4
b) CD8
c) MHC 1
d) MHC 2

18) which of the following has the most influence on antibody response carried out by B
lymphocytes?
a) A T cytotoxic cell surface molecule
b) Cytokines is released by TH cells
c) Anaphylactic reaction
d) Phagocyte presenting with T to TH cells

19. genes for immunoglobulin are unlike other human genes in that

nolan x soju
a.each polypeptide chain encoded by several intron
b.igG are composed of intron and exon
c.stomatic recombination occours before mRNA is trancripted
d.igG genes

20. lymphocytes occur their antigen specificity


a.they enter the tissue from the circulation
b.in the secondary lymphocyte organ
c.depend on which antigen are present
d.combination of v,d,j genes
25. Which of the following statements is FALSE?

a. An example of passive humoral immunity is treatment with horse anti-snake venin.


b. Antigen recognized by helper T cells must be associated with Class II MHC
molecules on the surface of professional APC .
c. Each lymphocyte has many antigen binding receptors, each receptor capable of
binding the same antigen.
d. Recognition and killing of virus-infected cells by cytotoxic T cells is an example of
adaptive immunity.
e. The innate immune system does not deal with endogenous antigen. Or
cytokines that derived development of t cell substance are not produced by antigen
presenting cells

26.complement
a.an antigen specific molecule
b.is an protein presented virus carrying into host cell
c.is an series of extracellular signalling protein
d.complement in circulaton is in inactive form

nolan x soju
27.immune system cell adherence molecules
a.dont allow macrophages to leave circulation
b.dont allow T cell to come pheriperal lymphoid tissue
c.help activated B cell producing high affinity antibody
d.help cytotoxic T cell to bind specific target

28.which is not characterize for both type 2 and type 3 hypersentitivity reaction
a.involve attack by tc cell
b.involve igG antibody
c.is likely to result is kidney damage
d.can be involved in exposure to pathogen.

29 circulating immune complexes are etiology factor of one of the following


a.goodpasteurs syndrome
b.farmers lungs
c.graves diseases

d.mesithenia gravi

30 b cell in the bone marrow undergo negative selection


a.binding antigen
b.do not bind the antigen
c.recognizing class 2 mhc presented
d.recognizing class 1 mhc presented
31 immune complex fragment cannot result from all except
a.presistant infection
b.inhalation of antigen
c.autoimmune disease
d.extradermal antigen

nolan x soju
32.failure to express class 2 HLA molecule on antigen presenting cells cause
a.tH cell deficiency
b.tc cell deficiency

c.b cell deficiency


d.nk cell deficiency
1. Mast cell responds to C3a by
Ans. secreting histamine and other allergic mediators

2. Identify the surface molecule expressed on the antigen presenting cells participating in
activation of the naïve T-cell?
Ans. B-7

3. Complement
Ans. Components are in the circulation in an inactive form

4. Which one of the following antibodies would be considered to have the greatest specificity for
an epitope?
Ans. The antibody with highest affinity for the epitope

5. Identify the component that does NOT actually help a phagocyte to attach to the surface of a
bacterial cell?
Ans. C3a

6. Phagocytes bind the pathogen using receptors for


Ans. Peptidoglycan

7. Antigenic determinant on Fc region of human g chain which is recognized by antibodies of


different person

nolan x soju
Ans. Isotype

8. Vaccination protects us from infectious disease by generating


Ans. Memory cells

9. If pathogens block the fusion of lysozymes with the phagocytic vesicle, they can be killed
through the effector function of
Ans. Cytotoxic T cells

10. Inflammation does NOT involve


Ans. Production and maturation of lymphocytes

11. A patient is admitted with multiple bacterial infections and is found to have a complete absence
of C6. which complement mediated function would be mostly affected in such a patient
Ans. Lysis of pathogen

12. Reaction to M tuberculosis and eggs of schistosomes is


Ans. A T cell mediated response

13. A transplant of tissue between individuals of different species is called a(n)


Ans. Xenograft
14. Hypersensitivity to gentamicin and hypersensitivity to nickel are both
Ans. Initiated by haptens

15. Which is the BEST method of reducing the effect of graft versus host disease in a bone marrow
recipient?
Ans. Removing mature T cells from the graft

16. Which of the following is NOT correct regarding autoimmune diseases?


Ans. Autoantibody production alone does not equal autoimmune disease

17. Which of the following clinical diseases is most likely to involve a reaction to a hapten in its
etiology?
Ans. Pollinosis

18. Hyperacute response differs from a first set response in that it involves
Ans. Antibodies which mark the tissue for ADCC

19. Anaphylactic reactions


Ans. May involve components of mast cells granule matrix

20. The tuberculin skin test is performed by injecting 0.ml of tuberculin purified protein derivative
(PPD) into the inner surface of the forearm. Histologically, the reaction site would MOST
probably show

nolan x soju
Ans. Th cells and macrophages

21. Why do allergens treat patients by injecting them with an allergen they are sensitive to?
Ans. Over a longer period of time there is a progressive decrease in IgE in the serum

22. Chemically induced tumors have tumor-associated transplantation antigens that


Ans. Are different for two tumors of different histological type even if induced by the same
carcinogen

23. An example of a known oncogenic virus is


Ans. Epstein Barr virus

24. Immunologic tolerance is defined as


Ans. Unresponsiveness of the immune system to an antigen which is induced by previous
exposure to that antigen

25. AIDS is caused by a human retrovirus which


Ans. Induces strong immune responses

26. Chronic granulomatous disease


Ans. Results from a defect of NADPH oxidase
27. Which cells are directed by HEV into the secondary lymphoid tissue?
Ans. T and B lymphocytes

28. Humans are most likely to mount a Th1 response to


Ans. Intravesicular pathogen

nolan x soju
1. Natural killer cells
Ans. Are stimulated to kill infected host cells with high levels of membrane MHC Class I

2. Lymphocytes are activated by antigen in the


Ans. Lymph nodes

3. Lymphocytes recirculation
Ans. Allows B cells to go to the secondary lymphoid organs

4. Which is associated with the expression of Class I MHC molecule


Ans. Stimulation of antibody production

5. An antigen binding signal at the membrane results in the mature B lymphocyte changing its
Ans. Gene expression

6. Signal transduction is the process of converting


Ans. A binding signal to a channel signal

7. When C3 binds to complement receptor on mast cell, the mast cell responds by
Ans. Apoptosis

8. A child stepped on a piece of glass 1 day ago, and an active inflammatory reaction is occurring in
her wound, with large numbers of neutrophils attracted to the inflammation site. Which of the

nolan x soju
following is the major chemotactic factor responsible for attracting neutrophils?
Ans. C5a

9. Antibody-dependent cell-mediated cytotoxicity (ADCC) is a process in which antibody-coated


cells are killed by
Ans. Complement

10. Somatic recombination occurs


Ans. In the progenitor cell as it is becoming a B cell

11. Loss of which of the following classes of molecules on the surface of an infected cell would
results in loss of susceptibility to killing by host CD8+ cells ?
Ans. MHC class I

12. Complement is
Ans. Present in the circulation in an inactive form

13. The alternative pathway of complement activation


Ans. Requires C3
14. Which one of the following is a fragment produced during complement activation that can
actually help a phagocyte to attach to the surface of a bacterial cell
Ans.C3b

15. Antigen entering the body in a subcutaneous injection activates its specific lymphocytes in the
Ans. Draining lymph nodes

16. Phagocytes bind antigen using receptors for


Ans. LPS

17. An inflammatory response


Ans. Recruits phagocytes to the infection site

18. Vaccination protects us from infectious disease by generating memory


Ans. Antigen

19. Allergy symptoms are protected when antigen binds to IgE on FcR on
Ans. Mast cells

20. One amino acid difference in the Fc region of different human g chains is the epitope recognized
by
Ans. Allotype

nolan x soju
21. Genes for immunoglobulins ARE UNLIKE other human genes in that
Ans. HLA-DR, -DP, and -DQ

22. The blood from an 8-year old boy was analyzed by flow cytometry. The exact number of B cells
was counted. Which of the following cell surface markers was likely used to identify the B cells
in this blood sample?
Ans. CD4

23. Administration of the diphtheria toxoid would stimulate which of the following types of
immunity?
Ans. Artificial passive

24. Antibody dependent cell-mediated cytotoxicity (ADCC) is not characteristic for


Ans. Cytotoxic T cells

25. The interaction that is not involved in naïve cell activation


Ans. TCR-MHC II peptide

26. Central (primary) lymphoid organs


Ans. Provide the microenvironment for maturation of T and B cells
27. Hematopoietic stem cells are pluripotent, which means that they are
Ans. Capable of developing into any blood cells

28. Toll-like receptors


Ans. Are activated by specific antigens
29. Peripheral lymphoid organs
Ans. Are designed to maximize contact between antigen and lymphocytes

30. The primary purpose of the adaptive immune system is to


Ans. Protect from disease upon re-infection with a specific pathogen

31. IgA can be secreted from the body because it


Ans. Binds poly-Ig receptor on mucosal epithelial cells

32. Human class I MHC alpha chain molecules are


Ans. HLA-A, -B and -C

33. Inflammatory cytokines produced by macrophages activate all of the following EXCEPT:
Ans. B cells to secrete acute phase proteins

34. TCR most closely resembles


Ans. Fab region of immunoglobulin

nolan x soju
35. Which classes of immunoglobulin has J chain?
Ans. IgA and IgM
1. Takes immune complex for removal (or) MHC II involves all except? – RBC
2. Syngeneic effect – same species
3. Prick’s test – type I hypersensitivity
4. C9 deficiency – decrease in Neisseria
5. Innate- macrophages & Neutrophils
6. Anti B bodies- A& O
7. Th1- viral
8. HIV -CCR5
9. AIDS – macrophages
10. Mycobacterium – granuloma
11. Massive histamine – wheezing and hypothermia
12. Case about NO serum immunoglobulins, and X-linked disorder-Bruton agammaglobulinemia
13. Germinal center – B lymphocyte
14. Double negative cell- upper cortex of thymus
15. Chemotaxis – C5a
16. B-chain synthesized first-………chain
17. Primary immune deficiency – infections
18. …………………. – NADPH oxidase
19. C56 – NK cell
20. You have purified some Fab from an IgG myeloma protein appropriate conditions; you could use
this Fab to generate antibodies – the isotype of this myeloma
21. Kidney transplant – AB group or MHC II presenting recipient antigen to recipient helper cells
22. Native B cell – spleen

nolan x soju
23. Anergic questions – unresponsiveness
24. J.Chain – IgA & IgM
25. Immune to chicken pox – IgG
26. Bee sting – systemic anaphylaxis
27. Hyperacute response differs from innate immune responses, hyperacute response includes –
preexisted antibodies
28. Artificial H3N1 influenza virus – artificial active
29. Graves disease – self-reactive lymphocytes
30. Microbes evade immune response – decrease production of MHC I in ER
31. Transport of immune complex to the site of removal in by – macrophages
32. Co stimulation signal for T cell – CD28
33. about person got – wasps
34. adoptive isotype is – hypervariable heavy and right chain
35. two identical animals (graft between animal) – syngeneic
36. patient with third degree burns and graft is required graft from which is less rejected – patient
37. choose correct answer – innate immune response does not induce memory
38. the isotype of antigen specific receptors found on fully mature B lymphocytes – IgM and IgD
39. which complement – mediated function would remain intact in such a patient – none of the
above
40. graft between genetically identical individuals – are not rejected even without immune
suppression
41. type III -complement depend
42. Type IV – delayed hypersensitivity
43. Which of the following immunoglobin has 2 division – IgA

nolan x soju
1. Rhesus hemolytic disease of the newborn involves
Ans. Antibody to cell surfaces

2. The Arthus reaction is characterized by an intense infiltration by:


Ans. Neutrophils

3. Type IV hypersensitivity is often referred to as:


Ans. Delayed

4. The major effector molecule involved in Type IV hypersensitivity reactions are.


Ans. Cytokines

5. Pigs have been proposed as a source of organs for xenotransplantation to humans


Ans. Their hearts are an appropriate size, and the pigs can be genetically modified to limit the
possibility of hyperacute rejection

6. The very rapid response to a second allogenic graft is:


Ans. Specific for antigens of the major histocompatibility complex (MHC)

7. Hyperacute graft rejection is caused by:


Ans. Preformed antibody

8. Which of the following allogenic grafts does not require immunosuppression:

nolan x soju
Ans. Cornea

9. Antigens normally expressed only on embryonic cells but also sometimes found on tumors are
known as:
Ans. Oncofetal antigens

10. Studies performed using tissue specimens from cancer patients have identified several types of
immune cell infiltrates that are associated with a good prognosis in different human
malignancies. Which one of these immune system cells infiltrate is most likely to be associated
with a poor prognosis?
Ans. Cytotoxic T cells

11. A 54-year old male is diagnosed with cancer and given a cytokine with anti-tumor activity part of
his treatment. Which of the following cytokines was most likely administered to this patient?
Ans. IFN-gamma

12. Chemically induced tumors have tumor-associated transplantation antigens that


Ans. Are different for two tumors of different histologic type even if induced by the same
carcinogen

13. Malignant lymphoid cells:


Ans. Show maturation arrest at characteristic stages in differentiation

14. CD44 is a molecule which may be involved in:


Ans. Metastatic spread

15. Which of the following are compatible with a diagnosis of auto-immunity:


Ans. Delayed type hypersensitivity in tuberculosis

16. Which of the following is not true about rheumatoid arthritis:


Ans. Rheumatoid factor is an antibody with reactivity to the heavy chain of IgG

17. A 67 year old woman develops graves disease, an autoimmune disease characterized by the
presence of antibodies directed against thyroid-stimulating hormone receptors in the thyroid
gland. Her condition results from:
Ans. Failure of her immune system to prevent activation of self-reactive lymphocytes

18. A 27 year old male presents with fatigue dyspnea and tachycardia. She has a decreased
hemoglobin level, and a coombs test result is positive for the presence of antibodies on
erythrocyte surfaces. The patient is currently taking an antibiotic for a urinary tract infection.
Her immune response is best described as a/an:
Ans. Type II hypersensitivity

19. Which of the following is a non-organ-specific (systemic) autoimmune disease:

nolan x soju
Ans. Systemic lupus erythematosus

20. Neonatal myasthenia gravis is thought to be caused by:


Ans. Transplacental transfer of maternal IgG against the acetylcholine receptor

21. In type 1 diabetes (insulin dependent diabetes mellitus), the target of the autoimmune attack is:
Ans. The b-cells in the islets of Langerhans

22. Which one of the following mast cell products is not performed and therefore has to be newly
synthesized?
Ans. Prostaglandin D2

23. Individuals lacking C8 or C9 are more prone infection with the following type of bacteria:
Ans. Neisseria

24. Specific immunity to Tuberculosis in mice can be transferred to naïve histocompatible recipients
by:
Ans. T-cells

25. Lepromatous leprosy is characterized by:


Ans. Poor T-cell responses
26. Viral antigen shift involves:
Ans. Interchange of large segments of the viral genome with other viruses

27. Which of the following is not true about interferons


Ans. Increase virus speed

28. Cytotoxic T-cells:


Ans. Restrict viral replication

29. Classical NK cells:


Ans. Can kill virally-infected cells

30. IgE levels are high in infection with:


Ans. Necator americanus

31. Inflammation is a defensive reaction initiated by infection or tissue injury which first causes:
Ans. Upregulation of adhesion molecules on endothelia cells and leukocytes

32. Type I hypersensitivity can be blocked using


Ans. Histamine

33. Which type of hypersensitivity CAN Not be transferred with serum antibody?
Ans. Type IV

nolan x soju
34. Anaphylaxis can be triggered by cross linking of IgE receptors on:
Ans. Mast cells

35. Defects in neutrophil NADPH oxidase system produce:


Ans. Chronic granulomatous disease

36. Paroxysmal nocturnal hemoglobinuria results from deficiency in:


Ans. Decay accelerating factor (DAF)

37. X-linked agammaglobulinemia results from a mutation in:


Ans. A tyrosine kinase gene

38. Di George syndrome results form a defect in:


Ans. Thymic development

39. HIV binds to:


Ans. CD4

40. Poor skin tests to a range of microbial antigens such as tuberculin and mumps indicatea
deficiency of:
Ans. T-cells

41. A 42 year old female has seasonal allergic rhinitis. Her symptoms result from allergen triggering
or release of vasoactive amines by:
Ans. Cross linking IgE on mast cell surfaces

42. A 26 year old male presents with persistent nasal congestion, rhinorrhea, sneezing and nasal
pruritus 6 months after moving in with a roommate who has a pet cat. An oral antihistamine has
revealed most of his symptoms. His signs and symptoms most likely result from processes
mediated by:
Ans. IgE

43. A 10 year old male comes into contact with poison ivy. He develops contact dermatitis. The
immune response involved is best described as:
Ans. Type IV hypersensitivity

44. A 33 year old female presents for evaluation of pain and swelling in both wrists that is worse in
the mornings. Both cell-mediated and humoral immunity are believed to be involved in her
condition. These findings best support a diagnosis of:
Ans. Rheumatoid arthritis

45. A 67 year old woman develops graves disease an autoimmune disease characterized by the
presence of antibodies directed against thyroid-stimulating hormone receptors in the thyroid

nolan x soju
gland. Her condition. These findings best support a diagnosis of:
Ans. Failure of her immune system to prevent activation of self reactive lymphocytes

46. A 27 year old male presents with fatigue, dyspnea and tachycardias. She has a decreased
hemoglobin level, and a Coombs test result is positive for the presence of antibodies on
erythrocyte surfaces. The patient is currently taking an antibiotic for a urinary tract infection.
Her immune response is best described as a/an:
Ans. Type II hypersensitivity

47. Which of the following is a non-organ specific (systemic autoimmune disease


Ans. Systemic lupus erythematosus

48. Neonatal myasthenia gravis thought to be caused by:


Ans. Transplacental transfer of maternal IgG against the acetylcholine receptor

49. In type I diabetes (insulin dependent diabetes mellitus), the target of the……. Attack is.
Ans. The b cells in the islets of Langerhans

50. Which one of the following mast cell products is not performed and therefore……..newly
synthesized?
Ans. Prostaglandin D2

51. Type I hypersensitivity can be blocked using:


Ans. Sodium cromoglycate

52. Which type of hypersensitivity CANNOT be transferred with serum antibody/


Ans. Type IV

53. Anaphylaxis can be triggered by cross linking of IgE receptors on:


Ans. Mast cells

54. Septic shock associated with gram-negative bacteria is primarily due to:
Ans. Liposaccharide

55. Neutrophil chemotaxis is mediated by:


Ans. IL-8

56. Opsonization of bacteria occurs through coating bacteria just with:


Ans. C3b

57. Extracellular bacteria are optimally killed by:


Ans. Macrophages plus complement

nolan x soju
58. Extracellular bacteria try to avoid killing by:
Ans. Synthesizing capsules

59. Toxins are neutralized by:


Ans. Antibody

60. CR1 complement receptors on phagocytic cells bind:


Ans. C3b

61. Secretory IgA protects external mucosal surfaces by:


Ans. Preventing microbial adherence to the mucosa

62. Studies performed using tissue specimens from cancer patients have identified several types of
immune cell infiltrates that are associated with a good prognosis in different human
malignancies. Which one of these immune system cell infiltrates is most likely to be associated
with a poor prognosis?
Ans. Cytotoxic cells

63. A 54 year old male is diagnosed with cancer and given a cytokine with anti-tumor activity as part
of his treatment. Which of the following cytokines was most likely administered to this patient?
Ans. IFN-gamma
64. Correct statements on immune deficiency syndrome are:
Ans. Patients with disorders of antibody formation are prone to infections with encapsulated
bacterial pathogens

65. A 30 year old male with a disease also has mycobacterium avium complex infection, candida
esophagitis and pneumocystis carinii pneumonia, which are opportunistic infections. He has a
history of repeated infections that do not respond well to standard treatments. What type of
disease might you suspect that this individual has?
Ans. An immune deficiency disease

66. A 4 year old male has impaired thymus development with greatly reduced numbers of
functional T cells. He has hypocalcemia. Cardiac abnormalities are also noted. The most likely
cause of the finding is:
Ans. Di George syndrome

67. A lack of development of secondary follicles in the lymph nodes, appendix and spleen indicates:
Ans. A deficient B-cell system

68. Examples of primary T-cell deficiency disease DO NOT include:


Ans. AIDS

69. Recently published studies suggest that immune system cells play a role in both the inhibition as
well as the progression of cancer growth. All of the factors below contribute to the complex

nolan x soju
nature of the human immune response to cancer except:
Ans. Human tumors are not immunogenic and do not stimulate the immune system

70. Detectable serum antibody against a T-independent pathogen is a good indication that
Ans. A functional B-cell system exists

71. Positive skin tests showing delayed-type hypersensitivity, such as for mumps or tuberculosis,
indicate that
Ans. A cell mediated immune response has occurred

72. A t cell deficiency associated with thymic hypoplasia leads to infection of the following type(S):
Ans. Viral and intracellular bacterial

73. Infantile, x linked agammaglobulinemia is associated with excessive infections of the following
type(s):
Ans. Extracellular bacterial (e.g., Staphylococcus)

74. Chemically induced tumors have tumor associated transplantation antigens that
Ans. Are different for two tumors of different histologic type even if induced by the same
carcinogen
75. Malignant lymphoid cells:
Ans. Show maturation arrest at characteristic stages in differentiation

76. CD44 is a molecule which may be involved in:


Ans. Metastatic spread

77. A graft between members of the same species is termed an:


Ans. Allograft

78. The human major histocompatibility complex:


Ans. Provokes the most intense allograft reactions

79. Non-specific suppression of graft rejection can be achieved with:


Ans. Anti CD34

80. Graft vs host disease often accompanies transplantation of:


Ans. Bone marrow

nolan x soju
4.natural killer cells:
A. are stimulated to kill infected host cells via antibody- dependant cell cytotoxicity
5.immune system cell co-stimulatory molecules
B-allows t and b cells to go to the secondary lymphoid organs
6.B lymphocyte recognize native antigen:
TRUE
7.individuals unable to make the J protein found in certain immunoglobulin would be
expected to have frequent infections of the:
intestinal tract
8.biological functions of the Fc fragment is:
Neutralization of viral particles
9.which immunoglobulin(s) has different subtypes?
IgA & igM
10.which of the following chiefly found on the surface of B cell receptor molecules and
is involved in cell activation?

nolan x soju
IgD
36.WHICH IS CORRECT
a) IgG is transferred across the placenta
b) IgM exists in serum in both monomeric and multimeric forms
c) IgM exists predominantly as membrane bound form
d) IgG is predominant Ig in primary responses
37.the antigenic determinant is the combining site of an antibody

38. DCs are able to present internalized antigen to T cells via MHC class I
39.MHC molecules present antigenic determinants to B cells

40.Which is correct
a) 1-5% of the progenitor T cells which enter the thymus leave as mature T cells
b) 95% of the progenitor T cells which enter the thymus leave as mature T cells
c) 50% of the progenitor T cells which enter the thymus leave as mature T cells
d) 10% of the progenitor T cells which enter the thymus leave as mature T cells
51.A cytokine that promotes cell-mediated immunity and is produced by Th1 cells is an
A. INF-Y
52.which of the following pairs of cytokines could be said to have antagonistic activities
C. IL-1 and TNF-a
53.production of IgG antibodies against thymus dependent antigens requires

B. Interaction of T and B cells in germinal center

54.which of the following best describes NK cell morphology

A. Large granular lymphocytes

55.which of the following does not contribute to the diversity of the TCRs
A. Junctional flexibility

46. Is it true or false about thymus independent antigens? Against them the secondary
immune response antibodies affinity are higher than in primary immune response?

nolan x soju
True
47. An anti inflammatory cytokine is an..
A. IL-4 B. IL-6 C. IL-10 D. IL-17 E. INF-gamma
48. A cytokine that promotes humoral immunity and is produced by Th2 cells is an..
A. IL6 B. IL4 C. INF D. IL10 E. IL12
49. CD40 on the surface of B lymphocytes binds to which of the following Th cell surface
molecule?
A. CD40L B. ICAM C.TcR D.TcR
50. Secondary immune response is rapid than primary response?
True
28. Which one of the following is a fragment produced during complement activation that
can actually help a phagocyte to attach to the surface of a bacterial cell?
A.C3 B.C3 convertase C.C3a D.C3b
29. Phagocytes bind antigen using receptors for
A.C5a B. Chemokines C. Glucose D.LPS E. Selectin
30. Isotype switching
A. Changes the leader sequence exon so the antibody is secreted
B. Improves the antigen binding specificity of an Ig molecule
C. Increases the affinity of antibodies in a process called antibody maturation
D. Increases the functional diversity of Ig molecules
E. Occurs randomly between switch regions

0.Which of the following cytokines secreted by TH1 cells “help” macrophages to become
more efficient phagocytic cells?
Interferon-V (gamma)

0. A 14-year-old girl was referred to a radiology because of abdominal pain and vague
respiratory symptoms. The patient’s stool was positive for ova of a common
parasite. What is likely to be elevated in this patient?
eosinophils

nolan x soju
0. A neonate is found to have a mutation in the fcv receptor gene. What immune
function is likely to be affected?
: Intracellular killing

0. F(ab)2 fragments prepared from an IgG antibody to sheep red blood cells should still
be able to carry out:
Agglutination

0. The major histocompatibility complex proteins function is to:


bind antigen fragments for presentation to T-cells

0. Which of the following cells may eventually differentiate into an


immunoglobulin-bearing memory cell
naïve B-cell
0. Which immunoglobulin is the principal one found in secretion such as milk?
: IgA
0. The immunoglobulin monomer is composed of
two identical heavy chains and two identical light chains

0. A plasma cell which secretes IgA dimers is also producing


J Chain

0.The immunoglobulin joining chain (j-chain) is


is produced by B cell
61) Antibodies are immunoglobins.

TRUE

62) Is it true or false about thymus independent antigens class switching does not take place?

TRUE

63) Is it true or false about thymus independent antigens, they are mostly proteins.

FALSE

nolan x soju
64) Is it true or false about thymus independent antigens, they activate B-1 cells.

TRUE

65) A T-cell located at the epithelium barrier of the gut is a,

(γδ) T cells

22. T cells that recognize MHC class I differentiate in the thymus to become which type of
cell?
A. CD8+ cytotoxic lymphocyte

// natural T killer cell


T helper 1
T helper 2 cell
(γδ) T cells

23. during the maturation of a B lymphocyte , 1st immunoglobin heavy chain synthesized is
the ;

μ heavy chain
24. genes for Immunoglobulins (antibodies ) are UNLIKE other human genes in that
DNA for antibody molecules is inherited from only one parent .

31. C3a and C5a can cause


B. Vascular permeability
32. The role of follicle dendritic cells during an antibody response is to
D. Present antigen to B cell and promote affinity maturation
33. Neutrophils are attracted to an infected area by
C. IL 8

37) Both class 1 and class 2 molecules are?


B - Expressed constitutively on all nucleated cells.

38) The ligand for TCR is?


C - MHC+ Peptide

nolan x soju
39) T cells are MHC-restricted in their ability to respond to antigen because?
D - TCR must recognise both antigen and MHC molecule
40. Human MHC class II molecules
C. Have an antigen binding site formed from the regions of 2 polypeptide chains.

25. Loss of which of the following classes of molecules on the surface of an infected cell
would result in loss of susceptibility to killing by host CD8+ cells?
Ans) D -MHC class 1
26)complement is?
Ans) E-present in circulation in an inactive form
27)The alternative pathway of complement activation?
Ans)D-requires C3
41. Only processes antigen is presented to T lymphocytes
42. The antibody response to a hapten such as DNP is usually polyclonal
43 Tick the correct answer
Ans)Microbial “superantigens” produce the symptoms of toxic shock sydrome by causing
excessive activation of CD8+ T cells
44) correct answr
MHC molecules present antigenic determinants to T cells
45. IGA
IgA is a secretory immunoglobulin

34. Somatic hypermutation

A –occur by somatic recombination

35. CD8 is a co-receptor on T cells that binds:

C –constant region of MHC Class 1

36.Which is true for antigen receptors on both B and t cells

D –specific for a single antigenic epitope

56. Co-stimulatory signal for T cell activation is:

D - TCR-MHC CLASS 1 + Ag

nolan x soju
57.Which of the following pairs of cytokines could be said to have antagonistic activities?

A– IL-10 and IFN-g

58. antibodies are glycoprotein

TRUE

59.antibodies are made up of 1 heavy and 2 light chains

.FALSE

60. differentiation of myeloid cells from committed progenitor to matured cells occurs primarily in:

B- BONE MARROW

1) The ___ T cell-APC interaction is MHC ___ -restricted, and the target cell
interaction is MHC___ -restricted.
Ans) [D] CD8+, Class II; CD4+, Class I
2) Match the immunoglobulin(s) with the functional description:
predominant in the primary (early) immune response.
Ans) [C] IgG & IgM
3) what class of MHC is not found on the surface of erythrocytes?
Ans) [C] Class I MHC
4) What types of cells is class II MHC found on?
Ans) [B] Dendritic cells
5) The invariant chain ___ the empty peptide-binding groove. After vesicle
fusion, the invariant chain is ___ and peptides can enter the MHC class 2
groove.
Ans) [D] Blocks; Degraded
66) A T cell located at the epithelial barrier of the gut is a
Ans) [C] (γδ) T cell

67) which of the following pairs of cytokines have redundant activities?


Ans) [A] IL-6 and IL-1

nolan x soju
68) is it true or false about thymus independent antigens,
There are lot of memory cells
Ans) False

1) Each of the following statements concerning class I MHC proteins is correct


EXCEPT:
a) They are cell surfaces proteins on virtually all cells
b) They are recognition elements for cytotoxic T cells
c) Are co-dominantly expressed
d) They are important in the skin test response to Mycobacterium tuberculosis

2) Clonal selection
a) Begins with inflammation
b) Occurs for all leukocytes
c) Occurs in response to self-antigens
d) Results in innate immunity
e) Results in proliferation of antigen-specific lymphocytes
3) An inflammatory response
a) In characterized by a decrease in vascular permeability
b) Is stimulated by cytokines produced by neutrophils
c) Occurs only during a secondary response
d) Recruits phagocytes to the infection site
e) Usually lasts for many weeks to ensure antigen is completely removed
16. Cross-presentation allows APCs to acquire antigens from infected cells

17. J chains are generally associated with

—b— Polymeric Immunoglobulins (More than two Fab´s).

18. Affinity is determined by

—The aminoacidsequence in the Fab regions of the antibody molecule

19- MHC molecules are polymorphic

nolan x soju
20. IgM and IgC can fix complement

10. Membrane IgM and IgD on the surface of individual B cells:

B— Have idetical specificities

11. An antigen binding signal and the membrane results in the mature B lymphocytes changing its

D— gene expression

12. Signal transduction is the process of converting

B— a binding signal to a chemical signal

13- When c3a is bind to complement receptor on mast cell the mast cell responds by:

A- apoptosis

B- presenting the antigen to the cell

C- secreting IgE
D-secreting histamine and other allergic mediators

E- Stimulating macrophage in the neutrophil phagocytosis of the coated antigen

14-A child stepped on a piece of glass one day ago and an active inflammatory reaction is a curing in
her wound with large numbers of neutrophils attracted to the inflammation site which of the
following is the major chemotactic Factor responsible for attracting neutrophils

A-C3b

B- C5a

C- IGM

D- I L2

E- lysozyme

15- A 60-year old alcoholic male with a streptococcus pneumoniae is brought to the emergency
department which of the following immune effector mechanism was most important incompletely
clearing this infection

A- Antibody-dependent cell cytotoxicity

B-natural killer cells

C- complement-mediated opsonization

nolan x soju
D- cytotoxic t-cell lymphocytes

----------------

21.

a) B lymphocyte recognize native antigen

b) Only processes antigen is presented to T lymphocyte

c) Hapten possess one antigenic determinant

d) T but not B lymphocytes express antigen-specific receptor of only one specificity

22.the CD3 complex of the T-cell receptor:

a) Binds complement

b) Functions to transduce a signal to the cell’s interior following binding of antigen

c) Causes the histamine release

d) Present antigenic determinant to T cells


23. Which of the following T cells would survive the process of negative selection?

a) Those bearing a TcR specific for self MHC + foreign antigen

b) Those bearing a TcR specific for self MHC + self antigen

c) Those bearing a TcR specific for foreign MHC + foreign antigen

d) Those bearing a TcR specific for foreign MHC + self antigen

24. Members of the immunoglobulin superfamily are mainly molecules with a receptor function.
Which of the following molecules have the MHC II as its natural ligand?

CD2

CD3

CD28

CD4

25. Antibodies are protective primarily against invading extracellular organisms

a)Used in combination with an antigen fragment to mark a cell for killing cytotoxic t cells

b) are used to participate and helper functions

nolan x soju
c) can not bind anantigen fragment

d) recognised by the cd4 molecules

// Lymphocyte recirculation

a. activates inflammatory cytokines to promote antigen presentation to T cells

b. allows T and B cells to go to the secondary lymphoid organs

C. circulates lymphokines efficiently throughout the body

d. occurs for both naïve and effector lymphocytes

e. only occurs during an infection

// A FUDAMENTAL difference between the antigen receptors on B cells (BCR) and on T cells (TCR) is
their

a. different requirements for antigen presentation.

b. function following antigen binding

c. heterogeneity from one lymphocyte to the next

d. heterogeneity on each lymphocyte


e. membrane location

// Natural killer cells are:

Able to kill virus-infected cells without prior sensitazation.

// Isotope switching" of immunoglobulin classes by B cells:

a. occurs in bone marrow

b. involves successive insertion of a VH gene adjacent to different CH genes

c. occurs before gene rearrangement

d. involves switching of light chain types

// The membrane IgM and IgD the surface of individual B cell:

a. have identical heavy chains but different light chains

b. have identical specificities

C. are identical except for their VH regions

d. have different VH and VL regions

nolan x soju
// The blood from an 8-year-old boy was analyzed by flow cytometry. The exact number of T helper
cells was counted. Which of the following cell surface markers was likely used to identify the Th cells
in this blood sample?

a. CD3

b. CD4

c. CDS

d. CD2

e. CDS6

// Administration of tetanus toxoid would stimulate which of the following types of immunity?

a. adoptive

b. artificial active

c. artificial passive

d. natural active

e. natural passive
// Which marker or markers is present on B cells and could be used to specifically identify such cells
flow Cytometric analyzer?

a. CD 3

b. CD8

c. CD 14

d. CD 16 and CD 56

e. CD 19 and CD 20

// Peripheral lymphoid organs

Are designed to maximize contact between antigen and lymphocytes

// the primary purpose of the adaptive immune system is to

Protect from disease upon re-infection with a specific pathogen

// IgA can be secreted from the body because it

a. binds poly-Ig receptor on mucosal epithelial cells

nolan x soju
b. has a Specialized H chain called secretory chain

c. has a special secretory idiotype.

d. is small enough to pass between mucosal epithelial cells and leave the body

e. is synthesized by mucosal epithelial cells and secreted directly into the intestinal lumen

// Human Class I MHC chain molecules are

a.82-microglobulin

b. H-2 D, K, and

c.H-2 IA and IE

d. HLA-A. -B, and-C

e. HLA-DR, -DP, and DO

// Inflammatory cytokines produced by macrophages activate all of the following EXCEPT:

a. B cells to secrete acute phase proteins


b. integrin on leukocytes to bind more strongly to vascular CAMS

C. neutrophils to be more cytotoxic

d. NK cells to kill virus-infected cells

e. vascular endothelium to increase expression of CAMs

// TCR most closely resembles

a. Class I MHC

b. Class I MHC

c. Fab region of immunoglobulin

d. Fe region of immunoglobulin

e. light chain of immunoglobulin

// Which classes of immunoglobulin has J chain?

a. IgA and IgM

b. IgG and IgM

c. IgD and IgE

nolan x soju
4. IgM and lgD

// rapid but non-antigen specific immune responses are produced by the

Innate immune system

// Vaccination protects us from infectious disease by generating memory

a. antigen

b. lymphocytes

c. macrophages

d. PMNs

e. stem cells

// allergy symptoms are produced when antigen binds to IgE on FcR on

Mast cells

// one amino acid difference in the Fc region of different human g chains is the epitope recognized
by anti-

Allotype
// which is associated with the expression of class 1 MHC molecules:

Stimulation of antibody production

// somatic recombination occurs

In the progenitor cell as it is becoming a B cell

// Administration of diphtheria toxoid would stimulate which of the following types of immunity?

Artificial passive

// Antibody-dependent cell-mediated cytotoxicity (ADCC) is not characteristics for

Cytotoxic t cells

// Toll like receptors

Are activated by specific antigens

// hematopoietic stem cells are pluripotent, which means that they are

Capable of developing into any blood cells

nolan x soju
// central (primary) lymphoid organs

Provide the micro environment for maturation of t and b cells

// Antigen entering the body in a subcutaneous injection activates its specific lymphocytes in the

a. blood circulation.

b. draining lymph nodes.

c. MALT.

d. skin.

e. spleen.

// Lymphocytes are activated by antigen in thee

a. blood stream.

b. bone marrow

c. liver

d. lymph nodes

e. skin
Which is the first of the following genes to be upregulated subsequent to T-cell activation?

IL-2 receptor

The αβ heterodimeric form of the IL-2 receptor

Binds IL-2 with high affinity

Which of the following is characteristically produced by the Th2 CD4 cells which provide
help for antibody production, but not by Th1 cells?
IL-4

What is the purpose secreting interleukin-2?


To activate Helper T cell

The immune cell responsible for the IL-2, IFN-gamma, TNF production is the
Th1

Listeria monocytogenes, a classic intracellular bacterium, would most likely elicit:


MHC-II antigen presentation and CD4Th1 cells

nolan x soju
High affinity B-cell clones in mammals are usually generated by *
Somatic hypermutation

The immune cell responsible for the IL-4, IL-5 production is the
Th2

Prior to class switching, B-cells express


BCR, IgM

Which of the following statements is TRUE of cytokines?


Cytokines can be pleiotropic

Which is the first of the following genes to be upregulated subsequent to T-cell activation?
il-2 receptor

A toxin that forms a bridge between CD4 T cell receptors and MHC class II molecules
causing T cells to divide and differentiate are called
Superantigen

What do Th1 cells secrete?


IL-2

Which of the following statements is TRUE of cytokines?


Cytokines can be pleiotropic

Prior to class switching, B-cells express


BCR, IgM

Which of the following is characteristic of B- but not T-cells?


Transmembrane surface immunoglobulin

What is the T-cell ligand binding B7 on a professional antigen-presenting cell?


CD28

When a resting naive T-cell engages its specific MHC–peptide complex displayed on the
surface of a fibroblast, it
Becomes anergic

nolan x soju
For production of secondary antibodies is required the following molecules
CD40

Cytotoxic T cells kill target cells _________


through insertion of perforins into the target's membrane

Dendritic cells can be driven from a resting state to an activated state by the T-cell surface
molecule
CD40L

Cytotoxic T cells kill target cells _________


through insertion of perforins into the target's membrane

IFNγ and TNF (TNFα) can act synergistically


To upregulate expression of MHC class II

What is the purpose secreting interleukin-2


To activate Helper T cell

High affinity B-cell clones in mammals are usually generated by


Somatic hypermutation
This cell helps B cells convert to plasma cells:
CD4TH2

Dendritic cells can be driven from a resting state to an activated state by the T-cell surface
molecule
CD40L

What do Th2 cells secrete?


IL-4

The αβ heterodimeric form of the IL-2 receptor


Binds IL-2 with high affinity

What is the T-cell ligand binding B7 on a professional antigen-presenting cell?


CD28

Tolerance is induced by
Simultaneously administration of antigen and adjuvant

In what other ways does IL-1 mediate interactions between the immune and nervous
systems?

nolan x soju
It causes an increase in body temperature and suppress appetite

Biological functions of the Fab fragment is:


All answers is correct
nolan x soju

Copy protected with Online-PDF-No-Copy.com

You might also like